You are on page 1of 220

EBD_8177

Corporate Office
DISHA PUBLICATION
45, 2nd Floor, Maharishi Dayanand Marg, Corner Market, Malviya
Nagar, New Delhi - 110017
Tel : 49842349 / 49842350

© Copyright Disha
No part of this publication may be reproduced in any form without prior permission
of the publisher. The author and the publisher do not take any legal responsibility
for any errors or misrepresentations that might have crept in. We have tried and
made our best efforts to provide accurate up-to-date information in this book.

All Right Reserved

Typeset by Disha DTP Team

Review this book at


Check latest updates for this book

www.dishapublication.com www.mylearninggraph.com
Books & Etests
ebooks for
School & for
Competitive Competitive
Exams Exams

Write to us at info@dishapublication.com
CONTENTS
SECTION A : VERBAL ABILITY

1. Vocabulary/One Word Substitution  A-1–13

2. Error Detection  A-14–18

3. Sentence Completion/Cloze Test  A-19–24

4. Reading Comprehension  A-25–32

SECTION B : NUMERICAL ABILITY


1. Number System/Simplification  B-1–9

2. Arithmetic  B-10–29

3. Algebra  B-30–37

4. Time, Speed and Distance  B-38–44

SECTION C : REASONING & MILITARY APTITUDE


VERBAL REASONING

1. Analogy & Odd One Out  C-1–12

2. Series / Coding-Decoding  C-13–14

3. Logical Deduction / Statement & Assumption  C-15–24

NON-VERBAL REASONING

4. Analogy / Odd One Out / Series  C-25–37

5. Completion of Figure/Embedded Figure  C-38–54

6. Spatial Ability/Image Analysis/Rotated Figure  C-55–60

SECTION D : GENERAL AWARENESS


1. History/Civics & Polity  D-1–8

2. Geography/General Science  D-9–16

3. Miscellaneous D-17–32
EBD_8177
PRACTICE SETS 
1. Practice Set-1  PS-1–10

2. Practice Set-2  PS-11–18

3. Practice Set-3  PS-19–28

4. Practice Set-4  PS-29–38

5. Practice Set-5  PS-39–48


Section-A : Verbal Ability

Vocabulary/One
1 Word Substitution
DIRECTIONS (Qs. 1 - 3) : In each of the following choose the 11. PERMEABLE means the same as
word most similar in meaning to the word given in capitals. (a) flexible (b) variable
(c) soluble (d) penetrable
[2011-I]
12. CONVOY means the same as
1. CAUSED (a) carry (b) flock
(a) Brought about (b) Brought forward (c) standard (d) escort
(c) Brought out (d) Brought over 13. CITE means the same as
2. PLACID (a) illustrate (b) reveal
(a) Plain (b) Clear (c) recollect (d) quote
(c) Poor (d) Calm 14. VOCATION means the same as
3. AUDACIOUS (a) hobby (b) occupation
(a) Obvious (b) Daring (c) post (d) designation
(c) Ardent (d) Affluent DIRECTIONS (Qs. 15 - 16) : Each of the following questions has
DIRECTIONS (Qs. 4 - 6) : Pick out the word that is most nearly an underlined/capitalized word. You are to indicate which one
the opposite in meaning to the word given in capitals. [2011-I] of the four choices most nearly means the same as the underlined/
4. MONOLOGUE capitalized word. [2011-I]
(a) Prologue (b) Epilogue 15. The benefits of the plan are likely to be transitory.
(c) Dialogue (d) Catalogue (a) significant (b) obvious
5. DELETE (c) temporary (d) cumulative
(a) Imbibe (b) Improve 16. The hikers found several crevices in the rocks.
(c) Insert (d) Inspire (a) cracks (b) minerals
6. AMBIGUITY (c) canals (d) puddles
(a) Certainty (b) Clarity DIRECTIONS (Qs. 17 - 19) : In each of the following choose the
(c) Rationality (d) Laxity word most similar in meaning to the word given in capitals.
DIRECTIONS (Qs. 7 - 14) : This is a test of your ability to 17. VENERATE [2011-II]
understand words. For each question four options are given. (a) Reject (b) Remove
There is only one correct answer for each question. Mark the (c) Love (d) Respect
correct answer. [2011-I] 18. VACILLATE
7. INCITE means the same as (a) Waver (b) Disintegrate
(a) short (b) delay (c) Relegate (d) Salute
(c) place (d) provoke 19. FELICITY
(a) Zeal (b) Excitement
8. SUCCUMB means the same as
(c) Happiness (d) Expertise
(a) aid (b) yield
DIRECTIONS (Qs. 20 - 23) : Pick out the word that is most nearly
(c) check (d) oppose
the opposite in meaning to the word given in capitals.
9. ANOMALOUS means the same as
20. HYSTERIA [2011-II]
(a) disgraceful (b) formless (a) Disease (b) Ceremony
(c) irregular (d) threatening (c) Serenity (d) Frenzy
10. FORTUITOUS means the same as 21. ABSTAIN
(a) accidental (b) conclusive (a) Indulge (b) Dismiss
(c) courageous (d) prosperous (c) Repel (d) Acquire
EBD_8177
A-2 Vocabulary/One Word Substitution
22. SEDENTARY 35. INSOLENT
(a) Inactive (b) Sluggish (a) Affable (b) Spotted
(c) Moving (d) Settled (c) Foolish (d) Mature
23. Relaxed means the same as the opposite of [2011-II] DIRECTIONS (Qs. 36 - 38) : In each of the following, choose the
(a) calm (b) angry word most similar in meaning to the word given in capitals.
(c) tense (d) sleep 36. “MEDDLE” [2013-I]
DIRECTIONS (Qs. 24 - 26) : In each of the following choose the (a) Disregard (b) Overlook
word most similar in meaning to the word given in capitals. (c) Interfere (d) Free
24. EMANCIPATE [2012-I] 37. “ABJURE”
(a) Set free (b) Exist (a) Renounce (b) Run off secretly
(c) Correct morally (d) Restrain (c) Abide (d) Discuss
25. DECEIT 38. “ESTRANGE”
(a) Simplicity (b) Gentility (a) Endanger (b) To become puzzling
(c) Sincerity (d) Dishonesty (c) Miscalculate (d) Alienate
26. ADMONITION
DIRECTIONS (Qs. 39 - 40) : In each of the following, choose the
(a) Thrash (b) Hindrance
word most nearly opposite in meaning to the word given in
(c) Warning (d) Exhort capitals. [2013-I]
DIRECTIONS (Qs. 27 - 28) : In each of the following choose the 39. “DEROGATORY”
word most nearly opposite in meaning to the word given in
(a) Conferred (b) Immediate
capitals. [2012-I]
27. VOCIFEROUS (c) Praising (d) Private
(a) Laudable (b) Quiet 40. “WANE”
(c) Dangerous (d) Powerful (a) Widen (b) Poor
28. IMPLICATE (c) Swell (d) Tight
(a) Involve (b) Exonerate DIRECTIONS (Qs. 41 - 42) : Choose the word which is nearest
(c) Corrupt (d) Accuse in meaning to the given word : [2014-I]
DIRECTIONS (Qs. 29 - 31) : In each of the following choose the 41. ‘TRANSGRESSOR’
word most similar in meaning to the word given in capitals. (a) Passenger (b) Law-breaker
29. OSMOSIS [2012-II] (c) Protector (d) Comrade
(a) Gradual acceptance (b) Slow recovery 42. ‘EGREGIOUS’
(c) Abrupt ending (d) Strength (a) Common (b) Social
30. OSTRACISE, (c) Plain (d) Atrocious
(a) Take away (b) Cut off DIRECTIONS (Qs. 43 - 45) : Choose the word which is nearly
(c) Include (d) Expedite opposite in meaning to the given word: [2014-I]
31. ALACRITY 43. ‘PREDILECTION’
(a) Hesitatingly (b) Eagerness (a) Oblivion (b) Objectivity
(c) Unwillingly (d) Laziness
(c) Aversion (d) Defeat
DIRECTIONS (Qs. 32 - 35) : In each of the following choose the 44. ‘CACOPHONOUS’
word most nearly opposite in meaning to the word given in
(a) Tamed (b) Harmonious
capitals. [2012-II]
32. PROFUSION (c) Domestic (d) Silent
(a) Travesty (b) Validity 45. ‘CALUMNY’
(c) Scarcity (d) Agitated (a) Apology (b) Eulogy
33. ESCHEW (c) Enjoyment (d) Reservation
(a) Vicious (b) Invite DIRECTIONS (Qs. 46 - 50) : Choose the word that best defines
(c) Use (d) Emanate the given phrase: [2014-I]
34. ABSTAIN 46. ‘Able to use both hands alike’
(a) Refuse (b) Oppose (a) Dexterous (b) Ambidextrous
(c) Run away (d) Permit (c) Skilful (d) Expert
Vocabulary/One Word Substitution A-3

47. ‘A written account of the life of an individual’ 61. Building castles in the air
(a) Autobiography (b) Epigraph (a) Making impossible plans
(c) Biography (d) Novel (b) Making tall promises
48. ‘The identification of a disease by its symptoms’ (c) Building skyscrapers
(a) Prescription (b) Prognosis (d) Structures without strong foundation
(c) Diagnosis (d) Biopsy 62. At the drop of a hat
49. ‘Prolonged inability to sleep’ (a) Willingly and softly
(a) Amnesia (b) Utopia (b) Willingly and immediately
(c) Nausea (d) Insomnia (c) Willingly and silently
50. ‘A style in which a writer makes a display of his knowledge’ (d) Slowly and silently
(a) Verbose (b) Pedantic DIRECTIONS (Qs. 63 - 66) : Which word or words explains the
(c) Ornate (d) Pompous meaning of the following idioms:- [2014-II]
DIRECTIONS (Qs. 51 - 54) : Choose the word which is nearest 63. Airy- Fairy
in meaning to the given word: [2014-II] (a) Most important (b) Nervous
51. LIBERALISE (c) Not practical (d) Confident
(a) Resist (b) Change 64. Be given the axe
(c) Function (d) Malfunction (a) To move fast (b) Carpenter
52. PRAGMATIC (c) Woodcutter (d) To lose job
(a) Theoretical (b) Suitable
65. To go like a bomb
(c) Realistic (d) Productive
(a) Loud explosion (b) To move fast
53. PRISTINE
(c) Terrorism (d) Not practical
(a) Fresh (b) Old
66. Bolt from the blue
(c) Preserve (d) Dirty
(a) Unexpected (b) Lighting
54. INTREPID
(c) Nervous (d) Sudden
(a) Middle (b) Tolerant
(c) Rude (d) Fearless DIRECTIONS (Qs. 67 - 70) : Choose the word which is nearest
in meaning to the given word : [2015-I]
DIRECTIONS (Qs. 55 - 58) : Choose the word which is nearly 67. Sporadic
opposite in meaning to the given word: [2014-II]
(a) Epidemic (b) Whirling
55. ANGELICAL (c) Occasional (d) Stagnant
(a) Magnanimous (b) Benlvolent 68. Genesis
(c) Diabolical (d) Critical (a) Style (b) Beginning
56. HAUGHTINESS (c) Movement (d) Relevant
(a) Affability (b) Unskilled 69. Intransigent
(c) Adduce (d) Abject (a) Authoritative (b) Impersonal
57. BELLOW (c) Strenuous (d) Unbending
(a) Tout (b) Whisper 70. Intimidate
(c) Stupour (d) Down (a) Mislead (b) Misplace
58. INEQUITY (c) Frighten (d) Demoralise
(a) Law (b) Illegal DIRECTIONS (Qs. 71 - 74) : Choose the word which is nearly
(c) Slander (d) Libel opposite in meaning of the given word: [2015-I]
DIRECTIONS (Qs. 59 - 62) : Choose the word that best defines 71. Clemency
the given phrase: [2014-II] (a) Corporal (b) Intolerance
59. Tickled pink (c) Compromise (d) Sensibility
72. Cajole
(a) Greatly pleased (b) Coloured (a) Nestle (b) Secede
(c) Deeply upset (d) Embarrassed
(c) Bully (d) Moisten
60. Split one's side
73. Malevolent
(a) Intense pain (b) To laugh a lot
(a) Kindly (b) Vacuous
(c) To be hurt (d) None of these
(c) Ambivalent (d) Primitive
EBD_8177
A-4 Vocabulary/One Word Substitution

74. Purgatory 85. The alpha and the omega


(a) Reward (b) Celestial (a) The begining and the end
(c) Flawless (d) Proximity (b) A nice act
(c) The starts and the moon
DIRECTIONS (Qs. 75 - 78) : Which word or words explains the
(d) Very costly
meaning of the following idioms : [2015-I]
86. A bull in a china shop
75. In a jiffy (a) A person who is very ugly but loves the beautiful
(a) Outstanding (b) Suddenly things of life
(c) In a fix (d) Appropriate (b) A person who takes a sadistic delight in harming
76. Upto the hilt innocent people
(a) Completely (b) Upto the mark (c) A person who becomes too excited where no excitement
is warranted
(c) Upto the final decision (d) None of these
(d) A rough and clumsy person at a place where skill and
77. Man of Letters care are required.
(a) Who writes too many letters
DIRECTIONS (Qs. 87 - 90) :Choose the word that best defines
(b) An important person
the given phrases: [2015- II]
(c) A politician
87. To hold something in leash
(d) A literary person
(a) To restrain (b) To disappoint
78. Sangfroid
(c) To dismiss (d) To discourage
(a) Composure (b) Go on leave
88. To talk through one's hat
(c) Changed suddenly (d) Make an attempt
(a) To speak fluently (b) To talk nonsense
DIRECTIONS (Qs. 79 - 82) : Choose the word that best defines
(c) To talk wisdom (d) To speak at random
the given phrases : [2015-I]
89. To throw up the sponge
79. A Curtain Lecture
(a) To surrender or give up a contest
(a) To speak plainly
(b) To offer a challenge
(b) Vulgar ideas
(c) To become utterly disappointed
(c) Private scolding of a husband by his wife
(d) Hate others (d) To maintain grit and enthusiasm until the end
80. Square pegs in round holes 90. To get into hot waters
(a) A genuinely helpful person (a) To be impatient
(b) A clever person (b) To suffer a huge financial loss
(c) People in the wrong jobs (c) To get into trouble
(d) To be perplexed (d) To be in a confused state of min
81. In weal and woe DIRECTIONS (Qs. 91 - 94) : Choose the word which is nearest
(a) By hook or crook in meaning to the given word : [2015- II]
(b) During illness 91. Anathema
(c) In prosperity and adversity (a) Religious Chant (b) Pun
(c) Musical Subject (d) Curse
(d) During the operation
92. Debilitate
82. Globetrotters
(a) Weaken (b) Attack
(a) People against global philosophy
(c) Surmount (d) Destroy
(b) People indulging in treachery 93. Penchant
(c) Intelligent minds (a) Liking (b) Eagerness
(d) Travellers around the world (c) Disability (d) Keenness
DIRECTIONS (Qs. 83 - 86) : Which word or words explains the 94. Nebulous
meaning of the following idioms: [2015- II] (a) Tiny (b) Vague
(c) Insignificant (d) Inadequate
83. A fool's errand
(a) A blunder (b) An impossible task DIRECTIONS (Qs. 95 - 98) : Choose the word which is nearly
opposite in meaning to the given word : [2015- II]
(c) A useless undertaking (d) None of these
95. Insolent
84. To flog a dead horse
(a) Sutten (b) Affable
(a) To act in a foolish way (c) Determined (d) Rich
(b) To waste one's efforts 96. Sceptic
(c) To revive into rest in an old subject (a) Cryptic (b) Believer
(d) To revive old memories (c) Support (d) Eminent
Vocabulary/One Word Substitution A-5

97. Refractory DIRECTIONS (Qs. 111 - 115) : Choose the word explains the
(a) Refreshing (b) Burdensome meaning of the following idioms: [2016 -I]
(c) Privileged (d) Manageable
98. Acrimonious 111. All and Sundry
(a) Legal (b) Severe (a) Everybody without distinction
(c) Cursive (d) Harmoniousd (b) Only rich person
DIRECTIONS (Qs. 99 - 102) : Choose the word/words that best (c) Together
defines the given phrases : [2016 -I] (d) Selected people
99. Beats me 112. To play second fiddle
(a) To hit someone (b) To quarrel with a person (a) To be happy, cheerful and healthy
(c) To not understand (d) To be silent (b) To be in a subordinate position to someone
100. Cloak and dagger
(c) To do back seat driving
(a) To fight to a finish
(d) To reduce importance of one's senior
(b) To get into unexpected trouble
113. A wolf in sheep's clothing
(c) To end the hostility
(d) Like a spy (a) A hypocrite
101. To be at the end of one's tether (b) A wolf in woolen clothes
(a) To feel offended (c) A coward
(b) To draw moral attention (d) A proud man
(c) To have no resources left 114. A blue stocking
(d) To think deeply (a) Stocking which is blue in colour
102. I Don't Buy it (b) A clean cheat
(a) To refuse to purchase something (c) An unreliable person
(b) To give up (d) A woman having literary tastes and learning
(c) To decline an offer 115. Yeoman's service
(d) To not be convinced (a) Lip-service (b) To serve like a slave
DIRECTIONS (Qs. 103 - 106): Choose the word which is nearest (c) To play god father (d) Efficient or useful help
in meaning to the given word:- [2016 -I]
DIRECTIONS (Qs. 116 - 120) Which word or words explains
103. Abdicate the meaning of the following idioms: [2016 - II]
(a) Join (b) Search
(c) Advance (d) Renounce 116. All Agog
104. Acrimony (a) Almighty
(a) Spleen (b) Courtesy (b) Unmoved
(c) Oriented (d) Scarce (c) Praise someone
105. Embellish (d) Full of interest and excitement
(a) Employ (b) Arrange 117. To the manner born
(c) Decorate (d) Design
(a) One of low birth (b) Naturally at ease
106. Damp Squib
(c) Place of one's birth (d) Caesarean birth
(a) Insignificant happening
(b) An old technique 118. Lose one's marbles
(c) Unsuccessful attempt (a) Lose something dear to you
(d) Short, shrill cry (b) Suffer a setback
DIRECTIONS (Qs. 107 - 110) : Choose the word which is nearly (c) Become insane
opposite in meaning to the given word:- [2016 -I] (d) Become drunk
107. Sporadic 119. Bolt from the blue
(a) Rare (b) Frequent (a) Sea swimming (b) Complete surprise
(c) Sharp (d) Coordinated (c) Reckless (d) Careless
108. Exonerate
(a) Reject (b) Contract 120. Be like a bear with a sore head
(c) Accuse (d) Admit (a) Be in a foul mood (b) Have headache
(c) Powerful and arrogant (d) Restless
109. Exodus
(a) Home-coming (b) Influx DIRECTIONS (Qs. 121 - 124) Choose the word which is nearly
(c) Restoration (d) Return opposite in meaning to the given word: [2016 - II]
110. Relinquish 121. Beguile
(a) Cede (b) Abdicate (a) Deceive (b) Honest
(c) Deny (d) Possess (c) Charm (d) Cheat
EBD_8177
A-6 Vocabulary/One Word Substitution

122. Ameliorate DIRECTION (Qs. 135 - 136): Choose the word which is nearly
(a) Deteriorate (b) Procrastinate opposite in meaning to the given word: [2017 - I]
(c) Stagnate (d) Deviate 135. Amending
123. Feckless (a) Alter (b) Ameliorate
(a) Spotted (b) Fatuous (c) Better (d) Debase
(c) Strong (d) Fawning 136. Entangle
(a) Untwist (b) Twist
124. Cacophonous (c) Hook (d) Impede
(a) Tamed (b) Harmonious
DIRECTIONS (Qs. 137 - 141) : Choose the word which is
(c) Silent (d) Domestic nearest in meaning to the given word: [2017 - II]
DIRECTIONS (Qs. 125 - 128) Choose the word which is nearest 137. Ameliorate
in meaning to the given word: [2016 - II] (a) Improve (b) Disapprove
125. Preposterous (c) Appreciate (d) Discourage
(a) Formal (b) Judicious 138. Haggle
(c) Ridiculous (d) Ceremonious (a) Struggle (b) Bargain
126. Dissemble (c) Advise (d) Acknowledge
(a) Disagree (b) Shake 139. Morose
(a) Ill-tampered (b) Well-behaved
(c) Impeccable (d) Conceal
(c) Kind-hearted (d) Cunning
127. Raucous 140. Taciturn
(a) Flavourful (b) Jarring (a) Talkative (b) Uncommunicative
(c) Boisterous (d) Evil (c) Clever (d) Bad-tampered
128. Abrogate 141. Cajole
(a) Put an end to (b) Elope (a) Deceive (b) Kind
(c) Gatecrash (d) Send away (c) Suspect (d) Humor
DIRECTIONS (Qs. 129 - 131) : Choose the word which is DIRECTIONS (Qs. 142 - 146): Choose the most appropriate
nearest in meaning to the given word: [2017 - I] option that explains the correct meaning of the following
129. Idiocy idioms: [2017 - II]
(a) Madness (b) Happiness 142. A man of straw
(c) Wisdom (d) Repentance (a) A man with no means
130. Harbinger (b) A generous man
(a) Remarkable (b) Omen (c) A man of character
(c) Meritorious (d) Overwhelming (d) A man of no substance
131. Passe 143. To catch a tartar
(a) Out of date (b) Forthcoming (a) To catch a dangerous person
(c) Fashionable (d) Predictable (b) To meet with disaster
DIRECTIONS (Qs. 132 - 134) : Choose the most appropriate (c) To make a deal
option that explains the meaning of the following idioms: (d) To hurry up
[2017 - I] 144. To have an axe to grind
132. To hit the nail on head (a) To have a strong personal opinion
(a) Find exactly the right answer (b) To fail to arouse interest
(b) To scold someone
(c) To have no result
(c) To show correct path to someone
(d) To work for both sides
(d) To be harsh in behaviour
145. To play second fiddle
133. Halcyon days
(a) Bad days (b) Sunny days (a) To be happy, cheerful and healthy
(c) Cloudy days (d) Happy days (b) To reduce importance of one's senior
134. To keep the wolf from the door (c) To support the role and view of another person
(a) To remain alert (d) To do back seat driving
(b) To question others in case of doubt 146. To go to the wall
(c) To have enough money to avert hunger or starvation (a) To be ruined (b) To be idle
(d) To chase others to get something (c) To be in grave trouble (d) To go for a holiday
Vocabulary/One Word Substitution A-7

DIRECTIONS (Qs. 147 - 152): Choose the word which is nearly 162. A fool's paradise
opposite in meaning to the given word: [2017 - II] (a) paradise of idiots
147. Relinquish (b) a state of illusory happiness
(a) Abdicate (b) Renounce (c) to live in the past
(c) Deny (d) Possess (d) to have happy dreams
148. Quiescent 163. Nip in the bud
(a) Active (b) Dormant (a) destroy in the beginning
(c) Weak (d) Unconcerned (b) extremely good start
149. Flagitious (c) striving from the beginning
(a) Innocent (b) Clever (d) nurture the bud to grow into flower
(c) Ignorant (d) Frivolous DIRECTIONS (Qs. 164-168) : In each of the following ques-
150. Gregarious tions, choose the correctly spelt word. [2018 - I]
(a) Antisocial (b) Glorious 164. (a) Properetry (b) Propriatory
(c) Horrendous (d) Similar (c) Proprietary (d) Proprietory
151. Hirsute 165. (a) Reharsal (b) Rehersal
(c) Rehearsal (d) Rehearsel
(a) Scaly (b) Bald
166. (a) Millionare (b) Millionaire
(c) Erudite (d) Quiet
(c) Milionaire (d) Millunaire
152. Urbane
167. (a) Fasist (b) Facicl
(a) Illiterate (b) Backward
(c) Facist (d) Fascist
(c) Discourteous (d) Orthodox
168. (a) Legendry (b) Legendary
DIRECTIONS (Qs. 153-157) : Select the word which means the (c) Legendery (d) Legandery
opposite of the given word. [2018 - I]
DIRECTIONS (Qs. 169-172): In the following questions four alter-
153. PROFANE natives are given for idioms/phrases in now. Choose the one that
(a) Sacred (b) Artless
(c) Rigid (d) Aspersion best expresses the meaning of the given idiom/phrase. [2018 - II]
154. OBLIGATORY 169. Take the spear
(a) Doubtful (b) Voluntary (a) To drink in the company of others
(c) Sincerely (d) Faithfully (b) To make an exaggerated statement
155. OBSCURE
(c) To fight fiercely till the end
(a) Suitable (b) Apt
(c) Thalamus (d) Clear (d) To accept full blame for something
156. MUTUAL 170. Grist to one's mill
(a) Reciprocal (b) Agreed (a) Harmful to somebody (b) Useful to somebody
(c) Common (d) Conjugal (c) Useless to somebody (d) Dreadful to somebody
157. EVIDENT
171. In a lather
(a) Prominent (b) Seen
(c) Observed (d) Quite clear (a) Encouraged (b) Tired
DIRECTIONS (Qs. 158-163) : Select the meaning of the given (c) Distressed (d) Refreshed
phrases/idioms. [2018 - I] 172. Mop down [2018 - II]
158. Man of letters (a) To clean something
(a) a person who writes letters (b) To misinterpret a statement
(b) a person who receives letters (c) To be dejected
(c) an illiterate person (d) To be uncertain
(d) a learned person
159. All Greek DIRECTIONS (Qs. 173-175): In the following questions, choose
(a) totally classical (b) totally unintelligible the word opposite in meaning to the given word. [2018 - II]
(c) totally impressive (d) totally original 173. UPEND
160. A live wire
(a) Overbear (b) Subdue
(a) a person who is full of energy
(b) an eminent person (c) Flourish (d) Lose
(c) an unruly person 174. TURF
(d) a critical person (a) Shelter (b) Oust
161. Writing on the wall (c) Injure (d) Evict
(a) graffiti 175. SPLEEN
(b) an event indicating impending danger
(c) announcement of an event (a) Caprice (b) Cheer
(d) a political slogan (c) Sway (d) Boisterous
EBD_8177
A-8 Vocabulary/One Word Substitution

DIRECTIONS (Qs. 176-178): Out of the four alternatives, choose 188. Dole out
the one which best expresses the meaning of the given word. (a) Allocate (b) Be effective
[2018 - II] (c) Turn up (d) Mismanage
176. BARGE DIRECTIONS (Qs. 189-191) : Choose the one which best ex-
(a) Shove (b) Shout presses the meaning of the given word. [2019 - II]
(c) Interpret (d) Plead 189. Allegiance
177. CHERUBIC (a) Loyalty (b) Violence
(a) Elderly (b) Lowness (c) Disloyalty (d) Treachery
(c) Adorable (d) Hardheaded
190. Abash
178. HUMMOCK
(a) Dauntless (b) Ashamed
(a) Tranquility (b) Slab
(c) Hammer (d) Knoll (c) Intrepid (d) Honest
191. Altruism
DIRECTIONS (Qs. 179-181): Out of the four alternatives, choose
(a) Selfishness (b) Deceitful
the one which best expresses the meaning of the given word.
(c) Selflessness (d) Scepticism
[2019-I]
DIRECTIONS (Qs. 192-194) : In the following questions, choose
179. POLTROON the word opposite in meaning to the given words. [2019 - II]
(a) Pusillanimous (b) Gallant
192. Adroit
(c) Gutsy (d) Wearied
(a) Honest (b) Adept
180. ROSTRUM
(a) Guardian (b) Podium (c) Gallant (d) Incompetent
(c) Device (d) Scheme 193. Vague
181. PROROGUE (a) Clear (b) Faint
(a) Adjourn (b) Convene (c) Obscure (d) Vogue
(c) Rally (d) Continue 194. Frugal
DIRECTIONS (Qs. 182-184): In the following questions, choose (a) miserly (b) gluttonous
the word opposite in meaning to the given word. [2019-I] (c) plentiful (d) extravagant
182. LUCRE DIRECTIONS (Qs. 195-198) : In the following questions, four
(a) Debt (b) Elegance alternatives are given for the Idiom/Phrase. Choose the alterna-
(c) Outlaw (d) Sissy tive which best expresses the meaning of the Idiom/Phrase.
183. RABBLE [2019 - II]
(a) Rag (b) Nobility 195. To bury the hatchet.
(c) Scanty (d) Sanction (a) To make peace with the enemy
184. COTERIE (b) To throw soil over something
(a) Loner (b) Socialize (c) To make someone cry
(c) Elite (d) Indecent (d) To kill someone
DIRECTIONS (Qs. 185-188): In the following questions four alter- 196. Strain every nerve.
natives are given for idioms/phrases in now. Choose the one that (a) To hurt someone
best expresses the meaning of the given idiom/phrase. [2019-I] (b) To make the greatest possible effort
185. Brain sauce (c) To count every nerve
(a) Foolish (d) To spread something everywhere
(b) Wisdom
197. At the eleventh hour
(c) Mentally ill
(d) Head full of thoughts (a) too late
186. Lynch law (b) too early
(a) Law imposed by the government (c) immediately
(b) Law of the mob (d) at the last moment
(c) A law that is supposed to be useless 198. To burn one’s fingers
(d) A rule that no one follows (a) to get hurt physically
187. Globetrotters
(b) to suffer financial losses
(a) Travellers around the world
(b) Sick people in hospital (c) to find work
(c) The people living in asylum (d) to suffer nervous breakdown
(d) World champions
Vocabulary/One Word Substitution A-9

DIRECTIONS (199-204): in the following questions, choose 204. What is the synonym of Inquisitive?
(a) Curious (b) intrigued
the correct synonym/antonym. [2020 - I]
(c) agog (d) all of these
199. What is the antonym of naive?
DIRECTIONS (Qs. 205-207): in the following questions four
(a) Unsophisticated (b) artless
(c) experienced (d) inexperienced alternatives are given for idioms/phrases. Choose the one that
200. What is the Antonym of Cajole? best expresses the meaning of the given idiom/phrase. [2020 - I]
(a) Persuade (b) wheedle
205. What is the meaning of Idiom – ‘Get into hot water’?
(c) coax (d) bully
(a) To swim in the deep water
201. What is the Synonym of Parochial? (b) To think over difficult problem
(a) conventional (b) Broad minded (c) To get into difficult situation
(c) cosmopolitan (d) liberal (d) To discuss a burning topic
202. What is the Antonym of Sporadic? 206. What is the meaning of Idiom—-‘rain cats and dogs’?
(a) Uneven (b) spasmodic (a) Rain heavily (b) To fight in rain
(c) frequent (d) intermittent (c) Cats and dogs fight (d) Rain water wastage
203. Antonyms of intrinsic? 207. What is the meaning of Idiom—-‘at daggers drawn’?
(a) Innate (b) natural (a) To throw daggers (b) To be bitterly hostile
(c) acquired (d) built in (c) To be confused (d) To think deeply

ANSWERS & EXPLANATIONS


1. (a) Brought about 24. (a) Emancipate means free from slavery or servitude; hence,
2. (d) Calm set free is the correct option.
3. (b) Daring 25. (d) The quality of being fraudulent.
4. (c) Dialogue 26. (c) admonition means cautionary advice about something
5. (c) Insert imminent (especially imminent danger or other unpleas-
6. (b) Certainty antness).
7. (d) ‘INCITE’ means to encourage illegal or unpleasant. 27. (b) Vociferous is opposite of quiet.
8. (b) ‘SUCCUMB’ means not to be able to fight. 28. (b) Implicate means involve in crime. So Exonerate is the
9. (c) ‘ANOMALOUS’ means different what from is normal antonym.
or expected. 29. (a) Osmosis means the process of gradual or unconscious
10. (a) ‘FORTUITOUS’ and accidental means happening by assimilation of ideas, knowledge, etc.
change. 30. (b) Ostracise means avoid speaking to or dealing with
11. (a) ‘PERMEABLE’ means allowing something to pass and cut off means the same.
through. 31. (b) Alacrity means liveliness and eagerness; hence, option
b is right.
12. (b) ‘CONVOY’ means a group of vehicles travelling
32. (c) Profusion means the property of being extremely
together.
abundant and its opposite should be scarcity.
13. (a) ‘CITE’ means to mention an example in order to support
33. (c) Eschew means avoid and stay away from deliberately;
what you are saying. stay clear of, hence, its opposite should be Use.
14. (d) ‘VOCATION’ and designation mean a type of work 34. (c) Abstain means choose not to consume and its opposite
that you believe is especially suitable for you. should be permit.
15. (c) ‘TRANSITORY’ means continuing for only a short time. 35. (a) Insolent means marked by casual disrespect while
16. (a) ‘CREVICES’ means cracks in a rock or wall. affable means diffusing warmth and friendliness.
17. (d) ‘VENERATE’ means regard with great respect. 36. (c) Meddle means to interfere in something that is not
18. (a) ‘VACILLATE’ means go back and forth. one's concern.
19. (c) Felicity means immense happiness. 37. (a) Renounce and abjure means to formally declare one's
20. (c) Hysteria means state of extreme upset. Its opposite abandonment.
meaning will be serenity. 38. (d) Estrange means to cause someone to be no longer
21. (a) Abstain means to hold back from doing. Its opposite is involved or connected with something.
indulging in something. 39. (c) Derogatory means showing a critical or disrespectful
22. (c) Sedentary means motionless or lazy. Its opposite attitude which is opposite of praising.
meaning will be moving. 40. (c) Wane means to diminish, weaken or lessen which is
23. (c) opposite to swell that means to grow.
EBD_8177
A-10 Vocabulary/One Word Substitution

41. (b) Transgressor is a person who breaks the law i.e. a law 62. (b) At the drop of a hat - immediately; instantly; on the
breaker. slightest signal or urging. So, (b) is the correct choice.
42. (d) Egregious means outstandingly bad and atrocious 63. (c) Airy-fairy means not practical or not useful in real
means outrageous. situations. Hence, (c) is the correct choice.
43. (c) Predilection means preference toward something 64. (d) Get the axe or also be given the axe means if a person
whereas aversion means opposition. gets the axe, they lose their job. Hence, (d) is the
44. (d) Cacophonous means harsh sounding, whereas quiet correct choice.
is calm and noiseless. 65. (b) Go like a bomb means if a vehicle goes like a bomb, it
45. (b) Calumny is the making of false statements that damage can move very fast. Hence, (b) is the correct choice.
another's reputation whereas eulogy means praise and
66. (d) A bolt from the blue or also a bolt out of the blue
acclamation.
means something that not expected to happen and that
46. (b) Ambidextrous means both hands. The ambidextrous
surprises someone very much. Hence, (d) is the correct
person can perform anaction with either hand having
equal dexterity in the action.Writing is themost strik- choice.
ing of these actions. 67. (c) Sporadic means, occuring at irregular intervals or any
47. (c) A written account of the life of an individual by himself in a few places. Ex. A sporadic fighting broke out.
is called a autobiography. And when it is written by 68. (b) Genesis is the origin or mode of formation of something.
other person, then it is called biography. The nearest meaning is beginning.
48. (c) Diagnosis refers to both the process of attempting to 69. (d) Intransigent defines unwilling or refusing to change
determine or identify a possible disease and to the one's views or to agree about something. Therefore
opinion reached by this process. unbending is the nearest meaning.
49. (d) Insomnia, or sleeplessness, is a sleep disorder in which 70. (c) Intimidate means frighten or overawe someone, in order
there is an inability to fall asleep or to stay asleep as to make them do what one wants.
long as desired. 71. (b) The word clemency means mercy or lenience. Here
50. (b) Pedantic mean overly concerned with minute details or intolerance is the nearly opposite meaning.
formalisms, especially in teaching. 72. (c) The word cajole means to persuade someone to do
51. (b) Liberalise - remove or loosen restrictions on something. something by sustained coaxing or flattery. Bully is
Hence, option (b) is correct choice. the nearly opposite meaning.
52. (c) Pragmatic - of or relating to a practical point of view or 73. (a) Malevolent defines, showing a wish to do evil to others.
practical considerations.Hence, option (c) is correct
Kindly is the most appropriate opposite.
choice.
74. (b) Purgatory means a place or state of suffering inhabited
53. (a) Pristine - in its original condition; unspoilt;clean and
by the souls of sinners who are expiating their sins
fresh as if new; spotless.Hence, option (a) is correct
choice. before going to heaven. Therefore celestial is the right
54. (d) Intrepid means fearless; adventurous. Hence, option opposite word.
(d) is correct choice. 75. (b) Jiffy means-a very short time, a moment which is used
55. (c) Meaning of angelical - having a sweet nature befitting in an informal way in English. Ex. I we'll be back in a
an angel or cherub. Meaning of Diabolical - concerning, jiffy. The other options are not appropriate.
or characteristic of the devil; satanic.Hence, antonym 76. (a) Upto the hilt means completely.
of angelical is diabolical. Ex. The building was mortgaged up to the hilt.
56. (d) Meaning of haughtiness- the appearance or quality of Other options give different meaning.
being arrogantly superior and disdainful. 77. (d) Literary persons are scholars or male authors known
57. (b) Meaning of bellow - a deep roaring shout or sound. as–man of letters. Ex. He wished to fashion for himself
Meaning of whisper - a soft or confidential tone of a career as a man of letters.
voice. Hence, antonym of bellow is whisper. Other options are not correct as they express different
58. (a) Mean ing of in equity- lack of fairness or meanings.
justice.Antonym for inequity is law according to given 78. (a) Sangfroid means composure or coolness shown in
options. danger or under trying circumstances.
59. (a) Tickled pink -very much pleased or entertained.So, (a) Ex. Offering the most welcoming stage for the talented,
is the correct choice. the city with equal sangfroid accepts the misery of
60. (b) Split one's sides or laugh one's head off- be extremely millions who fail to flourish. Other options are simply
amused, laugh uproariously. So, (b) is the correct choice. irrelevat so far the meaning is concerned.
61. (a) Build castles in the air or build castles in Spain- to 79. (c) The phrase a curtain Lecture means an instance of a
daydream; to make plans that can never come true. So, wife reprimanding her husband in private.
(a) is the correct choice. 80. (c) Square pegs in round holes means a misfit. So, people
in the wrong jobs is the correct option.
Vocabulary/One Word Substitution A-11

81. (c) The phrase means good and bad day. E.g., Weal and 102. (d) To not be convinced or disagree. Ex: I just don't buy
woe comes in everybody's life. Therefore, in prosperity the idea that you can swim that far.
and adversity is the correct option. 103. (d) Abdicate means to give up or renounce from one's
82. (d) Globetrotter is a person who travels widely. position.
83. (c) The idiom "a fool's errand" means an attempt to do 104. (a) Acrimony means bitterness or ill feeling; spleen means
something that has no chance of success. bad temper.
84. (b) 'To flog a dead horse' means to waste effort on 105. (c) Embellish means make more attractive by adding
something where there is no chance of succeeding. ornament, colour, etc.
85. (a) 'The alpha and the omega' means the beginning and 106. (a) Damp squib means an event which people think will be
the end.
exciting but which is disappointing when it happens.
86. (d) 'A bull in a China shop' means a person who breaks
107. (b) Sporadic means recurring in scattered and irregular or
things or who often makes mistakes or causes damage
unpredictable instances while its opposite is frequent.
in situations that requires careful thinking or behaviour.
108. (c) Exonerate means pronounce not guilty of criminal
87. (a) 'To hold something in leash' means to provide very
charges, its opposite is accuse.
little freedom to someone.
88. (b) "To talk through one's hat" means to talk nonsense. 109. (b) Exodus means mass departure; it opposite is influx
89. (a) 'To throw up the sponge' means to give up a contest or 110. (d) Relinquish means to give up; its opposite is possess.
to acknowledge defeat. 111. (a) All and Sundry means everybody without
90. (c) 'To get into hot waters' means to be in trouble or a discrimination.
difficult situation. 112. (b) To play second fiddle means to have a subordinate
91. (d) Anathema means a formal curse by a pope or a council role to someone or something.
of the church. Therefore, 'curse' is the nearest meaning 113. (a) A wolf in sheep's clothing denotes a person or thing
or synonym of Anathema. that appears friendly or harmless but is really hostile.
92. (a) Debilitate means to make someone weak and infirm. 114. (d) A blue stocking denotes an intellectual or literary
Therefore, weaken is the nearest in meaning to woman.
Debilitate. 115. (d) Yeoman's service means efficient or useful help in need.
93. (a) Penchant means a strong or habitual liking for 116. (d) The idiom 'all agog' means in high spirits or in eager
something. Therefore, liking is the nearest in meaning expectation.
to Penchant. 117. (b) The idiom 'to the manner born' means naturally at ease
94. (b) Nebulous means vague or ill-defined. Therefore, vague in a specified job or situation.
is the closest in meaning to Nebulous. 118. (c) The idiom 'lose one's marbles' means to lose one's mind.
95. (b) Insolent means showing a rude and arrogant behaviour 119. (b) The idiom 'bolt from the blue' means something
or lack of respect. Affable means good natured or easy important or unusual that happens suddenly or
to talk to. Therefore, Affable is the nearly opposite unexpectedly.
meaning of Insolent.
120. (a) The idiom 'be like a bear with a sore head' means to be
96. (b) Sceptic means a person who questions or doubts in a bad mood that causes you to treat other people
accepted opinions. Therefore, Believers which means badly.
a person who believes in the truth or existence of
121. (b) Beguile means to attract or delight someone, often in a
something is the nearly opposite in meaning to Sceptic.
deceptive way. So, the antonym of beguile is honest.
97. (d) Refractory means stubborn or unmanageable. 122. (a) Ameliorate means to make something better. So, the
Therefore, manageable is nearly the opposite in meaning antonym of ameliorate is to deteriorate.
to Refractory.
123. (c) Feckless means lacking initiative or strength of
98. (d) Acrimonious means full of anger and bitter feelings.
character; irresponsible. So, the antonym of feckless
Whereas, Harmonious means friendly and free from
is to be strong.
disagreement. Therefore, Harmonious is the antonym
124. (b) Cacophonous means producing a harsh, discordant
of Acrimonious.
mixture of sounds. So, the antonym of cacophonous is
99. (c) Beats me: To not know; to not understand. Ex: "It beats
to be harmonious.
me how they finished before us."
125. (c) Both preposterous and ridiculous mean contrary to
100. (d) Like a spy - Involving secrecy and plotting. Ex: Why reason or common sense; utterly absurd.
all these cloak-and-dagger (= secretive) meetings? We
126. (d) Dissemble means conceal or disguise one's true
should discuss the issuesopenly.
feelings or beliefs.
101. (c) To be so tired, worried, or annoyed by something that
127. (c) Both raucous and boisterous mean making or
you feel unable to deal with it any more
constituting a disturbingly harsh and loud noise.
Ex: After a day's hard work in the office, she was at the
128. (a) Abrogate means repeal or do away with.
end of her tether.
EBD_8177
A-12 Vocabulary/One Word Substitution

129. (a) Idiocy is a noun which means madness or extremely 159. (b) All greek means totally unintellingible
unusual behaviour. 160. (a) A live wire means an energetic and unpredictable
130. (b) Harbinger is a person or thing that announces or signals person.
the approach of something. Omen also means a 161. (b) The writing on the wall means the likelihood that
phenomenon that is believed to foretell the future. something bad will happen.
131. (a) Passe is an adjective which means no longer 162. (b) A fool's paradise means the state of being happy for
fashionable or out of date. foolish reasons.
132. (a) The idiom "To hit the nail on head" means to find the 163. (a) Nip in the bud means to end something before it
correct answer of something. develops into something larger.
133. (d) The idiom "Halycon days" means a period of peace 164. (c) 165. (c) 166. (b) 167. (d) 168. (b) 169. (d)
and happiness. 170. (b) 171. (c) 172. (a)
134. (c) The idiom "To keep the wolf from the door" means to 173. (d) Upend: to achieve a victory over.
have enough money to prevent hunger. 174. (a) Turf: force (someone) to leave somewhere.
135. (d) Amending means to make changes for improvement 175. (b) Spleen: bad temper; spite.
and debase means degrade in value or quality. Caprice: a sudden and unaccountable change of mood
136. (a) Entangle means twisted, opposite to the word untwist. or behaviour.
137. (a) Ameliorate is a verb which means make something Sway: rule; control.
better. Hence 'Cheer' is the correct antonym.
138. (b) Haggle means dispute or bargain repeatedly, especially 176. (a) Barge: move forcefully or roughly.
over the cost of something. Shove: push (someone or something) roughly.
139. (a) Morose is an adjective which means bad-tempered and Hence Barge and Shove are synonyms to each other.
sulky. 177. (c) Cherubic: having the innocence or plump prettiness
140. (b) Taciturn is an adjective which means a person who of a young child.
remains reserved or is uncommunicative in speech. Adorable: inspiring great affection or delight.
141. (a) Cajole means persuade someone to do something by Hence Cherubic and Adorable are synonyms to each
other.
false promises.
178. (d) Hummock: a very small hill or raised part of the ground;
142. (d) The idiom 'a man of straw' means a person undertaking
hillock.
a financial commitment without adequate means.
Knoll: a small hill or mound.
143. (a) The idiom 'to catch a tartar' means to nab a dangerous
Hummock and Knoll are synonyms to each other.
person.
179. (a) Poltroon: having or showing a shameful lack of
144. (a) The idiom 'to have an axe to grind' means to have a
courage.
strong personal opinion.
Pusillanimous: showing a lack of courage or
145. (c) The idiom 'To play second fiddle' means to have a determination; timid.
subordinate role to someone or something; be treated
Gutsy: having or showing courage, determination, and
as less important than someone or something.
spirit.
146. (a) The idiom 'To go to the wall' means to be ruined.
147. (d) Relinquish means voluntarily cease to keep or claim. Hence Poltroon and Pusillanimous are synonyms to
148. (a) Quiescent means to be in a state or period of inactivity each other.
or dormancy. 180. (b) Rostrum: a level usually raised surface.
149. (a) Flagitious means criminal. Podium: a small platform on which a person may stand
150. (a) Gregarious means sociable. to be seen by an audience.
151. (b) Hirsute means having hair on the body. Hence Rostrum and Podium are synonyms to each
152. (c) Urbane means a person who is polite and respectful. other.
153. (a) Profane means ‘unholy, not devoted to holy or religious 181. (a) Prorogue: to bring to a formal close for a period of
purposes’. So option ‘a’ (sacred) would be the right time.
antonym of this word. Adjourn: break off (a meeting, legal case, or game)
154. (b) The word ‘obligatory’ means compulsory, mandatory, with the intention of resuming it later.
required whose opposite is optional or voluntary. Convene: come or bring together for a meeting or
155. (d) The word ‘obscure’ means unclear whose opposite is activity.
clear.
156. (a) The word ‘mutual’ is used to describe feelings that
Hence Prorogue and Adjourn are synonyms to each
two or more people have for each other equally. Hence, other.
reciprocal is similar word in meaning to it. 182. (a) Lucre: monetary gain.
157. (d) The word ‘evident’ means obvious, clear, tangible, Debt: a sum of money that is owed or due.
distinct etc. which is nearest in meaning to ‘quite clear’. Outlaw: a person who has broken the law.
158. (d) Man of letters means a person devoted to literary or Sissy: a person regarded as effeminate or cowardly.
scholarly activities. Hence ‘Debt’ is the correct antonym.
Vocabulary/One Word Substitution A-13

183. (b) Rabble: a disorderly crowd; a mob. 193. (a) The word vague means not clear so, the words vague
Nobility: the quality of being noble in character; and clear are opposite to each other. The words faint
integrity. and obscure are similar to the given word. The word
Hence ‘Nobility’ is the correct antonym. vogue has no relevance.
184. (a) Coterie: a small group of people with shared interests or 194. (d) The words (frugal means economical) & (extravagant
tastes, especially one that is exclusive of other people. free spending) are opposite to eachother. The word
Loner: a person that prefers not to associate with 'gluttonous means excessively greedy' and the word
others. plentiful means abundant are not related.
Hence ‘Loner’ is the correct antonym. 195. (a) 196. (b) 197. (d) 198. (b)
185. (b) 186. (b) 187. (a) 188. (a) 199. (c) The antonym of naive is experienced. Other options
189. (a) Allegiance and loyalty means commitment to a superior are nearly same in meaning.
or to a group or cause. The words disloyalty and 200. (d) Bully
treachery are opposite to the given word; the word Other options are nearly same in meaning.
violence is not related. 201. (a) The word ‘conventional’ means having a limited or
190. (b) The word abash and ashamed are similar to each other. narrow outlook or scope.
The words intrepid and dauntless are nearly opposite Other words are nearly opposite in meaning.
to the given word and the word honest is not related. 202. (c) Frequent
191. (c) Here, the words altruism and selflessness are similar Other options are nearly similar in meaning to the word
to each other. The words selfishness and deceitful are ‘sporadic’.
nearly opposite in meaning to the given word and the 203. (c) Acquired
word scepticism is irrelevant. Other words are nearly similar in meaning to the given
192. (d) The words adroit and incompetent are opposite to word.
each other. 204. (d)
The word adept means skilled is similar to the given 205. (c) to get into difficult situation
word, remaining words are irrelevant. 206. (a) to rain heavily
207. (b) to be bitterly hostile
EBD_8177
A-14 Error Detection

2 Error Detection

DIRECTIONS (Qs. 1 - 5) : In each of the following questions, DIRECTIONS (Qs. 16 - 20) : In each of the following questions,
find out which part of the sentence has an error. If there is no find out which part has an error. [2012-II]
mistake, the answer is (d) “No error”. [2011-I] 16. If you will (a) / follow my instructions (b) / you will get
1. A person I met (a) / in the theatre (b) /was the playwright (c) / a suitable reward for this (d)
himself. (c)/ No error. (d) 17. Harshad, along with (a) / his brother (b) / Ashwani and six
2. They walked (a) / besides each other (b) / in silence (c) / No senior officials (c) / were arrested (d)
error. (d) 18. He received timely support (a) / from his elder brother (b)
/ who is working abroad (c) / for the last six years (d)
3. We returned to the guest house (a)/ impressed by (b) /
19. One of the drawbacks (a) / of modern education are (b) / that
What we had seen (c) / No error. (d)
it does not encourage original thinking (c) / No error (d)
4. The judge was convinced (a)/ that neither (b)/ of the five
20. Morphine and other (a) / narcotic drugs are valuable (b)/
accused was guilty (c)/ No error. (d)
medically, if misused (c) / it can cause irreparable damage (d)
5. The municipality is going (a)/ to built a new school (b)/ near DIRECTIONS (Qs. 21 - 25) : In each of the following questions,
the park (c)/ No error. (d) find out which part has an error. [2013-I]
DIRECTIONS (Qs. 6 - 10) : In each of the following questions, 21. The police has (a)/arrested the thief (b)/who broke into my
find out which part of the sentence has an error. If there is no house (c)/last night. (d)
mistake, the answer is (d) “No error”. [2011-II] 22. The man who (a)/they thought to be (b)/a gentleman turned
6. It was a year since (a) / I received any letter (b) / from my out (c)/to be a rogue. (d)
sister. (c) / No error. (d) 23. I told him on his face (a)/that he could not hope (b)/to pass
7. His family members may arrive (a) / any moment (b) / by car. the stringent (c)/Medical examination of the Services
(c)/ No error. (d) Selection Board. (d)
8. He went to office (a) / but returned back (b) / home 24. Mohan is one of those boys (a)/who has expressed (b)/
willingness for joining (c)/the education tour. (d)
immediately (c) / No error. (d)
25. I may spend (a)/ this summer vacations (b)/with one of my
9. The two brothers amicably divided (a) / their parent’s
friends (c)/ in the back waters of Kerala. (d)
property (b) /among them. (c) / No error. (d)
DIRECTIONS (Qs. 26 - 30) : Choose the correctly spelt word:
10. To attain a high academic standard (a) / in his college Sunil [2014-I]
worked hard (b) / since morning till night (c) / No error.
26. The school alumni gathering put us in a ______ mood.
DIRECTIONS (Qs. 11 - 15) : In each of the following questions,
(a) Remniscent (b) Reminisent
find out which part has an error. [2012-I]
(c) Reminiscent (d) Reminicent
11. The boos was irritated (a) / by him neglecting (b) / the duties 27. ‘God is Dead’ is a ___________ statement.
and (c) / not listening to his advice (d) (a) Blasphemus (b) Blaphemous
12. Each of the three (a) / beggars were (b) / asking for more (c)/ (c) Blasphemous (d) Blosphemos
food to eat (d) 28. The threat of an epidemic caused great alarm and _______.
13. My brother sent (a) / two pairs (b) / of shoe (c) / from America (a) Trepidation (b) Terpidation
(d). (c) Trepidition (d) Trepidattion
14. The young boy said (a) / that he (b) / neither liked me (c)/ 29. The din caused by the children howling is enough to ____
nor my wife (d). the dead.
15. He was (a) / congratulated for (b) / his success in (c) / the (a) Ressurect (b) Resurrect
100 m race (d) (c) Resurect (d) Resurecct
Error Detection A-15

30. Can you ___ the car into that parking spot? 49. (a) Hegamony (b) Hegemony
(a) Manuer (b) Manever (c) Hegemoney (d) Hegamoney
(c) Manoeuvre (d) Manuver 50. (a) Parpetuate (b) Perpetuete
(c) Perpetuate (d) Perpatuate
DIRECTIONS (Qs. 31 - 34) : Choose the correctly spelt word:
51. (a) Apocaleptic (b) Apocalaptic
[2014-II]
(c) Apocalyptic (d) Apacalyptic
31. (a) Parentheses (b) Parenthsis
DIRECTIONS (Qs. 52 - 55): Choose the correctly spelt word:
(c) Parentesis (d) Parenthses [2017 - I]
32. (a) Verstile (b) Versatile 52. (a) Accreditation (b) Acreditation
(c) Versetile (d) Versatele (c) Accredtation (d) Accrediation
33. (a) Hemmorrhage (b) Hemorrhage 53. (a) Vicarrious (b) Viccarious
(c) Haemorrhage (d) Hemmorrage (c) Vecarious (d) Vicarious
54. (a) Abbreviate (b) Abreviate
34. (a) Vetnerinarian (b) Veternarian
(c) Abrrviate (d) Abbreviat
(c) Vetrinarian (d) Veterinarian 55. (a) Acquainttance (b) Acqquaintance
DIRECTIONS (Qs. 35 - 38) : Choose the correctly spelt word : (c) Accquaintance (d) Acquaintance
[2015-I] DIRECTIONS (Qs. 56 - 60): Choose the correctly spelt word:
35. (a) Konnoisseur (b) Conoisseur [2017 - II]
(c) Connoisseur (d) Konoisseur 56. (a) Varmillion (b) Vermillion
36. (a) Munifisent (b) Muneficent (c) Varmilion (d) Vermilion
(c) Munificent (d) Munificient 57. (a) Scraptorium (b) Scriptorium
37. (a) Equanmity (b) Equannimity (c) Screptorium (d) Scriptoreum
58. (a) Thisaurus (b) Thaesaurus
(c) Equanimmisty (d) Equinimity
(c) Thesaurus (d) Thesaeurus
38. (a) Vetarinary (b) Veterinary 59. (a) Ultterior (b) Ultereor
(c) Vetennary (d) Vetniary (c) Ultarior (d) Ulterior
DIRECTIONS (Qs. 39 - 43) : Choose the correctly spelt word : 60. (a) Reconnaissence (b) Reconnaisance
[2015- II] (c) Reconnaissance (d) Reconaissance
39. (a) Dolorous (b) Dolurous DIRECTIONS (Qs. 61-64) : In these questions some of the
(c) Doloreus (d) Delorious sentences have errors and some have none. Find out which part
40. (a) Condiut (b) Conduit of a sentence has an error and indicate it corresponding to the
appropriate letters. If there is no error, indicate corresponding
(c) Connduit (d) Condeut
to the option of No Error. [2018 - II]
41. (a) Gragarious (b) Gragerious
61. The Allahabad High Court on Monday dismissed a plea
(c) Gregarious (d) Grigareous from private power producers seeking relieve from (A)/ an
42. (a) Intrensic (b) Intrinsic RBI diktat to banks to take cognizance of a stressed loan
(c) Entrensic (d) Entrinsec (B)/ if repayments were missed even by a day. (C)/ No Error
43. (a) Sepulchral (b) Sepalchrle (D)
(c) Sepalchral (d) Sepulchrle (a) A (b) B
DIRECTIONS (Qs. 44 - 47): Choose the correct spelt word: (c) C (d) D
[2016 -I] 62. The Turkish lira, which has lost (A)/ almost half its value
this year, is (B)/ another currency in doldrums. (C)/ No Error
44. (a) Omminous (b) Omineous
(D)
(c) Ominous (d) Omenous
(a) A (b) B
45. (a) Hindrance (b) Hinderance
(c) C (d) D
(c) Hindrence (d) Hinderence
63. The mandate of emerging market central banks in (A)/the
46. (a) Perjery (b) Perjury current scenario should be to let their currencies find (B)/
(c) Purjury (d) Perjary their true value in a smooth manner. (C)/ No Error (D)
47. (a) Reminiscene (b) Reminiscence (a) A (b) B
(c) Reminicence (d) Remeniscence (c) C (d) D
DIRECTIONS (Qs. 48-51) Choose the correctly spelt word:- 64. Developmental activities of the (A)/ government come to a
[2016 - II] standstill (B)/ due to paucity of funds (C)/ No Error (D)
48. (a) Ascendancy (b) Ascendncy (a) A (b) B
(c) Ascandency (d) Acsendancy (c) C (d) D
EBD_8177
A-16 Error Detection

DIRECTIONS (Qs. 65-68): In the following question, some part 70. It is feared that (A) / hundreds of people (B) / have lost
of the sentence may have errors. Find out which part of the lives. (C) No error (D).
sentence has an error and select the appropriate option. If a (a) A (b) B
sentence is free from error, select ‘No Error’. [2019-I]
(c) C (d) D
65. It was being hard to believe (A)/that my brother could be
71. He is (A) / one of those students (B) / who comes late
(B)/involved in anything so sinister. (C)/No error(D)
regularly. (C) / No error (D).
(a) A (b) C
(a) A (b) B
(c) C (d) D
(c) C (d) D
66. The average age at which (A)/people die of heart diseases
72. This Child (A) / was run over (B) / with a private bus. (C) /
(B)/are decreasing. (C)/No error (D)
No error (D).
(a) A (b) B
(a) A (b) B
(c) C (d) D
(c) C (d) D
67. Hardly had I stepped (A)/out of my house when (B)/I saw
them coming towards my house. (C)/No error(D) DIRECTIONS (Qs. 73-76): Directions: In each of the question
(a) A (b) B part, find out which part has an error. If there is no mistake, the
(c) C (d) D answer is ‘No error’. [2020 - I]
68. The last Mughal emperor was (A)/send into exile (B)/by the 73. (a) The poor (b) is oppressed
British. (C)/ No error (D). (c) All around the world (d) No error
(a) A (b) B 74. (a) Mohan and me are (b) going to learn
(c) C (d) D (c) many tricks (d) No error
DIRECTIONS (Qs. 69-72) : In questions no 11 to 14 some parts 75. (a) We shan’t (b) invite them
of the sentences have errors and some are correct. Find out
(c) won’t we? (d) No error
which part of a sentence has an error, If a sentence is free from
error, mark (d) in the answers. [2019 - II] 76. (a) the USA is aiming at
69. Our country (A) / does not lack (B) / in scientists of quality. (b) double its bilateral trade
(C) /No error (D). (c) with India by 2015
(a) A (b) B (d) no error
(c) C (d) D
Error Detection A-17

ANSWERS & EXPLANATIONS

1. (a) Here it should be the definite article 'the' person instead 15. (b) The preposition for should be replaced with on. Hence,
of indefinite 'a' person. the correct sentence should be he was congratulated
2. (b) Beside means next to, at the side of while besides means on his success in the 100 m race.
making an additional point; anyway. 16. (a) In the conditional sentence the correct patter should
3. (a) The construction of the sentence should be as be 'If you follow my instructions……..
'Impressed by what we had seen, we returned to the 17. (d) Along with, like in addition to, and together with, is
guest house. often employed following the subject of a sentence or
clause to introduce an addition. The addition, however
4. (d) The sentence is correct. When neither, a singular form,
does not alter the tense of the verb, which is governed
is followed by a prepositional phrase with a plural
by the subject. The king (singular), along with two
object, there is a tendency, esp. in speech and less
aides, is expected in an hour. Hence, were is to be
formal writing, to use a plural verb and pronoun: replaced with was.
Neither of the guards were at their stations. In edited
18. (c) Present perfect continuous tense talks of an action or
writing, however, singular verbs and pronouns are actions that started in the past and continued until
more common: Neither of the guards was at his station. recently or that continue into the future. Hence, the
This use of a singular verb and pronoun is usually correct structure of the sentence should be - who has
recommended by usage guides. been working abroad.
5. (b) The infinitive form of the verb is always in the first 19. (b) Here the verb 'are' is to be replaced with is; hence, the
form and preceded by to (e.g., to run, to dance, to correct sequence of the sentence is 'one of the
think). Hence here the municipality is going to build a drawbacks of modern education is …….
new school' should be correct option. 20. (d) Since morphine and other narcotic drugs is in plural
6. (a) Here 'was a' should be replaced with 'has been'. Thus so, the pronoun 'it' should be replaced with 'they'.
the sentence should be 'It has been a year since I 21. (a) The police have arrested the thief who broke into my
received any letter from my sister.' house last night. Here the verb 'has' is to be replaced
7. (a) Here 'may' should be replaced with 'can'. Thus the by 'have' because the subject is plural.
sentence should be 'His family members can arrive
22. (d) Replace 'to be a rogue' with 'to a rogue'
any moment by car'.
8. (b) Here 'back' should be removed from the sentence. 23. (b) Here 'could' is to be replaced by 'should'. Could is used
Return means to go backwards to where you have left in a hypothetical situation, where the speaker wants to
before. express ability instead of willingness. Whereas 'should'
9. (c) Among should be replaced with between. Between expresses advisability.
should be used where the relationship is distinctly 24. (d) Replace 'education tour' with 'educational tour'
one-to-one. Whereas Among should be used where 25. (b) Here vacation is to be used as a singular because the
the entities are considered as a group.
noun 'friend' present after the pronoun is a plural.
10. (c) Here 'since' should be replaced with 'from'. From is
used to indicate a specified place or time as a starting 26. (c) 27. (c) 28. (b)
point. 29. (b) 30. (c)
11. (b) The correct sentence should be the boss was irritated 31. (a) Parentheses. It's a plural form of parenthesis.
with him for neglecting the duties and not listening to 32. (b) Versatile
his advice. 33. (c) Haemorrhage
12. (c) The correct sentence should be 'each of the three beg- 34. (d) Veterinarian
gars was asking for more food to eat, each is often 35. (c) Connoisseur is an expert judge in matters of taste. E.g.,
followed by a prepositional phrase ending in a plural A connoisseur of music.
word (Each of the cars), thus confusing the verb choice.
36. (c) Munificent in its adjective form means characterised
Each is always singular and requires a singular verb.
by or displaying great generosity.
Each of the students is responsible for doing his or
37. (a) Equanimity means calmness and composure, especially
her work in the library.
in a difficult situation.
13. (c) The correct sentence is 'My brother sent two pairs of
shoes from America'. 38. (c) Veterinary : Relating to the diseases, injuries and
treatment of farm and domestic animals.
14. (c) The correct sentence should be 'The young boy said
that he liked neither me nor my wife. 39. (a) 'Dolorous' is the correctly spelt word which means
feeling or expressing great sorrow or distress.
EBD_8177
A-18 Error Detection
40. (b) 'Conduit' is the correctly spelt word which means a 60. (c) Reconnaissance means military observation of a region
channel for conveying water or other fluid.
41. (c) 'Gregarious' is the correctly spelt word which means to locate an enemy or ascertain strategic features.
fond of company or sociable. 61. (a) Replace verb 'relieve' with noun 'relief'. Relief (noun):
42. (b) 'Intrinsic' is the correctly spelt word which means financial or practical assistance given to those in
belonging naturally or essential. special need or difficulty.
43. (a) 'Sepulchral' is the correctly spelt word which means 62. (c) Replace 'in doldrums' with correct idiomatic expression
relating to a tomb.
'in the doldrums' which means 'in a state of stagnation;
44. (c) Ominous means giving the worrying impression that
lacking activity or progress'.
something bad is going to happen; threateningly
inauspicious. 63. (d) No Error.
45. (a) Hindrance means something that interferes with (or 64. (b) Replace 'come' by 'have come'. As the sentence doesn't
delays) action or progress. imply a regular habit, universal truth or general fact
46. (b) Perjury means criminal offence of making false but a time based event hence, Perfect Tense is
statements under oath. required.
47. (b) Reminiscence means the act or process of recollecting 65. (a) “Being” should be removed because two forms of
past experiences or events. “be” in a complement should not be used.
48. (a) Ascendancy means occupation of a position of
66. (c) When we talk about “age” and if we need to talk
dominant power or influence.
49. (b) Hegemony means leadership or dominance quality of about the context related to lessening, the word
a person. “reduce” should be used.
50. (c) Perpetuate means to make something continue 67. (d) No Error
indefinitely.
51. (c) Apocalyptic means describing or prophesying the 68. (b) As the sentence is in passive and in passive voice
complete destruction of the world. structured sentence, we use “be + v3” and the 3rd
52. (a) Accreditation is a process of validation in which form of “send” is “sent” thus “sent” should be used
colleges, universities and other institutions of higher in place of “send”.
learning are evaluated. 69. (c) Remove “in” from the (C) part.
53. (d) Vicarious means experienced in the imagination through
70. (c) Use “their before” lives-use their before lives
the feelings or actions of another person.
54. (a) Abbreviate is a verb which means to put shorten a 71. (c) Use come” in place of “comes”
word, phrase or text. 72. (c) Use “by” in place of “with”.
55. (d) Acquaintance is a noun which means to have 73. (b) the word ‘poor’ is a plural here, hence the helping
knowledge or experience of something.
verb should be in plural (are) form.
56. (d) Varmillion means brilliant red pigment made from
mercury sulphide (cinnabar). 74. (a) the word ‘me’ is an objective case it should be re-
57. (b) Scriptorium means a room set apart for writing, placed with nominative case ‘I”.
especially one in a monastery where manuscripts were
75. (c) the options (a) and (b) are sentences and the option
copied.
58. (c) Thesaurus means a reference work that lists words (c) is question tag which is not correct. The correct
grouped together according to similarity of meaning in question tag is ‘shall we?’ Hence, the option (c) is the
contrast to a dictionary. correct answer.
59. (d) Ulterior means existing beyond what is obvious or
76. (b) The word ‘double’ should be replaced with ‘doubling’
admitted; intentionally hidden.
Sentence Completion/Cloze Test A-19

Sentence Completion/
3 Cloze Test
DIRECTIONS (Qs. 1 - 5) : Pick out the most effective word from DIRECTIONS (Qs. 11 - 15) : Pick up the most effective word from
the given words to fill in the blank to make the sentence the given words to fill in the blanks to make the sentence
meaningfully complete. [2011-I] meaningfully complete. [2012-I]
1. His actions had ________ pain and suffering on thousands 11. Sanjay was______________ with divine vision to see the
of people. great battle.
(a) affected (b) imposed (a) demure (b) authorized
(c) inflicted (d) deplored (c) endowed (d) uttered
2. The Government will ______ all resources to fight poverty. 12. There was so much__________ material in the essay that it
(a) collect (b) exploit was difficult to get the author’s message.
(c) harness (d) muster (a) variegated (b) superficial
3. The children ______ crackers to celebrate the victory of (c) extraneous (d) exemplary
their team. 13. The world is so constructed that if you wish to enjoy its
(a) burst (b) fired pleasures, you must also______ its pains.
(c) shot (d) released (a) deny (b) neglect
4. I am ______ forward to our picnic scheduled in the next (c) ignore (d) endure
month. 14. Indian press did not give ____________ to the British
(a) seeing (b) looking publicity.
(c) planning (d) thinking (a) credence (b) scion
5. I hope you must have _____ by now that failures are the (c) augury (d) opportunity
stepping stones to success. 15. Travellers ____________their reservations well in advance
(a) known (b) felt if they want to travel during the Diwali holidays.
(c) decided (d) realized (a) has better to get (b) had better get
DIRECTIONS (Qs. 6 - 10) : Pick out the most effective word from (c) had to get better (d) had better got
the given words to fill in the blank to make the sentence DIRECTIONS (Qs. 16 - 20) : Pick up the most effective word
meaningfully complete. [2011-II] from the given words to fill in the blanks to make the sentence
6. Mohini is an independent and innovative thinker, it is best meaningfully complete. [2012-II]
to grant her a good deal of __________ with regard to the 16. His book was marked by many ______ remarks which made
direction of her research. us forget its main theme.
(a) leverage (b) interest (a) irrelevant (b) objective
(c) assistance (d) money (c) slanted (d) digressive
7. The __________ of meat in your refrigerator does not 17. Some people have the capacity for learning foreign lan-
necessarily indicate that you are a vegetarian. guages but they have no ________ to speak.
(a) presence (b) absence (a) interest (b) ability
(c) amount (d) colour (c) fondness (d) inclination
8. Due to the rise of new media technology, many people predict 18. The dispute among the parties became so _______ that
newspapers will soon be __________ . there was every likelihood of a free exchange of blows
(a) obsolete (b) ubiquitous (a) complicated (b) acrimonious
(c) commonplace (d) widespread (c) bellicose (d) aggressive
9. Attention to detail is the __________ of a fine craftsman. 19. The judge decided to resign when he was _______ for pro-
(a) hallmark (b) stamp motion to Chief Justice.
(c) authenticity (d) show (a) passed by (b) passed out
10. Although the two sisters are twins, they look somewhat (c) passed off (d) passed over
__________. 20. Questions will be answered by a _______ of experts.
(a) alike (b) unique (a) staff (b) panel
(d) different (d) related (c) bunch (d) band
EBD_8177
A-20 Sentence Completion/Cloze Test

DIRECTIONS (Qs. 21 - 25) : Pick up the most effective word 33. (a) feels (b) offers
from the given words to fill in the blanks to make the sentence (c) owes (d) acknowledges
meaningfully complete. [2013-I] 34. (a) amount (b) fund
21. According to the weather __________ it is going to be (c) loan (d) debt
cloudy today. 35. (a) labours (b) discoveries
(a) announcement (b) indication (c) achievements (d) successes
(c) prediction (d) forecast 36. (a) strange (b) useful
22. The villagers __________ the murder of their leader by (c) advantageous (d) profitable
burning the police van. 37. (a) decrease (b) disappears
(a) protested (b) avenged (c) alleviate (d) belittle
(c) mourned (d) consoled DIRECTIONS (Qs. 38 - 43) : Select the most appropriate word
23. While on the routine fight, the aircraft was hit by a missile from the options against each number : [2015-I]
and __________ into flames. As home entertainment, television is rapidly becoming more (38)
(a) fired (b) burst than any other form. A news broadcast becomes more immediate
(c) caught (d) engulfed when people (39) actually see the scene (40) question and the
24. Hari got the company car for a __________ price as he was movement of the figures. Films could be viewed in the (41) of the
the senior most employee in the company. home and a variety of shows are also available. One of the
(a) reduced (b) discounted advantages of travel programmes is the (42) of faraway places
(c) fixed (d) nominal which many viewers would not (43) see.
38. (a) interesting (b) popular
25. The unruly behaviour of the soldiers ___________ their (c) powerful (d) purposeful
commander. 39. (a) could (b) would
(a) clashed (b) aggrieved (c) might (d) shall
(c) incensed (d) impeached 40. (a) of (b) with
DIRECTIONS (Qs. 26 - 31) : Select the most appropriate word (c) as (d) in
from the options against each number : [2014-I] 41. (a) surroundings (b) assistance
(c) comfort (d) privilege
Those living in the slums are (26) and tough because they are 42. (a) glimpses (b) image
totally (27) to the vagaries and hardships of life. The rising sun (c) portrait (d) picture
(28) the day and the setting sun closes the day for them. It is like 43. (a) possible (b) rather
a drama where the curtain (29) up in the morning and comes down (c) else (d) otherwise
in the evening. They don’t (30) hypertension and heart attacks DIRECTIONS (Qs. 44 - 49) : Select the most appropriate word
because there is, after all (31) to worry about. from the options against each number: [2015- II]
26. (a) Poor (b) Sick Life is an (44) series of challenges and opportunities to be seized.
(c) Hardy (d) Weak You have to plan for exercising the right career choices and (45) the
27. (a) Dependent (b) Independent right opportunities. Planned (46) rather than a hasty decision is (47)
(c) Exposed (d) Tried as far as your career is concerned. You need to (48) what occupational
groups, (49) and type of organisation are suitable for you.
28. (a) Heralds (b) Herald 44. (a) enticing (b) exciting
(c) Bring (d) Brings (c) encharming (d) enhancing
29. (a) Hangs (b) Hand 45. (a) catching (b) offsetting
(c) Goes (d) Shines (c) grabbing (d) conceiving
46. (a) delay (b) hindrance
30. (a) Know (b) Get (c) application (d) execution
(c) Think (d) Have 47. (a) desirable (b) deciphered
31. (a) Everything (b) Something (c) inevitable (d) acceptable
(c) Somewhere (d) Nothing 48. (a) check (b) classify
(c) divide (d) analyse
DIRECTIONS (Qs. 32 - 37) : Select the most appropriate word 49. (a) specifications (b) limitations
from the options against each number: [2014-II] (c) qualifications (d) identification
Science has made an (32) contribution to the relief of human DIRECTIONS (Qs. 50 - 54) : Today most businessmen are very
suffering and humanity (33)a deep (34) of gratitude to scientists worried. To begin with, they are not used to competition.In the
whose (35) and sacrifices have led to many (36) discoveries and past they sold whatever ...(50)... produced at whatever prices
inventions which have done so much to (37) human pain and they chose. But ...(51)... increasing competition, customers began
misery. to ...(52)... and choose. Imports suddenly became ...(53)...
32. (a) excessive (b) enormous available and that too at cheaper ...(54)...
(c) intensive (d) active 50. (a) it (b) he
(c) they (d) we
Sentence Completion/Cloze Test A-21

51. (a) with (b) by 66. I have recently used the services of his ______________
(c) after (d) from agency to book a cruise in the Mediterranean.
52. (a) buy (b) take (a) progress (b) deportation
(c) pick (d) want (c) travel (d) transfer
53. (a) hardly (b) easily 67. They would like local authorities to be given greater
(c) frequently (d) conveniently ________________ as to how the money is spent.
54. (a) costs (b) returns (a) affairs (b) function
(c) dividend (d) prices
(c) omission (d) discretion
DIRECTIONS (Qs. 55 - 59): Select the most appropriate word 68. In a 10-billion-year-old galaxy there should have been ample
from the options against each number: [2016 - II]
________________ for at least one species to escape its
About sixty percent of the human body is water. If you could own mess, and to spread across the stars, filling every niche.
(55) out a human being like a (56). you would obtain about fifty (a) negligence (b) opportunity
litres of water. This water which is not like (57) water because of
the substance it (58) is necessary to the life of the human (c) surveillance (d) supply
being. About a gallon of it is in the blood vesseis and is (59) 69. A true ________________ of the resources involved in
circulating by temperature. sport would include the unpaid labour services.
55. (a) work (b) wrench (a) growth (b) consideration
(c) press (d) squeeze (c) guidance (d) estimation
56. (a) mango (b) lemon DIRECTIONS (Qs. 70-72) : In the following passage some of the
(c) fruit (d) banana words have been left out. Read the passage carefully and select
57. (a) usual (b) ordinary the correct answer for the given blank out of the four alternatives.
(c) common (d) normal [2018 - II]
58. (a) dissolves (b) carries Tibet (70) _________ up images of a mystic land. Snow-capped
(c) contains (d) includes
mountain peaks pierce the blue sky and fierce chilly winds sweep
59. (a) continued (b) made
the rolling grasslands. Maroon-robed Buddhist monks pray in
(c) got (d) kept
remote monasteries and (71) ________ horsemen pound the
DIRECTIONS (Qs. 60-64) : In the following passage there are rugged earth. People in this high plateau perform punishing rituals
some numbered blanks. Fill in the blanks by selecting the most
like prostrating hundreds of miles in tattered clothes on pilgrimage.
appropriate word for each blank from the given options.
Spirits, spells and flying apparitions are part of the Tibetan world.
[2018 - I]
In short, Tibet remains an (72).
In tropical countries, certain crops are grown ............(60)............
70. (a) molds (b) conjures
the year. These countries have ............(61)............ rainfall for the
crops. They also have plenty of sunshine that ............(62)............ (c) puts (d) toil
the crops. More food than is ............(63)............ can be grown in 71. (a) sturdy (b) wobbly
these places. But there are other countries in the world where it is (c) devilish (d) drained
............(64)............ to grow crops. 72. (a) inspiration (b) abhorrent
60. (a) Along (b) Over (c) exotica (d) heaven
(c) Through out (d) Across DIRECTIONS (Qs. 73-75) : In the following passage some of the
61. (a) Sufficient (b) Little words have been left out. Read the passage carefully and select
(c) Plenty (d) Inadequate the correct answer for the given blank out of the four alternatives.
62. (a) Opens (b) Gathers [2019-I]
(c) Destroys (d) Ripens Some scholars, while exploring the forests of America, discovered
63. (a) Cooked (b) Required
some buildings that were in ruins. These buildings were
(c) Planted (d) Used
(73)_________ ruined by encroaching forest. They were
64. (a) Difficult (b) Rough
(c) Smooth (d) Impossible remnants of a (74)________ civilization. The scholars got
interested. They excavated more and discovered
DIRECTIONS (Qs. 65-69): Given below are sentences with a
(75)___________ their utmost surprise the remains of a
blank in each. Identify the most suitable alternative among the
flourishing civilization-the Mayas as they named it.
five given that fits into the blank to make the sentence logical
and meaningful. [2018 - II] 73. (a) reasonably (b) surely
(c) apparently (d) perfectly
65. In the same amount of time it would take me to correct all the 74. (a) great (b) invisible
________________ in your report, I could write a better
report myself. (c) static (d) ordinary
75. (a) at (b) for
(a) mistakes (b) problems
(c) by (d) to
(c) accuracies (d) obstacles
EBD_8177
A-22 Sentence Completion/Cloze Test

DIRECTIONS (Qs. 76-80) : In the following questions, sentences 86. (a) being delivered (b) to be delivered
are given with blanks to be filed in with an appropriate word(s). (c) to have delivered (d) to deliver
Four alternatives are suggested for each question. Choose the 87. (a) less quickly
correct alternative out of the four as your answer. [2019-I] (b) too quickly
(c) so quickly that
76. Grandmother has a good memory; she can remember things
(d) as quickly as
which __________ many years ago. 88. (a) back out (b) check out
(a) had happened (b) have happened (c) come in (d) rely on
(c) happened (d) happens to be 89. (a) to (b) for
(c) at (d) on
77. I _________ her among the crowd just now.
(a) have glimpsed (b) had glimpsed DIRECTIONS (Qs. 90-93): In the following questions, the
(c) have been glimpsing (d) glimpsed sentence given with blank to be filled in with an appropriate
word. Select the correct alternative out of the four and indicate
78. Lost time is ________ again, and what we call time enough
it by selecting the appropriate option. [2020 - I]
always proves little enough.
(a) found never (b) find never 90. A leaderless police force........... the path of least resistance
since there was no one in authority to spur it to action.
(c) never found (d) never been found
(a) Take (b) Took
79. To such a degree ____________ that people rebuked him. (c) Tread (d) Negotiate
(a) he made a noise (b) did he make a noise 91. this has provided a ray of hope and there is no dearth of
(c) he had made a noise (d) did he make noise data as far as cervical cancer and prevention are.............
(a) Concerned (b) Forgotten
80. I’m going to adopt her as ________ as Julie and I get
(c) Discernible (d) Detectable
married.
92. the program has ......... a DNA based HPV test and offered it
(a) sooner (b) quickly for free to these women.
(c) earlier (d) soon (a) Relate (b) Deploying
DIRECTIONS (Qs. 81-84) : In questions no. 15 to 18, sentences (c) Deployed (d) Refrain
are given with blanks to be filled in with an appropriate word 93. There were four instances when Delhi Police gave short
(S). Four alternatives are suggested for each question. Choose shrift to the laws of the land and, indeed, to the very
the correct alternative out of the Four options. [2019 - II] Constitution they are ............. to uphold.
(a) Swear (b) Sworn
81. After the treatment, he was relieved ___________ the pain
(c) Sword (d) Firm
(a) For (b) Of
(c) From (d) With DIRECTIONS (Qs.94-96): In these questions, in the following
82. Several items have been left____________ from the list passage some of the words have been left out. Read the passage
(a) Out (b) Within carefully and choose the correct out of four alternatives. [2020 - I]
(c) Behind (d) Off For the rule of law to be well .....65........ and enforced, we need to
83. I have not paid my school fees_________ start at the very beginning — mobilising our local communities. A
(a) Already (b) Now strong, mobilized, civil society is a necessary condition to hold the
(c) Then (d) Yet State to account. It is not about one seemingly revolutionary
protest, even if the well-meaning antagonists of the CAA ......66.......
84. My father has _________ returned home from work.
believe so. When violence and lawlessness spill over into the
(a) Now (b) Never streets and begin to constrain our work-a-day lives, as it has across
(c) Defense (d) Just India; when the police are routinely challenged by those on the
DIRECTIONS (Qs. 85-89) : Read the following passage carefully wrong side of the .....67.....; when the police force is overworked
and answer the questions. [2019 - II] and overwhelmed by political subterfuge, our society will be on
the brink. As a society, we are currently engaged in a negative-sum
The postal service is the government agency — (85) —- handles
game. The rule of law cannot, unlike riots, be engineered. It can
the mail. Its job is — (86) — letters and packages to people and only be practiced if we understand that liberty has to be balanced
businesses all over the world. Its goal is to see that your mail with equality; and pluralism entails a measure of negative liberty.
gets to its destination — (87) — possible. People —- (88) —-the 94. (a) Establish (b) Establishing
postal service to deliver important letters and even valuables, — (c) Established (d) None of these
95. (a) genuine (b) genuinely
(89) — time and to the right person.
(c) wrong (d) right
85. (a) the fact that (b) whether 96. (a) face (b) Law
(c) of which (d) that (c) Police (d) None of these
Sentence Completion/Cloze Test A-23

ANSWERS & EXPLANATIONS


1. (c) Inflicted which means make (someone) do something 26. (c) In the sentence, hardy is used along with the word
unpleasant; e.g. "The teacher inflicted his rage on the tough to describe the people living in slums.
students. Other options do not correspond. 27. (c) In the sentence, the author tells how slum people are
2. (c) Harness means exploit the power of some one or some subject to uncertainties and hardships of life.
28. (d) Using the third person singular present form of
thing.
bringinstead of infinitive form.
3. (a) Burst. If you are bursting crackers then you are setting 29. (c) In the sentence, comes is used with evening. And goes
off fireworks. is the opposite of come which will thus come along
4. (b) The correct phrase is look forward to. morning.
5. (d) Realize means perceive (an idea or situation) mentally 30. (d) It is the most appropriate word from the given options.
which is correct filler. 31. (d) The sentence begins with don't, which means nega-
6. (a) Leverage means to use (something) to maximum tion.
32. (b) 33. (c) 34. (d) 35. (a) 36. (b)
advantage.
7. (b) Absence is the most appropriate word because a 37. (c)
contrast is made here. 38. (b) The word rapidly can match popular, not with other
8. (a) Obsolete means no longer in use. options.
9. (b) Here stamp is being used as a trademark or a signature 39. (a) Could shows the ability.
style. 40. (d) In question' means being discussed or considered ex.
10. (c) Although is used to show a contrast. Twins are usually On the days in question there were several serious
considered similar looking but using although makes questions.
the sentence contrasting. 41. (c) In the comfort of home' defines in the home atmosphere
11. (c) The correct option is endowed which means provided without going out for the purpose of entertainment.
or supplied or equipped with (especially as by inherit- 42. (a)
ance or nature). 43. (d) Otherwise television this would not have possible.
12. (c) Extraneous means not pertinent to the matter under 44. (b) 45. (c) 46. (d) 47. (a) 48. (d)
consideration. 49. (a) 50. (c) 51. (a) 52. (c) 53. (b)
13. (d) Endure means put up with something or somebody 54. (d)
unpleasant. 55. (d) Sueeze means to take out something forcefully.
14. (a) Credence means the mental attitude that something is 56. (b) Lemon can be squeezed to take out juice.
believable and should be accepted as true. 57. (d) Normal water means water not having anything
15. (b) The correct filler is 'had better get'. Hence, the correct dissolved in it.
sentence should be the travellers had better get their 58. (c) Water contains essential elements required by the
reservations well in advance if they want to travel dur- human body.
ing the Diwali holidays. 59. (d) Water is kept circulating by temperature.
60. (c) 61. (a) 62. (d) 63. (b) 64. (a)
16. (d) The correct filler here should be digressive.
17. (d) Here the right option should be 'inclination'. 65. (a) The most appropriate word that would fill the blank is
18. (b) Acrimonious is the most effective word among the 'mistakes' which means an act or judgement that is
given options. misguided or wrong. All the other words do not fill the
19. (d) Pass over means to leave out, disregard. blank appropriately, hence option (a) is the most
20. (b) Panel rightly corresponds with the experts. suitable answer choice. Accuracies means the quality
21. (d) Forecast is used to predict or estimate a future event.
or state of being correct or precise. Obstacles means a
22. (b) Avenged means to inflict harm in return for an injury
or wrong done to oneself or another. thing that blocks one's way or prevents or hinders
23. (b) Burst means tobreak open or apart suddenly and progress.
violently, especially as a result of an impact (here the 66. (c) The most appropriate word that would fill the blank is
impact is hit by a missile).
24. (b) Being the senior most employee, Hari got a discount 'travel' which means journeys, especially abroad. All
on the company car. The discount was proportional the other words do not fill the blank appropriately,
to his long service for the company. hence option (c) is the most suitable answer choice.
25. (c) Incensed means extremely angry. Thus the unruly Deportation means the action of deporting a foreigner
behaviour of the soldiers made their commander
from a country. Transfer means move from one place
extremely angry.
to another.
EBD_8177
A-24 Sentence Completion/Cloze Test

67. (d) The most appropriate word that would fill the blank is 79. (b) Sentences beginning with ‘To such a point, ‘To such
'discretion' which means the freedom to decide what a degree’, ‘To such an extent’ take inversion form i.e.
should be done in a particular situation. All the other “To such a degree + H.V. (auxiliary) +Subject + M.V.”
words do not fill the blank appropriately, hence option Moreover ‘make a noise’ is the correct idiomatic
(d) is the most suitable answer choice. Omission means expression not ‘make noise’.
someone or something that has been left out or 80. (d) Soon means ‘a short time after then’ i.e. in or after a
excluded. Affairs means an event or sequence of short time.
events of a specified kind or that has previously been Early means ‘near the beginning of a period of time we
referred to. are talking about’. Early does not mean soon.
Moreover, comparative degree is not needed.
68. (b) The most appropriate word that would fill the blank is
'opportunity' which means a time or set of While we use quickly to refer to the speed with which
something is done.
circumstances that makes it possible to do something.
All the other words do not fill the blank appropriately, Hence option D is the correct choice.
hence option (b) is the most suitable answer choice. 81. (b) With “relived” “of ”, is the correct word.
Negligence means failure to take proper care over hence (B) is the correct choice.
82. (a) Left out means to go out of or away from.
something. Surveillance means close observation,
83. (d) Yet is the most appropriate option because it means to
especially of a suspected spy or criminal.
up until the present or a specified or implied time; by
69. (d) The most appropriate word that would fill the blank is now or then.
'estimation' which means a judgement of the worth or 84. (d) Just is the correct answer.
character of someone or something. All the other 85. (d) that
Here that is the suitable relative pronoun used for
words do not fill the blank appropriately, hence option postal service
(d) is the most suitable answer choice. Consideration 86. (d) to deliver
means careful thought, typically over a period of time. 87. (d) as quickly as means as soon as
Guidance means advice or information aimed at 88. (d) rely on means to put trust in with confidence is the
right choice
resolving a problem or difficulty, especially as given
89. (d) Preposition on should be the right choice before time
by someone in authority. 90. (b) The word ‘took’ is correct usage contextually.
70. (b) 71. (a) 72. (c) 73. (c) 74. (a) 91. (a) the word ‘concerned’ is right in forming passive.
75. (d) 92. (c) ‘deployed’ is the right after the word ‘has’.
76. (c) Subject + V2(past form of verb)+ Object……………..+ 93. (b) the word ‘sworn’ the past participle form of ‘swear’ is
ago. the correct usage contextually.
77. (d) When ‘just now’ means a very short time ago. It takes 94. (c) the word ‘established’ is the correct word usage.
past form of the verb i.e.V2. 95. (b) the word ‘genuinely is an adverb. Hence, it is the right
78. (c) ‘Never’, ‘Seldom’, ‘Always’ are used before the ‘Main word to be used before the verb ‘believe’.
verb’. Also the sentence is in ‘passive voice’. Hence 96. (b) law
option C is the correct choice.
Reading Comprehension A-25

Reading
4 Comprehension
DIRECTIONS (Qs. 1- 5) : Read the following passage carefully DIRECTIONS (Qs. 6-9) : Read the following passage carefully
and answer the questions given below it. [2011-I] and answer the questions given below it. [2011-II]
We stand poised precariously and challengingly on the razor’s Educational planning should aim at meeting the educational needs
edge of destiny. We are now at the mercy of atom bombs and the of the entire population of all age groups. While the traditional
like which would destroy us completely if we fail to control them structure of education as a three layer hierarchy from the primary
wisely. And wisdom in this crisis means sensitiveness to the basic stage to the university represents the core, we should not overlook
values of life; it means a vivid realization that we are literally living the periphery which is equally important. Under modern
in one world where we must either swim together or sink together. conditions, workers need to rewind, or renew their enthusiasm, or
We cannot afford to tamper with man’s single minded loyalty to strike out in a new direction, or improve their skills as much as any
peace and international understanding. Anyone, who does it is a university professor. The retired and aged have their needs as
traitor not only to man’s past and present, but also to his future, well. Educational planning, in other words, should take care of the
because he is mortgaging the destiny of unborn generations. needs of everyone.
1. From the tone and style of the passage it appears that the Our structures of educational have been built up on the
writer is assumption that there is a terminal point of education. This basic
(a) a prose writer with a fascination for images and defect has become all the more harmful today. A UNESCO report
metaphors. entitled ‘Learning to Be’ prepared by Edgar Faure and other in
(b) a humanist with a clear foresight. 1973 asserts that the education of children must prepare the future
(c) a traitor who wishes to mortgage the destiny of future adult for various forms of self-learning. A viable education system
generations. of the future should consist of modules with different kind of
(d) unaware of the global power situation. functions serving a diversity of constituents. And performance,
2. The best way to escape complete annihilation in an atomic not the period of study, should be the basis for credentials. The
war is to writing is already on the wall.
(a) work for international understanding and harmony. In view of the fact that the significance of a commitment of lifelong
(b) invent more powerful weapons. learning and lifetime education is being discussed only in recent
(c) turn to religion. years even in educationally advanced countries, the possibility
(d) ban nuclear weapons. of the idea becoming an integral part of educational thinking seems
3. The phrase ‘razor’s edge of destiny’ means a/an to be a far cry. For, to move in that direction means so much more
(a) enigma that cuts through the pattern of life like the
than some simple rearrangement of the present organisation of
edge of a razor.
education. But a good beginning can be made by developing
(b) critical situation that foreordains the future.
Open University programmes for older learners of different
(c) sharp line of division that marks the alternative courses
categories and introducing extension services in the conventional
of action in the future.
colleges and schools. Also, these institutions should learn to
(d) destiny with sharp edges.
cooperate with numerous community organisations such as
4. According to the writer, ‘wisdom’ on the razor’s edge of
destiny means libraries, museums, municipal recreational programmes, health
(a) awareness that we stand poised precariously on the services etc.
razor’s edge of destiny. 6. What is the main thrust of the author?
(b) determination to ban nuclear weapons. (a) Traditional systems should be strengthened.
(c) responsibility to the ‘unborn generations’. (b) Formal education is more important than non-formal.
(d) awareness of the basic values of life. (c) One should never cease to learn.
5. The author is concerned about the threat of nuclear (d) It is impossible to meet the needs of everyone.
weapons because he feels that 7. What should be the major characteristic of the future
(a) a nuclear war will destroy human civilization. educational system?
(b) all countries are interlinked and one cannot escape the (a) Different modules with same function.
consequences of what happens to another country. (b) Same module for different groups.
(c) the world is on the brink of disaster. (c) No modules but standard compulsory programme for all.
(d) his country is threatened by a nuclear war. (d) None of the above
EBD_8177
A-26 Reading Comprehension

8. According to the author, what measures should open DIRECTIONS (Qs. 15- 17) : Read the following passage
university adopt to meet modern conditions? carefully and answer the questions given below: [2012-II]
(a) Develop various programmes for adult learners. The development and widespread use of computer technology
(b) Open more colleges in traditional lines. and the internet have transformed how we communicate, how
(c) Cater to the needs of those who represent ‘cone’. business is conducted, how information is dispersed, and how
(d) Primary education should be under the control of open society is organised. Prior to 1980, in-depth information about
universities. any one subject matter was attained through laborious research
9. In the context of the passage, what is the meaning of the involving countless visits to libraries and via repeated interviews
sentence “The writing is already on the wall”? with persons of known reputation and reputable expertise. Now, a
(a) Everything is uncertain now-a-days. great deal of information is available at the click of a mouse button,
(b) Changes have already taken place. all attainable from within the confines of one's own home or from
(c) The signs of change are already visible. the use of a computer in an office. Previous labour-intensive
(c) You cannot change the future. support Jobs. such as loading and unpacking of trucks', luggage
DIRECTIONS (Qs. 10-14) : Read the following passage carefully handling at airports, and food manufacturing, once performed by
and answer the questions given below: [2012-I] a large middle-class workforce, are now performed routinely by
Pablo Picasso showed his truly exceptional talent from a very robots which are monitored by computer-controlled systems. Our
young age. His first word was lapiz (Spanish for pencil) and he lives have been simplified but these benefits which have been
learnt to draw before he could talk. He was the only son in the ushered in by the technology revolution have had an adverse
family and very good-looking, so he was thoroughly spoilt. He effect on the core of our interpersonal-relationships. Mere
hated school and often refused to go unless his doting parents communication is no longer via postal mail or face-to-face
allowed him to take one of his father’s pet pigeons with him. contact, but rather via electronic email, personal internet message
Apart from pigeons, his great love was art and when in 1891 his boards and by virtue of hand-held personal electronic assistants.
father, who was an amateur artist, got a job as a drawing teacher at Although computer technology has brought us to within a
a college, Pablo went with him to the college. He often watched mouse-click of any sought-after piece of information, this
his father paint and sometimes was allowed to help. One evening technology boom has sequestered us to the confines of our
his father was painting a picture of their pigeons when he had to
computer desks and homes and has removed us away from those
leave the room. He returned to find that Pablo had completed the
traditional settings where personal and communication skills are
picture, and it was so amazingly beautiful and lifelike that he gave
his son his own palette and brushes and never painted again. developed.
Pablo was just thirteen. 15. The author's attitude the advent of computer technology
10. As a boy Pablo Picasso was can be best summarised as
(a) ordinary looking but talented. (a) optimistic and thankful
(b) handsome and talented. (b) appreciative but reserved
(c) handsome and studious. (c) candid and reverent.
(d) handsome and hardworking. (d) understanding and obsessive
11. He was spoilt mostly because he was 16. The author would agree with which of the following
(a) a smart boy. statements?
(b) loved by one and all. (a) The advent of computer technology has decreased
(c) the only son in the family. access to libraries
(d) always surrounded by notorious boys. (b) Because of advancements in robotics, labour-inten-
12. Picasso went to school only when sive jobs are more plentiful
(a) his friends accompanied him.
(c) Although heralded as a great leap forward, the widespread
(b) his father went with him.
use of computer technology is not without its setbacks
(c) he was allowed to paint at school.
(d) Of all the benefits ushered in by the use the internet.
(d) he was allowed to carry a pet with him.
13. When his father painted in the college, Pablo electronic email is the most beneficial
(a) occasionally helped him.(b) rarely helped him. 17. The author's primary purpose in writing this passage is
(c) always helped him. (d) invariably helped him. most likely which of the following?
14. Pablo’s father gave up painting because he (a) To downplay the need for the internet
(a) did not like the job. (b) To explain how robotics and the internet have had both
(b) retired from the college. a positive and negative influence on how we live
(c) was impressed by his son’s talent. (c) To pave way for the next great technology revolution
(d) lost interest in painting. (d) To showcase the wonders of recent technology ad-
vancements
Reading Comprehension A-27

DIRECTIONS (Qs. 18-22) : Read the following passage carefully 23. Gandhi came like a powerful current of fresh air and
and answer the questions given below: [2013-I] (a) awakened us to the plight of the masses in the grip of
oppressors
We shall go on the end, we shall fight in France, we shall fight on (b) made us patriotic
the seas and oceans, we shall fight with the growing confidence (c) emboldened us to attack and destroy the oppressors
and strength in the air, we shall defend our island, whatever the (d) praised our culture
cost may be, we shall fight on the beaches, we shall fight on the 24. The rise of Gandhi
landing grounds, we shall fight in the fields and in the streets, we (a) shocked people
shall fight in the hills. We shall never surrender, and even if this (b) made India powerful
island or a large part of it was subjugated and starving, then our (c) made the condemnation of the exploiter final
empire beyond the seas would carry on the struggle, until the (d) made women feel secure
New World steps forth to the rescue and the liberation of the Old. 25. Gandhi fought the
18. On the basis of the passage which of the following statements (a) rich (b) oppressor
may be said to be correct? (c) apathetic masses (d) unjust system
26. The conspicuous role of Gandhi is that of a
(a) The speaker is encouraging his men for the conquest
(a) father (b) reformer
of France (c) teacher (d) liberator
(b) The speaker is an aggressive and maniacal war-monger
DIRECTIONS (Qs. 27–29) : Read the following passage
(c) The speaker is not satisfied with the conquest of the island
carefully and answer the questions given below it: [2014-II]
(d) The speaker is a patriot urging the defence of his
Language is often used for one of the following three purposes,
motherland
namely, to inform, to convince and to persuade. The first requiring
19. The speaker in the passage wants to go on fighting because talent of telling what we know, is a matter of little difficulty. The
(a) he is a raving lunatic second demands reasoning. The third, besides reasoning,
(b) he is in a state of utter despair demands all the aid that we can obtain from the use of figures of
(c) he expects help from other quarters speech or figures of rhetoric, which means the power of persuasion.
(d) he is the leader of a suicide squad 27. Rhetoric is the
20. Which of the following pair of the phrases helps best to (a) art of reasoning
bring out the intension of the speaker? (b) use of figure of speech
(a) “Go on to the end”, “shall never surrender” (c) power of persuasion
(b) “Growing confidence”, “subjugated and starving” (d) means of communicating information
28. The art of persuasion requires the use of
(c) “Subjugated and starving”, “fighting and landing around”
(a) information and talent feels
(d) “Fighting in the streets”, “subjugated and starving” (b) reasoning and information
21. The passage consists of repetitive patterns in syntax and (c) figure of speech
vocabulary. The effect of this style is that it (d) reasoning and figure of speech
(a) reveals the speaker’s defects in giving a speech 29. The above passage is
(b) produces the impression of bad poetry (a) informative (b) persuasive
(c) conveys the speaker’s helpless situation (c) convincing (d) rhetorical
(d) reinforces the speaker’s basic intention DIRECTIONS (Qs. 30–32) : Read the following passage carefully
22. The tone of the speaker is and answer the questions given below it : [2015-I]
(a) pleading and urging In spring, polar bear mothers emerge from dens with three months
(b) inspiring and encouraging old cubs. The mother bear has fasted for as long as eight months
(c) discouraging and gloomy but that does not stop the young from demanding full access to
(d) menacing and bullying her remaining reserves. If there are triplets, the most persistent
DIRECTIONS (Qs. 23–26) : Read the following passage stands to gain an extra meal at the expense of others. The smallest
carefully and answer the questions given below it: [2014-II] of the cubs forfeits many meals to stronger siblings. Females are
protective of their cubs but tend to ignore family rivalry over
And then Gandhi came. He was like a powerful current of fresh air food. In 21 years of photographing polar bears. I have only once
that made us stretch ourselves and take deep breaths, like a beam seen the smallest of triplets survive till autumn.
of light that pierced the darkness and removed the scales from our 30. With reference to the passage, the following assumptions
eyes, like a whirlwind that upset many things but most of all the have been made :
working of people’s minds. He did not descend from the top; he I. Polar bears fast as long as eight months due to non
seemed to emerge from the millions of India, speaking their availability of prey.
language and incessantly drawing attention to them and their II. Polar bears always give birth to triplets.
appalling condition. Get off the backs of these peasants and Which of the assumptions given above is/are true?
workers, he told us, all of you who live by their exploitation; get (a) I only (b) II only
rid of the system that produces this poverty and misery. (c) Both I and II (d) Neither I nor II
[2014-I]
EBD_8177
A-28 Reading Comprehension
31. Female polar bears give birth during 36. In the passage the term "urbanisation" has been used to
(a) Spring (b) Summer refer to
(c) Autumn (d) Winter
32. Mother bear (a) the destruction of the greenery in the city
(a) Takes sides over cubs (b) construction of concrete structures on a large scale
(b) Lets the cubs fend for themselves (c) a change over from the rural life to the city life
(c) Feeds only their favourites (d) the movement of people from villages to cities
(d) Sees that all cubs get an equal share 37. The phrase "absentee house sparrow" refers to
DIRECTIONS (Qs. 33-35) : Read the following passage carefully (a) the sparrow that makes infrequent visits
and answer the questions given below it : [2015- II] (b) the sparrow that has forgotten its habitat
Patience is better than wisdom; An ounce of patience is worth a (c) the sparrow that has gone away from the urban areas
pound of brains. All men praise patience, but few can practise it. It (d) the sparrow that has become extinct
is a medicine which is good for all diseases, but it is not every 38. According to the passage, the birds have made a comeback
garden that grows the herbs to make it with. Many people are
to the suburb because they
born crying, live complaining and die disappointed. They think
(a) have been hunted down by the village people
every other person's burden to be light and their own feathers to
be heavy as lead, and yet if the truth were known, it is their fancy (b) Love to be amidst human beings
rather than their fate that makes things go so hard with them. (c) get food only where human beings live
Many would be well off than what they think of. (d) they have been deliber ately brought back by
33. Which of the following is the most suitable explanation to biologists
the author's remark that "anounce of patience is worth a DIRECTIONS (Os. 39–41) Read the following passage carefully
pound of brains"? and answer the questions given below it: [2016 - II]
(a) Wisdom can only be attained by practising patience
The great Acharyas have said that having discovered a great
(b) Patience comes first wisdom next
goal, surrender yourself to that goal and act towards it, drawing
(c) Patience is essential for every achievement
your inspiration from that goal whereby you will get a new col-
(d) Without patience wisdom cannot be used in like
property umn of energy. Do not allow this energy to be dissipated in the
34. Which one is the most likely explanation, among the futile memories of past regrets of failure, not in the excitement of
following, of the author's metaphorical statement that "It is the present, and thus bring that entire energy focussed into activ-
not every garden that grows the herbs to make it with"? ity. That is the highest creative action in the world outside. Thereby
(a) Patience is a must for solving all our problems in life the individual who is till now considered as most inefficient finds
(b) Patience is a rare herb that cures all diseases his way to the highest achievement and success. This is said very
(c) Patience is virtue of a highest order easily in a second. But in order to train our mind to this attitude
(d) It is only a small number of people that are found needs considerable training because 7we have already trained
observing patience in life the mind wrongly to such an extent that we have become perfect
35. The writer's remarks, "They think every person's burden to in imperfections. Not knowing the art of action, we have become
be light and their own feathers to be as heavy as lead" is master artist in doing wrong things. The totality of activity will
very significant. It means bring the country to a wrong end, indeed. The point is, intellect is
(a) They are always worried and dejected very powerful and everyone is driving but nobody seems to know
(b) They consider their own problems to be difficult to
how to control the mental energy and direct it properly, or guide it
solve as compared with problems of other people
(c) They feel that they alone face serious problems while to the proper destination.
others have a nice time 39. Which of the following is the source of energy?
(d) They remain very much worried about their own (a) A column that supports a building.
problem. (b) Stimulation obtained from a set aim.
DIRECTIONS (Qs. 36-38) : Read the following passage carefully (c) Highest creative action.
and answer the questions given below it [2016 -I] (d) Proper training of the mind to achieve perfection.
Recently, a newspaper article mourned the total disappearance of 40. The author's chief concern is
the common house sparrow. (a) Establishment of socialistic pattern.
This was a comment on the city's perceptible move towards edging (b) Discovery of great goal in the life.
out the flora and fauna of the city. In the rapid urbanization, multi- (c) Regulation of energy in the proper direction.
storied apartments grew and large scale felling of trees became (d) Training of the mind.
necessary. Last week, however, seven pairs of these sparrows 41. What is the effect of training of the wrong mind?
were spotted in a suburb. Possibly the greenery of this place has (a) We have become perfect in all aspects.
created a new habitat for these birds which like their proximity to
human beings, and have made a comeback. A systematic (b) Art of action is too much emphasized.
development of trees and shrubs all over the city could woo the (c) Each of us could become a master artist.
absentee house sparrow to our midst. (d) None of these.
Reading Comprehension A-29

DIRECTION (Qs. 42-44): Read the following passage and answer grains of uniqueness (The Russians have never failed that
the following questions: [2017 - I] characteristic in themselves; they have twice experimented with
completely different ideologies, Communism and Capitalism both
At this stage of civilisation, when many nations are brought in to in the space of a century).
close and vital contact for good and evil, it is important, as never 45. Which of the following statements is correct according to
before, that their gross ignorance of one another should be the passage?
diminished, that they should begin to understand a little of one (a) India focused on the weak faults of Russian policies
another's historical experience and resulting mentality. It is the and system.
fault of the English to expect the people of other countries to react (b) India seriously commended the achievement of Russia,
as they do, to political and international situations. i.e., 100% per cent literacy and rapid industrialization.
Our goodwill and good intentions are often brought to nothing, (c) The process of industrialization had already started
because we expect other people to be like us. This would be when Russian revolution took place in 1917.
corrected if we knew the history, not necessarily in detail but in (d) The literature, art and music received a setback during
broad outlines, of the social and political conditions which have the communist regime in Russia.
given to each nation its present character. 46. The West did not pay heed to:
42. According to the author, 'Mentality' of a nation is mainly (a) Rapid growth of nuclear weapons in Russia
product of its (b) Massive human rights violation by the Soviet state
(a) Present character (b) International position on its people
(c) Politics (d) History (c) Planned uprooting and mass migration of ethnic people
43. According to the author, his countrymen should in the name of industrialization.
(a) read the story of other nations (d) Both b and c
(b) not react to other actions 47. India's perception towards USSR was always
(c) have a better understanding of other nations (a) Negative (b) Counter-reactionary
(d) have vital contacts with other nations (c) Applauding (d) Neutral
44. The need for a greater understanding between nations 48. The passage given above is
(a) is more today than ever before (a) Descriptive (b) Paradoxical
(b) was always there (c) Analytical (d) Thought provoking
(c) is no longer there DIRECTIONS (Qs. 49-52) : Read the following passage and
(d) will always be there answer the questions given after it. [2018 - I]
DIRECTIONS (Qs. 45-48): Read the following passage and Dr. Carver was an American Negro slave, who by dint of his
answer the following questions: [2017 - II] ability became a scientist and educator of world-wide fame. A
The most important reason for this state of affairs is that India national monument has now been erected to honour him. This
was the only country in the world to truly recognise the monument has been built at his birth place in the United States of
achievements of the Soviet Union-rather than merely focus on America. Carver's life and achievements prove the American saying
the debilitating faults that Communism brought to its people. The : "You can't keep a great man down." From childhood he showed
people of India realised that the achievement of one hundred per qualities which gave promise of his genius. He would get up
cent literacy in a country much, much larger than its own and with before sunrise to study the wonders of nature before the break of
similarly complicated ethnic and religious groupings, the rapid dawn in the east. His guardians wanted to educate him, but were
industrialization of a nation that was a primarily agrarian society too poor to do so. So he left home. He was hardly ten when he
when the Bolshevik revolution took place in 1917, the attendant began to work at small jobs to earn a little money for his school
revolutionary steps in science and technology, the accessibility expenses. He continued to do so even when he was at college.
of health care (primeval according to Western standards, perhaps, Thus, he passed his M.Sc. examination and became a professor.
but not according to Indian ones) to the general population, and There he wrote several books on science subjects. His chief desire
despite prohibition of the government of the time the vast was to do the greatest good to the greatest number of people. He
outpourings in literature, music, art, etc. are momentous and left all his life's savings to found scholarships for research in
remarkable feats in any country. Agricultural Chemistry. He knew this research, was bound to
In contrast, all that the West focused on were the massive human benefit farmers all over the world. Though world famous, he never
rights violations by the Soviet State on its people, the deliberate felt proud of his discoveries. "I discovered nothing," he once
uprooting and mass migrations of ethnic peoples from one part of said, "I am God's agent—the instrument through which he works."
the country to another in the name of industrialization, the end of 49. What can you say about the early life of Dr. Carver?
religion. In short, all the tools of information were employed to (a) He was born with silver spoon in his mouth.
condemn the ideology of Communism, so much at variance with (b) He was brought up in an orphanage.
capitalist thinking. The difference with the Indian perception, I (c) He had to struggle a lot as his parents were poor.
think here is, that while the Indians reacted as negatively to what (d) He was brought up and educated by wealthy parents.
the Soviet governments did to its people in the name of good 50. Which of the following statements show that he was a great
governance (witness the imprisonment of Boris Pasternak and lover of mankind?
the formation of an international committee to put pressure for his (a) He offered charitable services to the poor.
release with Jawaharlal Nehru at its head), they took the pain not (b) He desired to do the greatest good to the greatest number
to condemn the people of that broad country in black and white of people.
terms; they understood that mingled in the shades of grey were (c) He opened several colleges and institutes.
(d) He donated all his life savings.
EBD_8177
A-30 Reading Comprehension

51. Find out the statement showing that he was humble. 56. Modern medicine is primarily concerned with:
(a) He never felt proud of his discoveries. (a) promotion of good health
(b) He always respected the women. (b) people suffering from imaginary illnesses
(c) He was very polite in his attitudes. (c) people suffering from real illnesses
(d) He admired his own achievements. (d) increased efficiency in work
52. What was of Dr. Carver by profession? 57. A healthy man should be concerned with:
(a) Doctor (b) Politician (a) his work which good health makes possible
(c) Scientist (d) Professor (b) looking after his health
DIRECTIONS (Qs. 53-55) : Study the paragraph and answer (c) his health which makes work possible
the questions that follow: [2018 - II] (d) talking about health
58. Our great concern with health is shown by?
Judiciary has become the centre of controversy, in the recent
(a) free medicine distribution in hospitals
past, on account of the sudden 'Me' in the level of judicial
(b) free education to medical students
intervention. The area of judicial intervention has been steadily
(c) taking yoga classes
expanding through the device of public interest litigation. The
(d) the health articles in popular magazines
judiciary has shed its pro-status-quo approach and taken upon
itself the duty to enforce the basic rights of the poor and vulnerable DIRECTIONS (Qs. 59-60): Read the following passage carefully
sections of society, by progressive interpretation and positive and answer the questions. [2019 - II]
action. The Supreme Court has developed new methods of Plants need sunlight and water to make their own food. You can
dispensing justice to the masses through the public interest do an experiment to test if this is true. Place a bucket over a patch
litigation. Former Chief Justice PN. Bhagwat, under whose of green grass. After a few days, lift the bucket. You will see that
leadership public interest litigation attained a new dimension the grass is not as green anymore. If you leave the bucket in
comments that "the Supreme Court has developed several new place for a week, the grass will become very dull. This happens
commitments. It has carried forward participative justice". because the grass cannot make food in the dark. Remove the
53. The steady expansion of judicial intervention is the result bucket. In a few days, the grass will start turning green again.
of 59. What do plants need?
(a) excessive laws (a) Buckets are bad
(b) public interest litigation (b) Plants need sunlight
(c) Supreme Court's new methods of dispensing justice (c) Grass is not pretty
(d) new commitments of Supreme Court (d) Plants needs air
54. According to the author, judiciary has become the center of 60. What is the experiment?
controversy because of (a) a way to grow grass
(a) problems arising in dispensing justice in the recent past (b) a way to test an idea
(b) public interest litigation (c) a way to use a bucket
(c) sudden 'Me' in the level of judicial intervention (d) a way to use a sunlight
(d) Supreme Court's supremacy
55. According to Justice PN. Bhagwat, Supreme Court has DIRECTIONS (Qs. 61-65): A passage is given with questions
developed following it. Read the passage carefully and choose the best
(a) judicial intervention answer to each question out of the four alternatives. [2020-I]
(b) various new commitments
(c) participative judicial approach to dispense justice My brother, David, was always close to our grandmother. Both
(d) public interest litigation of them shared a love of Mother Nature and of food that they had
DIRECTIONS (Qs. 56-58): Read the passage carefully and choose grown themselves. Whenever his schedule permitted, he would
the best answer to each question out of the four alternatives. drop in for a short visit and a cup of coffee. One day, when he
[2019-I] found no one home, he left a chunk of dirt on her porch. This
started what was later to be known as his “calling card”.
In the world have we made health an end in itself? We have
forgotten that health is really a means to enable a person to do Grandmother would come home occasionally and instantly know
his work and do it well. A lot of modern medicine is concerned that Dave had been by when she spotted the chunk of dirt on her
with promotion of good health. Many patients as well as many porch. Although Grandmother had a poor upbringing in Italy,
physicians pay very little attention to health; but very much she managed to do well in the United States. She was always
attention to health makes some people imagine that they are ill. healthy and independent and enjoyed a fulfilling life. Recently
Our great concern with health is shown by the medical columns she had a stroke and died. Everyone was saddened by her death.
in newspaper, the health articles in popular magazines and the David was disconsolate. His life-long friend was now gone.
popularity of the Television programme and all those books on 61. Which of the following is the synonym of the word
medicine we talk about health all the time. Yet for the most only ‘upbringing’?
result is more people with imaginary illnesses. The healthy man (a) Devastated
should not be wasting any time talking about health, he should
(b) Hilarious
be using health for work, the work he does and the work that
(c) raising
good health makes possible.
(d) surrounding
Reading Comprehension A-31

62. The adjective form of the word ‘saddened’ is— 64. Grandmother enjoyed a _____ life.
(a) sadly (b) sad (a) healthy but sickly (b) good and healthy
(c) suddenly (d) sudden (c) rich but sickly (d) poor and healthy
63. What was condition of grandmother earlier? 65. Grandmother’s death made everyone
(a) rich in Italy but poor in the United States (a) sad including David
(b) in the United States but is now in Italy (b) disconsolate excluding David
(c) poor earlier but became rich later on (c) happy and disconsolate
(d) rich earlier but now poor (d) sad excluding David

ANSWERS & EXPLANATIONS


1. (b) The writer appears to be a humanist with a clear 19. (c) The speaker wants a change, for which he is expecting
foresight according to the passage. his people (countrymen) and people from other quarters
2. (d) The best option to escape complete annihilation in an to come forth and fight for it.
atomic war is to ban nuclear weapons. 20. (a) The phrase "Go on to the end, shall never surrender"
3. (b) The phrase 'razor's edge of destiny' implies here the means turning all odds to save his motherland from France.
critical situation that foreordains the future.
21. (d) Repetition of something means putting pressure and
4. (d) 'Wisdom' on the razor's edge of destiny in the chapter
signifies awareness of the basic values of life. highlighting it. In this paragraph, speaker is
5. (a) The author is concerned about the threat of nuclear highlighting his intentions of fighting for the country
weapons because he feels that a nuclear war will till the very end.
destroy human civilization. 22. (b) Speaker is encouraging his countrymen to fight for
6. (c) The author highlights the importance and need of their island and his words are inspiring.
lifelong learning for everyone from different spheres 23. (a) The author states how Gandhi showed the mirror to
of life and age groups. the masses on how the cruel system is deteriorating
7. (d) The major characteristic of the future educational the country and urged them to stand up for themselves.
system includes modules with different kind of 24. (b) The rise of Gandhi empowered the people which thus
functions serving a diversity of constituents. made the country strong.
8. (a) Developing different programmes for adult learners in 25. (c) Gandhi fought the unjust system that was exploiting
conjunction with cooperation from various community the peasants and workers.
organisation like libraries, museums etc. 26. (d) As seen in the paragraph, Gandhi is seen as a liberator.
9. (b) the meaning of the idiom "The writing is already on the
A liberator is a person who liberates a person or place
wall" is to know that something is about to happen.
from imprisonment or oppression.
10. (b) As a boy Pablo Picasso was handsome and talented.
27. (c) Rhetoric is the power of persuasion.
11. (c) Picasso was spoilt mostly because he was the only
28. (d) The art of persuasion requires the use of reasoning
son in the family.
and figure of speech.
12. (d) Picasso went to school only when he was allowed to
29. (d) 30. (a) 31. (d)
carry a pet with him.
32. (b) The sentence means, Mother Bear never interfers in
13. (a) When his father painted in the college, Pablo occa-
the food sharing of her cubs.
sionally helped him.
33. (d) 34. (d) 35. (c)
14. (c) Pablo's father gave up painting because he was im-
36. (b) Urbanisation refers to construction of concrete
pressed by his son's talent.
structures on a large scale.
15. (b) The author's attitude the advent of computer technology
37. (c) Absentee house sparrow here refers to the sparrow
can be best summarised as the appreciative but reserved.
that has gone away from the urban areas.
16. (c) The sentence '….but these benefits which have been
38. (b) According to the passage, the birds have made a
ushered in by the technology revolution have had an
comeback to the suburb because they love to be amidst
adverse effect on the core of our interpersonal-
human beings.
relationships' affirms that the author would agree with
39. (b) The source of energy is the stimulation obtained from
the widespread use of computer technology is not
a set aim.
without its setbacks.
40. (c) The author is concerned about using energy in right
17. (b) The author's primary purpose in writing this passage
direction, bringing positive result.
is to explain how robotics and the internet have had
41. (d)
both a positive and negative influence on how we live.
42. (d) Mentality of a nation is reflected by the historical
18. (d) The speaker is a patriot who is urging to fight against
experience of that nation. For better relation, countries
France for saving his motherland at any cost.
EBD_8177
A-32 Reading Comprehension

should understand one another's historical experience 51. (a) Though world famous, he never felt proud of his
and resulting mentality. discoveries.
43. (c) According to the author, his countrymen should have 52. (c) Dr. Carver was an American Negro slave, who by dint
a better understanding of other nations. Better of his ability became a scientist and educator of world-
understanding averts political tension between nations wide fame.
and promote peaceful atmosphere. 53. (b) 54. (c) 55. (b)
44. (d) There will always be need for understanding between 56. (a) Refer to, “A lot of modern medicine is concerned with
nations for better relations. promotion of good health.”
45. (b) India appreciated the achievement of Russia in the field 57. (a) Refer to, “The healthy man should not be wasting
of literacy and rapid industrialization. any time talking about health, he should be using
46. (d) health for work, the work he does and the work that
47. (c) India was the only country in the world to truly good health makes possible.
recognise the achievements of the Soviet Union-rather 58. (d) Refer to, “Our great concern with health is shown by
than merely focus on the debilitating faults that the medical columns in newspaper, the health articles
Communism brought to its people. in popular magazines and the popularity of the
48. (c) The passage given above is analytical as analytical Television programme and all those books on medicine
thinking is required to answer the questions from the we talk about health all the time”.
passage. 59. (b) 60. (b)
49. (c) His guardians wanted to educate him, but were too 61. (c) raising
poor to do so. So he left home. He was hardly ten,
when he began to work at small jobs to earn a little 62. (b) sad
money for his school expenses. He continued to do so 63. (c) 64. (b) 65. (a)
even when he was at college.
50. (b) His chief desire was to do the greatest good to the
greatest number of people.
Section-B : Numerical Ability

Number System/
1 Simplification
1. In a 225 meter long yard 26 trees are planted at equal distance, (a) 22 (b) 23
one tree being at each end of the yard. What is the distance (c) 24 (d) 26
between two consecutive trees ? [2011-I] 10. A student was asked to divide a number by 3. But, instead
(a) 10 meters (b) 8 meters of dividing it he multiplied it by 3 and got 29.7 as the answer.
(c) 12 meters (d) 9 meters What was the correct answer had he not made the mistake ?
2. A bonus of ` 1000 is divided among three employees. Rohit gets [2011-II]
twice the amount Sachin gets. Sachin gets one fifth of what (a) 3.3 (b) 9.3
Gagan gets. How much amount does Gagan get ? [2011-I] (c) 9.8 (d) 9.9
(a) ` 500 (b) ` 625 11. Which of the following fraction is the smallest ? [2011-II]
(c) ` 750 (d) ` 120 (a) 9/13 (b) 17/26
3. A boy was asked to multiply a number by 25. Instead, he (c) 28/39 (d) 33/52
multiplied the number by 52 and got the answer 324 more 12. Which of the following fractions are in ascending order ?
than the correct answer. The number to be multiplied was [2012-I]
[2011-I] (a) 2/3, 3/5,7/9,9/11,8/9 (b) 3/5, 2/3, 9/11, 7/9, 8/9
(a) 12 (b) 15 (c) 25 (d) 32 (c) 3/5, 2/3, 7/9, 9/11, 8/9 (d) 8/9, 9/11, 7/9, 2/3, 3/5
4. The value of (?) in the equation 365.089 – ? + 89.72 = 302.35 is 13. 337.62 + 8.591 + 34.4 = ? [2012-I]
[2011-I] (a) 370.611 (b) 380.511
(a) 152.456 (b) 152.459 (c) 380.611 (d) 426.97
(c) 153.456 (d) 153.459
14. Find the sum of :- 1/9+1/6+1/12+1/72 [2012-I]
5. A sum of ` 312 is divided among 60 boys and some girls in
(a) 3/5 (b) 3/2
such a way that each boy gets ` 3.60 and each girl gets
` 2.40. The number of girls are- [2011-I] (c) 3/8 (d) 4/7
(a) 35 (b) 60 15. ? % of 932 + 30 = 309.6 [2012-I]
(c) 40 (d) 65 (a) 25 (b) 30
6. The number of girls in a class in five times the number of (c) 35 (d) 40
boys. Which of the following cannot be the total number of 16. The difference between a number and its two-fifth is 510.
children in the class ? [2011-II] What is 10% of that number ? [2012-I]
(a) 24 (b) 30 (a) 12.75 (b) 85
(c) 35 (d) 54 (c) 204 (d) None
7. Ram went to a shop to buy 50 kg of rice. He bought two 17. If 4/5th of an estate is worth ` 16,800, then the value of 3/7th
varieties of rice which cost him ` 4.50 per kg and ` 5 per kg. of the estate is [2012-I]
He spent a total of ` 240. What was the quantity of the (a) ` 9000 (b) ` 21000
cheaper rice purchased by him ? [2011-II] (c) ` 72000 (d) ` 90000
(a) 20 Kg (b) 25 Kg 18. If a/b = 3/4 and 8a + 5b=22, then the value of ‘a’ is [2012-I]
(c) 30 Kg (d) None of these (a) 1 (b) 1/2 (c) 3/2 (d) 3/4
8. A man has ` 640 in the denominations of one rupee, five 19. If (a – b) is 6 more than (c + d) and (a + b) is 3 less than
rupee and ten rupee notes. The number of each type of (c – d), then the value of (a – c) is [2012-I]
notes are equal. What is the total number of notes he has ? (a) 0.5 (b) 1.0
[2011-II] (c) 1.5 (d) 2.0
(a) 60 (b) 150 (c) 90 (d) 120 20. The number whose square is equal to the difference of the
9. A man has few hens and cows. If the total number of heads squares of 40 and 32 is [2012-II]
are 48 and the total number of feet are 140, then the number (a) 45.09 (b) 24
of hens are [2011-II]
(c) 25 (d) 28
EBD_8177
B-2 Number System/Simplification

21. 15 buckets of water fill a tank when the capacity of each 33. Find two natural numbers whose sum is 85 and the least
bucket is 7 litres How many buckets will be needed to fill the common multiple is 102. [2014-I]
same tank, if the capacity of the bucket is 5 litres ?[2012-II] (a) 30 and 55 (b) 17 and 68
(a) 12 (b) 24 (c) 35 and 55 (d) 51 and 34
(c) 21 (d) 30 34. In a fort there was sufficient food for 200 soldiers for 31
22. A vessel, full of water, weighs 27.5 kg. when the vessel days. After 27 days, 120 soldiers left the fort. For how many
is 1/4 full, it weighs 12.26 kg. Find the weight of empty vessel extra days will the rest of the food last for the remaining
? [2012-II] soldiers ? [2014-I]
(a) 7.18 kg (b) 6.54 kg (a) 12 days (b) 10 days
(c) 2.75 kg (d) 2 5 kg (c) 8 days (d) 6 days
23. If a+b = 10 and ab = 21, find the value of a 3+b3 [2012-II] 35. 10 is added to a certain number, the sum is multiplied
(a) 370 (b) 210 by 7, the product is divided by 5 and 5 is subtracted from
(c) 730 (d) 598 the quotient. The remainder left is half of 88. What is the
24. If a/(a+b)=17/23, what is (a+b)/(a – b) equal to ? [2012-II] number ? [2014-I]
(a) 13/7 (b) 23/11 (a) 21 (b) 20 (c) 25 (d) 30
(c) 14/5 (d) 25/9 36. A bag contains 25 paise, 50 paise and 1 ` coins. There are
2 2 220 coins in all and the total amount in the bag is ` 160. If
(598 + 479) - (598 - 479) there are thrice as many 1 ` coins as there are 25 paise coins,
25. [2012-II]
(598 ´ 479) then what is the number of 50 paise coins ? [2014-I]
(a) 2 (b) 6 (a) 60 (b) 40 (c) 120 (d) 80
(c) 4 (d) 8 37. The sum of two numbers is equal to thrice their difference. If
26. A jar contains black and white marbles. If there are ten marbles the smaller of the numbers is 10 find the other number.
in the jar, then which of the following could not be the ratio [2014-I]
of black to white marbles ? [2012-II] (a) 15 (b) 30
(a) 9:1 (b) 7:3 (c) 40 (d) None of these.
(c) 1:10 (d) 6:4
27. The number whose square is equal to the difference of the 69 ´ 69 ´ 69 - 65 ´ 65 ´ 65
38. Simplify: [2014-I]
squares of 37 and 23 is [2013-I] 69 ´ 69 + 69 ´ 65 + 65 ´ 65
(a) 45.09 (b) 28.98 (a) 1 (b) 4
(c) 47.09 (d) 28 (c) 0.216 (d) 0.164
28. A vessel, full of water, weights 24 kg. When the vessel is 39. 7 is added to a certain number, the sum is multiplied by 5;
1/4 full, it weighs 9 kg. Find the weight of empty vessel. the product is divided by 9 and 3 is subtracted from the
[2013-I] quotient. The remainder left is 12. What is the number ?
(a) 4 kg (b) 5 kg [2014-II]
(c) 8 kg (d) 3 kg (a) 20 (b) 30
29. If a – b = 4 and ab = 45 find the value of a 3 – b3. [2013-I] (c) 40 (d) 5
(a) 604 (b) 370 40. How many digits will be there to the right of the decimal
(c) 253 (d) 199 point in the product of 95.75 and 0.02554 ? [2014-II]
30. If a/(a + b) = 15/21, what is (a + b)/(a – b) equal to ? [2013-I] (a) 5 (b) 6
(a) 13/9 (b) 23/11 (c) 7 (d) Insufficient data
(c) 14/5 (d) 21/9 41. The average weight of 50 boys in a class is 45 kg. When one
boy leaves the class, the average reduces by 100g. Find the

31.
( 798 + 579 )2 - ( 798 - 579 )2 =? [2013-I]
weight of the boy who left the class. [2015-II]
(a) 50 kg. (b) 50.8 kg
( 798 ´ 579 ) (c) 49 kg (d) 49.9 kg
(a) 2 (b) 6 42. In Arun's opinion, his weight is greater than 65 kg but less
(c) 4 (d) 8 than 72 kg. His brother doest not agree with Arun and he
32. 18 buckets of water fill a tank when the capacity of each thinks that Arun's weight is greater than 60 kg but less than
bucket is 8 litres. How many buckets will be needed to fill the 70 kg. His mother's view is that his weight cannot be greater
same tank, if the capacity of the bucket is 12 litres ?[2013-I] than 68 kg. If all are them are correct in their estimation, what
(a) 12 is the average of different probable weights of Arun?
(b) 13.5 [2016-I]
(c) 24 (a) 67 kg (b) 68 kg
(d) can not be determined due to insufficient data (c) 69 kg (d) data inadequate
Number System/Simplification B-3

43. Present age of X and Y are in the ratio 7:5 respectively. Four (a) 178 (b) 174
years hence, the ratio of their ages will become 11:9 (c) 158 (d) 168
respectively. What is the present age of Y? [2016-I] 53. If 381A is divisible by 9, what is the least natural number A?
(a) 1 year (b) 7 year [2017 - I]
(c) 5 year (d) None of these (a) 6 (b) 1
3 (c) 9 (d) 4
44. In an examination, a student was asked to find of a 54. If a number exceeds 40% of itself by 56, then what is the
14
number? [2017 - II]
3 (a) 150 (b) 130
certain number. By mistake, he found of it. His answer
4 (c) 160 (d) 140
was 150 more than the correct answer. Find the number. 55. Average weight of 15 students increased by 1.5 kg when
[2016 - II] one person of 40 kg is replaced by a new man. Then weight
(a) 180 (b) 280 of the new man is [2017 - II]
(c) 380 (d) 480 (a) 62.5 kg (b) 58.5 kg
(c) 72.5 kg (d) 54.5 kg
45. A crate of mangoes contains one bruised mango for every
56. Average temperature of Tuesday to Thursday was 38 degree
30 mangoes in the crate. If 3 out of every 4 bruised mangoes
celsius and that Wednesday to Friday was 37 degree celsius.
are considered unsalable, and there are 12 unsalable
If the temperature on Friday was 39 degree celsius. The
mangoes in the crate, how many mangoes are there in the
temperature on Tuesday was? [2017 - II]
crate? [2016 - II]
(a) 38 degree celsius (b) 36 degree celsius
(a) 480 (b) 500
(c) 42 degree celsius (d) 39 degree celsius
(c) 420 (d) 520 57. A men spend 1/3 of his income on food,1/4 of the rest on
46. The mean of 50 observations was 36. It was found later that rent and 1/5 of the rest on clothes. He still has Rs.1760 left
an observation 48 was wrongly taken as 23. The corrected with him. Find his income? [2017 - II]
new mean is [2016 - II] (a) 4400 (b) 4200
(a) 35.2 (b) 34.1 (c) 4800 (d) 5200
(c) 36.5 (d) 39.1 58. Two numbers are in the ratio 12 : 13. If 20 is subtracted from
47. The average score of a cricketer for ten matches is 38.9 runs. each, the new numbers are in the ratio 2 : 3. The smaller
If the average for the first six matches is 42, the average for number is: [2017 - II]
the last four matches is [2016 - II] (a) 26 (b) 24
(a) 33.25 (b) 33.5 (c) 32 (d) 28
59. The sum of two numbers is 36 and their H.C.F and L.C.M. are
(c) 34.25 (d) 35
3 and 105 respectively. The sum of the reciprocals of two
48. If A/B = 4/3 then what is the value of (9A–3B)/(9A+3B) is numbers is [2018 - I]
[2017 - I]
2 3 4 2
(a) 3/5 (b) 5/3 (a) (b) (c) (d)
(c) 2/5 (d) 3/7 35 25 35 25
49. If one student got 77/100, 96/150 and 103/350 in various 60. A teacher wants to arrange his students in an equal number
of rows and columns. If there are 1369 students, the number
subjects, what is the percentage average? [2017 - I]
of students in the last row are [2018 - I]
(a) 15.33% (b) 12.25%
(a) 37 (b) 33 (c) 63 (d) 47
(c) 16.33% (d) 18.15% 61. A farmer divides his herd of n cows among his four sons, so
50. In an election between two candidates, Biku got 55% votes that the first son gets one–half the herd, the second one–
of the total valid votes, 25% of the votes were declared
1
invalid. If total populations of the village were 6000. Find fourth, the third son
and the fourth son 7 cows. Then the
5
the valid votes that Mahtre got, was [2017 - I]
value of n is [2018 - I]
(a) 1575 (b) 2025
(a) 240 (b) 100 (c) 180 (d) 140
(c) 2075 (d) 2175
62. The average of 5/16 and 3/8 is__? [2018 - II]
51. If 30% of a number is subtracted from 91 gives the same
(a) 0.5425 (b) 0.2585
number, find the number. [2017 - I] (c) 0.3475 (d) 0.4385
(a) 80 (b) 90
63. Find the value of 0.0081 + 0.0064 ? [2018 - II]
(c) 70 (d) 60
52. If 50% of a number is added to 84 gives the same number, (a) 0.27 (b) 0.7
(c) 0.17 (d) 0.4
what is the number? [2017 - I]
EBD_8177
B-4 Number System/Simplification

64. Find the value of 2160.16 × 160.18 ? [2018 - II] 72. If A = x% of y and B = y% of x, then which of the following
(a) 4 (b) 6 is true? [2020 - I]
(a) A is smaller than B.
(c) 8 (d) 2
(b) A is greater than B
65. If 3(a +8) = 27(2a +1) , then find 'a'? [2018 - II] (c) None of these
(d) If x is smaller than y, then A is greater than B.
(a) 1 (b) 0
73. If 20% of A = B, then B% of 20 is the same as: [2020 - I]
(c) –1 (d) 0.5 (a) 4% of A (b) 5% of A
66. If 2 = 1.4142, find the value of (c) 20% of A (d) None of these
74. The sum of ages of 5 children born at the intervals of 3 years
1 1 each is 50 years. What is the age of the youngest child?
2 2+ 2+ + [2019-I]
2+ 2 2-2 [2020 - I]
(a) 4 years (b) 5 years
(a) 1.4144 (b) 2.8284
(c) 8 years (d) 10 years
(c) 28.284 (d) 2.4142 75. The average weight of 8 person’s increases by 2.5 kg when
3 a new person comes in place of one of them weighing 65 kg.
67. In an examination, a student was asked to find of a certain
14 What might be the weight of the new person? [2020 - I]
3 (a) 76kg (b) 76.5kg
number. By mistake, he found of it. His answer was 150
4 (c) 85kg (d) Data inadequate
more than the correct answer. Find the number. [2019 - II] 76. Present ages of Sameer and Anand are in the ratio of 5 : 4
(a) 180 (b) 280 respectively. Three years hence, the ratio of their ages will
(c) 380 (d) 480
become 11 : 9 respectively. What is Anand’s present age in
5 6 8 3 3 1 7 years? [2020 - I]
68. ¸ ´ ? - ¸1 + ´3 = 2 [2019 - II]
6 7 9 5 4 3 9 (a) 24 years (b) 27 years
7 6 (c) 40 years (d) Cannot be determined
(a) (b) 77. What should come in place of both x in the equation
6 7
(c) 1 (d) None of these
x 162
69. The whole number which is a multiple of every number is = [2020 - I]
[2019 - II] 128 x
(a) 0 (a) 12 (b) 14
(b) 1 (c) 144 (d) 196
(c) There is no such number
(d) None of these 0.1 ´ 0.1 ´ 0.1 + 0.02 ´ 0.02 ´ 0.02
70. Highest common factor of 42 and 162 is ___. [2019 - II] 78. The value of is?
0.2 ´ 0.2 ´ 0.2 + 0.04 ´ 0.04 ´ 0.04
(a) 6 (b) 4
(c) 12 (d) 2 [2020 - I]
71. The ratio between two numbers is 3 : 4. If each number is increased (a) 0.0125 (b) 25
by 6 the ratio becomes 4 : 5. The difference between the numbers is
(c) 0.25 (d) 0.125
[2019 - II]
(a) 1 (b) 3
(c) 6 (d) 8
Number System/Simplification B-5

ANSWERS & EXPLANATIONS


222 33
1. (d) Distance between two consecutive trees = = meters. Hence, is the smallest fraction.
25 52
2. (b) According to question 2
R = 2S 12. (c) = 0.67
3
1 3
S = G Þ G = 5S = 0.6
5 5
R + S + G = 1000
7
2S + S + 5S = ` 1000 = 0.7
8S = 1000 9
S = 125 9
Hence, Gagan’s get 5 × 125 = ` 625. = 0.81
11
3. (a) Let the number be x.
25x + 324 = 52x 8
= 0.88
52x – 25 x = 324 9
27x = 324 Correct ascending order = 3 < 2 < 7 < 9 < 8
x = 12 5 3 9 11 9
4. (b) 365.089 –? + 89.72 = 302.35 13. (c) 337.62 + 8.591 + 34.4 = 380.611
? = 365.089 + 89.72 – 302.35 1 1 1 1 8 + 12 + 6 + 1 27 3
? = 152.459 14. (c) + + + Þ Þ =
9 6 12 72 72 72 8
5. (c) Let the number of girls be x. 15. (b) ? % of 932 + 30 = 309.6
60 × 3.60 + x × 2.40 = 312 ? % of 932 = 309.6 – 30
x = 40
?
6. (c) Let the number of boys in class be x. ´ 932 = 279.6
Therefore the number of girls in class be 5x. 100
Total number of children in class = x + 5x = 6x. ? = 30.
Hence, 35 cannot be the total number of children as it’s 16. (b) Let the number = x
not the multiple of 6. According to question
7. (a) Let one variety of rice be x kg. 2
x - x = 510
Another quantity = (50 – x) kg 5
According to question x × 4.50 + (50 – x) 5 = 240 3x
4.5x + 250 – 5x = 240 = 510
5
0.5x = 10 ; x = 20 510 ´ 5
Hence, the quantity of cheaper rice was 20 kg. x=
8. (d) Let the number of each type of notes be x. According 3
to question 1 × x + 5 × x + 10 × x = 640 x = 850
16x = 640 ; x = 40 10
Total number of notes = 40 + 40 + 40 = 120 10% of x = ´ 850 = 85
100
9. (d) Let hens and cows are x and y respectively 17. (a) Let estate cost is ` x
x + y = 48 ....(1) 4
2x + 4y = 140 ´ x = 16800 ; x = 21000
x + 2y = 70 ....(2) 5
After solving eq. (1) & (2) 3
Value of ´ 21000 == ` 9000
9000
y = 22 7
\ x = 26 a 3 4
10. (a) Let the number = x 18. (c) = Þb= a
According to question b 4 3
3x = 29.7 x = 9.9 8a + 5b = 22
9.9 4
Correct answer = = 3.3 8a + 5 ´ a = 22
3 3
9 17
11. (d) = 0.692; = 0.654 24a + 20a
13 26 = 22
28 33 3
= 0.717; = 0.634 44a = 66
39 52
EBD_8177
B-6 Number System/Simplification

66 3 x2 = (37)2 – (23)2
a= = x2 = 1369 – 529
44 2
19. (c) a – b = (c + d) + 6 ...(1) x = 28.98
a+b=c–d–3 ...(2) 28. (a) Let ‘x’ be the weight of empty vessel and ‘y’ be the
Adding eq. (1) and (2) weight of full vessel.
2a = 2c + 3 x + y = 24 ....(1)
2a – 2c = 3 y
x+ = 9 ....(2)
3 4
a–c= = 1.5 Solving eq. (1) and (2)
2
20. (b) (40) – (32)2 = 1600 – 1024 = 576
2 x = 4 kg.
29. (a) a – b =4
Hence, 24 is the required number. ab = 45
21. (c) Less capacity, more buckets. (Indirect proportion) (a–b)2 = a2 + b2 – 2ab
\ 5 : 15 : : 7 : x (4)2 = a2 + b2 – 2 × 45
5 7 16 + 90 = a2 + b2
= a2 + b2 = 106
15 x
a3 – b3 = (a – b) (a2 + b2 + ab)
x = 21
Hence, 21 buckets will be needed to fill the same tank. = (4) (106 + 45) = 604
22. (a) Let ‘x’ be the weight of empty vessel and ‘y’ be the a 15
30. (d) =
weight of full vessel. a + b 21
x + y = 27.5 ...(1) 21a = 15a + 15b
6a = 15b
y
x+ = 12.26 2a = 5b
4 5
4x + y = 4 × 12.26 ...(2) a= b
Subtracting equation (1) from (2) 2
5
4x – x = 49.04 – 27.5 b+b
3x = 21.54 a+b 2 5b + 2b 7 3 21
= =
x = 7.18 kg a-b 5 5b - 2b = 3 ´ 3 = 9
b-b
23. (a) Given, a + b = 10 2
Squaring on both sides
a2 + b2 + 2ab = 100 ( 798 + 579)2 - ( 798 - 579)2
31. (c)
a2 + b2 = 100 – 2 × 21 ( 798 + 579)
a2 + b2 = 58
Now, a3 + b3 = (a + b) (a2 + b2 – ab)
Þ
( 798)2 + (579)2 + 2 ´ 798 ´ 579 -( 798)2 - (579)2 + 2 ´ 798 ´ 579
= (10) (58 – 21) = 10 × 37 = 370 798 ´ 579
a 17
24. (b) = 4 ´ 798 ´ 579
a + b 23 Þ =4
23a = 17a + 17b 798 ´ 579
6a = 17b 32. (a) More capacity, less buckets (Indirect proportion)
17 12 : 18 : : 8 : x
a= b
6 12 8 18 ´ 8
= Þx= = 12
17 17 18 x 12
b+b +1
a+b 6 33. (d) By using option (d) is
= = 6 a + b 23
=
a - b 17 17 Þ correct answer in which
b-b -1 a - b 11 51 + 34 = 85
6 6
and LCM of 51 & 34 is 102.
( 598 + 479 )2 - ( 598 - 479 )2 34. (b) Let rest of the food last for the x days.
25. (c)
598 ´ 479 \ 200 × 4 = (200 – 120) × x
200 × 4 = 80 × x
(598)2 + (479)2 + 2 ´ 598 ´ 479 – (598)2 – (479)2 + 2 ´ 598 ´ 479
598 ´ 479 800
x= = 10 days.
80
4 ´ 598 ´ 479
Þ Þ4 35. (c) Let the number be = x
598 ´ 479
26. (c) 1 : 10 could not be the ratio of black to white marbles. ( x + 10) ´ 7 88
\ –5=
27. (b) Let the number = x 5 2
7x + 70 – 25 = 220
Number System/Simplification B-7

7x = 220 – 45 42. (a) Let Arun's weight by X kg.


7x = 175 According to Arun, 65 < X < 72
According to Arun's brother, 60 < X < 70.
x = 25 According to Arun's mother, X < = 68
\ Number is 25. The values satisfying all the above conditions are 66,
36. (a) Let 25 paise coins = x 67 and 68.
1 ` : 50 P : 25 P æ 66 + 67 + 68 ö æ 201ö
3x : 220 – 4x : x (Ratio in number of coins) \ Required average = çè ÷ø = çè ÷ = 67 kg.
3 3 ø
220 – 4 x x 43. (c) Let present age of X and Y are 7x and 5x respectively
3x : : (Ratio in amount)
2 4 7x + 4 11
x Now, =
\ 3x + 110 – 2x + = 160 5x + 4 9
4
x 63x + 36 = 55x + 44
x + 110 + = 160 8x = 8
4 x= 1
4x + 440 + x = 640
So, present age of y = 5 × 1 = 5 year.
5x = 200
44. (b) Let the number be x
x = 40
ATQ
\ 50 paise coins = 220 – 4 × 40 = 220 – 160 = 60
37. (d) Let largest no. = x 3 3x
x– < 150
\ x + 10 = 3 (x – 10) 4 14
10 + 30 = 3x – x 3x é 1 1 ù
40 = 2x ê – ú < 150
2 ëê 2 7 ûú
x = 20
\ other number = 20 é 7 – 2ù
xê ú < 100
69 ´ 69 ´ 69 – 65 ´ 65 ´ 65 ëê 14 ûú
38. (b) é5ù
69 ´ 69 + 69 ´ 65 + 65 ´ 65 x ê ú < 100 ; x = 280
ëê14 ûú
a3 - b3 45. (a) Let the total no. of Mangoes in the crate be x. Then the
Using =a–b x
a 2 + ab + b 2 no. of bruised Mango =
30
\ 69 – 65 = 4 é 1 ù 3
39. (a) Let the number be x ê as th Mango is bruisedú and every th is unsalable
êë 30 úû 4
5(7 + x) of brused.
– 3 =12
9 3æ x ö x
5(7 + x) Then the unsalable Mangoes = ççç ÷÷÷ <
=15 4 è 30 ø 40
9
15 ´ 9 x
7 + x= = 27 < 12 \ x = 480
5 40
x = 27 – 7 = 20 46. (c) Correct sum = (36 × 50 + 48 – 23) = 1825.
40. (b) 95.75 × 0.02554 = 2.445455 1825
There are 6 digits to the right of the decimal point in the Correct mean = < 36.5
50
product of 95.75 and 0.02554.
41. (d) Let the weight of the boy who left the class is x. 47. (c) Average score of 10 matches = 38.9
Total weight = 50 × 45 = 2250 kg Total score = 38.9 × 10 = 389
when one boy left the class total weight is (2250 – x) Average score of 6 matches = 42
and no. of student left is 49. Total score = 42 × 6 = 252
According to question – Total score for remaining 4 matches = 389 – 252 = 137
137
2250 - x æ 100 ö Average for 4 matches = < 34.25
= ç 45 - ÷ kg 4
49 è 1000 ø
A 4
2250 - x 48. (a) Here, =
= 44.9 B 3
49 (9A - 3B) (9 ´ 4 - 3 ´ 3)
2250 – x = 2200.1 \ (9A + 3B) = (9 ´ 4 + 3 ´ 3)
x = 2250 – 2200.1
x = 49.9 kg 36 - 9 27 3
= = =
36 + 9 45 5
EBD_8177
B-8 Number System/Simplification

49. (a) Total marks in all subjects = (100 + 150 + 350) = 600 b + c + friday = 111
Total marks obtained by student = 77 + 96 + 103 = 276 \ Temperature of friday = 39
\ b + c + 39 = 111
276 ´100 \ b + c = (111– 39) = 72
\ Obtain marks in percentage = = 46%
600 \ a + b + c = 114
a + 72 = 114
46 \ a = (114 – 72) = 42
\ Percentage average = = 15.33%
3 \ Temperature on tuesday = 42°C
57. (a) Let total amount be x
25
50. (b) Total valid votes = 6000 – 6000 × = 4500 x
100 Amount spent on food =
3
55 2x
Votes obtained by Biku = 4500 ´ = 2475 Amount left =
100 3
Votes obtained by Mahtre = (4500 – 2475) = 2025. 2x 1 2x
51. (c) Let the number be x Amount spent on rent = ´ =
3 4 12
According to question,
6x
x ´ 30 Amount left =
91 - =x 12
100
9100 – 30x = 100x 6x 1 6x x
Amount spent on cloth = ´ = =
130x = 9100 12 5 60 10
9100 Therefore,
\x= = 70
130 x 2x x
x= + + + 1760
52. (d) Let the number be x 3 12 10
According to question,
x x 2x x
x ´ 50 - - - = 1760
84 + =x 1 3 12 10
100
8400 + 50x = 100x 24x
= 1760
100x – 50x = 8400 60
50x = 8400 \ x = 4400
8400 58. (b) Let the numbers be 12x and 13x
\ x= = 168
50 12x - 20 2
53. (a) A number is divisible by 9 when the sum of its digit is Then, =
13x - 20 3
divisible by 9. Þ 3(12x – 20) = 2(13x – 20)
So, (3 + 8 + 1 + A) = must be divisible by 9. Þ 36x – 60 = 26x – 40
Thus smallest natural number be 6. Þ 10x = 20
or, (3 + 8 + 1 + 6) = 18, this is divisible by 9.
\ x= 2
54. (d) Let number be x, then
\ Smallest number = 12x
æ x ´ 40 ö = 12 × 2 = 24
x +ç ÷ - x = 56
è 100 ø 59. (c) Let the numbers be 3x and 3y.
\ 3x + 3y = 36
2x Þ x + y = 12 ...(i)
x+ - x = 56
5 and 3xy = 105 ...(ii)
2x Dividing equation (i) by (ii), we have
= 56 x y 12 1 1 4
5 + = + =
Þ
56 ´ 5 3xy 3xy 105 3y 3x 35
\ x= = 140
2
55. (a) Total weight increased by = 15 × 1.5 = 22.5 kg Shortcut Method:
\ Weight of new man = 40 + 22.5 = 62.5 kg 1 1 x+y
56. (c) Let temperature of tuesday = a, temperature of + =
x y xy
wednesday = b and temperature of thursday = c
Sum of temperatures of tuesday to thursday = 38 × 3 60. (a) If they are equal number of rows and columns then,
=114 1369 = 37
\ a + b + c = 114
61. (d) According to the question,
Sum of temperatures of wednesday to friday = 37 × 3
= 111 n n n
+ + +7 = n
2 4 5
Number System/Simplification B-9

10n + 5n + 4n 35 25 5 5 10 5
Þ +7=n Û x= + - = -
20 36 9 9 2 3 2
19n 19n 35 5 æ 5 36 ö 6
Þ +7 =n Þ n- =7 Û x= Û x=ç ´ ÷ =
20 20 36 6 è 6 35 ø 7
69. (b)
n 70. (a) 42 = 2 × 3 × 7
Þ = 7 Þ n = 20 ´ 7 = 140
20 162 = 2 × 3 × 3 × 3 × 3
Highest common factor of 42 and 162 is 6
5 3 5+ 6
+ 71. (c) Let the number are 3x and 4x.
62. (c) Average = 16 8 = 16 = 11 = 0.3475.
2 2 32 3x + 6 4
=
4x + 6 5
0.0081 + 0.0064 = ( 0.09 ) + ( 0.08)
2 2
63. (c)
15x + 30 = 16 x + 24
= 0.09 + 0.08 = 0.17 x= 6
64. (a) 2560.16 × 160.18 Number are 18 and 24.
(28)0.16 × (24)0.18 Hence, required difference is 6.
(2)1.28 × (2)0.72
æ x ö æ y ö
= 21.28+0.72 72. (c) x % of y = ç ´ y÷ =ç ´ x ÷ = y% of x
= 22 = 4 è 100 ø è 100 ø
65. (a) According to question, \ A = B.
(a+8) (2 a+1) ( a+8) 3(2 a+1) . 20
3 = 27 Þ3 =3 73. (a) 20% of A = B Þ A = B.
Equating the powers, we get (a+8) = 3(2a+1) or 'a' = 1 100
66. (b) Expression: \ B% of 20
1 1 æ B ö æ 20 1 ö 4
=2 2+ 2+ + =ç ´ 20 ÷ = ç A´ ´ 20 ÷ = A
2+ 2 2-2 è 100 ø è 100 100 ø 100
æ 1 1 ö 74. (a) Let the ages of children be x, (x + 3), (x + 6), (x + 9) and
=2 2+ 2+ç + ÷ (x + 12) years.
è2+ 2 2 - 2ø Then, x + (x + 3) + (x + 6) + (x + 9) + (x + 12) = 50
æ ö Þ 5x = 20
-2 + 2 + 2 + 2 ÷
=2 2 + 2 +ç Þ x = 4.
è (
ç 2+ 2 )(
2 -2 ÷
ø ) \ Age of the youngest child = x = 4 years.
75. (c) Total weight increased = (8 × 2.5) kg = 20 kg.
2 2 Weight of new person = (65 + 20) kg = 85 kg.
=2 2+ 2+ =2 2+ 2- 2 =2 2
2-4 76. (a) Let the present ages of Sameer and Anand be 5x years
= 2 × 1.4142 = 2.8284 and 4x years respectively.
67. (b) Let the number be x
ATQ, 5 x + 3 11
Then, =
3 3x 4x + 3 9
x– < 150 Þ 9(5x + 3) = 11(4x + 3)
4 14
Þ 45x + 27 = 44x + 33
3x é 1 1 ù
– = 150 Þ 45x – 44x = 33 – 27
2 êë 2 7 úû Þ x = 6.
é7 – 2ù \ Anand’s present age = 4x = 24 years.
xê ú = 100
ë 14 û
x 162
é5ù 77. (a) Let =
x ê ú = 100 128 x
ë14 û
x = 280 Then x2 = 128 ´162
5 6 8 8 3 10 25 = 64 ´ 2 ´18 ´ 9 = 82 ´ 62 ´ 32
68. (b) Let ¸ ´ x - ¸ + ´ = .
6 7 9 5 4 3 9
Then = 8 × 6 × 3 = 144
5 7 8 5 3 10 25 \ x = 144 = 12.
´ ´x- ´ + ´ =
6 6 9 8 4 3 9 (0.1)3 + (0.02)3 1
35 5 5 25 78. (d) Given expression = = = 0. 125
Û x- + = 3 3
2 [(0.1) + (0.02) ] 3 8
36 9 2 9
EBD_8177
B-10 Arithmetic

2 Arithmetic

1. The average age of 35 students in a class is 16 years. Out of 9. A cricketer has an average of 30 runs in 14 innings. How
these students the average age of 21 students is 14 years. many runs should he score in his next innings to achieve an
The average age of remaining students is [2011-I] average of 32 runs ? [2011-II]
(a) 65 (b) 60
(a) 15 years (b) 17 years
(c) 55 (d) 50
(c) 20 years (d) 19 years
10. The ratio of three numbers is 3 : 4 : 5 and the sum of their
2. After replacing an old member by a new member, it was squares is 1250. The sum of the three numbers is [2011-II]
found that the average age of five members of a club is the
(a) 30 (b) 50
same as it was 3 years ago. What is the difference between
(c) 60 (d) 90
the age of replaced member and new member ? [2011-I]
11. The average age of three boys is 25 years and their ages are in
(a) 2 years (b) 8 years the proportion 3 : 5 : 7. What is the age of the youngest boy ?
(c) 15 years (d) 25 years [2011-II]
3. The average salary of all the workers in a workshop is Rs. (a) 15 years (b) 18 years
8000. The average salary of seven technicians is `12000 (c) 21 years (d) 13 years
and average salary of others is ` 6000. The total number of 12. Find the average of all the numbers between 6 and 34 which
workers in the workshop are- [2011-I] are divisible by 5. [2012-I]
(a) 20 (b) 21 (a) 18 (b) 20
(c) 24 (d) 30
(c) 22 (d) 23
13. The average of first 80 natural numbers is [2012-I]
4. The price of a scooter and a TV are in the ratio of 7 : 5. If the
(a) 40 (b) 41
scooter costs `8000 more than a TV set, then the price of TV
(c) 40.5 (d) 142
set is- [2011-I]
14. If the sum of a few numbers is 450 and their mean is 50 and
(a) ` 20000 (b) ` 24000 if another number 100 is included, the mean would become
(c) ` 32000 (d) ` 28000 [2012-I]
5. The speed of three cars is in the ratio of 5 : 4 : 6. The ratio (a) 55 (b) 60
between the time taken by them to travel the same distance is (c) 75 (d) 150
[2011-I] 15. Two numbers are in the ratio 7 : 8. If 3 is added to each of
(a) 5 : 4 : 6 (b) 6 : 4 : 5 them their ratio becomes 8:9. The numbers are [2012-I]
(c) 10 : 12 : 15 (d) 12 : 15 : 10 (a) 14, 16 (b) 24, 27
(c) 21, 24 (d) 16, 18
6. The ratio between two numbers is 3 : 4. If each number is 16. The sum of three numbers is 98. If the ratio of the first to the
increased by 6 the ratio becomes 4 : 5. The difference second is 2 : 3 and that of the second to the third is 5 : 8, then
between the numbers is [2011-I] the second number is [2012-I]
(a) 1 (b) 3 (a) 20 (b) 30
(c) 6 (d) 8 (c) 48 (d) 58
17. A certain amount was divided between Sita and Gita in the
7. The average of five consecutive odd number is 61. What is ratio 9 : 8. If Sita’s share was ` 4500 then the amount is
the difference between the highest and lowest number ? [2012-I]
[2011-II] (a) ` 9000 (b) ` 8500
(a) 2 (b) 5 (c) ` 6750 (d) ` 9025
(c) 8 (d) 12 18. A man’s average monthly expenditure for the first four
months of the year was ` 225.25. For the next five months,
8. Jayesh is twice as old as Vijay and half as old as Suresh. If the average monthly expenditure was ` 20.75 more than
the sum of Vijay’s age and Suresh’s age is 85 years what is what it was during the first four months. If the person spent
the age of Jayesh ? [2011-II] ` 700 in all during the remaining three months of the year,
(a) 34 years (b) 36 years find what percntage of his annual income of ` 3500 he saved
(c) 68 years (d) 24 years in the year ? [2012-II]
Arithmetic B-11

(a) 10% (b) 15% students is reduced by ` 1. What was the initial expenditure
(c) 19.11% (d) 25% on food per day ? [2014-I]
19. The ratio of the present age of P and Q is 2.3. The ratio of (a) ` 432 (b) ` 442
their age after 18 years will be 4.5. What is the present age of (c) ` 420 (d) ` 400
Q? [2012-II] 29. There were 24 students in a class. One of them, who was 18
(a) 26 years (b) 25 years years old, left the class and his place was filled up by a new
(c) 24 years (d) 27 years comer. If the average of the class was thereby lowered by 1
20. The mean temperature of Monday to Wednesday was 37ºC month, the age of new comer is [2014-I]
and of Tuesday to Thursday was 34°C. If the temperature (a) 14 years (b) 15 years
on Thursday was 4/5th that of Monday, the temperature on (c) 16 years (d) 17 years
Thursday’ was [2012-II] 30. 19 persons went to a hotel for a combined dinner party. 13 of
(a) 36.5°C (b) 36ºC them spent ` 79 each on their dinner and the rest spent
(c) 35.5°C (d) 34°C ` 4 more than the average expenditure of all the 19. What
21. A sum of money is to be distributed among P, Q and R in the was the total money spent by them ? [2014-I]
ratio of 6:19:7. If R gives ` 200 from his share to Q, the ratio (a) 1628.4 (b) 1534
of P, Q and R becomes 3:10:3, what is the total sum ? (c) 1492 (d) None of these
[2012-II] 31. A factory employs skilled workers, unskilled workers and
(a) ` 3200 (b) ` 12800 clerks in the proportion 8: 5: 1 and the wages of a skilled
(c) ` 6400 (d) data inadequate worker as unskilled worker and a clerk are in the ratio 5 : 2 : 3.
When 20 unskilled workers are employed, the total daily
22. In what ratio should tea worth ` 10 per kg be mixed with tea wages of all, amount to ` 318. What is the daily wages in
worth ` 14 per kg so that the average price of the mixture ` paid to each category of employees ? [2014-II]
may be ` 11 per kg ? [2012-II] (a) 240, 57, 19 (b) 210, 70, 13
(a) 2:1 (b) 3:1 (c) 230, 65, 12 (d) 240, 60, 18
32. A cat takes 5 leaps for every 4 leaps of a dog, but 3 leaps of
(c) 3:2 (d) 4:3 the dog are equal to 4 leaps of the cat. What is the ratio of
23. The average age of 8 men is increased by 4 years when one the speed of the cat to that of the dog ? [2014-II]
of them whose age is 30 years is replaced by a new man. (a) 13 : 14 (b) 15 : 11
What is the age of new man ? [2013-I] (c) 17 : 15 (d) 15 : 16
(a) 55 years (b) 62 years
33. Ram Shiv and Ganesh assemble for a contributory party.
(c) 42 years (d) 69 years Ram brings 3 apples while Shiv brings 5. Since Ganesh did
24. A man’s average monthly expenditure for the first four not bring any, he contributed ` 8/-. How many rupees should
months of the year was ` 231.25. For the next five months, Ram and Shiv respectively get, assuming each of the three
the average monthly expenditure was ` 22.75 more than consumes an equal portion of the apples ? [2014-II]
what it was during the first four months. If the person spent (a) 1, 7 (b) 2, 5
` 605 in all during the remaining three months of the year,
find what percentage of his annual income of ` 3500 did he (c) 5, 3 (d) 2, 6
save in the year ? [2013-I] 34. The average weight of 5 men is increased by 2 Kg when one
of the men whose weight is 60 Kg is replaced by a new man.
(a) 10% (b) 15% The weight of the new man is [2015-I]
(c) 20% (d) 25% (a) 50 Kg (b) 65 Kg
25. The average age of students of a class is 15.8 years. The (c) 68 Kg (d) 70 Kg
average age of boys in the class is 16.4 years and that of the 35. Gauri went to the stationers and bought things worth ` 25,
girls is 15.4 years. The ratio of the number of boys to the out of which 30 pasie went on sales tax on taxable purchases.
number of girls in the class is [2013-I] If the tax rate was 6%, then what was the cost of the free
(a) 1 : 2 (b) 2 : 3 items? [2015-II]
(c) 3 : 4 (d) 3 : 5 (a) ` 15 (b) ` 15.70
26. Divide ` 80 in the production of 3 : 6 : 7. [2013-I] (c) ` 19.70 (d) ` 20
(a) ` 10, ` 35, ` 40 (b) ` 15, ` 30, ` 35 36. An automobile financier claims to be lending money at simple
(c) ` 15, ` 35, ` 30 (b) ` 10, ` 40, ` 35 interest, but he includes the interest every six month 5 for
calculating the principal. If he is charging an interest of
27. In a mixture of 60 litres, the ratio of milk and water is 2 : 1. 10%, the effective rate of interest becomes: [2015-II]
What amount of water must be added to make the ratio of (a) 10% (b) 10.25%
milk and water as 1 : 2 ? [2013-I] (c) 10.5% (d) None of these
(a) 42 Litres (b) 56 Litres 37. In a certain store, the profit is 320% of the cost. If the cost
(c) 60 Litres (d) 77 Litres increases by 25% but the selling price remains constant,
28. There were 35 students in a hostel. If the number of approximately what percentage of the selling price is the
students be increased by 7, the expenditure on food profit? [2015-II]
(a) 30% (b) 70%
increases by ` 42 per day while the average expenditure of
(c) 100% (d) 250%
EBD_8177
B-12 Arithmetic
38. A man lent a sum of money at the rate of simple interest of (a) ` 4000 (b) ` 4950
4%. If the interest for 8 years is ` 340 less than the principal, (c) ` 5000 (d) ` 5150
the principal is [2015-II] 48. The difference between simple and compound interests
(a) ` 500 (b) ` 520
(c) ` 540 (d) ` 580 compounded annually on a certain sum of money for 2 years
39. A alone can do a piece of work in 6 days and B alone in 8 at 4% per annum is `1. The sum (in `) is: [2016-I]
days. A and B undertook to doo it for ` 3200. With the help (a) 625 (b) 630
of C, they completed the work in 3 days. how much is to (c) 640 (d) 650
paid to C? [2015-II] 49. A man rows to a place 48 km distant and come back in 14
(a) ` 375 (b) ` 400 hours. He finds that he can row 4 km with the stream in the
(c) ` 600 (d) ` 800
same time as 3 km against the stream. The rate of the stream
40. In a mixture of 35 litres, the ratio of milk and water is 4:1. If
7 litres of water is added to the mixture, the ratio of milk and is: [2016-I]
water of the new mixture will be [2015-II] (a) 1 km/hr (b) 1.5 km/hr
(a) 2 : 1 (b) 1 : 2 (c) 2 km/hr (d) 2.5 km/hr
(c) 4 : 3 (d) 5 : 4 50. If the incomes of Ram & Shyam are in the ratio of 3:4 and
41. A shopkeeper sells one transistor for ` 840 at a gain of 20% their expenditures in the ratio of 4:5, find the ratio of their
and another for ` 960 at a loss of 4%. His total gain or loss savings, given that Shyam saves a third of his income.
percent is: [2015-II]
[2016-I]
15 15 (a) 10:15 (b) 13:20
(a) 5 % loss (b) 5 % gain
17 17 (c) 15:20 (d) 13:15
2 51. A does 1/2 as much work as B and C does 1/2 as much work
(c) 6 % gain (d) None of these
3 as A and B together. If C alone can finish the work in 40
days, then together all will finish the work in [2016-I]
1 1
42. A can do of a piece of work in 8 days while B can do the (a) 30 days (b) 13 1/3 days
2 3
(c) 15 days (d) 20 days
of the same work in 8 days. In how many days can both do
it together? [2015-II] 52. A container contains 40 litres of milk. From this container 4
(a) 9.6 days (b) 10.5 days litres of milk was taken out and replaced by water. This
(c) 1.2 days (d) 16 days process was repeated further two times. How much milk is
43. If the selling price is doubled, the profit triples. The profit now contained by the container? [2016-I]
percent is [2015-II] (a) 26.34 litres (b) 27.36 litres
(a) 40 minutes (b) 1 hour (c) 28 litres (d) 29.16 litres
(c) 100% (d) 90%
53. A boat travels upstream from B to A and downstream from A
44. Anmol sold two items for ` 1000 each. On one, he gained
to B in 3 hours. If the speed of the boat in still water is 9 km/
10% and on other, he lost 10%. How much did he gain or
h and the speed of the current is 3 km/h, the distance between
lose in the whole transaction? [2016-I]
A and B is [2016-I]
(a) profit,0.95% (b) loss,1%
(a) 9 km (b) 10 km
(c) profit,10% (d) loss,10%
(c) 11 km (d) 12 km
45. The price of an item is decreased by 25%. What percent
54. A sum of money lent out at simple interest amounts to ` 720
increase must be done in new price to get the original price?
[2016-I] after 2 years and to ` 1020 after a further period of 5 years.
The sum is [2016-I]
1 3 (a) ` 500 (b) ` 600
(a) 33 % (b) 43 %
3 4 (c) ` 700 (d) ` 750
2 1 55. An amount of ` 735 was divided between A, B and C. If each
(c) 55 % (d) 65 % of them had received ` 25 less, their shares would have
3 4
46. A's salary is 50% more than B's. How much percent is B's been in the ratio of 1 : 3 : 2. The money received by C was
salary less than A's? [2016-I] [2016 - II]
(a) ` 195 (b) ` 200
1 1
(a) 63 % (b) 43 % (c) ` 225 (d) ` 245
4 4 56. 20 litres of a mixture contains milk and water in the ratio 5 : 3.
1 1 If 4 litres of this mixture is replaced by 4 litres of milk, the
(c) 53 % (d) 33 % ratio of milk to water in the new mixture would be[2016 - II]
3 3 (a) 2:1 (b) 7:3
47. A sum was invested for 3 years at simple interest at a certain (c) 8:3 (d) 4:3
rate. Had it been invested at 4% higher rate of interest, it 57. A can do a piece of work in 10 days. He works at it for 4 days
would have fetched `600 more. The sum is [2016-I] and then B finishes it in 9 days, in how many days can A and
B together finish the work? [2016 - II]
Arithmetic B-13

(a) 6 days (b) 8 days 68. A shopkeeper sold two articles at ` 12000 each. One sold at
1 1 a loss of 25% and another one at a gain 20%. What is net
(c) 8 days (d) 7 days
2 2 total loss and gain? [2017 - I]
58. How long will it take for a sum of money invested at 5 % p.a. (a) 4% (b) 8.5 %
at simple interest to increase its value by 40 %?[2016 - II]
(c) 6.5% (d) 7.6%
(a) 5 years (b) 6 years
(c) 7 years (d) 8 years 69. A shopkeeper sell a T.V for ` 3000 and makes a profit of 15%,he
1 sells a different T.V at a loss of 10%. Over all, he neither gains
59. On a certain sum, the simple interest at the end of 6 years
4 nor loses. What is cost price of the second T.V? [2017 - I]
3
becomes th of the sum. The rate percent is [2016 - II] (a) 3913 (b) 3923
8 (c) 3833 (d) 3723
(a) 7% (b) 6%
70. The ratio of the number of boys and girls in a school is 3:2.
1 If 20% of the boys and 30% of the girls are scholarship
(c) 5% (d) 5 %
2 holders, the percentage of the students who are not
60. A horse is sold at a profit of 25 %. If both the cost price and scholarship holders is: [2017 - II]
selling price are ` 200 less, the profit will be 5 % more. The (a) 80 (b) 72
cost price is [2016 - II] (c) 58 (d) 76
(a) ` 1100 (b) ` 1200 71. A is thrice as efficient as workman as B and therefore is able
(c) ` 1000 (d) ` 900 to finish a job in 40 days less than B to complete a work.
61. By selling a chair for ` 368. a man lost 8 %. For how much
should he have sold it to gain 15 %? [2016 - II] Working together, they can do it in: [2017 - II]
(a) ` 450 (b) ` 475 (a) 15 days (b) 18 days
(c) ` 460 (d) ` 500 (c) 24 days (d) 12 days
62. If 3 men or 4 women can plough a field in 43 days, in how 72. A and B together can complete a piece of work in 12 days, B
many days 7 men and 5 women can plough the same field? and C can do it in 15 days and A and C can do it in 20 days.
[2016 - II] A, B and C together can complete it in [2017 - II]
(a) 18 days (b) 10 days (a) 12 days (b) 16 days
(c) 12 days (d) 15 days (c) 10 days (d) 8 days
63. A man sold two watches for ` 3750 each; on one he gained 73. A shopkeeper sell a watch for Rs. 575 and makes a profit of
5 % and on the other he lost 5 %. What was his total gain or 15%, he sells a different watch at a loss of 10%. Over all, he
loss as a percentage? [2016 - II] neither gains nor loses. What is cost price of the second
(a) 0.25 % loss (b) 2.5 % loss watch? [2017 - II]
(c) 25 % gain (d) 12.5 % gain
(a) 550 (b) 625
64. 10 years ago the average age of a family of 4 members was (c) 650 (d) 675
24 years. Now two more babies were born with age difference 74. If 1/5th tank is full and if 22 litres fuel poured in it, then 3/4th
of 2 years and average age of the family is still 24. What is tank is full. Find the capacity of tank? [2017 - II]
the present age of the youngest child? [2017 - I] (a) 40 litres (b) 35 litres
(a) 4 years (b) 6 years (c) 50 litres (d) 45 litres
(c) 3 years (d) 5 years 75. A person distributes his pens among four friends A, B, C
and D in the ratio 1/3 : 1 / 4 : 1/5 : 1/6. What is the minimum
65. 3 years back the average age of a family of 4 members was
number of pens that the person should have ? [2017 - II]
24 years. A baby having been born, the average age of the (a) 57 (b) 48
family is still same. What is the present age of the baby? (c) 62 (d) 38
[2017 - I] 76. A sum is lent at simple interest at a certain rate for 4 years.
(a) 14 years (b) 16 years Had it been put at 3% more rate, it would have earned ` 78
(c) 12 years (d) 10 years more. Find the sum? [2017 - II]
66. If a sum of money placed at compound interest doubles (a) 750 (b) 650
(c) 575 (d) 350
itself in 4 years, In how many years will it amount to 4 times?
77. A cloth merchant says that due to slump in the market, he
[2017 - I] sells the cloth at 10% loss but he uses a false meter scale
(a) 6 (b) 8 and actually gains 15%.Find the actual length of the scale.
(c) 10 (d) 12 [2017 - II]
67. A man bought 240 chocolates at 9 per dozen. If he sold all of (a) 78.25 cm (b) 75cm
them at ` 1 each, what was his loss or gain percent? (c) 85cm (d) 72.25cm
[2017 - I] 78. The average salary of all the workers in a workshop is
1 ` 8,000. The average salary of 7 technicians is ` 12,000 and
(a) 33 % (b) 25% the average salary of the rest is ` 6,000. The total number of
3
(c) 20% (d) 15% workers in the workshop is [2018 - I]
(a) 20 (b) 21 (c) 22 (d) 23
EBD_8177
B-14 Arithmetic
79. The monthly salaries of A and B together amount to ` 40,000. (a) 2600 (b) 2500
A spends 85% of his salary and B, 95% of his salary. If now (c) 2800 (d) 2950
their savings are the same, then the salary (in `) of A is 90. A person sells two horses for rupees 1200/ each. On the first
[2018 - I] at a profit of 20% and second at a loss of 20%. The overall
(a) 10,000 (b) 12,000 (c) 16,000 (d) 18,000 profit/loss in percentage is__ ? [2018 - II]
80. The price of table depreciates every year by 20%. If the
value of the table after 2 years will be ` 32000, then what is (a) 4% loss (b) 4% profit
the present price (in `) of the table? [2018 - I] (c) 5% loss (d) 5% profit
(a) 48000 (b) 44000 (c) 50000 (d) 51000 91. Efficiency of A, B and C is in the ratio 4:5:6. What is the ratio
81. Krishna purchased a number of articles at `10 for each and of the time in which they complete the work? [2018 - II]
the same number for ` 14 each. He mixed them together and (a) 5:4:3 (b) 15:12:10
sold them for `13 each. Then his gain or loss percent is (c) 15:10:12 (d) 10:12:15
[2018 - I] 92. Someone purchase 5 dozen of egg in ` 100. Out of which
1 2 20% eggs were found broken. At what rate he should sell
(a) Loss 8 % (b) Gain 8 %
3 3 eggs so that he gets 10% profit? [2018 - II]
2 1 (a) 2.29 (b) 3.25
(c) Loss 8 % (d) Gain 8 %
3 3 (c) 2.75 (d) 3.75
82. The price of an article is first decreased by 20% and then 93. A bank give 16% interest per annum compounded semi
increased by 30%. if the resulting price is ` 416, the original annually. What interest a man get on amount of ` 10000 in 2
price of the article is. [2018 - I]
(a) ` 350 (b) ` 405 years? [2018 - II]
(c) `400 (d) ` 450 (a) 12665 (b) 13205
83. A sum of ` 12,000, deposited at compound interest becomes (c) 14515 (d) 13605
double after 5 years. How much will it be after 20 years ? 94. If there is 25% increase in the cost of sugar by what %
[2018 - I] consumption should be decreased in order to maintain
(a) ` 1,44,000 (b) ` 1,20,000 expenditure? [2018 - II]
(c) ` 1,50,000 (d) ` 1,92,000 (a) 25% (b) 20%
84. A sum becomes ` 2,916 in 2 years at 8% per annum (c) 32.5% (d) 15%
compound interest. The simple interest at 9% per annum for
3 years on the same amount will be [2018 - I] 95. Average marks of a class are 70. If average marks of fail
(a) ` 625 (b) ` 600 students are 40 and pass students are 80 marks. Find
(c) ` 675 (d) ` 650 percentage of pass students? [2018 - II]
85. ` 700 is divided among A, B, C in such a way that the ratio (a) 25% (b) 50%
of the amount of A and B is 2 : 3 and that of B and C is 4 : 5. (c) 65% (d) 75%
Find the amounts in ` each received, in the order A, B, C. 96. The population of a village increase 5% per annum. It's
[2018 - I] population at the end of 2016 was 1852200. What was its
(a) 150, 250, 300 (b) 160, 240, 300 population in 2014? [2018 - II]
(c) 150, 250, 290 (d) 150, 240, 310
86. A and B can complete a piece of work in 8 days, B and C can (a) 1680000 (b) 1640000
do it in 12 days, C and A can do it in 8 days. A, B and C (c) 1720000 (d) 1560000
together can complete it in [2018 - I] 97. 250 ml of mixture contains milk and water in the ratio of 7:2.
(a) 4 days (b) 5 days How much more milk must be added to get a new mixture
(c) 6 days (d) 7 days containing milk and water in the ratio of 4:1? [2018 - II]
87. Pipe A alone can fill a tank in 8 hours. Pipe B alone can fill it (a) 50 ml (b) 42 ml
in 6 hours. If both the pipes are opened and after 2 hours
(c) 28 ml (d) 32 ml
pipe A is closed, then the other pipe will fill the tank in
[2018 - I] 98. The average age of 25 students is 16years. If a teacher is
added the average age becomes 18 years. What is the age
1
(a) 6 hours (b) 3 hours of teacher? [2018 - II]
2 (a) 68 years (b) 62 years
1
(c) 4 hours (d) 2 hours (c) 64 years (d) 70 years
2 99. A and B can do a piece of work in 10 days. B and C can do it
88. P is four times as efficient as Q.P can complete a work in 45
days less than Q. If both of them work together, then in in 12 days. A and C can do it in 15 days. How long will A take
how many days the work will be completed? [2018 - I] to do it alone? [2018 - II]
(a) 10 (b) 12 (a) 20 days (b) 24 days
(c) 15 (d) 30 (c) 30 days (d) 40 days
89. The difference between compound interest and simple 100. The price of onions has been increased by 50% in order to
interest accrued on an amount at the end of 3rd year at a rate keep the expenditure on onions the same, the percentage of
of 10% is 77.5 rupees. What is amount? [2018 - II] reduction in consumption has to be. [2018 - II]
Arithmetic B-15

1 (a) 10 (b) 15
(a) 50% (b) 33 (c) 25 (d) 50
3
111. 60% of the cost price of an article is equal to 50% of its
(c) 33% (d) 30%
101. The ratio of the numbers of males and females in a club is 5 : selling price. Then the percentage of profit or loss on the
6. If 22 females leave the club, the ratio becomes reversed. cost price is- [2019-I]
The number of males in the club is [2018 - II] 2
(a) 20% loss (b) 16 % profit
(a) 40 (b) 50 3
(c) 55 (d) 60 (c) 20% profit (d) 10% loss
102. Average age of n students who promoted in class VIII is Y 112. There are in all, 10 balls; some of them are red and the others
years. Three more students included in class whose ages white. The average cost of all balls is Rs. 28. If the average
are Y–1, Y–2 and Y+3 years. Find their average age when cost of red balls is Rs. 25 and that of white balls is Rs. 30, the
they promoted in class Xth. [2019-I] number of white balls is: [2019-I]
(a) Y (b) Y-3 (a) 3 (b) 5
(c) Y+2 (d) Y+5 (c) 6 (d) 7
103. The average monthly rainfall is 2.7inch, the average of first
113. Either 8 men or 17 women can paint a house in 33 days. The
7 months rainfall is 1.1 less than the average of yearly rainfall
and rainfall of other four months is 20.3 what is the average number of days required to paint three such houses by 12
rainfall of the last month? [2019-I] men and 24 women working at the same rate is : [2019-I]
(a) 0.9 (b) 10 (a) 44 (b) 43
(c) 2.1 (d) 1.3 (c) 34 (d) 66
104. X, Y and Z have some monkey in the ratio 4:3:8. If 2 monkey 114. The difference between simple and compound interest on a
run away from X and 4 monkey run away Z than the ratio sum of money at 5% p.a. for 2 years. is Rs. 100. The sum of
become 3:3:8 how many monkey they initially had? [2019-I] money must be. [2019-I]
(a) 10 (b) 20 (a) Rs. 35, 000 (b) Rs. 41, 000
(c) 40 (d) 30 (c) Rs. 40, 000 (d) Rs. 45, 000
105. A is 5 times efficient as of B. A completes a piece of work in 115. An alloy contains copper, zinc and nickel in the ratio of 5 : 3 : 2.
60 days less than B, how many time will they take The quantity of nickel in kg that must be added to 100 kg of
individually? [2019-I] this alloy to have the new ratio 5 : 3 : 3 is [2019-I]
(a) 15 days,75 days (b) 13 days, 65 days (a) 8 (b) 10
(c) 15 days, 60 days (d) 17 days, 85 days (c) 12 (d) 15
106. A man bought watch and pen-drive at 1564 each. And one 116. The ratio of the ages of Ram and Rahim 10 years ago was 1 :
sold for 23% profit and other sold for 23% loss. What is 3. The ratio of their ages five years hence will be 2 : 3. Then
overall profit or loss? [2019-I] the ratio of their present ages is [2019-I]
(a) 0% (b) 23% (a) 1 : 2 (b) 3 : 5
(c) 46% (d) 0% (c) 3 : 4 (d) 2 : 5
107. A and B earn in the ratio 2:1. They spend in the ratio 5:3 and 117. The population of a town is 145530. If it increases at the rate
save in the ratio 4:1. If the total monthly savings of both A of 5% per annum, what was it 2 yr ago? [2019 - II]
and B are Rs.5000, the monthly income of B is- [2019-I] (a) 132000 (b) 142000
(a) Rs. 7,000 (b) Rs. 14,000 (c) 136000 (d) 139000
(c) Rs. 5,000 (d) Rs. 10,000 118. Marshall, Randy and Roger have some coins. Marshall’s 5
times coin is equal with Randy’s 8 times coin. Marshall’s 8
108. 240 men can finish a work in 20 days working 5 hours a day. times coin is equal with Roger’s 5 times coin. Find the
To finish the work within 10 days working 8 hours a day, the minimum coin the three have in total. [2019 - II]
minimum number of men required is- [2019-I] (a) 21 (b) 129
(a) 310 (b) 300 (c) 90 (d) None of these
(c) 315 (d) 312 119. A seller says he sells goods at 10% loss , but he actually
109. While selling, a businessman allows 40% discount on the uses the wrong measuring scale and gains 15%.what is the
marked price and there is a loss of 30%. If it is sold at the length of a meter scale? [2019 - II]
marked price, profit per cent will be – [2019-I] (a) 0.7628 (b) 0.6728
(c) 0.8762 (d) 0.7826
(a) 10% (b) 20%
120. Someone borrowed 7000 at some interest rate for 3 years
(c) 16.68% (d) 16.25%
and after 3 years he again borrowed 3000 with same rate . If
110. The average salary of all the staff in an office of a corporate he end up giving simple interest of 4615 after 8 years then
house is Rs. 5,000. The average salary of the officers is Rs. what was his interest rate? [2019 - II]
14,000 and that of the rest is Rs. 4,000. If the total number of (a) 6.5% (b) 7.2%
staff is 500, the number of officers is– [2019-I] (c) 7.8% (d) None of these
EBD_8177
B-16 Arithmetic

121. A man earns a commission at 10%. When he sells product 129. The cost price of 20 articles is the same as the selling price
of value more than 20k, he earns bonus of extra 5% on the of x articles. If the profit is 25%, then the value of x is?
excess of sales above 20000, if he earned 2600 total [2020 - I]
commission, find bonus amount? [2019 - II] (a) 15 (b) 16
(a) 400 (b) 200 (c) 18 (d) 25
(c) 600 (d) 800 130. In a certain store, the profit is 320% of the cost. If the cost
122. A family consists of grandparents, parents and 4 increases by 25% but the selling price remains constant,
grandchildren. If average age of grandparents is 67, average approximately what percentage of the selling price is the
age of parents is 35 and average age of grandchildren is 6, profit? [2020 - I]
then what is the average age of the family? [2019 - II] (a) 30% (b) 70%
(a) 25 (b) 26 (c) 100% (d) 250%
(c) 28 (d) 28.5
131. Seats for Mathematics, Physics and Biology in a school are
123. 6 boys and 3 men can do 5 times the work done by 1 man
in the ratio 5 : 7 : 8. There is a proposal to increase these
and 1 boy in an hour. Calculate the ratio of work done by
seats by 40%, 50% and 75% respectively. What will be the
men and boy? [2019 - II]
ratio of increased seats? [2020 - I]
(a) 2:1 (b) 1:2
(a) 2 : 3 : 4 (b) 6 : 7 : 8
(c) 2:3 (d) None of these
(c) 6 : 8 : 9 (d) None of these
124. A can finish a work in 20 days. A works for 5 days and leave,
then B finish the remaining work in 10 days. How much time 132. Salaries of Ravi and Sumit are in the ratio 2 : 3. If the salary of
will be required to do the work if both A and B work together? each is increased by ` 4000, the new ratio becomes 40 : 57.
[2019 - II] What is Sumit’s salary? [2020 - I]
(a) 7 days (b) 8 days (a) ` 17,000 (b) Rs. 20,000
(c) 8.5 days (d) 9 days (c) ` 25,500 (d) ` 38,000
125. If on P, R% rate per annum is compounded half yearly then 133. In a mixture 60 litres, the ratio of milk and water 2 : 1. If this
after an year we get an amount Q. If on P simple interest of ratio is to be 1 : 2, then the quantity of water to be further
S! per annum if applied, we get amount Q. What is the relation added is: [2020 - I]
between R and S? [2019 - II] (a) 20 litres (b) 30 litres
æ R ö æ R ö (c) 40 litres (d) 60 litres
(a) S = R ç1 + ÷ (b) S = R ç 1– ÷ 134. There is 60% increase in an amount in 6 years at simple
è 400 ø è 400 ø
interest. What will be the compound interest of ` 12,000
æ R2 ö after 3 years at the same rate? [2020 - I]
(c) S = R çç 1 + 400 ÷÷ (d) None of these (a) 2160 (b) 3120
è ø (c) 3972 (d) 6240
126. If the cost of rice increases by 25 percent then expenditure 135. An accurate clock shows 8 o’clock in the morning. Through
of family increase by 10 percent. If family was consuming how may degrees will the hour hand rotate when the clock
40kg rice before how much family is consuming now? shows 2 o’clock in the afternoon? [2020 - I]
[2019 - II] (a) 144° (b) 150°
(a) 32.2 kg (b) 28.6 kg (c) 168° (d) 180°
(c) 35.2 kg (d) None of these
136. A alone can do a piece of work in 6 days and B alone in 8
127. The age of captain of a cricket team is 30 yrs and
days. A and B undertook to do it for ` 3200. With the help of
wicketkeeper is 1 year older. The average of the team
C, they completed the work in 3 days. How much is to be
excluding captain and wicketkeeper is 1 less than the average
paid to C? [2020 - I]
of the whole team. What is average of whole team?
(a) ` 375 (b) ` 400
[2019 - II]
(a) 25 (b) 26 (c) ` 600 (d) ` 750
(c) 30.5 (d) None of these 137. The ratio between the perimeter and the breadth of a
128. A can do a job in 20 days. He works for 5 days and finishes rectangle is 5 : 1. If the area of the rectangle is 216 sq. cm,
1/4 of the total work. Then B comes and finishes the what is the length of the rectangle? [2020 - I]
remaining work in 15 days. How many days A and B together (a) 24 (b) 18
will take to finish the job? [2019 - II] (c) 16 (d) None of these
(a) 17.5 days (b) 15 days
(c) 10 days (d) None of these
Arithmetic B-17

ANSWERS & EXPLANATIONS


1. (d) Total sum of ages of 35 students = 35 × 16 30 ´ 14 + required run
Total sum of ages of 21 students = 21 × 14 9. (b) New Average =
15
35 ´16 - 21 ´ 14 32 × 15 – 30 × 14 = Required run
The average of remaining students = Required run = 60
14
10. (c) Let the numbers are 3x, 4x and 5x respectively.
= 19 years
2. (c) Age decreased = (5 × 3) years = 15 years
According to question (3x)2 + (4x)2 + (5x)2 = 1250
So, the required difference = 15 years
9x2 + 16x2 + 25x2 = 1250
3. (b) Let the total number of workers be x. Then,
8000x = (12000 × 7) + 6000 (x – 7) 50x2 = 1250
8000 x = 84000 + 6000 x – 42000 x2 = 25
2000x = 42000 \ x = 21 x= 5
4. (a) Let the price of a scooter and a TV be 7x and 5x Numbers are 15,20 and 25.
respectively. Sum = 15 + 20 + 25 = 60
According to question 11. (a) Let the ages of three boys are 3x, 5x and 7x.
7x = 5x + 8000 3x + 5x + 7x
2x = 8000 Average age =
3
x = 4000 25 × 3 = 15x
Hence, the price of TV = ` 20,000 x= 5
5. (d) Speed of cars is 5 : 4 : 6
The age of youngest boy = 15 yr.
1 1 1 12 :15 :10 12. (b) Numbers are 10, 15, 20, 25 and 30.
Time ratio = : : =
5 4 6 60
The ratio between the time taken by them to travel the 10 + 15 + 20 + 25 + 30
Required average =
same distance is 12 : 15 : 10 5
6. (c) Let the number are 3x and 4x. 100
= = 20
3x + 6 4 5
= (n) (n + 1)
4x + 6 5 13. (c) Average = (2) ´ n
15x + 30 = 16 x + 24
Where ‘n’ be the natural number
x= 6
Number are 18 and 24. 80 ´ (80 + 1) 81
Therefore, average = = = 40.5
Hence, required difference is 6. 2 ´ 80 2
7. (c) Let the numbers are Sum of all numbers
x, x +2, x + 4, x + 6 and x + 8. 14. (a) 50 =
number of observations
x + x +2+ x +4+ x +6+ x +8 450
Average = 50 =
5 Number of observations
61 × 5 = 5x + 20 450
5x = 305 – 20 Number of observations = =9
50
5x = 285
x = 57 450 + 100 550
New mean = = = 55
First number = 57 10 10
Last number = 65 15. (c) Let the numbers are 7x and 8x
Required difference = 65 – 57 = 8 7x + 3 8
8. (a) Let the age of Jayesh = x yr. =
8x + 3 9
x 63x + 27 = 64x + 24
Therefore age of Vijay = yr .
2 x= 3
And, age of Suresh = 2x yr.
Numbers are 21 and 24
According the question
16. (b) Let the numbers are a, b and c
x a : b= 2 : 3
+ 2x = 85
2 b: c= 5 : 8
5x = 85 × 2 ; x = 34 yr.
EBD_8177
B-18 Arithmetic

a:b:c
2:3 22. (b) Tea 1 Tea 2
5:8 10 14

10 : 15 : 24
11
10x + 15x + 24x = 98
49x = 98
x= 2 3 1
Second number = 15 × 2 = 30
Hence, the ratio should be 3 : 1.
9
17. (b) Sita’s share = ´ Amount 23. (b) Total age increased = (8 × 4) = 32 years
17
Age of new man = 30 + 32 = 62 years
17 ´ 4500 24. (c) Total expenditure spent on first four months
\ Amount = = ` 8500 = 4 × 231.25 = ` 925
9
18. (c) Total expenditure spent on four months Total expenditure spent on next five months
= 5 × (231.25 + 22.75)
= 4 × 225.25 = ` 901
= 5 × 254 = ` 1270
Total expenditure spent on next five months Total expenditure for 12 months
= 5 × (225.25 + 20.75) = 5 × 246 = ` 1230 = 925 + 1270 + 605 = ` 2800
Total expenditure for 12 months 3500 - 2800
Required % = ´100 = 20%
= ` [901+ 1230 + 700] = 2831 3500
Hence, saving = 3500 – 2831 = 669
25. (b) Boys Girls
669 16.4 15.4
Required % = ´ 100 = 19.11%
3500
19. (d) Let the present age of P and Q be 2x and 3x.
15.8
According to question

2x + 18 4
=
3x + 18 5 0.4 0.6
10x + 90 = 12x + 72
Boys to girls ratio = 0.4 : 0.6 or 2 : 3
2x = 18
26. (b) 3x + 6x + 7x = 80
x= 9
16x = 80
Present age of Q = 9 × 3 = 27 years x= 5
Mon + Tues + Wed Numbers are 15, 30, 35
20. (b) = 37°C ...(1)
3 2
27. (c) Milk = ´ 60 = 40l
3
Tues + Wed + Thurs
= 34°C ...(2)
3 1
Water = ´ 60 = 20l
Subtracting equation (2) from (1) 3
Mon – Thurs = 37 × 3 – 34 × 3 Let ‘x’ be the amount of the water to be added
5 40 1
Thurs – Thurs = 9 = = 80 = 20 + x
4 20 + x 2
Thurs = 36°C 60 = x
21. (c) Going by options, 28. (c) Let expenditure per day = x
If the total sum is 6400 then distribution among P, Q x x + 42
\ = +1
and R was 1200, 3800 and 1400. 35 42
After giving 200 to Q by R. \ x = 420
Hence, the initial expenditure on food per day = ` 420
New sum would be 1200, 4000 and 1200.
Or, 1200 : 4000 : 1200 = 3 : 10 : 3
Arithmetic B-19

35. (c) Let the amount taxable purchase be ` X.


1
29. (c) Age of new comer = 18 – 24 × = 16 years. 30
12 Then, 6% of x =
100
30. (d) Let average of all persons = x
\ (13 × 79) + 6(x + 4) = 19 × x æ 30 100 ö
Þx=ç ´ =5
è 100 6 ÷ø
13 × 79 + 6x + 24 = 19x
\ Cost of tax free items = ` [25 – (5 + 0.30)] = ` 19.70
13 × 79 + 24 = 13x
36. (b) Let the sum be ` 100 Then.
13 ´ 79 + 24
x= = 80.25 æ 10 0 × 10 + 1 ö
13 S.I for first 6 months = ` çè ÷ =` 5
Total money spent = 1536 1 00 × 2 ø
31. (d) Skilled workers : Unskilled workers : Clerks = 8:5:1
Ratio of the respective wages = 5:2:3 æ 105 ×10 + 1 ö
S.I for last 6 month = ` ç ÷ = ` 5.25
Hence, the amount must be paid in the ratio è 100× 2 ø
8×5 : 5×2 : 1×3 = 40 : 10 : 3 So, amount at the end of 1 year = ` (100 + 5 + 5.25)
Sum of the ratios = 40 + 10 + 3 = 53
If the total amount is ` 53, the skilled workers get = ` 110.25
` 40. \ Effective rate = (110.25 –100) = 10.25%
If the total amount is ` 318, the skilled worker will get 37. (b) Let C.P. = ` 100 Then, Profit = ` 320 S.P. = ` 420.
40 New C.P. = 125% of ` 100 = ` 125
= ´ 318 = ` 240
53
New S.P. = ` 420
10
Unskilled workers get = ´ 318 = ` 60 Profit = ` (420 –125) = `295.
53
æ 295 ö 1475
3 \ Required percentage = çè ´ 100÷ % = % = 70 %
and clerk get = ´ 318 = ` 18 420 ø 21
53
(approximately).
32. (d) 3 leaps of dog = 4 leaps of Cat
38. (a) Let the principle amount be ‘P’
16
\ 4 leaps of dog = leaps of Cat P.r .t
3 S.I. =
100
16 According to question –
\ the rate of dog : rate of cat = :5 =16 :15
3 P.4.8
rate of cat : rate of dog = 15:16 P – 340 =
100
8 8P
33. (a) Each one receive apple P – 340 =
3 25
8 1 8P 17P
Ram gave = 3 – = apple P– = 340 Þ = 340
3 3 25 25
8 7 P = 500 `
Shiv gave = 5 – = apples
3 3 1 æ 1 1ö 1 7 1
39. (b) C'S 1 day's work = -ç + ÷ = - =
1 7 3 è 6 8 ø 3 24 24
\ Ram : Shiv = : =1: 7
3 3 1 1 1
A'S wages : B's wages: C's wages = : : = 4 : 3 :1
1 6 8 24
Ram got = 8× = ` 1
8 æ 1 ö
7 \ C'S share (for 3 days) = ` çè 3 ´ ´ 3200÷ø = ` 400
Shiv got = 80 × = ` 7 24
8 40. (a) In 35 litre mixture
34. (d) Let total weight of 5 men be x kg and weight of new
1
man y kg. water is = 35 ´ = 7 litre
5
x -60 + y x x y x milk is = 35 – 7 = 28 litre
= + 2 Þ - 12 + = + 2
5 5 5 5 5 after adding 7 litre of more water, water is 7+7 = 14 litre.
Þ y = 70 kg 28 2
weight of new man = 70 kg so ratio of milk and water in new mixture is = = =2 : 1
14 1
EBD_8177
B-20 Arithmetic

æ 100 ö 48. (a) Let the sum be Rs. x. Then,


41. (b) C.P. of 1st transistor = ` çè × 840÷ = ` 700
120 ø é æ 4 ö
2 ù æ 676 ö 51
C.I. = ê x ç1 + ÷ - xú = ç x - x÷ = x
æ 100 ö ë è 100 ø û è 625 ø 625
C.P. of 2nd transistor = ` çè × 960÷ = ` 1000
96 ø
æ x ´ 4 ´ 2 ö 2x
So, total C.P = `(700 + 1000) = ` 1700 S.I. = ç ÷= .
Total S.P = ` (840 +960) = ` 1800. è 100 ø 25

æ 100 ö 15 51x 2x
\ Gain % = çè ´ 100÷ % = 5 % \ - =1
1700 ø 17 625 25
42. (a) If A is doing half of the work is 8 days then he will Þ x = 625
complete it in 16 days. 49. (a) Suppose he move 4 km downstream in x hours. Then,
1 æ 4ö
If B is doing rd of the work in 8 days then he will Speed downstream = ç ÷ km/hr..
3 èxø
complete it is 8 ×3 = 24 days.
16 ´ 24 æ 3ö
days required = Speed upstream = ç ÷ km/hr..
16 + 24 èxø
16 ´ 24 48 48 1
(when A & B both are working ) = = 9.6 days \ +
40 ( 4 / x ) ( 3 / x ) = 14 or x = 2
So, Speed downstream = 8 km/hr, Speed upstream
43. (c) Profit (P) = Selling price (S) – Cost price (C)
= 6 km/hr.
P =S-C ...(1)
1
According to question – Rate of the stream =(8 – 6) km/hr = 1 km/hr..
3P = 2S – C ...(2) 2
Subtracting eq (1) from eq (2) 50. (b) Income Expenditure
2P=S Ram 3x 4y
Putting S in eq (1) Shyam 4x 5y
P= 2P–C Þ C=P ....(3) 1
Q Shyam's saving = of his income
3
(Selling price - Cost price) 1
profit % = ´ 100 4x – 5y = × 4x
Cost price 3
æ S - Cö (2P - P) 4
= çè ÷ ´ 100 = ´ 100 =100% 4x – x = 5y
C ø P 3
8
(10)2 100 x = 5y
44. (b) Net loss% = = =1 3
100 100
1 15
45. (a) Since price of an item is decreased by times. So it x= y ...(i)
4 8
1 Ram's Savings 3x – 4y
must be increased by times to get the original price. Now, =
3 Shyam's Savings 4x – 5y
1 1
= 33 % 15
3 3 3 ´ y – 4y
8 13
1 = =
46. (d) A' salary is times more than B'salary. So B' salary 15 20
2 4 ´ y – 5y
1 8
would be times, less than A.
1+ 2 Hence ratio of their savings = 13 : 20.
1 1
i.e., = 33 % 51. (b) C alone can finish the work in 40 days
3 3
1
47. (c) For 3 years Þ `600 more Q C does as much as A and B together
for 1 years Þ ` 200 more 2
So, A and B can finish the work in 20 days.
Now4% of principal = 200 So, together they will finish in
200 ´ 100
Principal = 40 ´ 20 40 1
4 = days = 13 days
40 + 20 3 3
Principal = `5000
Arithmetic B-21

52. (d) Amount of milk after 3 operations 57. (a) A can do a work in 10 day
é æ 1
4 ö ù
3
ê - A can do work in 1 day = part of work
= 40 ç 1 ÷ ú litres 10
ë è 40 ø û
Let B can do in x days
æ 9 9 9 ö 1
= ç 40 ´ ´ ´ ÷ = 29.16 litres in 1 day he can do = part of work
è 10 10 10 ø x
53. (d) Let the distance = d A working for 4 day and B for 9 days work is completed
Speed of upstream = 9 – 3 = 6 km/h
4 9
Speed of down stream = 9 + 3 = 12 km/h ie ∗ <1
10 x
d d
Now, + =3 9 6
6 12 <
x 10
3d x = 15 days
=3
12
1 1
d = 12 km Then Both A and B can do in 1 day = + part of
Hence distance = 12 km 10 15
54. (b) Principal + S.I for 2 years = `720 ...(1) 1
Principal + S.I for 7 years = `1020 ...(2) work =
6
Clearly S.I for 5 years = 1020 – 720 = `300
Or they both will complete work in 6 days
2
NowS.I for 2 years = 300 ´ = `120 2x
5 58. (d) Let the sum be x. then SI = 40% of x = Rate = 5%.
from (1) 5
Principal = 720 – 120 = `600
é100 ≥ 2x 1 ù
55. (d) Let A got = x + 25 Time = êê ≥ ú
B got = 3x + 25 ë 5 5x úû
C got = 2x + 25 é ù
ATQ ê T < 100 ≥ S.I ú = 8 years
ê P ≥ R úû
x + 25 + 3x + 25 + 2x + 25 = 735 ë
6x = 735 – 75
3P
6x = 660 59. (b) Let principal be = P, SI =
x = 110 8
Money received by C = 2x + 25 = 2 × 110 + 25 = ` 245 1
R = R, T = 6 year
3 4
56. (b) Amount of water in mixture = ≥ 20 < 7.5 l
3∗5
PTR
SI <
5 100
amount of Milk in mixture = ≥ 20 < 12.5 l
8
3P P ≥ 25 ≥ R
When 4 litre is drawn out of mixture remaining Mixture <
8 100 ≥ 4
= 16l
R = 6% P. a
3
Water amount = ≥16 < 6l 60. (b) Let CP of Horse be = ` x
8
125
5 Then SP after getting 25% profit = x
Milk amount = ≥16 < 10 l 100
8
When CP is 200 Less CP = ` (x – 200)
After adding 4 litre of milk
Profit = 30%
Total mixture = 20 l
Milk = 10 + 4 = 14 l 130
SP = (x – 200)
Water = 6 l 100
ATQ
M 14
Ratio = < < 7:3 130 125
W 6 (x – 200) = x – 200
100 100
EBD_8177
B-22 Arithmetic

130 125 G - 40
x – 260 = x – 200 = 24
100 100 4
G = 24 × 4 + 40 = 136 years
5x Let present age of youngest child = x
< 60
100 Present age of older child = x + 2
x = ` 1200 G+x+x+2
= 24
6
61. (c) SP of chair = ` 368, Let CP = `x
Loss% = 8% 136 + x + x + 2
= 24
6
é100 – 8 ù
ê ú x < 368 2x + 138 = 24 × 6
ëê 100 ûú 2x = 144 – 138 = 6
368 \ x = 6/2 = 3 years
x= ≥ 100 < 400 \ Present age of youngest child = 3 years
92 65. (c) Let total age of family be G years.
When, profit = 15% 3 years back, total age = G – 3 × 4 = G – 12
115 According to question,
[ SP = ≥ 400 < ` 460
100 G ,12
< 24
62. (c) 3 men can Plough Field in 43 days, 1 man will plough in 4
43 × 3 \ G = 24 × 4 + 12 = 108 years
4 women can plough in 43 days 1 women will plough in Let present age of baby = x
= 4 × 43 days.
G+x
Work done by 1 men and 1 women in 1 day = 24
5
1 1
= + 108 + x
43 ≥ 3 43 ≥ 4 = 24
5
Work done by 7 men and 5 women in 1 day
Þ 108 + x = 120
7 5 \ x = 120 – 108 = 12 years
= 43 ≥ 3 ∗ 43 ≥ 4 66. (b) Let principal = 100
Amount = 200
1 é 28 ∗ 15 ù ..
43 1 Rate = r%
= ê ú< <
ê
43 ë 12 û ..ú 43 ≥ 12 12 Time = 4 years
Now,
They will do work in 12 days.
t
3750 æ r ö
63. (a) CP of first watch = ≥100 A = P ç1 + ÷
105 è 100 ø
3750 4
CP of second watch = ≥100 æ r ö
95 200 = 100 ç1 + ÷
è 100 ø
\ Total CP
3750 ≥100 æç 1 1 ö 375000 æç 40 ö÷ 4
ç ∗ ÷÷÷ =
æ r ö
= ç ç ÷÷ 2 = ç1 + ÷ ............(i)
5 è 21 19 ø 5 èç 21 ≥19 ø÷ è 100 ø
If sum become 4 times in the time n years
3000000
= 21≥19 < 7518.80 n
then, 4 = æç1 +
r ö
÷
Total SP = 3750 + 3750 = 7500 è 100 ø
Loss = 18.80 n
æ r ö
18.80 22 = ç1 + ÷ .............(ii)
Loss% = ≥100 » . 25% loss è 100 ø
7500
Using eqn (i) and (ii) we get
64. (c) Let total age of family be G years.
10 years ago, total age = G – 10 × 4 = G – 40 éæ
2
r ö ù
4 n
According to question, æ r ö
êç1 + ÷ ú = ç 1 + ÷
êëè 100 ø úû è 100 ø
Arithmetic B-23

æ r ö æ
8
r ö
n æ3 ö
ç1 + ÷ = ç1 + ÷ If difference of time is 40 days, B takes ç ´ 40 ÷ = 60 days
è 100 ø è 100 ø è2 ø
Thus, n = 8 years So, A takes 20 days to do the work.
67. (a) CP of 12 chocolates = ` 9 1
A's 1 day's work =
9 20
CP of 1 chocolate = = 0.75
12 1
Now, B's 1 day's work =
60
SP = ` 1, Profit = 0.25
æ 1 1 ö æ 3 +1 ö
Profit% =
0.25 1
´ 100 = 33 % (A + B)'s 1 day's work = ç + ÷ = ç ÷
è 20 60 ø è 60 ø
0.75 3
68. (d) SP1 = 12, 000 4 1
= =
12, 000 60 15
CP1 = = 16, 000
0.75 \ A and B together can do the work in 15 days.
SP2 = 12, 000 72. (c) According to question,

12, 000 1
(A + B)'s 1 day's work =
CP2 = = 10, 000 12
1.2
So, total CP = 16000 + 10000 = 26, 000 1
total SP = 24, 000 (B + C)'s 1 day's work =
15
So loss = 2000
1
2000 (A + C)'s 1 day's work =
\ loss percentage = ´100 = 7.6% 20
26, 000
æ 1 1 1 ö
100 \ (A + B) + (B + C) + (A + C) = ç + + ÷
69. (a) CP of 1st TV = 3000 ´ = 2608.69 è 12 15 20 ø
115
Profit = SP – CP = 3000 – 2608.69 = 391.31 æ 5 + 4 + 3 ö æ 12 ö æ 1 ö
Loss on 2nd TV = 391.31
\ 2 (A + B + C) = ç ÷= ç ÷ = ç ÷
è 60 ø è 60 ø è 5 ø
Loss percentage = 10%
1
Loss \ (A + B + C)'s 1 day's work =
Loss percentage = ´ 100 10
CP
\ A, B and C together can do the work in 10 days.
391.31 73. (d) Let CP = x
10 = ´ 100 According to question
CP
391.31 ´100 æ 100 ö
\ CP = Þ 575 + x = 500 + x ç ÷
10 è 90 ø

= 3913.10 @ 3913 10x


Þ 575 + x = 500 +
70. (d) Let the number of boys and girls be 3x and 2x. 9
Number of those who are not scholarship holders x
75 =
æ 80 70 ö æ 12x 7x ö 19x 9
= ç 3x ´ + 2x ´ ÷Þ ç + ÷=
\ x = 75 × 9 = 675
è 100 100 ø è 5 5 ø 5
74. (a) Let capacity of tank = x
æ 19x ö According to question,
\ Required percentage = ç ´100 ÷
è 5 ´ 5x ø 3x x
- = 22
= 76% 4 5
71. (a) Ratio of times by A and B = 1 : 3
The time difference is (3 – 1) = 2 days while B takes 3 15x - 4x
= 22
days and A takes 1 day. 20
If difference of time is 2 days, B takes 3 days
EBD_8177
B-24 Arithmetic

OR
11x
= 22 15 5
20 A´ =´
100 100
22 ´ 20 \A: B= 1 : 3
\ x= = 40
11
1
\ Capacity of tank is 40 liters Salary of A = 40000 × = 10000
4
1 1 1 1
75. (a) Ratio among A, B, C and D = : : : 32000
3 4 5 6 80. (c) Present price of table = 2
= 20 : 15 : 12 : 10 æ 20 ö
çè1 - ÷
So, minimum number of pens = 20 + 15 + 12 +10 = 57 100 ø
76. (b) Let sum be x
5 5
Original Rate = R = 32000 ´ ´ = 50000
1st Condition: 4 4
Rate of interest = R 10 + 14
Time = 4 years 81. (d) Average cost of = = 12
2
2nd Condition: SP = 13
Rate of interest increased by 3 = (R + 3)%
Time = 4 years 13 - 12 1
P% = ´ 100 = 8 % (gain).
Therefore, 12 3
82. (c) If the original price of article be ` x, then
P ´ (R + 3) ´ 4 P ´ R ´ 4
- = 78 80 130
100 100 x× ´ = 416
4PR + 12P – 4PR = 7800 100 100
12P = 7800 416 ´ 100 ´ 100
Þx= = ` 400
7800 80 ´ 130
P= = 650
12 T 5
æ R ö æ R ö
83. (d) A = P ç1 + Þ 24000 = 12000 çè1 +
77. (a) Here C. P. is not equal to selling price because he sells è 100 ÷ø ÷
100 ø
the cloth at 10% loss.
Using the formula, æ R ö
5
æ R ö
20
100 ∗ g True scale Þ 2 = ç1 + Þ 24 = çè1 +
è 100 ÷ø ÷ : = 16 times
< 100 ø
100 ∗ x False scale
i.e. The sum amounts to `192000.
Here, overall gain G = 15% and loss = – 10%
2
Let false scale length = l cm æ 8 ö
(c) 2916 = P ç1 +
è 100 ÷ø
84.
100 ∗ 15 100 90
[ < Þ ≥100 = 78.25 cm
100 – 10 l 115 2916
78. (b) Let total number of workers be n P= = 2500
(1.08)2
Total salary of all workers = 8000 n
Total salary of 7 technicians = 7 × 12000 = 84,000 2500 ´ 9 ´ 3
Total salary of remaining workers = (n – 7) × 6000 S.I = = 675
100
84000 + (n – 7) × 6000 = 8000 n 85. (b) A : B = 2 : 3 = 8 : 12
84 + 6n – 42 = 8n B : C = 4 : 5 = 12 : 15
42 = 2n \ A : B : C = 8 : 12 : 15
n = 21 Sum of ratio = 35
79. (a) Let the monthly salary of A be x,,
8
monthly salary of B is (40000 – x). \ A’s share = ´ 700 = ` 160
Savings of A = (100 – 85)% of x = 0.15x 35
Savings of B = (100 – 95)% of (40000 – x) 12
= 0.05 (40000 – x) B’s share = × 700 = ` 240
35
0.15 x = 0.05 (40000 – x)
0.15x + 0.05x = 40000 × 0.05 15
C’s share = × 700 = ` 300
0.2x = 2000 35
x = 10000
Arithmetic B-25

1 100
86. (c) (A + B)’s 1 day’s work = 90. (a) Cost price of first horse = 1200 ´ = 1000
8 120
1 100
(B + C)’s 1 day’s work = Cost price of second horse = 1200 ´ = 1500
12 80
Sum of cost price of two horses = 1000 + 1500 = 2500
1
(C + A)’s 1 day’s work = Sum of selling price of two horses = 1200 + 1200 = 2400
8
Loss = 2500 – 2400 = 100
On adding,
100
2 (A + B + C)’s 1 day’s work Overall percentage loss = ´ 100 = 4% ( loss )
2500
1 1 1 3+ 2+ 3 8 1 91. (b) We know that efficiency is inversely proportional to
= + + = = =
8 12 8 24 24 3 the time
So, ratio of time in which they complete the work
1
\ (A + B + C)’s 1 day’s work = 1 1 1
6 = : : = 15 : 12 : 10
4 5 6
Hence, the work will be compeleted in 6 days.
So, ratio of time = 15 : 12 : 10
87. (d) Part of the tank filled by both pipes in two hours
92. (a) 5 dozen = 60
æ1 1ö æ3+4 ö 7 Now, cost price of 60 eggs = `100
= 2ç + ÷ = 2ç ÷=
è8 6ø è 24 ø 12 80
Number of unbroken eggs = 60 ´ = 48
7 5 100
Remaining part = 1 - = to get 10% profit selling price of 48 eggs
12 12
Time taken by B in filling the remaining part 110
= 100 ´ = 110
100
5 5 1
= ´ 6 = = 2 hours 110
12 2 2 Selling price of each egg = = 2.292 / eggs.
48
88. (b) According to question,
16
If P can complete a work in 1 day, Q can complete the 93. (d) Rate of interest (semi annually) = % = 8%
2
same work in 4 days.
Time (in semi year) = 4
Hence, if the difference is 3 days, Q can complete the
t
work in 4 days æ r ö
Now, compound interest = P ç1 +
Þ If the difference is 45 days, Q can complete the work è 100 ÷ø
in 60 days 4
æ 8 ö
= 10000 ç1 +
è 100 ÷ø
1 = 13605
\ Q's 1 day's work =
60
94. (b) Let cost price of 1 kg sugar is `100
1 1 After increase in cost price by 25%
\ P's 1 day's work = 4 ´ =
60 15 Cost price of 1 kg sugar = 125
\ (P + Q)'s 1 day's work Amount of sugar we can purchase in `100
æ1 1 ö ( 4 + 1) 5 1 100 4
=ç + ÷ = = = = = kg
è 15 60 ø 60 60 12 125 5
\ P and Q together can do work in 12 days. Percentage decrease in consumption
89. (b) From formula, difference between C.I. and S.I.
at the end of 3 years. æ 4ö
ç 1- 5 ÷
Pr 2 æ 300 + r ö =ç ÷ ´100 = 20%
D= ç ÷ ç 1 ÷
(100)2 è 100 ø è ø
2
æ 10 ö æ 300 + 10 ö 95. (d) Let number of students fail and pass in the class are x
\ 77.5 = P ç
è 100 ÷ø çè 100 ÷ø and y respectively.
P æ 31ö ATQ,
77.5 = ç ÷
100 è 10 ø 70 (x + y) = 40x + 80y
77.5 ´ 1000 30x = 10y Þ y = 3x
\P= =` 2500
31 y
Now, = 3
x
EBD_8177
B-26 Arithmetic

y 3 36 x – 25 x = 132
= {By componendo & devedendo}. 11 x = 132
x + y 1+ 3
Þ x = 12
3
\ Percentage of pass students = ´ 100 = 75% The number of males = 5x = 5×12 = 60
4 102. (c) Total age of students who promoted in class VIII is
96. (a) Let population of the village in 2014 is N.
= n. Y years.
2
æ 5 ö When 3 more students included this college.
Then, 1852200 = N ç1 +
è 100 ÷ø = n.Y + Y – 1 + Y – 2 + Y + 3 = ny + 3y
After two years when they are in Xth total age
1852200
\N = = 1680000. = n (Y + 2) + 3(Y + 2)
1.05 ´ 1.05
n ( Y + 2) + 3( Y + 2)
97. (c) Amount of milk in the mixture = 250 ´ 7 = 194.4 ml Avg age =
9 ( n + 3)
Amount of water in the mixture = 55.6
Let x ml of milk is added in the mixture then, Ratio of
( n + 3)( Y + 2)
milk to water. = =Y+ 2
n +3
194.4 + x 4
= = 103. (a) Total Rain fall of 12 months = 2.7 × 12 = 32.4
55.6 1
Total Rain fall of first 7 months = 7 × (2.7 – 1.1) = 11.2
(194.4 + x) = 55.6 × 4 Rain fall of last month
194.4 + x = 222.4
= 32.4 – (11.2 + 20.3) = 32.4 – 31.5 = 0.9
x = 28 ml.
104. (d) X : Y : Z
98. (a) Let the Teacher's age is x year.
ATQ, 4 : 3 : 8
18 × 26 = 16 × 25 + x 4k - 2 3
468 = 400 + x =
3k 3
\ Teacher's age = 468 – 400
= 68 years. 4k – 2 = 3k
1 k= 2
99. (b) (A + B)'s 1 day's work = Initially monkey = (4 + 3 + 8) k = 15 k = 15 × 2 = 30
10
105. (a) According to question,
1
(B + C)'s 1 day's work = A : B
12 Efficiency 5 : 1
1 Time 1 : 5
(C + A)'s day's work = 5 x – x = 60
15
On adding, 4 x = 60
x = 15
1 1 1 6+5+ 4 1
2(A + B + C)'s 1 day's work = + + = = Time take by A = 15 days
10 12 15 60 4
B = 75 days
1 106. (d) CP = 1564 + 1564 = 3128
\(A + B + C)'s 1 day's work =
8
1564 ´ 123 1564 ´ 77
SP = +
1 1 3- 2 1 100 100
A's 1 day's work = = - = =
8 12 24 24
1564 ´ 200
\ A alone will complete the work in 24 days. = = 3128
100. (b) The percentage of reduction in consumption: 100
CP = S.P
100 ´ 50 1
= = (100 × 50)/150 = 33 % So, no profit no loss.
100 + 50 3
107. (a) Let the monthly income of B be Rs. x.
101. (d) Ratio of males and females = 5 : 6
Then , Monthly income of A = Rs. 2x and
Let the males are 5x and females are 6x
Now, Saving of A =5000 × 4/(4 + 1)
22 females leave the club = Rs. 4000
5x : ( 6x – 22) = 6 : 5 Saving of B = Rs.1000
(6x – 22) 6 = 5x × 5 Now , we have
36x – 132 = 25x
Arithmetic B-27

(2x – 4000)/(x–1000) = 5/3 115. (b) Let x kg of nickel be mixed.


Þ 6x – 12000 = 5x – 5000 20 + x 3
x = Rs.7000 \ =
100 + x 11
So, Monthly income of B = 7000 Þ 220 + 11x = 300 + 3x
108. (b) M1D1H1 = M2D2H2 Þ 11x – 3x = 300 – 220
Required no. of men = (240 × 5 × 20)/(8 × 10) Þ 8x = 80
= 300 Þ x = 10 kg
109. (c) Let the M.P. be Rs.100. 116. (b) Let the ages of Ram and Rahim 10 years ago be x and
We know S.P = (100-40) = `60 3x years respectively.
After 5 years from now,
and C. P. = 60 × 100/(100 – 30) = `600/7 = 85.7
So, Required % profit = (100 – 85.7) /85.7 × 100 x + 15 2
Þ =
= 14.3/85.7 × 100= 16.68% 3x + 15 3
110. (d) Let the number of officers be x. Þ 6x + 30 = 3x + 45
Q 5000×500 = 14000x + 4000(500 – x) Þ 3x = 45 - 30 Þ 3x = 15
\ 2500000 =14000x + 2000000 – 4000x Þ x=5
\ x = 500000/10000 = 50 \ Ratio of their present ages
111. (c) Let the cost price be `100. = (x + 10) : (3x + 10)
= 15 : 25 = 3 : 5
Q S.P × 50/100 = 100× (60)/100
117. (a) Population 2 yr ago
\ S.P = (60×100)50
= Rs. 120 145530
=
\ Required % profit = (120 – 100)/100 × 100 = 20% æ 5 ö
2
112. (c) Let the number of white balls be x. ç1 + ÷
è 100 ø
\ Number of red balls = (10 – x)
\ 10 × 28 = x × 30 + 25 (10 – x) = æç 145530 ´ 20 ´ 20 ö÷ = 132000
Þ 280 = 30x + 250 – 25x è 21 21 ø
= 280 = 5x + 250 118. (b) Let Marshall, Randy and Roger have x, y and z number
Þ 5x = 280 – 250 = 30 of coins respectively.
Þ x=6 then, According to the question.
So, The number of white balls = 6 5x
5x= 8yÞ y =
8
113. (c) \ 8 men = 17 women
8x
17 51 and 8x = 5z Þ z =
Þ 12 men º ´ 12 = women 5
8 2 5x
As y is integer, then, for y =
\ 12 men + 24 women 8
5´8
=
51
+ 24 =
99 for x = 8; y = =5
women 8
2 2
8x
M1D1 M 2 D2 Again z is integer, for z =
By = 17 ´ 33 99 ´ D 2 5
Þ =
W1 W2 1 2´3 8´5
for x = 5, z = =8
5
17 ´ 33 ´ 6 \ x = L.C.M of (8, 5) = 40
D2 = = 34 days When x = 40, y = 5 × 5 = 25
99
and z = 8 × 8 = 64.
Pr 2 \ Minimum of (x + y + z) = 40 + 25 + 64 = 129.
114. (c) Difference = 119. (d) Let the price of 1m cloth is ` 100.
(100)2 then selling price of 1 m cloth on 10% loss
æ 100 –10 ö
P´5´5 = 100 ´ ç ÷ =` 90.
Þ 100 = è 100 ø
100 ´ 100
But, actually he got the profit of 15%
P then, length of cloths, he sold in ` 90, to get 15% profit.
Þ = 100
400 100
= 90 ´ = 78.26 cm
Þ P = Rs.40000 (100 + 15)
Hence, length of meter scale is 0.7826m.
EBD_8177
B-28 Arithmetic

120. (a) Let the interest rate is r% 124. (b) Amount of work done by A in 5 days
prt 5 1
= =
then, S.I. = 20 4
100
1 3
7000 ´ 3 ´ r 10000 ´ 5 ´ r Remaining work = 1– = work.
4615 = + 4 4
100 100
4615 = 210 r + 500r 3
work is done by B in 10 days.
4615 = 710 r 4
\ r = 6.5%
10 ´ 4
121. (b) Commission earned on sales of amount 20,000 \ whole work is done by B is = days.
3
10
= 20,000 ´ =` 2000 40
100 = days.
Remaining amount earn in commission 3
= 2600 – 2000 = `600 Time require to complete whole work by A and B together
Ratio of Percent of Commission 10 2 40
= = 20 ´
Bonus Percent 5 1
= 3 = 20 ´ 40 ´ 3 = 8days.
40 3 ´ (60 + 40)
1 20 +
\ Amount earned as Bonus = 600 ´ 3
(1 + 2)
125. (a) As interest is compounded half yearly
600
= =` 200. R%
3 \ R% = , time = 2 half year..
2
122. (d) total age of grandparents = 2 × 67 = 134 years
total age of parents = 2 × 35 = 70 years 2
æ ö
total age of 4 childrens = 4 × 6 = 24 years ç ÷ 2
Amount Q = P ç1 + R æ R ö
total age of the family = 134 + 70 + 24 = 228 years = P 1 + (i)
ç 2 ÷÷ ç
è 200 ø
÷
Number of members in the family = 2 + 2 + 4 = 8
è 100 ø
Average age of the family = 228 = 28.5 years
8 When interest is simple interest.
123. (a) Let 1 man and 1 boy can complete the work in x and y p.s.t æ 5 ö
hours respectively. Q =P+ Þ Q = P ç1 + ÷ (ii)
100 è 100 ø
then, Amount of work done by 1 man and 1 boy in 1
From (i) and (ii)
1 1
hour = + 2
x y æ R ö æ S ö
P ç1 + ÷ = P ç 1 + 100 ÷ .
Amount of work done by 3 men and 6 boys in 1 hour è 200 ø è ø
3 6 R2
= + S= + R.
x y 400
According to the question, 126. (c) Let initial expenditure of the family was `100 for 40 kg
Rice.
3 6 æ1 1ö
+ = 5ç + ÷ 100 ´ 10
x y èx yø Increase in expenditure = 100 + = 110.
100
6–5 5–3 100 10
=
y x Initial Rate of rice = = ` = per kg.
40 4
1 2 After increasing price by 25%
=
y x 10 æ 25 ö 25
Rate of rice = ´ 1+ =` per kg
4 çè 100 ÷ø 8
x 2
= Amount of rice one can purchase in `110
y 1
100 110 ´ 8
Hence, ratio of work done by 1 man to 1 boy = 2 : 1. = = = 35.2 kg.
25 25
8
Arithmetic B-29

127. (b) Age of captain = 30 years. 132. (d) Let the original salaries of Ravi and Sumit be ` 2x and
Age of wicketkeeper = 31 years. ` 3x respectively.
Let average age of whole team = x years.
then average age of 9 players (excluding captain & 2 x + 4000 40
Then, =
w.k). = (x – 1). 3 x + 4000 57
According to the question. Þ 57(2x + 4000) = 40(3x + 4000) Þ 6x = 68,000
11 × x – 9(x – 1) = (30 + 31). Þ 3x = 34,000
2x + 9 = 61. Sumit’s present salary = (3x + 4000) = `(34000 + 4000)
x = 26 years = ` 38,000.
Average age of the team = 26 years.
128. (c) Work done by A in 5 days. æ 2ö
133. (d) Quantity of milk = ç 60 ´ ÷ litres = 40 litre.
è 3ø
5 1
= = work. Quantity of water in it = (60 – 40) litres = 20 litres.
20 4 New ratio = 1 : 2
1 3 Let quantity of water to be added further be x litres.
Remaining work = 1 – = work.
4 4 æ 40 ö
Then, milk : water = ç ÷
3 è 20 + x ø
Now, time taken by B to finish work = 15 days
4 æ 40 ö 1
Now, ç ÷=
15 ´ 4 è 20 + x ø 2
Time taken by B to finish full work = = 20 days
3 Þ 20 + x = 80 Þ x = 60
Time Require to finish whole work, when A and B work \ Quantity of water to be added = 60 litres.
together 134. (c) Let P = `100. Then, S.I ` = 60 and T = 6 years.
20 ´ 20 æ 100 ´ 60 ö
= = 10 days. \ R= ç ÷ = 10% p.a.
20 + 20 è 100 ´ 6 ø
129. (b) Let C.P. of each article be ` 1 Now, P = ` 12000, T = 3 years & R = 10% p.a
C.P. of x articles = ` x.
S.P. of x articles = ` 20. é ìïæ 10 ö
3 üï ù
ê
\ C.I = ` ê12000 ´ í ç 1 + ÷ - 1 ýú
Profit = ` (20 – x).
ë ï
î è 100 ø ïþ úû
æ 20 - x ö
ç ´ 100 = 25 ÷
è x ø æ 331 ö
= ` ç 12000 ´ ÷ = 3972
Þ 2000 – 100x = 25x è 100 ø
125x = 2000 Þ x = 16 135. (d) Angle traced by the hour hand in 6 hours
130. (b) Let C.P.= ` 100. Then, Profit = ` 320, S.P. = `420. °
New C.P. = 125% of ` 100 = ` 125 æ 360 ö
= ç ´ 6 ÷ = 180°
New S.P. = ` 420. è 12 ø
Profit = ` (420 – 125) = ` 295. 1 æ1 1ö 1 7 1
136. (b) C’s 1 day’s work = -ç + ÷ = - =
æ 295 ö 1475 3 è 6 8 ø 3 24 24
\ Required percentage ç ´100 ÷ % = % = 70%
è 420 ø 21 1 1 1
A’s wages : B’s wages : C’s wages = : :
131. (a) Originally, let the number of seats for Mathematics, 6 8 24
Physics and Biology be 5x, 7x and 8x respectively. = 4 : 3 : 1.
Number of increased seats are (140% of 5x), (150% of
æ 1 ö
7x) and (175% of 8x). \ C’s share (for 3 days) = ` ç 3 ´ ´ 3200 ÷ = `400
è 24 ø
æ 140 ö æ 150 ö æ 175 ö
Þç ´ 5x ÷ , ç ´ 7 x ÷ and ç ´ 8x ÷ 2(l + b) 5
è 100 ø è 100 ø è 100 ø 137. (b) =
b 1
21x 2l + 2b = 5b Þ 3b = 2l
Þ 7 x, and 14 x 2
2 b= l
3
21x Then, Area = 216 cm2
\ The required ratio = 7x : : 14x
2 2
Þ 2 : 3 : 4. Þ l × b = 216 Þ l ´ l = 216
3
l2 = 324 Þ l = 18 cm.
EBD_8177
B-30 Algebra

3 Algebra

1. In an examination a candidate has to get 35% of total marks 10. What percentage of profit should be added in the cost price
to pass. In one paper he gets 62 out of 150 and in the second of an item so as to gain a profit of 33% after allowing 5%
35 out of 150. How many marks should he get out of 200 discount to the customer ? [2011-II]
marks in the third paper to pass ? [2011-I]
(a) 61 (b) 68 (a) 45 (b) 40
(c) 70 (d) 78 (c) 52 (d) 48
2. The salary of A & B together amounts to ` 2000. A spends 11. If the manufacturer gains 10%, the wholesale dealer gains
95% of his salary and B 85% of his salary. If their savings 15% and the retailer gains 25%, find the cost of production
are same what is the salary of A ? [2011-I] of a table. The retail price of table is ` 1265 [2011-II]
(a) ` 750 (b) ` 1250 (a) ` 800 (b) ` 1000
(c) ` 1500 (d) ` 1600 (c) ` 950 (d) ` 1180
3. Out of the 1000 inhabitants of a town, 60% are male of whom 12. A loss of 19% on a shirt gets converted into a profit of 17%
20% are literate. If, amongst all the inhabitants, 25% are when the selling price is increased by ` 162. What is the
literate, then what percentage of the females of the town are cost price of the shirt ? [2011-II]
literate ? [2011-I]
(a) 22.5 (b) 32.5 (a) ` 540 (b) ` 450
(c) 27.5 (d) 37.5 (c) ` 600 (d) ` 360
4. A trader mixes 26 kg of rice at ` 20 per kg with 30 kg rice of 13. In an examination 75% of the total students passed in English
another variety costing ` 36 per kg. If he sells the mixture at and 65% passed in Mathematics, while 15% failed in English
` 30 per kg his profit will be- [2011-I] as well as Mathematics. If a total of 495 candidates who
(a) –7% (b) 5% passed in both exams. Find the total number of students
(c) 8% (d) 10% who appeared in the exam. [2011-II]
5. The difference between the cost price and sale price is ` (a) 850 (b) 900
240. If the profit is 20%, the selling price is [2011-I] (c) 1000 (d) 1050
(a) ` 1200 (b) ` 1440 14. When the price of a product was increased by 15%, the
(c) ` 1800 (d) ` 2440 number of items sold was decreased by 20%. What was the net
6. Samant bought a microwave oven and paid 10% less than effect ? [2011-II]
Maximum Retail Price(MRP). He sold it with 30% profit
on his purchase cost. What percentage of profit did he earn on (a) 10% gain (b) 6% loss
MRP ? [2011-I] (c) 8% loss (d) 4% gain
(a) 17% (b) 20% 15. A mixture of 40 litres of milk and water contains 10% water.
(c) 27% (d) 32% How much water should be added to this mixture so that the
7. ` 800 becomes ` 956 in 3 years at a certain rate of interest. new mixture contains 20% water ? [2011-II]
If the rate of interest is increased by 4% what amount will (a) 4 litres (b) 5 litres
` 800 become in 3 years ? [2011-I] (c) 6.5 litres (d) 7.5 litres
(a) ` 1020 (b) ` 1052 16. A certain sum of money becomes three times of itself
(c) ` 1282 (d) ` 1080 in 20 years at simple interest. In how many years will
8. How much time will it take for an amount of ` 450 to gain the initial sum become double at the same rate of simple
` 81 as interest, if rate of interest is 4.5% p.a on simple interest ? [2011-II]
interest ? [2011-I] (a) 8 (b) 10
(a) 4.5 years (b) 3.5 years
(c) 12 (d) 14
(c) 5 years (d) 4 years
17. Ram borrows ` 8000 at 12% p.a. simple interest and Mohan
9. At what rate of annual simple interest will `10000 double in borrows ` 9100 at 10% p.a. simple interest. In how many years
15 years ? [2011-I] will their borrowed amounts (debt) be equal ? [2011-II]
(a) 5.5% (b) 8% (a) 18 (b) 20
(c) 6.67% (d) 7.25% (c) 22 (d) 24
Algebra B-31

18. Reena took a loan of ` 1200 with simple interest for a certain (a) 9% (b) 8.25%
numbers of years. The number years are same as the interest (c) 9% (d) 9.25%
rate. If she has paid ` 432 as interest at the end of the loan 30. Sixty five pupils from a school entered for an examination
period, what was the rate of interest ? [2011-II] and 80% of them passed. Another school entered 10 more
(a) 3.6 (b) 6 pupils than the first school and four more pupils passed.
(c) 12 (d) None of these The % of pass in the second school was [2012-II]
19. Hari’s income is 20% more than Madhu’s income. Madhu’s
(a) 75% (b) 84%
income is less than Hari’s income by [2012-I]
(a) 15% (b) 16.66 % (c) 72% (d) 74.6%
(c) 20% (d) 22.25% 31. Calculate the amount on ` 1250 for 2 years at 4% per annum.
20. A sum of money lent out at simple interest doubled itself in compounded yearly. [2012-II]
20 years. In how many years will it triple itself ? [2012-I] (a) ` 676 (b) ` 1352
(a) 28 yrs (b) 30 yrs (c) ` 1778 (d) ` 255
(c) 40 yrs (d) 35 yrs
32. A sum of ` 3200 is lent out into two parts, one at 6% and
21. One litre of water is evaporated from 6 litres of a solution
containing 5% salt. The percentage of salt in the remaining another at 4%. If the total annual income is ` 176, find the
solution is [2012-I] money lent at 6%. [2012-II]
(a) 16% (b) 5% (a) ` 2400 (b) ` 800
(c) 4% (d) 6% (c) ` 1600 (d) ` 3200
22. A shopkeeper professes to sell all things at a discount of 33. Srinivasan invests two equal amounts in two banks
10% but increases the selling price of each article by 20%. giving 8% and 12% rate of interest respectively. At the end
His gain on each article is [2012-I] of year the interest earned is ` 1500. Find the sum invested
(a) 6% (b) 8% in each. [2013-I]
(c) 10% (d) 12% (a) ` 8500 (b) ` 15000
23. If the selling price of an article is 4/3rd of its cost price, the (c) ` 7500 (d) ` 17000
profit in transaction is [2012-I] 34. The simple interest accrued on a sum of money at the end of
(a) 16.75% (b) 20.50% four years is 1/5th of its principal. What is the rate of interest
(c) 25.50% (d) 33.33% per annum ? [2013-I]
24. If selling price is doubled, the profit triples. Find the profit (a) 4% (b) 5%
percent. [2012-I] (c) 6% (d) Inadequate data
(a) 66.66 (b) 100 35. A sum of ` 2600 is lent out into two parts, one at 9% and
another at 7%. If the total annual income is ` 206, find the
(c) 105.33 (d) 120
money lent at 7%. [2013-I]
25. Srinivasan invests two equal amounts in two banks giving (a) ` 1400 (b) ` 900
10% and 12% rate of interest respectively. At the end of (c) ` 1600 (d) ` 1200
year the interest earned is ` 1650. Find the sum invested in 36. In an election between two candidates, 70% of the voters
each. [2012-II] cast their votes, out of which 2% of the votes were declared
(a) ` 8500 (b) ` 15000 invalid. A candidate got 7203 votes which was 60% of the
(c) ` 7500 (d) ` 17,000 total valid votes. Find the total number of voters enrolled in
that election. [2013-I]
26. The simple interest on sum of money is 1/9 of the sum. The (a) 18050 (b) 17500
number of years is numerically equal to the rate percent per (c) 17000 (d) 7203
annum. The rate percent per annum is [2012-II] 37. If the price of kerosene be raised by 11%, find by how much
(a) 3.33 (b) 5 percent a house holder must reduce his consumption of
(c) 6.66 (d) 10 kerosene so as not to increase his expenditure ? [2013-I]
27. A sum of ` 10.000 is lent partly at 8% and the remaining at (a) 11% (b) 9.9%
(c) 11.09% (c) 8.25%
10% per annum, If the yearly interest on the average is
38. 75 pupils from a school appeared for an examination and
9.2%. the money lent at 10% is [2012-II] 80% of them passed. Another school entered 10 more pupils
(a) ` 6000 (b) ` 5500 than the first school and five pupils less than the first school
(c) ` 5000 (d) ` 4500 passed. The pass % of in the second school was [2013-I]
28. In an election between two candidates, 60% of the voters (a) 75% (b) 84%
cast their votes, out of which 4% of the votes were declared (c) 72% (d) 64.7%
39. Calculate the amount on ` 1875 for 2 years at 4% per annum,
invalid. A candidate got 7344 votes which were 75% of the
compounded yearly. [2013-I]
total valid votes. Find the total number of votes enrolled in (a) ` 676 (b) ` 776
that election. [2012-II] (c) ` 1778 (d) ` 2028
(a) 1700 (b) 17590 40. If x varies as y and x = 8 when y = 15 then the values of x
(c) 17000 (d) 7344 when y = 10 is [2013-I]
29. If the price of kerosene be raised by 9%, find how much (a) 5 (b) 15/8
percent a house holder must reduce his consumption of (c) 8/15 (d) 16/3
kerosene so that not to increase his expenditure [2012-II]
EBD_8177
B-32 Algebra

41. A sum of ` 10,000 is lent partly at 6% and the remaining at x% of the total number of students. How many boys are
10% p.a. If the yearly interest on the average is 9.2%, the there in the school ? [2014-II]
money lent at 10% is [2013-I] (a) 51 (b) 65
(a) ` 2000 (b) ` 8500 (c) 60 (d) 95
(c) ` 5000 (d) ` 8000 52. 'A' scored 30% marks and failed by 15 marks. 'B' scored 40%
42. An article costs ` 50 presently. The rate of inflation is 300%. marks and obtained 35 marks more than those required to
What will be cost of this article after two years ? [2014-I] pass what is the pass percentage ? [2014-II]
(a) ` 200 (b) ` 600 (a) 33% (b) 40%
(c) ` 800 (d) ` 1000 (c) 34% (d) 48%
53. A banker lent `6000/- at 10% and `5000/- at 12% at the same
43. A dishonest shopkeeper professes to sell his groceries at
time and for same period of time. The banker received `2400
his cost price, but uses a false weight of 900 grams for each
as total interest on both loans. Find the period for which the
kilogram. Find his gain percentage. [2014-I] banker had lent the amount. [2014-II]
(a) 91/9 % (b) 100/9% (a) 3 years 6 months (b) 3 years
(c) 100/11% (d) 95/9% (c) 2 years 6 months (d) 2 years
44. A man purchased a bullock and a cart for ` 1800. He 54. If a sum becomes double in 16 years, how many times will it
sold the bullock at a profit of 20% and the cart at a profit of be in 8 years? [2015-I]
30%. His total profit was 155/6 %. Find the cost price of 1 1
bullock. [2014-I] (a) 1 times (b) 1 times
2 3
(a) ` 650 (b) ` 750 3 1
(c) ` 900 (d) ` 800 (c) 1 items (d) 1 times
4 4
45. If a person repaid ` 22500 after 10 years of borrowing a loan, 55. In how many years will a sum of ` 800 at 10% per annum
at 10% per annum simple interest find out what amount did compounded semi-annually become ` 926.10? [2015-I]
he take as a loan ? [2014-I] 1 1 1 1
(a) 11,225 (b) 11,250 (c) 10,000 (d) 7,500 (a) 1 (b) 1 (c) 2 (d) 2
3 2 3 2
46. A sum of money invested at simple interest triples itself 56. A sell 2 TV sets, one at a loss of 15% and another at a profit
in 8 years. How many times will it become in 20 years of 15%. Find the loss/gain percentage in the overall
time ? [2014-I] transaction? [2015-I]
(a) 8 times (b) 7 times (c) 6 times (d) 9 times (a) 2.25% (b) 3%
47. Two-third of a consignment was sold at a profit of 5% and (c) 4% (d) No profit, no loss
the remainder at a loss of 2% if the total profit was `400, 57. The price of sugar increases by 20% due to the festive
what was the value of the consignment ? [2014-II] season. by what percentage should a family reduce the
(a) `13,000/- (b) `17,000/- consumption of sugar so that there is no change in the
(c) `15,000/- (d) `40,000/- expenditure ? [2015-I]
48. In three annual examinations' of which the aggregate marks 1 2 1
of each was 500, a student secured average marks 45% and (a) 20% (b) 18 % (c) 16 % (d) 16 %
3 3 3
55% in the first and the second yearly examinations 58. A's salary is 20% lower than B's salary, which is 15% lower
respectively. To secure 60% average total marks, it is than C's salary. By how much percent is C's salary more
necessary for him in third yearly examination to secure than A's salary? [2015-I]
___ marks. [2014-II] (a) 44.05% (b) 45.05%
(a) 300 (b) 350 (c) 46.05% (d) 47.05%
(c) 355 (d) 400 1 2 1
49. A towel was 50 cm broad and 100 cm long. When bleached, it 59. If 2x – = 6, then the value of x + is [2018 - I]
was found to have lost 20% of its length and 10% of its
2x 16x 2
breadth. Find the percentage of decrease in area ?[2014-II] 19 17 18 15
(a) 32% (b) 28% (a) (b) (c) (d)
2 2 3 2
(c) 33% (d) 24% 60. x4 + 4 is equal to: [2019 - II]
50. A man deposited a total sum of ` 88400/- in the name of his
(a) (x2 + 2) (x2 – 2) (b) (x2 + 2x + 2) (x2 – 2x + 2)
two sons aged 19 and 17 years so that at the age of 21, both
2
(c) (x + 2) 2 (d) (x – 1) (x + 1) (x + 2) (x – 2)
will get equal amounts. If the money is invested at the rate
of 10% compound interest per annum what are the shares of 61. If A - B = 3 and A2 + B2 = 29, find the value of AB. [2020 - I]
his two sons ? [2014-II] (a) 10 (b) 12 (c) 15 (d) 18
(a) `48200/- (b) `48400
(c) `42600/- (d) `44200 5x 1 æ 1ö
62. If 2
=, then the value of çè x + ÷ø is
51. The sum of the number of boys and girls in a school is 150. 2x + 5x + 1 3 2x
If the number of boys is x, then the number of girls becomes (a) 15 (b) 10 (c) 20 (d) 5
Algebra B-33

ANSWERS & EXPLANATIONS


1. (d) Total marks = 150 + 150 + 200 = 500
æ 800 ´ 10.5 ´ 3ö
35% of 500 = 175 New, S.I = ` çè ÷ø = ` 252
175 = 62 + 35 + x 100
x = 78 \ New amount = 800 + 252 = 1052
2. (c) Let the salary of A and B are x and (2000 – x) resp. P´R´T
According to question 8. (d) SI =
100
x – 95% of x = [(2000 – x – 85% of (2000 – x)]
é 95 ù é 85 ù 450 ´ 4.5 ´ T
x ê1 - = ( 2000 - x ) ê1 - 81 =
ë 100 úû ë 100 úû
100
5x = (2000 – x) 15 100 ´ 81
T= = 4 years
20x = 30000 450 ´ 4.5
x = 1500 9. (c) S.I = 2P – P = P
Hence, the salary of A is ` 1500.
3. (b) Total number of males those are literate = 60% of 20% P × R ×15
P=
of 1000 100
60 20 100
= ´ ´ 1000 = 120 R= = 6.67%
100 100 15
Total number of males in the town = 600 10. (b) Let the C.P = ` 100
Total number of females in the town = 400 The, S.P = `133
Total number of all inhabitants those are literate = 25% Let the marked price ` x
of 1000 = 250 Then, 95% of x = 133
Remaining females those are literates = 250 – 120 = 130 133 ´ 100
x= = 140
130 95
\ Required % = ´ 100 = 32.5%
400 Marked price = 40% above C.P.
4. (b) C. P. of 56 kg rice 11. (a) Let the cost of production of a table = ` x.
= (26 × 20 + 30 × 36) = ` (520 + 1080) = ` 1600 110 115 125
S. P. of 56 kg rice = 56 × 30 = ` 1680 x´ ´ ´ = 1265
100 100 100
80
Profit % = ´ 100 = 5% 1265 ´ 1000000
1600 x= = ` 800
110 ´ 115 ´ 125
5. (b) Profit = S. P – C. P
Profit = 240 12. (b) Let C. P. = x
Loss = 19%
Profit
Profit % = ´ 100 æ 100 –19 ö 81
C.P. then S.P. is ç ÷ x = 100 x
è 100 ø
240
20 = ´ 100 Þ C.P. = 1200 æ 100 + 17 ö 117
C.P. If profit = 17% then S. P. = ç ÷ x = 100 x
Therefore, S.P = 1200 + 240 = ` 1440 è 100 ø
6. (a) Let the original price = `100 According to question
Then, C.P. = ` 90 81 117
æ 130 ö x + 162 = x
S. P. = 130% of 90 = ` ç ´ 90 ÷ 100 100
è 100 ø \ x = 450
= ` 117 13. (b) Let A and B represent the sets of students who passed
\ Required percentage = (117 – 100) in English and Mathematics respectively.
= 17% If 15% of candidates failed in both, then 85% passed at
7. (b) S.I. = ` (956 – 800) = ` 156; least one of the exams.
P = 800, T = 3 yrs. Then, the total number of students passed in one or
æ 100 ´156 ö both subjects
Q R = ç 800 ´ 3 ÷ % = 6.5% = n ( A È B ) = n ( A ) + n ( B) = n ( A Ç B )
è ø
New rate = (6.5 + 4) = 10.5% 0.85= 0.75 +0.65 – n ( A Ç B)
EBD_8177
B-34 Algebra

n ( A Ç B) = 1.40 – 0.85 = 0.55 21. (d) Amount of salt in the solution = 5% of 6 l = 0.3 l
0.55% of number of students = 495 0.3
Percentage of salt in the remaining solution = ´ 100
5
495
\ Number of students = ´ 100 = 900 = 6%
55
22. (b) Let S. P. = 100%
xy Rate increases of 20% on S.P makes it 100% + 20% = 120%
14. (c) Net effect = x + y +
100
æ 120 ö
15 ´ ( -20 ) Later, the 10% discount make it 120% – ç ÷%
= 15 – 20 + = – 5 – 3 = –8 è 10 ø
100
= 120% – 12%
Negative sign indicates that there is a loss of 8%. = 108%
15. (b) Milk contains in mixture = 36 liters. Thus gain is 108 – 100 = 8%
Water contains in mixture = 4 liters 4x
23. (d) Let C. P. = ` x, then S.P. = `
Let ‘x’ be the water added to the mixture. 3
36 80
= æ 4x ö x
4 + x 20 G ain = ` ç - x÷ =
`
36 = 16 + 4x è 3 ø 3
20 = 4x
\ x = 5 litres æx 1 ö
\ G ain % ç ´ ´ 100 ÷ = 33.33%
16. (b) Let Principal = x è3 x ø
Amount = 3x
Simple interest = 2x 24. (b) Let C.P. = ` x, S.P. = ` y
Profit = y - x
2x ´ 100
Rate = = 10% According to question
x ´ 20 Profit = S.P. - C.P.
x ´ 100 3 (y - x) = 2y - x
Now, required time = = 10 years
x ´ 10 3y - 3x = 2y - x
Þ y = 2x
8000 ´12 ´1
17. (c) Simple interest for Ram = = 960 Profit = 2x - x = x
100
æx ö
9100 ´ 10 ´ 1 Profit % = ç ´ 100÷ % = 100%
= 910 èx ø
Simple interest for Mohan = 25. (c) Let ‘x’ be the sum invested in the bank.
100
Let ‘x’ be the years when borrowed amount be equal. According to question
8000 + 960 x = 9100 + 910 x x ´10 ´1 x ´12 ´1
50x = 9100 – 8000 + = 1650
100 100
50x = 1100
x = 22 years 10x + 12x = 165000
18. (b) Let Time = T years and Rate = T% 22x = 165000
x = ` 7500
1200 ´ T ´ R
Then, = 432 1
100 26. (a) Let sum = ` x, S.I. = x
9
432 ´ 100
T2 = = 36 Rate = R%
1200 Time = R yrs.
T=6
19. (b) If Hari’s income is 20% more than Madhu, then 1 x´R´R
x=
Madhu’s income is less than Hari by 9 100
æ 20 ö R2 =
100
= çè ´ 10÷ % = 16.66%
20 + 100 ø 9
20. (c) Let P = x, S. I = x R = 3.33%
SI ´ 100 x ´ 100 8% 10%
Rate = = = 5% 27. (a)
P´T x ´ 20
Now, P = x, S.I = 2x, Rate = 5% 9.2%
SI ´ 100 2x ´ 100
Time = = = 40 yrs
P´R x ´5 0.8% 1.2%
Algebra B-35

The ratio in which money lent is 0.8 : 1.2 or 2 : 3 x´R´4


P´R´T 1
Therefore, money lent at 10% S.I. = Þ x=
100 5 100
3
= ´ 10000 = 6000 100
5 R= = 5%
28. (c) Let the total number of votes enrolled be x. 5´ 4
Then, number of votes cast = 60% of x 35. (a) Let the sum lent at 7% be ` x and that lent at 9% be
Valid votes = 96% of (60% of x) ` (2600 – x).
75% of [96% of (60% of x)] = 7344 Then,
75 96 60 x ´ 7 ´1 ( 2600 - x ) ´ 9 ´1
´ ´ ´ x = 7344 + = 206
100 100 100 100 100
7x + 9 × 2600 – 9x = 20600
7344 ´ 100 ´ 100 ´ 100
x= – 2x = 20600 – 9 × 2600 Þ x = `1400
75 ´ 96 ´ 60 36. (b) Let the total number of votes enrolled be x. Then, number
x = 17000 of votes cast = 70% of valid votes = 98% of (70% of x)
29. (b) If the price of a commodity increases by R%, then the 60% of [98% of 70% of x] = 7203
reduction in consumption. So as not to increase the
70 98 60
expenditure is ´ ´ ´ x = 7203
100 100 100
é R ù 9
ê100 + R ´100ú % = 100 + 9 ´100 7203 ´100 ´100 ´100
ë û x= ; x = 17500
= 8.25% 70 ´ 98 ´ 60
30. (d) Pupils from first school appeared for an examination, 37. (b) If the price of a commodity increases by R%, then the
reduction in consumption so as not to increase the
The no. of passed student is = 80% of 65 = 52 expenditure is
Pupils from second school appeared for an examination,
é R ù 11
The no. of passed student is = 52 + 4 = 56 êë100 + R ´ 100úû % = 100 + 11 ´ 100 = 9.9%
Total pupils in second school = 65 + 10 = 75
38. (d) Pupils from first school passed the examination
56 = 80% of 75 = 60
Pass % of second school = ´ 100 = 74.6%
75 Pupils from second school passed the examination
Time = 55
æ Rate ö Total pupils in second school = 75 + 10 = 85
31. (b) Amount = Principal ç 1 + ÷
è 100 ø 55
2 Pass % of second school = ´ 100 = 64.7%
æ 4 ö 85
= 1250 ç 1 + ÷ T
è 100 ø æ R ö
39. (d) A = P ç1 + ÷
26 26 è 100 ø
= 1250 ´ ´ = ` 1352 Where A = Amount, P = Principal, R = Rate % per annum
25 25
T = Time in years
32. (a) Let the sum lent at 6% be ` x and that lent at 4% be
` (3200 – x). Then, 26 26
A = 1875 ´ ´
25 25
x ´ 6 ´1 ( 3200 - x ) 4 ´1
+ = 176 A = ` 2028.
100 100 40. (d) x = ky
6x + 4 × 3200 – 4x = 17600 8 = 4 k × 15
2x = 17600 – 4 × 3200
2x = 17600 – 12800 = 4800 8 8 16
k= ; x = ky ; x = ´ 10 ; x =
x = ` 2400 15 15 3
33. (c) Let the sum invested be ` x. 41. (d) Let x be the money lent at 10%
x ´ 8 ´ 1 x ´12 ´ 1
+ = 1500
(10000 - x ) ´ 6 + x ´10
920 =
100 100 100
8x + 12x = 150000 92000 = 60000 – 6x + 10x
20x = 150000 Þ x = 7500 32000 = 4x Þ x = ` 8000
1 42. (c) Cost of article = ` 50
34. (b) Let sum = ` x, S.I = x Inflation = 300%
5 2
Time = 4 years, Rate = ? æ 300 ö
After 2 years cost = 50 ç1 + ÷ = 50 × 16 = ` 800
è 100 ø
EBD_8177
B-36 Algebra

1000 – 900 100 100 400 ´150


43. (b) Gain %= ´ 100 = ´ 100 = % x= =15000
900 900 9 4
44. (b) Let CP of bullock = ` x Value of consignment was ` 15,000
48. (d) Let in third yearly examination he Secure x %
x ´ 120
SP = 45 + 55 + x
100 Then, = 60
CP of cart = (1800 – x) 3
P = 30% 100 + x = 180
x = 80
(1800 – x ) ´ 130
SP = To secure 60% average, he has to get 80%
100
æ 155 ö 80
1800 ´ ç100 + 80% of 500 = ´ 500 = 400 marks
è ÷ 100
6 ø
Total SP = = ` 2265 49. (b) Area of towel = l×b =100 cm × 50cm = 5000 cm2
100
Now, length decreased by 20% and breadth decreased
120 x (1800 – x ) ´ 130 by 10%
\ + = 2265 l' = 100 – 20% of 100 = 80cm
100 100
\ x = 750 b' = 50 – 10% of 50 = 45cm
Hence, cost price of bullock = ` 750 New area = l' × b' = 80cm × 45cm = 3600cm2
45. (b) Let P = ` x Change in area = (5000 – 3600) cm2 = 1400 cm 2
1400
x ´ 10 ´ 10 % change in area = ´100 = 28%
SI = =x 5000
100
50. (b) Let son aged 19 years getting `x and son aged 17 years
A = P + SI
getting (88400 – x).
22500 = x + x
At the age of 21, both will get equal amount
2x = 22500
x = 11250 2 4
æ 10 ö æ 10 ö
\ He took `11,250 as a lone. x ç1 + ÷ = (88400 - x) ç1 + 100 ÷
46. (c) Let P = x è 100 ø è ø
A = 3x 121x 121 121
SI = 2x Þ = (88400 - x) ´ ´
100 100 100
T = 8 years
Þ 100 x =88400 ´121–121x
2 x ´ 100 Þ 221x = 88400 × 121
R= = 25%
x´8
88400 ´121
Þ x= = 48400
x ´ 25 ´ 20 221
Now SI = = 5x
100 x = ` 48400
A = 5x + x = 6x 51. (c) If number of boys is x, then number of girls is (150–x)
\ In 6 years it becomes 6 times (150 – x ) = x % of 150
47. (c) Let value of consignment was ` x x 3x 5x
150 – x = ´150 = Þ =150
rd 100 2 2
æ2ö 2x
ç3÷ consignment costs
è ø 3 150 ´ 2
Þ x= = 60
rd
5
æ2ö Number of boys is 60
Selling price of ç ÷ consignment
è3ø 52. (a) Difference in percentage of A and B = (40–30) = 10%
Difference in marks = 50
2x 5 2x 7 Let maximum marks be x
= + ´ = x
3 100 3 10 50 ´100
rd
10% of x = 50 x = = 500
æ 1ö x 2 x 49 10
S.P of ç ÷ consignment = – ´ = x A scored 30% of 500 means 150 marks
è 3ø 3 100 3 150
minimum marks required to pass = 150 + 15 = 165
49 x 7 x 49 x + 105 x 154 x 165
Total S.P = + = = ´100 = 33%
150 10 150 150 Pass % =
500
Profit = S.P – C.P 53. (d) Let time Period be x years
154 x 4x 6000 ´10´ t 5000 ´12 ´ t
400 = – x= + = 2400
150 150 100 100
Algebra B-37

Þ 600t + 600t = 2400 58. (d) A = B – 20% of B = 0.8 B


2400 B = C – 15% of C = 0.85 C
Þt= =2 A = 0.8 × 0.85 C = 0.68 C
1200
Time period is 2 years C-A C - 0.68C 32
´ 100 = ´ 100 = ´ 100 = 47.05%
54. (a) S.I. = 2P – P = P A 0.68C 68
P ´ R ´ 16 1
P= 59. (a) 2x - =6
100 2x
25 1
R= % Þx- = 3 [on dividing by 2]
4 4x
25
P´ ´8 P 1 1
4 = Þ x2 + - 2´ x ´ =9
(S.I) For 8 years = 16x 2 4x
100 2
[On Squaring]
P 3P
Amount = P + = 1 1 19
2 2 Þ x2 + =9+ =
Amount increased by 1½ times. 16x 2 2 2
60. (b) (x4 + 4) = (x2)2 + (2)2+ 2x2 × 2 –(2x)2
2t
æ 10 ö Þ (x2 + 2)2 – (2x)2
ç ÷ Þ (x2+ 2x + 2) (x2 – 2x + 2)
55. (b) 926.1 = 800 ç 1 + 2 ÷ 61. (a) 2AB = (A2 + B2) – (A – B)2
ç 100 ÷
è ø = 29 – 9 = 20
Þ ab = 10.
2t
9261 æ 21ö 5x 1
=ç ÷ 62. (d) =
8000 è 20 ø 2
2x + 5x + 1 3
3 2t Dividing numerator & denominator by 2x
æ 21ö æ 21ö 3 1
çè ÷ø = çè ÷ø Þ t = years or 1 years. 5
20 20 2 2
2 1
56. (d) Let x be the cost price of T.V. =
= 5 1 3
loss = 15% x+ +
then, S.P1 = x – 15% of x = 0.85x 2 2x
Profit = 15% 5
then, S.P2 = x + 15% of x = 1.15x 1
total S.P = 0.85 x + 1.15x = 2x 2 =
= 1 5 3
Profit = 2x – 2x = 0 x+ +
No profit, no loss 2x 2
57. (c) Let x and y be the rate of sugar per Kg and quantity of
æ 1 ö 5 15
sugar. Þ çx+ ÷+ =
è 2x ø 2 2
æ 20 ö
xy = ç x + ´ x ÷ y' æ 1 ö 15 – 5 æ 1 ö
è 100 ø Þ çx + ÷= Þ çx + ÷=5
è 2x ø 2 è 2x ø
6x
xy = y'
5
5 y
y' = y = y -
6 6
100 2
Reduction in consumption = = 16 %
6 3
EBD_8177
B-38 Time, Speed and Distance

Time, Speed and


4 Distance
1. Two typists of varying skills can do a typing job in 6 minutes 9. The speed of a boat in still water is 10 Kmph. If it can travel
if they work together. If the first typist typed alone for 4 26 Km downstream and 14 Km upstream in the same time,
minutes and then the second typist typed alone for 6 minutes, the speed of the stream is [2015-I]
they would be left with 1/5 of the whole work. How many (a) 2 Kmph (b) 2.5 Kmph
minutes would it take the slower typist to complete the typing (c) 3 Kmph (d) 4 Kmph
job working alone ? [2014-I] 10. A man travelled from a point A to B at the rate of 25 Kmph
(a) 10 minutes (b) 15 minutes and walked back at the rate of 4 Kmph. If the whole journey
(c) 12 minutes (d) 20 minutes took 5 hrs 48 minutes, the distance between A and B is
2. A train covers a distance of 10 km in 12 minutes. If its speed [2015-I]
is decreased by 5 kmph, what is the time taken train to cover (a) 30 km (b) 24 km
the same distance ? [2014-II] (c) 20 km (d) 51.6 km
(a) 14 min. 33 sec (b) 13 min. 33 sec 11. A train travelling at a uniform speed clears a platform 200 m
(c) 13 min. 20 sec (d) 15 min. 20 sec long in 10 seconds and passes a telegraph post in 5 seconds.
3. 'A' is thrice as good a workman as 'B' and takes 10 days less The speed of the train is [2015-I]
to do a piece of work than 'B' takes. How many days will 'B' (a) 36 km/h (b) 144 km/h
take to complete if he works alone ? [2014-II] (c) 72 km/h (d) 78 km/h
(a) 21 days (b) 15 days 12. A and B can do a piece of work in 18 days; B and C can do
(c) 18 days (d) 24 days it in 24 days, A and C can do it in 36 days. In how many days
4. 'A' and 'B' can do a piece of work in 30 days while 'B' and 'C' B alone can finish the work? [2015-I]
can do the same work in 24 days and 'C' and 'A' in 20 days. (a) 48 days (b) 45 days
They all work for 10 days and 'B' and 'C' leave. How many 4
days more will 'A' take to finish the work ? [2014-II] (c) 28 days (d) 144 days
(a) 12 days (b) 18 days 5
(c) 20 days (d) 22 days 13. A train covers a distance of 12 km in 10 min. If it takes 6 sec
5. A train is moving at a speed of 132 kmph. If the length of the to pass a telegraph post, the length of the train is [2015-II]
train is 110 meters, how long will it take to cross a railway (a) 90 m (b) 100 m
platform 165 m long ? [2014-II] (c) 120 m (d) 140 m
(a) 6.0 secs (b) 7.5 secs 14. A train can travel 50% faster than a car. Both start from point
A at the same time and reach point B 75 kms away from A at
(c) 7.0 secs (d) 8.5 secs
the same time. On the way, however, the train lost about 12.5
6. Two pipes 'A' and 'B' can fill a tank in 20 and 30 minutes minutes while stopping at the stations. The speed of the car
respectively. If both the pipes are used together, then how is: [2015-II]
long are will it take to fill the tank ? [2014-II] (a) 100 Kmph (b) 110 Kmph
(a) 12 Min (b) 15 Min (c) 120 Kmph (d) 130 Kmph
(c) 25 Min (d) 50 Min 15. A boatman goes 2 km against the current of the stream in 1
7. Speed of a boat in still water is 9 kmph and the speed of the hour and goes 1 km along the current in 10 minutes. How
stream is 1.5 kmph. A man rows to a place at a distance of long will it take to go 5 km in stationary water? [2015-II]
105 kms and comes back to the starting point. What will be (a) 40 minutes (b) 10 hour
the total time taken by him ? [2014-II] (c) 1 hr 15 min (d) 1 hr 30 min
(a) 16 Hrs (b) 18 Hrs 16. A steamer moves with a speed of 4.5 km/h in still water to a
(c) 24 Hrs (d) 28 Hrs certain upstream point and comes back to the starting point
1 in a river which flows at 1.5 km/h. The average speed of
8. A man can row 9 Kmph in still water and finds that it steamer for the total journey is [2015-II]
3
takes him thrice as much time to row up than as to row down (a) 12 km/h (b) 9 km/h
the same distance in the river. The speed of the current is (c) 6 km/h (d) 4 km/h
[2015-I] 17. A train 150 m long is running with a speed of 68 km/h. In
1 1 what time will it pass a man who is running at 8 km/ h in the
(a) 3 Kmph (b) 3 Kmph same direction in which the train is going? [2016-I]
3 9 (a) 8 sec (b) 8.5 sec
2 1 (c) 9 sec (d) 9.5 sec
(c) 4 Kmph (d) 4 Kmph
3 3
Time, Speed and Distance B-39

18. A worker may claim ` 1.5 for each km which he travels by 28. A train passes a man standing on a platform in 10 sec. and
taxi and 50 paise for each km he drives his own car. If in one passes the platform in 22 sec. completely. If the platform is
week he claimed ` 50 for travelling 80 km, how many kms did 150 meters long, how long is the train? [2017 - II]
he travel by taxi? [2016 - II] (a) 125 m (b) 115 m
(a) 20 km (b) 14 km (c) 135 m (d) 145 m
(c) 12 km (d) 10 km 29. On a journey across Kolkata, a taxi averages 50 km per hour
19. A man rows upstream a distance of 9 km or downstream a for 50% of the distance. 40 km per hour for 40% of it and 20
distance of 18 km taking 3 hours each time. The speed of the km per hour for the remaining. The average speed in km/
boat in still water is [2016 - II] hour, for the whole journey is : [2018 - I]
1 1 (a) 42 (b) 40 (c) 35 (d) 45
(a) 7 km/h (b) 6 km/h 30. Two trains 108 m and 112 m in length are running towards
2 2 each other on the parallel lines at a speed of 45 km/hr and 54
1 1 km/hr respectively. To cross each other after they meet, it
(c) 5 km/h (d) 4 km/h will take [2018 - I]
2 2
20. A man can row 30 km upstream and 44 km downstream in 10 (a) 10 sec (b) 12 sec (c) 9 sec (d) 8 sec
hours. He can also row 40 km upstream and 55 km 1
downstream in 13 hours. Find the rate of current, [2016 - II] 31. A boat goes 15 km upstream and 10 km downstream in 3
2
(a) 3 km/h (b) 2 km/h hours 15 minutes. It goes 12 km upstream and 14 km
(c) 4 km/h (d) 5 km/h downstream in 3 hours. What is the speed of the boat in still
21. Two trains travel in opposite directions at 36 km/h and 45 water? [2018 - I]
km/h respectively. A man sitting in slower train passes the (a) 4 (b) 6 (c) 10 (d) 14
faster train in 8 seconds. The length of the faster train is 32. A bus started its journey from Pune and reached Mumbai in
[2016 - II] 44 minutes at its average speed of 50 km/hr. If the average
(a) 80 m (b) 120 m speed of the bus is increased by 5km/hour, how much time
(c) 160 m (d) 180 m will it take to cover the same distance? [2018 - II]
22. A train 110 metres long is running with a speed of 60 km/h.
In what time will it pass a man who is running at 6 km/h in (a) 40 minutes (b) 38minutes
the direction opposite to that of train? [2016 - II] (c) 36 minutes (d) 31minutes
(a) 5 sec (b) 6 sec
33. Car A is travelling at 60 kmph towards northwest creating an
(c) 7 sec (d) 10 sec
23. A man rows 1 km in 10 minutes, along the stream and 1km in angle 420 to north and Car B is travelling towards South
20 minutes against the stream. The speed of the stream is West at 80 kmph creating an angle 480 degree to South. Both
[2017 - I] are started from same point. Find distance between A and B
(a) 1 km/hr (b) 1.5 km/hr after one hour? [2019-I]
(c) 3 km/hr (d) 2.5 km/hr (a) 100 km (b) 120 km
24. The speed of a boat in standing water is 8 km/hr and the (c) 150 km (d) 90 km
speed of the stream is 2km/hr. If boat takes 3hr 12 minutes to
34. Difference between two stations X and Y is 500 km one train
a place and come back, then what is the distance?
starting from X move toward Y with 20 km/h and another
[2017 - I]
train move toward X from Y with the speed 30km/h. What is
(a) 14 km (b) 10 km
the distance of the point where both train cross each other
(c) 24 km (d) 12 km
from point X. [2019-I]
25. A train 110m long running at the speed of 60km/hr, how long
it will take to cross a man running in the opposite direction (a) 400 km (b) 200 km
at the speed if 10km/hr. [2017 - I] (c) 300 km (d) 100 km
(a) 6 seconds (b) 8 seconds 35. Gabbar kidnapped Basanti and placed him a tunnel which
(c) 12 seconds (d) 10 seconds has a length of 500 m. He tied her from 200 m distance from
26. A man whose speed is 4.5 kmph in still water rows to a rarer side. Veeru heroically saved Basanti and at the same
certain upstream point and back to the starting point in a time a train was approaching the tunnel. They ran at a speed
river which flows at 1.5 kmph, find his average speed for the of 10m/s to the nearer end and missed the collision very
total journey? [2017 - II] closely. Find the speed of the train (in km/hr). [2019 - II]
(a) 8 kmph (b) 4 kmph (a) 25 (b) 70
(c) 2 kmph (d) 10 kmph (c) 90 (d) 80
27. A train moving at a cover a rate of 90 kmph. Crosses a tunnel 36. A boat takes 22 hours to go downstream from point A to
in 36 seconds. and other train which is 100 metres shorter point B then to point C (which is midway between A and B),
moving at a cover a rate of 45 kmph. How much time will it find the distance between A and B if speed of boat in still
take to go through the tunnel? [2017 - II] water is 16 km/hr and that of stream is 4 km/hr ? [2019 - II]
(a) 54 seconds (b) 64 seconds (a) 200 km (b) 120 km
(c) 50 seconds (d) 58 seconds (c) 240 km (d) None of these
EBD_8177
B-40 Time, Speed and Distance

37. Alok’s office is 100 km from his home. Once he started late trains was 220km and speed of bird was 80kmph what is the
from his home by 1 hour. He increased his speed by 5kph total distance travelled by bird? [2019 - II]
and reached office on time, find his changed speed. (a) 160 km
[2019 - II] (b) 180 km
(a) 18 kmph (b) 20 kmph (c) 140 km
(c) 24 kmph (d) None of these (d) None of these
38. A man daily goes to temple which is 4km away from his 41. If a person rows a boat at the speed of 8kmph and the speed
home at a speed of 4kph. Once he decided to take his dog of stream is 1.4 kmph then it takes 2 hrs to reach somewhere
with him . The dog reached the temple and returned back to and coming back. If he increased his speed by 25percent
his owner then turn back again and ran towards temple, this how will time be affected? [2019 - II]
goes on and on. Find the distance covered by dog in the (a) 21 % increase
direction of temple if the speed of dog is 8kph. [2019 - II] (b) 21 % decrease
(a) 5.5 km (b) 6 km (c) 18 % increase
(c) 6.5 km (d) None of these (d) None of these
39. A train has 7 compartments and two compartments have 42. A man complete a journey in 10 hours. He travels first half of
not same number of passengers. Average number of the journey at the rate of 21 km/hr and second half at the rate
passengers is 15. And no compartment has number of of 24 km/hr. Find the total journey (in km). [2020 - I]
passengers more than the average number of passenger. (a) 220km (b) 224km
except compartments 7th. Find the least number of (c) 245km (d) 278km
passengers in 7th compartment. [2019 - II] 43. Two trains running in opposite directions cross a man
(a) 30 (b) 32 standing on the platform in 27 seconds and 17 seconds
(c) 35 (d) None of these respectively and they cross each other in 23 seconds. The
40. Two trains start towards each other at same time with speeds ratio of their speeds is? [2020 - I]
50kmph and 60kmph, a bird perched first train and starting (a) 1:3 (b) 3:2
from first train it went to second train and came back to first (c) 2:3 (d) None of these
train it went back to second train. The distance between

ANSWERS & EXPLANATIONS


1. (b) Let first complete the job in x minutes 1 1 1
Second complete the job in y minutes + = ...(2)
y z 24
1 1 1 1 1 1
\ + = + = ...(3)
x y 6 z x 20
...(1) adding equ (1) , (2) and (3)
4 6 1 4
and + = 1 - = æ 1 1 1ö 1 1 1 1 1
x y 5 5 2ç + + ÷ = Þ + + = ...(4)
è x y zø 8 x y z 16
...(2)
By (1) and (2) A,B and C together complete the work in 16 days.
x = 10, y = 15 10 5
Hence slower typist complete the jobs in 15 minutes. In 10 days they completed = Part
16 8
10km 5 3
2. (c) Speed of train = = 50km/h Remaining work = 1– =
12/60 h 8 8
Speed is decreased by 5 kmph then new Speed of the Subtracting equ (2) from (4)
train will be 45 kmph 1 1
10 2 we get, = or x = 48
Time taken to cover 10 km, t = = h x 48
45 9 A alone can finish the Remaining work in
Time taken = 13min. 20 sec 3
´ 48 =18days
3. (b) Let A finish the work in x days 8
B finish same work in 3 x days 5 110
3x – x = 10 Þ x = 5 days 5. (b) Speed of train = 132 kmph = 132 × m/s = m/s
18 3
B finish the work in 3×5 = 15 days
Total distance = 110m + 165m = 275m
4. (b) Let A,B and C individualy complete the work in x,y and 275ms
z days respectively. Time taken to cover the distance = = 7.5s.
110m
1 1 1 275
+ = ...(1) ´ 3 = 7.5 sec.
x y 30 110
Time, Speed and Distance B-41

æ 1 ö
th According to question
6. (a) In one minute A can fill ç ÷ Part x = s × 5 sec. ...(2)
è 20 ø Puting 'x' value in (1) from (2)
th
æ 1 ö (5s + 200) = 105
In one minute B can fill ç ÷ Part 200 = 10s – 5s = 5s
è 30 ø
In one minute A and B together can fill s = 40 m/s
8
1 1 æ1ö
th 40 ´ 18
+ = ç ÷ Parts s= = 144 km/h
20 30 è 12 ø 5
A and B together can fill the tank in 12 minutes None of the option is matching.
7. (c) Speed of upstream = 9 – 1.5 = 7.5 Kmph 1 1 1
Speed of down stream = 9 + 1.5 = 10.5 Kmph 12. (c) + = ...(1)
A B 18
105
Time taken for up stream = =14h 1 1 1
7.5 + = ...(2)
105 B C 24
Time taken for down stream = =10h
10.5 1 1 1
+ = ...(3)
Total time taken = 10 + 14 = 24 hours C A 36
8. (c) Distance covered by man = D Km Adding eqn. (1), (2) and (3)
Speed of Man in still water = x Kmph
æ1 1 1ö 1 1 1
28 2ç + + ÷ = + +
Speed of current = Kmph è A B C ø 18 24 36
3
According to question, 1 1 1 1
+ + = ...(4)
æ ö A B C 16
D ç D ÷ Subtract eqn. (3) from (4)
= 3ç ÷
28 1 1 1 5
-x ç 28 + x ÷ = - =
3 è 3 ø B 16 36 144
28 æ 28 ö 28 144 4
Þ + x = 3 ç - x÷ Þ 4x = 2 ´ B alone can finish work in = 28 days
3 è 3 ø 3 5 5
13. (c) Let length of the train is x.
14 2 Distance = 12 km = 12000 m
Þ x= or 4 Kmph
Time = 10 min = 10 × 60 = 600 sec
3 3
9. (c) Let speed of steam be x Kmph. 12000
Then, Speed of train = = 20 m / sec
600
26 14 when train passes a telegraph post it covers distance
= equal to its length.
10 + x 10 - x
260 – 26x = 140 + 14x using distance = time × speed
40x = 120 x = 6 × 20 = 120 m
x= 3 14. (c) Let speed of the car be x kmph
Speed of steam 3 Kmph. 150 æ3 ö
Then speed of the train = x = ç x÷ kmph
10. (c) Let D Km be the distance between A and B. 100 è2 ø
D D 48 75 75 125
+ =5 \ - =
25 4 60 x (3 / 2) x 10 ´ 60
4D + 25D 29 75 50 5
- Þ - =
100 5 x x 24
29D 29
= æ 25 ´ 24 ö
Þx=ç = 120 kmph
100 5 è 5 ÷ø
D = 20 Km
11. (b) Let 'x' be the length of the train. æ1 ö
15. (c) Rate downstream = çè ´ 60÷ø km/hr = 6km/hr.
Let 's' be the speed of train. 10
Distance travelled by train to cross the platform Rate upstream =2 km/hr.
= ( x + 200)m 1
According to question Speed in still water = (6 + 2) km/ hr = 4 k/ hr .
2
(x + 200) = s × 10 sec. ...(1)
æ 5ö 1
Distance travelled by train to cross he telegraph post \ Required time = çè ÷ø hrs = 1 hrs = 1hr15min .
=x m 4 4
EBD_8177
B-42 Time, Speed and Distance
16. (d) Speed of steamer in down stream = 4.5 + 1.5 = 6km Distance travelled by train while passing man = length
speed of steamer in up stream = 4.5 – 1.5 =3km of train.
let the distance be x. 45
time taken in down stream = x / 6 Length of train = ´ 8 < 180m Q (D = ST)
time taken in up stream = x / 3 2
22. (b) Relative speed of train for man = speed of train + speed
total distance of man = (60 + 6) km/hr
Avg. speed =
total time æ 5 ö÷ 55
= ççç66 ´ ÷÷ m / sec = m / sec
x+x 2x è 18 ø 3
Þ = 4 km/h
x x
+ ( 3x / 6 ) Distance travelled by train while crossing man = length
6 3 of train.
17. (c) Net relative speed = (68 – 8) km/h Length of train = 110 metres
D 110
= 60 km/h = 60 ´ m/s =
5 50
m/s Time taken = = ´ 3 < 6 sec
18 3 S 55
150 1
Time = = 9 sec. 23. (b) Speed of current = (Rate of downstream – Rate of
50 2
upstream)
3
18. (d) Let Distance travelled by taxi x km and by our car be 1 1
y km = × (6 – 3) = × 3 = 1.5 km/hr
2 2
ATQ 24. (d) According to question
x + y = 80 ......(i)
1.5x + .5y = 50 .....(ii) d d 12
+ =3
Multiplying Eq(i) by .5 and subtracting Eq(ii) from (i) (8 + 2) (8 - 2) 60
x = 10
Distance travelled by taxi is 10 Km d d 1
+ =3
10 6 5
9
19. (d) Speed of boat upstream = = 3km/hr 3d + 5d 16
3 =
30 5
18
Speed of boat down stream = = 6 km/hr 8d 16 16 ´ 30
3 = \d= = 12km
Let speed of boat be = x km/hr 30 5 8´ 5
Let speed of stream be = y km/hr 25. (a) Distance = 110m
ATQ Relative speed = 60 + 10 = 70 kmph (since both the train
x+y= 6 and the man are moving in opposite direction)
x–y=3 5 350
By subtracting = 70 ´ = m / sec
2x = 9 18 18
\ Time taken to pass the man
9 1
x = or 4 km / hr 18
2 2 = 110 ´ = 5.65 @ 6 seconds
20. (a) Let rate of upstream = x kmph and rate of downstream 350
= y kmph 26. (b) Here, Speed of man = 4.5 km/hr
Then Speed of current = 1.5 km/hr
\ Speed of man in downstream = (4.5 + 1.5) = 6km/hr
30 44 Speed of man in upstream = (4.5 – 1.5) = 3km/hr
∗ < 10
x y 2´6´3 2´6´3
40 55
\ Average speed = (6 + 3) = = 4 km/hr
9
and ∗ < 13
x y 5
On solving we get x = 5, y = 11 27. (b) Speed of 1st train = 90 km/hr = 90 ´ = 25 m/sec
18
So, rate of upstream = 5Km/hr and rate of downstream Time taken by 1st train = 36 seconds
= 11Km/hr \ Distance = speed × time = 25 × 36 = 900m
1 Now
Rate of current = Ζ11 – 5∴ < 3 km / hr 2nd train is 100 meters shorter, then
2
21. (d) Relative speed of both trains = (36 + 45) km/hr \ Distance (900 – 100) = 800m
5
Speed of 2nd train = 45 ´ = 12.5 m/sec
æ 5ö 18
= ççç81´ ÷÷÷ m / sec =
45
m / sec
è 18 ø 2 800
\ Time = = 64 seconds.
12.5
Time, Speed and Distance B-43

28. (a) According to question Distance between A & B after one hour
Let the length of train = x meter.
Therefore, = 602 + 802 = 3600 + 6400 = 10000 = 100 km.
x + 150 x
=
22 10 34. (b) 500
12x = 1500 X Y
O
1500
x= = 125 meters. D 500 – D
12
\So, length of the train = 125 meters. Suppose they meet at point 'O' at the same time
29. (b) Total distance = 100 km.
50 40 10 1 5 D ( 500 - D )
Total time = + + = 1 + 1 + = hours Time = =
50 40 20 2 2 20 30
100 ´ 2 Þ 30D = (500 – D) × 20
\ Average speed = = 40 kmph
5 Þ 50D = 500 × 20
99 ´ 5 D = 200 km
30. (d) Relative speed = ( 45 + 54) = 99km / hr = m / sec
18 Both the train meet 200 km from point 'X'.
Distance covered in crossing each other
= (108+112) = 220m Train Basanti 200 m
35. (c)
220 18 Position
Required time = ´ = 8sec
99 5 500 m
31. (c) Let speed of the boat in still water be x km/h and speed Speed with which Basanti and Veeru ran away
of current be y km/h. = 10 m/s
Then,
upstream speed = (x – y) km/h 200
Time to cover 200 m distance = = 20sec.
and down stream speed = (x + y) km/h 10
Now, Distance covered by the train in 20 sec.
15 21 1 = 500 m.
+ =3 ...(i)
( x - y) 2 ( x + y ) 4 500
Speed of the train = = 25m / sec.
12 14 20
+ =3 ...(ii)
( x - y) ( x + y ) = 25 ´
18
= 90km / hr.
From Equation (i) and (ii) 5
x = 10 km/hr and y = 4 km/hr. 36. (c) Let the distance from point A to point B is d km
32. (a) Distance = (44/60) × 50 = (x/60) ×55 downstream velocity = 16 + 4 = 20 kmph.
\ x= 40 minutes upstream velocity = 16 – 4 = 12 kmph.
total time of travell = 22 hours.
d d/2 d d
A \ + = 22 Þ + = 22
33. (a) N 20 12 20 24
60k
m (6 + 5)d
42°
= 22.
W E 120
90°
22 ´ 120
S d= = 240 km
48° 11
km 37. (d) Let Alok's changed Speed is S kmph.
80 then, Alok's normal Speed is (S – 5) Kmph.
B According to the question,
100 100
– = 1.
Distance travelled by A in one hour = 60 km (S – 5) (S)
Distance travelled by B in one hour = 80 km 100 (S – S + 5) = S (S – 5)
Angle between them = 90° 500 = S2 – 5S
S2 – 5 S – 500 = 0
Using Pythagoras theorm.
(S – 25) (S + 20) = 0
\ changed speed S = 25 Kmph.
EBD_8177
B-44 Time, Speed and Distance

38. (b) Home •_______________• Temple It the speed of the bird is b kmph, then distance
Time taken by dog to reach the temple b.d
travelled by the bird = .
4 1 (u + v)
= = hour.
8 2 Here, distance travelled by the bird
1 80 ´ 220 80 ´ 220
Distance Covered by man in hr. = = = 160 km.(Approx)
2 (50 + 60) 110
1 41. (b) Let the person row a boat to a distance d km.
= 4´ = 2 km.
2 d d
then, + = 2.
Now, Let the dog meets man after t hour (8 + 1.4) 8 – 1.4)
then, 8 × t + 4 × t = (4 – 2).
12t = 2 d d
+ = 2.
1 9.4 6.6
t= hour
6 d(3.3 + 4.7)
Distance covered by the dog to reach the temple after = 2.
31.02
1 4
first meeting = 8 ´ = km. 31.08 ´ 2
6 3 d= = 7.755 km
8
Similarly, distance covered by the dog to reach the
Now, when speed is increases by 25%
temple after second meeting = 4 ´ 1 = 4 km.
3 3 9 new speed = 8 + 8 ´ 25 = 10 kmph.
100
4 1 4 Now, total time taken to come back the journey
after third meeting = ´ = km.
9 3 27
......................so on. 7.755 7.755
= +
total distance covered by the dog towards the temple (10 + 1.4) (10 – 1.4)
4 4 4 7.755 7.755
= 4+ + + + ........¥. = + = 0.68 + 0.90 = 1.58 hours.
3 9 27 11.4 8.6
4 4 4´3 æ 2 –1.58 ö
= = = = 6 km.
1 2 2 Percentage decrease in time = ç ÷ ´ 100
1– è 2 ø
3 3
= 21%
39. (a) Total number of Compartments = 7.
Average number of passengers in each (1 / 2) x (1 / 2) x
42 (b) + = 10
compartments = 15 21 24
total number of passenger in 7 compartment
x x
= 15 × 7 = 105. Þ + = 20
For the least number of passenger in 7 th compartment, 21 24
we have maximum number of passenger in first 6 Þ 15x = 168 × 20
compartments. æ 168 ´ 20 ö
Maximum passenger in first compartment = 15. Þx= ç ÷ = 224 km.
è 15 ø
Maximum passenger in second compartment = 14.
Maximum passenger in third compartment = 13. 43. (b) Let the speeds of the two trains be x m/sec and y m/sec
Maximum passenger in fourth compartment = 12. respectively.
Maximum passenger in fifth compartment = 11. Then, length of the first train = 27x metres, and length
Maximum passenger in sixth compartment = 10. of the second train = 17y metres.
total number of passengers in 6 compartment = 75. 27 x + 17 y
\ Min. number of passenger in 7th compartment \ = 23
x+ y
= 105 – 75 = 30.
40. (a) Let the speed first and Second train is u kmph and Þ 27x + 17y = 23x + 23y
v kmph and two trains are d km apart. Þ 4x = 6y

d x 3
Þ =
then time when two train collide = . y 2
(u + v)
Section-C : Reasoning & Military Aptitude

Analogy &
1 Odd One Out
1. BOOK is to CHAPTER as BUILDING is to [2011-I] 13. GRAIN : SALT [2011-II]
(a) ELEVATOR (b) LOBBY (a) Shard : Pottery (b) Shred : Wood
(c) ROOF (d) STOREY (c) Blades : Grass (d) Chips : Glass
2. CARROT is to VEGETABLE as [2011-I] 14. WAITER : TIP [2011-II]
(a) DOGWOOD is to OAK (b) FOOT is to PAW (a) Student : Marks (b) Worker : Bonus
(c) PEPPER is to SPICE (d) SHEEP is to LAMB (c) Employee : Wages (d) Clerk : Bribe
3. CONCAVE is to CONVEX as [2011-I] 15. PAIN : SEDATIVE [2011-II]
(a) CAVITY is to MOUND (b) HILL is to HOLE (a) Comfort : Stimulant (b) Grief : Consolation
(c) OVAL is to OBLONG (d) ROUND is to POINTED (c) Trance : Narcotic (d) Ache : Extraction
4. GOWN is to GARMENT as GASOLINE is to [2011-I] 16. SILENCE : NOISE [2011-II]
(a) COOLANT (b) FUEL (a) Quiet : Peace (b) Baldness : Hair
(c) OIL (d) LUBRICANT (c) Talk : Whisper (d) Singer : Dance
5. HYPER- is to HYPO- as [2011-I] 17. WAN : COLOUR [2011-II]
(a) DIASTOLIC is to SYSTOLIC (a) Corpulent : Weight (b) Insipid : Flavour
(b) OVER is to UNDER (c) Pallid : Complexion (d) Enigmatic : Puzzle
(c) SMALL is to LARGE 18. PORK : PIG [2011-II]
(d) STALE is to FRESH (a) Rooster : Chicken (b) Mutton : Sheep
6. IMMIGRATION is to EMIGRATION as [2011-I] (c) Steer : Beef (d) Lobster : Crustacean
(a) ARRIVAL is to DEPARTURE 19. AFTER : BEFORE [2011-II]
(b) FLIGHT is to VOYAGE (a) First : Second (b) Present : Past
(c) LEGAL is to ILLEGAL (c) Contemporary : Historic (d) Successor :
(d) MIGRATION is to TRAVEL Predecessor
7. OCTAGON is to SQUARE as HEXAGON is to [2011-I] 20. EAST : ORIENT [2011-II]
(a) POLYGON (b) PYRAMID (a) North : Polar (b) South : Capricorn
(c) RECTANGLE (d) TRIANGLE (c) West : Indian (d) West : Occident
8. TELL is to TOLD as
21. DISTANCE : MILE [2011-II]
(a) RIDE is to RODE (b) SINK is to SANK
(a) Liquid : Litre (b) Bushel : Corn
(c) WEAVE is to WOVE (d) WEEP is to WEPT
(c) Weight : Scale (d) Fame : Television
9. SHEEP is to LAMB as HORSE is to [2011-I]
22. TEN : DECIMAL [2011-II]
(a) COLT (b) DOE
(a) Seven : Seplet (b) Four : Quartet
(c) FAWN (d) MARE
(c) Two : Binary (d) Five : Quince
10. IGNORE is to OVERLOOK as [2011-I]
23. MUNDANE : SPIRITUAL [2011-II]
(a) AGREE is to CONSENT (b) CLIMB is to WALK
(a) Common : Ghostly (b) Worldly : Unworldly
(c) DULL is to SHARPEN (d) LEARNis toREMEMBER
(c) Routine : Novel (d) Secular : Clerical
11. FREQUENTLY is to SELDOM as [2011-I]
(a) ALWAYS is to NEVER 24. LAWYER : COURT [2011-II]
(b) EVERYBODY is to EVERYONE (a) Businessman : Market (b) Chemist : Laboratory
(c) GENERALLY is to USUALLY (c) Labourer : Factory (d) Athelete : Olympics
(d) OCCASIONALLY is to CONSTANTLY 25. ARMY : LOGISTICS [2011-II]
(a) Business : Strategy (b) Soldier : Students
DIRECTIONS (Qs. 12-33) : Each question consists of two words (c) War : Logic (d) Team : Individual
which have a certain relationship to each other, followed by 26. GRAVITY : PULL [2011-II]
four pairs of related words. Select the pair which has the same (a) Iron : Metal (b) North pole : Directions
relationship. (c) Magnetism : Attraction (d) Dust : Desert
12. STORY : NOVEL [2011-II] 27. FILTER : WATER [2011-II]
(a) Sea : Ocean (b) School : University (a) Curtail : Activity (b) Expunge : Book
(c) Book : Dictionary (d) Poetry : Drama (c) Edit : Text (d) Censor : Play
EBD_8177
C-2 Analogy & Odd One Out

28. HOPE : ASPIRES [2011-II] 46. Plant : Flower


(a) Love : Elevates (b) Film : Flam (a) Face : Eye (b) Stem : Tree
(c) Fib : Lie (d) Fake : Ordinary (c) Chair : Sofa (d) Blades : Grass
29. SADIEST : PAIN [2011-II] 47. Protoplasm : Cell
(a) Killer : Death (b) Teacher : Pupil (a) Fibre : Plastic (b) Coin : Money
(c) Injury : Bandage (d) Alcohol : Dipsomaniac (c) Chemistry : Elements (d) Chain : Link
30. SYMPHONY : COMPOSER [2011-II] 48. Nalanda : Takshshila
(a) Leonardo : Music (b) Fersco : Painter (a) Venus : Mars (b) University : College
(c) Colours : Pallet (d) Art : Appreciation (c) Office : Department (d) Ship : Cargo
31. CURATOR : MUSEUM [2011-II] DIRECTIONS (Qs. 49-53) : In the following questions the first
(a) Wit : Wisdom (b) Bank : Teller word is related to the second in the same way as the third word
(c) Manager : Office (d) Doctor : Patient is related to the fourth. In the given problems either the third or
32. Which word in each set of four is the odd one out ? [2011-II] the fourth word is missing and is left blank. You have to choose
(a) Look (b) See the correct word to fill in the blank. [2012-I]
(c) Watch (d) Face 49. Foot is to Man as Hoof is to _______.
33. This test requires you to identify the relationship between (a) Dog (b) Cow
two words. Shoe is to foot as sock is to [2011-II] (c) Cat (d) Rabbit
(a) wind (b) hand 50. Broad is to Narrow as ________ is to Lane.
(c) foot (d) leg (a) Footpath (b) Field
(c) Pavement (d) Road
DIRECTIONS (Qs. 34-38) : Find the odd one out. [2012-I]
51. Back is to Backbone as Belly is to _________.
34. (a) Advice (b) Counsel (a) Throat (b) Ribs
(c) Direct (d) Suggest (c) Heart (d) Navel
35. (a) Tumble (b) Topple 52. Ankle is to Knee as Wrist is to _____________.
(c) Crumble (d) Sprain (a) Elbow (b) Finger
36. (a) Sobriquet (b) Alias (c) Hand (d) Foot
(c) Pseudonym (d) Anonymous 53. Sting is to Bee as ______ is to Snake.
37. (a) Mumbai (b) Goa (a) Slithering (b) Rats
(c) Visakhapatnam (d) Thiruvananthapuram (c) Poison (d) Fangs
38. (a) Petrol (b) Acetone DIRECTIONS (Qs 54-63) : In the following questions the words
(c) Mercury (d) Kerosene qiven bear a certain relationship. Find out from the choices the
DIRECTIONS (Qs. 39-48) : In the following questions the words words with the same relationship. [2012-II]
given bear a certain relationship. Your task is to find out from 54. Surgeon : Scalpet
the choices the words with the same relationship. [2012-I] (a) Musician : Instrument (b) Sculptor : Chisel
39. Hope : Despair (c) Carpenter : Cabinet (d) Baker : Oven
(a) Work : Failure (b) Worship : Adore 55. Creche : Infants
(a) School : Pupils (b) Deck : Sailors
(c) Cow : Milk (d) Encourage : Dishearten
(c) Cottage : Guests (d) Aircraft : Crew
40. Army : Logistics
56. Pesticide : Plant
(a) War : Logic (b) Soldiers : Students
(a) Injection : Disease (b) Teacher : Student
(c) Business : Strategy (d) Team : Individual (c) Medicine : Cure (d) Vaccination : Body
41. Bouquet : Flower 57. Stare : Glance
(a) Skin : Body (b) Chain : Link (a) Gulp : Sip (b) Story : tell
(c) Product : Factory (d) Page : Book (c) Hunt : Stalk (d) Step : Walk
42. Revenge : Vendetta 58. Fish : Aquarium
(a) Sleep : Dream (b) Sun : Moon (a) Student : Hostel (b) Bird : Forest
(c) Envy : Jealousy (d) Heaven : God (c) Goods : Consignment (d) Bee : Apiary
43. Refine : Style 59. Ecstasy : Pleasure
(a) Retouch : Photograph (b) Paint : Wall (a) Hatred : Affection (b) Rage : Anger
(c) Compose : Song (d) Author : Book (c) Joy : Grief (d) Mumble : Speak
44. Fear : Tremble 60. Necromancy : Ghosts
(a) Hand : Shake (b) Heat : Perspire (a) Romance : Stories (b) Magic : Amulets
(c) Distance : Walk (d) Evening : Star (c) Alchemy : Gold (d) Sorcery : Spirit
45. Condone : Offence 61. Coin : Mint
(a) Punish : Criminal (b) Mitigate : Penitence (a) Grain : Field (b) Hay : Stable
(c) Overlook : Aberration (d) Ignore : Loyalty (c) Wine : Brewery (d) Book : Publisher
Analogy & Odd One Out C-3

62. Oak : Coniferous 79. Liquor : Drink


(a) Chimpanzee : Ape (b) Animals : Carnivore (a) Bread : Butter (b) Tea : Beverage
(c) Fish : Sea (d) Tree : Grove (c) Sniff : Inhale (d) Water : Sip
63. Onam: Kerala 80. Evaporation : Cloud
(a) Christmas : Christians (b) Bhangra : Punjab (a) Mountain : Snow (b) Book : Pages
(c) Kathak : Uttar Pradesh (d) Bihu : Assam (c) Pressure : Atmosphere (d) Tension : Breakdown
81. Barrel : Vial
DIRECTIONS (Qs. 64-68) : In each of the following questions, a (a) Book : Pamphlet (b) Book : Reader
group of three inter related words is given. Choose a word from the (c) Brochure : Complier (d) Length : Height
given alternatives, that belongs to the same group. [2012-II] 82. Cream : Cosmetics
64. Calendar : Dates : : Dictionary : ? (a) Tiger : Forest (b) Mountain : Valley
(a) Vocabulary (b) Language (c) Magazine : Editor (d) Teak : Wood
(c) Words (d) Book 83. Carnivore : Herbivore
65. Heed : Neglect : : Pacify : ? (a) Flesh : Plant (b) Horse : Lion
(c) Camel : Giraffe (d) Animal : Bird
(a) Incite (b) Allay
(c) War (d) Victory DIRECTIONS (Qs. 84 - 93) : Three words in bold letters are
66. Malaria : Disease : : Sword : ? given in each question, which have something in common among
(a) Wound (b) Spear themselves. Out of the four given alternatives, choose the most
(c) Weapon (d) Rifle appropriate description about these three words. [2013-I]
67. Eye : Wink : : Heart: ? 84. Hamlet : Macbeth : Faustus
(a) Move (b) Throb (a) They are princes
(c) Pump (d) Quiver (b) They are plays by Shakespear
68. Earth : Sun : : Moon : ? (c) They are characters from various dramas
(d) They were romantic hereoes.
(a) Orbit (b) Sky
85. Vesuvius : Etna : Kilimanjaro
(c) Star (d) Earth (a) These are sites of volcanoes
DIRECTIONS (Qs. 69-73) : Choose the word which is least like (b) These are island countries.
the other words in the group. [2012-II] (c) These are hills of Italy.
69. (a) Rose (b) Lotus (d) These lie in polar region.
(c) Marigold (d) Tulip 86. Knot : Watt : Fathorn
(a) The terms are used by sailors
70. (a) Book (b) Sharpener
(b) The terms are used for installing electricity
(c) Pencil (d) Paper
(c) The terms are connected with rope
71. (a) Copper (b) Tin (d) They are units of measurement.
(c) Brass (d) Zinc 87. Barauni : Digboi : Ankleshwar
72. (a) Kiwi (b) Eagle (a) They are famous for oil fields.
(c) Emu (d) Penguin (b) They are famous for religious places.
73. (a) Raniganj (b) Jharia (c) They are tourist places of South India.
(c) Bokaro (d) Baroda (d) They are famous for handlooms.
DIRECTIONS (Qs. 74 -78) : Choose the word which is least like 88. Knight : Rook : Bishop
the other words in the group. [2013-I] (a) These are missionaries (b) They are churchmen.
(c) These are chessmen. (d) These are ranks of military.
74. (a) Garo (b) Khasi 89. Abhi Bhattacharya : Utpal Dutt : Satyajit Ray
(c) Kangra (d) Jayantia (a) They are character actors.
75. (a) Virgo (b) Pisces (b) They are directors of Bengali movies.
(c) Cancer (d) Orion (c) They are famous poets and writers.
76. (a) Shoulder (b) Foot (d) These personalities belong to Bengali
(c) Finger (d) Elbow 90. Voodoo : Sorcery : Necromancy
77. (a) Canoe (b) Dingy (a) They are ancient arts found in sculptures.
(c) Yatch (d) Igloo (b) They are terms connected with black magic.
78. (a) Inn (b) Club (c) They are ancient scripts.
(c) Motel (d) Hostel (d) They are means of communication of pre-historic age.
91. Rourkela : Bokaro : Durgapur
DIRECTIONS (Qs. 79 - 83) : The following questions consist of (a) They are steel plants.
two words that have a certain relationship to each other, (b) They have coal mines.
followed by four alternatives. Select the best alternative that has (c) They have atomic plants
the same relationship as the original pair of words. [2013-I] (d) They are on the sea coast.
EBD_8177
C-4 Analogy & Odd One Out

92. Spinach : Fenugreek : Celery 104. Firm : Flabby : : Piquant : ?


(a) These are cactus plants (a) Salty (b) Pleasant
(b) These are wild plants (c) Bland (d) Smell
(c) These are wild flowers. 105. Funk : Vitamins : : Curie : ?
(d) These are leafy vegetables. (a) Uranium (b) Radioactivity
93. Yeats : Ghalib : Kabir (c) Photography (d) Radium
(a) They were social reformers. 106. Contamination : Food : : Infection : ?
(b) They were famous poets. (a) Diseases (b) Body
(c) They were saints. (c) Germs (d) Microbes
(d) They were yoga instructors DIRECTIONS (Qs. 107 - 111) : In each of the following questions
DIRECTIONS (Qs. 94 -96) : Choose the most appropriate word: find out the alternative which will replace the question mark.
[2014-I] [2014-II]
94. ‘RUSTLE’ is to ‘LEAVES’ as ‘PATTER’ is to________ 107. Carbon : Diamond Corundum : ...... ? .....
(a) Snow (b) Wind (a) Garnet (b) Ruby
(c) Rain (d) Storm (c) Pukhraj (d) Pearl
95. ‘INDISCREET’ is to ‘IMPRUDENT’ as ‘INDISPOSED’ is to 108. Architect Building Sculptor : ...... ? .....
_______ (a) Museum (b) Stone
(a) Concerned (b) Reluctant (c) Chisel (d) Statue
(c) Crucial (d) Clear 109. Eye : Myopia Teeth ..... ?.......
96. ‘ACCIDENT’ is to ‘CAREFULNESS’ as ‘DISEASE’ is to (a) Pyorrhoea (b) Cataract
_________ (c) Trachoma (d) Eczema
(a) Sanitation (b) Treatment 110. Conference : Chairman Newspaper : ..... ?......
(c) Medicine (d) Doctor (a) Reporter (b) Distributor
DIRECTIONS (Qs. 97 & 98) : Choose the word which is least (c) Printer (d) Editor
like the other words in the group. [2014-I] 111. Safe : Secure Protect ..... ?......
(a) Lock (b) Sure
97. (a) Tsangpo (b) Hazaribagh
(c) Guard (d) Conserve
(c) Kanha (d) Bandipur
98. (a) Wheat (b) Rice DIRECTIONS (Qs. 112 -116) : In each of the following questions,
(c) Mustard (d) Gram five words have been given out of which four are alike in some
manner, while the different fifth one is Choose the word which is
DIRECTIONS (Qs. 99 - 103) : The following questions consist of
different from the rest. [2014-II]
two words that have a certain relationship to each other, followed
by four alternatives. Select the best alternative that has same 112. (a) Potassium (b) Silicon
relationship as the original pair of words. [2014-I] (c) Zirconium (d) Gallium
113. (a) Tea (b) Cinchona
99. Creche : Infants
(c) Rubber (d) Chalk
(a) School : Pupils (b) Deck : Sailors
114. (a) Hanger (b) Platform
(c) Cottage : Guests (d) Aircraft : Crew
(c) Dock (d) Park
100. Pesticide : Plant
115. (a) Deck (b) Quay
(a) Injection : Disease (b) Vaccination : Body
(c) Stern (d) Bow
(c) Medicine : Cure (d) Teacher : Student
116. (a) Tall (b) Huge
101. Stare : Glance
(c) Thin (d) Sharp
(a) Gulp : Sip (b) Confide : tell
(c) Hunt : Stalk (d) Step : Walk DIRECTIONS (Qs. 117 - 121) : In each of the following questions
102. Ecstasy : Pleasure find out the alternative which will replace the question mark.
(a) Hatred : Affection (b) Rage : Anger [2015-I]
(c) Joy : Grief (d) Mumble : Speak 117. East : Orient :: ? : ?
103. Necromancy : Ghosts (a) North : Polar (b) North : Tropic
(a) Romance : Stories (b) Magic : Amulets (c) South : Capricorn (d) West : Occident
(c) Alchemy : Gold (d) Sorcery : Spirit 118. ignominy : Disloyalty :: ? : ?
DIRECTIONS (Qs. 104 -106) : There is certain relationship (a) Dealth : Victory (b) Martyr : Man
between two given words on one side of : : and one word is given on (c) Fame : Heroism (d) Destruction : Victory
another side of : : while another word is to be found from the given 119. Loath : Coercion :: ? : ?
alternatives, having the same relation with this word as the given (a) Detest : Caressing (b) Irritate : Caressing
pair has. Select the best alternatives/ relationship. [2014-I] (c) Irate : Antagonism (d) Reluctant : Persuasion
Analogy & Odd One Out C-5

120. Trilogy : Novel :: ? : ? DIRECTIONS (Qs. 137-141): In each of the following questions,
(a) Rice : Husk (b) Milk : Cream find out the alternative which will replace the question mark.
(c) Serial : Episode (d) Gun : Cartridge [2016 -I]
121. Wife : Marriage :: ? : ?
137. Cerebrum : Brain :: ? : ?
(a) Bank : Money (b) Nationality : Citizenship
(a) Aorta : Hand (b) Ligament : Blood
(c) Service : Qualification (d) Attendance : Register
(c) Ventricle : Heart (d) Country : Universe
DIRECTIONS (Qs. 122- 126) : In each of the following questions, 138. Wife : Marriage :: ? : ?
four words have been given out of which three are alike in some (a) Bank : Money (b) Nationality : Citizenship
manner, while the fourth one is different. Choose the word which (c) Service : Qualification (d) Attendance : Register
is different from the rest. [2015-I] 139. Sadist : Injury :: ? : ?
122. (a) Othello (b) King Lear (a) Opportunist : Generosity (b) Priest : Church
(c) Oliver Twist (d) Macbeth (c) Dentist : Teeth (d) Thief : Robbery
123. (a) Nimitz (b) Yamamoto 140. Lecherous : Carnal :: ? : ?
(c) Nelson (d) Montgomery (a) Virile : Feeble (b) Diatribe : Abuse
124. (a) Blaze (b) Glint (c) Impede : Begin (d) Sunder : Link
(c) Simmer (d) Shimmer 141. lgnominy : Disloyalty :: ? : ?
125. (a) Aravalli hills (b) Shivalik hills (a) Death : Victory (b) Martyr : Man
(c) Mole hills (d) Satpura hills (c) Fame : Heroism (d) Destruction : Victory
126. (a) Beaver (b) Alpaca DIRECTIONS (Qs. 142-146) : In each of the following questions,
(c) Walrus (d) Koala four words have been given out of which three are alike in some
DIRECTIONS (Qs. 127-131) : In each of the following questions manner. while the fourth one is different. Choose the word which
find out the alternative which will replace the question mark. is different from the rest. [2016 -I]
[2015- II] 142. (a) Moth (b) Bee
127. Zenith : Nadir :: ? : ? (c) Lizard (d) Aphid
(a) Indigent : Poverty (b) Zeal : Eagerness 143. (a) Tomato (b) Gourd
(c) Success : Peak (d) Genuine : Phony (c) Brinjal (d) Potato
128. Graceful : Clumsy :: ? : ? 144. (a) Gold (b) Silver
(a) Horror : Sympathy (b) Laugh : Cry (c) Bronze (d) Iron
(c) Recapitulate : Synopsis (d) Encumber : Burden 145. (a) Mendicant (b) Ascetic
129. Blizzard : Freeze :: ? : ? (c) Pious (d) Hermit
(a) Insult : Humiliation (b) Bad : Immoral 146. (a) Bardoli (b) Bhadravati
(c) Caution : Careless (d) Jealousy : Respect (c) Porbandar (d) Champaran
130. Capriciousness : Reliability :: ? : ? DIRECTIONS (Qs. 147-151) : In each of the following questions,
(a) Heated : Boiling four words have been given out of which three are a like in some
(b) Tenacious : Practicality manner, while the fourth one is different. Choose the word which
(c) Arbitrary : Whimsical is different from the rest: [2016 - II]
(d) Extemporaneous : Predictability 147. (a) Bogota (b) Sydney
131. Pedagogy : Teaching :: ?:?
(c) Doha (d) Brussels
(a) Telepathy : Emotions (b) Radiology : Sound 148. (a) Retina (b) Pupil
(c) Mycology : Fossils (d) Entomology : Insects (c) Vision (d) Cornea
DIRECTIONS (Qs. 132-136) : In each of the following questions, 149. (a) Udayagiri and Khandagiri
four words have been given out of which three are alike in some (b) Badami
manner, while the fourth one is different. Choose the word which (c) Elephanta
is different from the test: [2015- II] (d) Doddabetta
132. (a) Brass (b) Bronze 150. (a) Tibia (b) Cortex
(c) Silver (d) Steel (c) Cranium (d) Cerebellum
133. (a) Documentary (b) Puppet 151. (a) Saltoro Kangri (b) Kongka
(c) Commentary (d) Feature (c) Zoji La (d) Kumbharli Ghat
134. (a) Deccan (b) Kaas DIRECTIONS (Qs. 152-156) : In each of the following ques-
(c) Chhota Nagpur (d) Nanda Devi tions, find out the alternative which will replace the question
135. (a) Niagara (b) Alamere mark:- [2016 - II]
(c) Kempty (d) Andes 152. Anatomy : Zoology :: Paediatrics : ?
136. (a) Small Betrayals (b) We Indians (a) Chemistry (b) Medicine
(c) Shooting from the Hip (d) Surviving Men
(c) Palaeontology (d) Mechanics
EBD_8177
C-6 Analogy & Odd One Out

153. Eccrinology : Secretions :: Selenography : ? 171. (a) Eye (b) Hand


(a) Sun (b) Mantle (c) Nose (d) Ear
(c) Crust (d) Moon DIRECTIONS (Qs. 172 - 176) : In each of the following questions,
154. Virology : Virus :: Semantics : ? select the related letters/word/number from the given alternative.
(a) Amoeba (b) Language [2017 - II]
(c) Nature (d) Society 172. Editor : Magazine : ? : ?
155. Tectonics : Building :: Taxidermy : ? (a) Novel : Writer (b) Poem : Poet
(a) Classification (b) Conserving (c) Chair : Carpenter (d) Director : Film
(c) Stuffing (d) Collecting 173. Taste : Tongue : : Walk : ?
156. Annihilation : Fire :: Cataclysm : ? (a) Pavement (b) Crutch
(a) Emergency (b) Tribulation (c) Legs (d) Walking sticky
(c) Anxiety (d) Flood 174. Chimney : Smoke : : _____ : ____
(a) Gun : Bullet (b) House : Roof
DIRECTIONS (Qs. 157 - 161) : In each of the following questions,
(c) Clay : Ceramic (d) Tea : Kettle
find the odd word /number/letters from the given alternatives.
175. DCGH : LKQP : : FEJI : ?
[2017 - I] (a) MLSR (b) NMRQ
157. (a) Cow (b) Hen (c) ONTS (d) QPUT
(c) Lioness (d) Horse 176. Paw : Cat :: Hoof : ?
158. (a) Chennai (b) Hyderabad (a) Lamb (b) Elephant
(c) Mohali (d) Kolkata (c) Lion (d) Horse
159. (a) Red (b) Yellow DIRECTIONS (Qs. 177-181) : In each of the following questions,
(c) Blue (d) Green select the related word/letters/ number from the given alternatives.
160. (a) Australia (b) Asia [2018 - I]
(c) Africa (d) USA 177. Flexible : Rigid : : Confidence : ?
161. (a) Assam (b) Mizoram (a) Diffidence (b) Indifference
(c) West Bengal (d) Nagaland (c) Cowardice (d) Scare
DIRECTIONS (Qs. 162 - 166) : In each of the following questions, 178. Mirage : Desert : : ?
select the related letters/word/number from the given alternative. (a) Sky : Illusion (b) Rainbow : Sky
[2017 - I] (c) Rain : Rainbow (d) Image : Mirror
162. Coconut : Shell : Letter : ? 179. Anaemia : Blood : : Anarchy : ?
(a) Disorder (b) Monarchy
(a) Envelope (b) Post
(c) Government (d) Lawlessness
(c) Postal Stamp (d) Letter Box
180. Symphony : Composer : : Painter : ?
163. Bismilla Khan : Clarinetist : : Birju Maharaj : ? (a) Fresco (b) Colours
(a) Kathak (b) Bharatnatayam (c) Art (d) Leonardo
(c) Music (d) Sitar 181. Influenza: Virus :: Ringworm: ?
164. 678: U: : 456 : ? (a) Bacteria (b) Fungi
(a) P (b) O (c) parasite (d) Protozoa
(c) Q (d) R
DIRECTIONS (Qs. 182-186) : In question, find the odd number/
165. CALL : ACLL :: COOL : ? letters/ number pair from the given alternatives. [2018 - I]
(a) LOOC (b) LCOO
(c) OOLC (d) OCLO 182. (a) Annoy (b) Distress
(c) Harass (d) Ravage
166. Punjab : Chandigarh :: Assam
183. (a) Hurdle (b) Disease
(a) Dispur (b) Guwahati (c) Barrier (d) Obstacle
(c) Shillong (d) Imphal 184. (a) Rooster (b) Buck
DIRECTIONS (Qs. 167 - 171) : In each of the following questions, (c) Gander (d) Peahen
find the odd word /number/letters from the given alternatives. 185. (a) fastidious (b) firm
[2017 - II] (c) grave (d) agreeable
167. (a) Intimacy (b) Attachment 186. (a) Soldier – Barrack (b) Principal – School
(c) Artist – Troupe (d) Singer – Chorous
(c) Friendship (d) Enmity
168. (a) Parrot (b) Bat DIRECTIONS (Qs. 187-189) : In each of the following questions
(c) Crow (d) Sparrow find out the alternative which will replace the question mark.
169. (a) Grapes (b) Pomegranate [2018 - II]
(c) Cardamon (d) Mango 187. Sheep : Lamb :: Insect : ?
170. (a) Water (b) Pool (a) Cub (b) Larva
(c) Lake (d) Pond (c) Bull (d) Tadpole
Analogy & Odd One Out C-7

188. Monk : Nun :: Bachelor : ? 201. 5 : 125 : : 7 : ?


(a) Spinster (b) Woman (a) 343 (b) 512
(c) Lady (d) Man (c) 243 (d) 729
189. Dark : Fear :: Honesty : ? DIRECTIONS (Qs. 202-203) : In the following question, select
(a) Personality (b) Money the one which is different from the other three responses.
(c) Treachery (d) Trust [2019-I]
DIRECTIONS (Qs. 190-191) : The following questions consist 202. (a) Daring : Timid (b) Beautiful : Pretty
of two words each that have a certain relationship to each (c) Clear : Vague (d) Youth : Adult
other, followed by four pairs of words. Select the pair that has 203. (a) Fish : Shoal (b) Cow : Herd
the same relationship as the original pair of words. [2018-II] (c) Sheep : Flock (d) Man : Mob
190. Food : Hungry DIRECTIONS (Qs. 204-206) : For the following questions Find
(a) Thought : Politics (b) Water : River the odd word / letter / number from the given alternative.[2019-I]
(c) Rest : Weary (d) Wine : Intoseication
204. (a) Rival (b) Opponent
191. Ampere : Current
(c) Foe (d) Ally
(a) Sound : Waves (b) Newton : Force
205. (a) POCG (b) KLIZ
(c) Speed : Time (d) Distance : Mile
(c) BUDX (d) FQMV
DIRECTIONS (Qs. 192-194) : Choose the odd man from the 206. (a) Farmer (b) Blacksmith
words given below in the question. [2018-II] (c) Cobbler (d) Helper
192. (a) APRIL (b) JUNE DIRECTIONS (Qs. 207-210) : In each of the following questions,
(c) JULY (d) SEPTEMBER select the related letter/word/ number from the given
193. (a) Hill Myna (b) House Sparrow alternatives. [2019 - II]
(c) Emerald Dove (d) Imperial Eagle
207. Pituitary: brain :: thymus :?
194. (a) Anther (b) Retina
(a) Throat (b) Chest
(c) Ovary (d) Petal
(c) Spinal cord (d) Larynx
DIRECTIONS (Qs. 195-196) : In the following question, select 208. Forecast : Future : : Regret : ?
the one which is different from the other three responses. (a) Present (b) Atone
[2018-II] (c) Past (d) Sins
195. (a) Mason : Wall (b) Cobbler : Shoe 209. Family : Home :: Colleagues : ?
(c) Farmer : Crop (d) Chef : Cook (a) Work (b) Office
196. (a) Bottle : Wine (b) Cup : Tea (c) Building (d) Friends
(c) Pitcher : Water (d) Ball : Bat 210. INDIA : PEACOCK :: USA : ?
DIRECTIONS (Qs. 197-199) : In each of the following questions, (a) Red Kite (b) Bald Eagle
select the related letter/word/ figure/ number from the given (c) Mute swan (d) Robin
alternatives. [2019-I] DIRECTIONS (Qs. 211-213) : Find the odd word/letters/number
197. Microphone : Loud :: Microscope : ? pair/number from the given alternatives. [2019 - II]
(a) Elongate (b) Investigate 211. (a) Albatross (b) Ostrich
(c) Magnify (d) Examine (c) Pelican (d) Penguin
198. Sound : Medium : : Light : ? 212. (a) Internet Banking
(a) Air (b) Vacuum (b) Mobile Banking
(c) Water (d) Glass (c) Automated Teller Machine
199. Democracy : India : : Communism : ? (d) Personal Banking
(a) France (b) China 213. (a) Cathedral (b) Mosque
(c) Britain (d) America (c) Temple (d) Monastery
DIRECTIONS (Qs. 200-201) : In the following question, select DIRECTIONS (Qs. 214-218): Three of the words will be in the
the related word pair from the given alternatives. [2019-I] same classification, the remaining one will not be. Your answer
200. School : Education : : ? : ? will be the one word that does NOT belong in the same
(a) Scalpel : Teacher classification as the others. [2020-I]
(b) Hospital : Treatment
(c) Teacher : School 214. (a) Parsley (b) Basil
(c) Dill (d) mayonnaise
(d) Class : College
EBD_8177
C-8 Analogy & Odd One Out

215. (a) Tulip (b) Rose


(c) Bud (d) Daisy 222.
216. (a) Rye (b) Sourdough
(c) Pumpernickel (d) Loaf
217. (a) Scythe (b) Knife
(c) Pliers (d) Saw
218. (a) Biology (b) Chemistry
(c) Theology (d) Zoology
DIRECTIONS (Qs. 219-222): In each of the following questions ,
select the related word/letter/figure from the given alternatives.
[2020-I]

219. Cup is to coffee as bowl is to


(a) Dish (b) Soup
(c) Spoon (d)
220. Play is to actor as concert is to
Food
(a) (b) (c) (d)
(a) Symphony
(b) Musician (a) a (b) b
(c) c (d) d
(c) Piano
(d) Percussion 223. Select the related word pair from the given alternatives.
221. Palette Easel Brush [2020-I]
Textbook Lessonplan ? EXPLORE : DISCOVER
(a) Artist (b) Teacher (a) Read : Skim (b) Research : Learn
(c) Report Card (d) Paint (c) Write : print (d) Think : Relate
Analogy & Odd One Out C-9

ANSWERS & EXPLANATIONS


1. (d) Chapter is a part of book, in the same way storey is a 27. (d) Filter is used to remove impurities from water similarly
level of building. censor removes objectionable scenes from play.
2. (c) As carrot is a vegetable, in the same way pepper is a 28. (c) Both words have nearly same meaning.
spice. 29. (b) Sadist is the one who enjoys giving pain to others
3. (a) while teacher pupil relation is the one where teacher
Convex enjoys white teaching to pupil.
Concave 30. (b) Fresco is prepared by painter similarly symphony is
prepared by composer.
Similarly, 31. (c) First one manages the second one.
Mound 32. (d)
Cavity
33. (d)
4. (b) Gown is type of garment. Similarly, Gasoline is a type 34. (c) Direct is different from the other three words. Direct
of fuel. means 'to give a formal order or command.'
5. (b) Hayper is the antonym of hype. Similarly, over is the 35. (d) Sprain means 'to injure a joint in the body, especially a
antonym of under. wrist or an ankle by suddenly twisting it so that there is
6. (a) Immigration (arrival) is the antonym of emigration pain and usually swelling.'
(departure). Crumble means to break into very small pieces, 'to begin
7. (d) Square is a four sided figure while octagon is a eight to fail or lose strength', "to come slowly to an end.'
sided figure, i.e., four side more. Similarly hexagon has Topple means ‘to move from side to side and fail, ‘to
three more sides than triangle. cause somebody to lose their position of power and
8. (d) Told is the third form of the verb tell, similarly, wept is authority.
the third form of weep. Trimble means ‘to make fall,’ to move or rush in the
9. (a) Lamb is young sheep. Similarly colt is a young horse. specified direction.
10. (c) Ignore is the antonym of overlook. Similarly, dull is the 36. (a) Anonymous means : with a name that is not known or
antonym of sharpen. not made public; written or given by somebody whose
11. (a) Frequently is the antonym of seldom. name is not known or revealed; having no outsanding
or unusual features; not particularly noticeable.
Similarly always is the antonym of never.
Alias and pseudonym refer to a name by which a per-
12. (a) First is the shorter form of the second.
son is called at other times or in other places.
13. (d) First is the splitted parts of the second.
Sobriquet refers to other name of a place.
14. (b) Second is the additional money given for good service
37. (d) Except Thiruvananthapuram, all others are ports.
to first.
38. (c) Except Mercury all others are organic compounds.
15. (b) Second is the relieve of first.
Mercury is a metal.
16. (b) As silence is opposite to noise, Similary Baldness is
39. (d) Hope is Antonym of Despair. Similarly, Encourage is
opposite to Hair. Antonym of Dishearten.
17. (c) Pallid refers to pale complexion as wan refers to pale 40. (c) The second is necessary for the first to be successful.
colour. 41. (b) The second is the unit of the first.
18. (b) First is a kind of meat of second. 42. (c) Revenge and Vendetta are synonymous. Similarly. Envy
19. (d) First one refers to later on stage and second one and Jealousy are synonymous.
previous stage. 43. (a) Style can be refined to make it more beautiful and
attractive. Similarly, photograph is made beautiful by
20. (d) First and second have similar meaning.
retouching it.
21. (a) First is the quantity and second is its unit. 44. (b) Tremble means 'to shake from fear, cold, weakness’,
22. (c) Decimal has base 10 and binary has base 2. Perspire means ‘to give out sweat through the skin'.
23. (b) 45. (c) Condone means 'to accept wrong behaviour or to treat
24. (b) As Lawyer practices in court similarly chemist practices it as if it were not serious', 'to ignore something'.
in laboratory. The first is the act of neglecting the second.
25. (a) Second is required for first to be successful. 46. (a) The second is a part of the first.
47. (b) The first is a vital part of the second.
26. (c) Second is the property of the first.
EBD_8177
C-10 Analogy & Odd One Out

48. (a) Nalanda ad Taxila are ancient seats of learning. Simi- 87. (a) They are famous for oil fields.
larly, Venus and Mars are planets. 88. (c) These are chessmen.
49. (b) Foot is the lower part of human’s leg. Similarly, hoof is 89. (d) These personalities belong to Bengal.
the lower part of cow’s leg. 90. (b) These are terms connected with black magic.
50. (d) Broad refers to spacious thing. Similarly, road is wider 91. (a) These are places having steel plants.
than lane. 92. (d) These are the leafy vegetables.
51. (b) Back of a human being rests on backbone. Similarly, 93. (b) They were the famous poets.
belly rests on ribs. 94. (c) Rustle is sound of leaves and Patter is the sound of rain.
52. (a) Ankle is analogous to wrist (hand) knee is analogous 95. (b) The words in each pair are synonyms of each other.
to elbow.
96. (a) Lack of carefulness causes accident similarly lack of
53. (d) The first is used by the second to bite. sanitation causes disease.
54. (b) Second is the tool used to first. 97. (a) All expect Tsangpo are national parks.
55. (b) Infants are kept in Creche like wise sailors. 98. (b) All except rice are rabi crops.
56. (d) Plant is given pesticide for cure and body is given 99. (d)
vaccination.
100. (b) Plant is given pesticide for cure and body is given
57. (a) First is bigger than the second. vaccination.
58. (d) Second is the place where first lives. 101. (a) First is of higher intensity than the second.
59. (b) First is more intense form of second. 102. (b) First is more intense form of second.
60. (d) Necromancy is the practice to deal with ghosts similarly 103. (d) Necromancy deals with ghosts. Similarly sorcery deals
Sorcery deals with spirits. with spirits.
61. (c) Second is the place where first is manufactured. 104. (c) Words in each pair are antonym to each other.
62. (a) First is a kind or type of second. 105. (d) Funk discovered vitamins and curie discoverd radium.
63. (d) Onam is a festival of Kerala and Bihu is of Assam. 106. (b) Food is contained by germs and body is infected by
64. (c) Calendar consists of dates and dictionary consists of germs.
words. 107. (b) Carbon is a chemical element found in diamond.
65. (a) The words in pair are opposite to each other. Similarly, corundum is a form of alumina found in ruby.
66. (c) Second is the class to which first belongs. 108. (d) An archietect is a person whose job is designing
67. (b) Second is the activity done by first. building similarly,
68. (d) First revolves around the second. sculptor is a person whose job is creating statue.
69. (b) Lotus grow in water and rest grow on land. 109. (a) Myopia is a disease of eye.
70. (b) All except sharpener are used in writing. Similary, Pyorrhoea is a disase of teeth.
71. (c) Brass is an alloy and rest are metals. 110. (d) A chairman is a person who is in charge of a conference.
72. (b) All except eagle are flightless birds. Similarly, an editor is a person who is in charge of a
73. (d) All except Baroda are famous for coal fields. Newspaper.
74. (c) Garo, Khasi, Jainti are hills and Kangra is a valley. 111. (d)
75. (d) All expect orion are Zodiac signs while orion is a 112. (a) All except potassium are metal used in semiconductor
cosntellaiton. devices.
76. (b) Expect foot all are parts of hand. 113. (d) All except chalk are abtained from crops.
77. (d) Igloo is a type of house whereas rest are type of boats. 114. (d) All others, except park are halting places where
78. (b) Club is the place of entertainment and rest are places aeroplanes, ships and trains are kept.
of living. 115. (b) All except quay are parts of a ship.
79. (b) Liquor is a drink. Similarly tea is a beverage. 116. (d) All except sharp are related to dimension
80. (d) Tension causes the breakdown as evaporation forms 117. (d) Second is another name for the first.
the cloud. 118. (a) Ignominy is the result of disloyalty. Fame is the result
81. (a) Second one is the bigger form of first. of heroism.
82. (d) First is a kind of second. 119. (d) Loath and Reluctant are synonyms. Coercion and
83. (a) Carnivore are flesh eating animals and herbivores are Persuasion are synonyms.
plant eating animals. 120. (c) Second is the past of first.
84. (c) They were characters from various dramas. 121. (b)
85. (a) These are sites of volcanoes. 122. (c) All except Oliver Twist are works of Shakespeare, while
86. (d) They are units of measurement. Oliver Twist is a work of Charles Dickens.
Analogy & Odd One Out C-11

123. (d) All except Montgomery were Admirals. Nimitz was U.S. 160. (d) Except (d), all are continents.
Admiral, Yamamoto was Japanese Admiral and Nelson 161. (c) Except (b), all are North-Eastern states of India.
was British Admiral, while British Admiral, while 162. (a) As 'Coconut' is inside the 'Shell', similarly , 'Letter ' is
Montgomery was British Field Marshal. inside the "Envelope'.
124. (c) All except Simmer are connected with light, while 163. (a) As 'Bismillah Khan' is a famous 'Clarinetist', similarly,
simmer is a way of cooking. 'Birju Maharaj' is a famous 'Kathak' dancer.
125. (c) All others are hills located in India. 164. (b) 687 = 6 + 7 + 8 = 21 is the place value of U in alphabetical
126. (c) All except Walrus are fur-bearing animals. sequence. Similarly, 456 = 4 + 5 + 6 = 15 is the place
value of O.
127. (d) First is the antonym of second.
165. (d) In CALL, first two letters are interchanged among
128. (b) Both are opposites. themselves and last two letters are interchanged among
129. (a) First is the cause of second. themselves to get ACLL. Similarly, COOL can be
130. (d) Capricious behaviour is not characterized by reliability. rearranged as OCLO.
Extemporaneous actions are not characterized by 166. (a) Punjab's capital is Chandigarh. Similarly, Assam's
predictability. capital is Dispur
131. (d) Pedagogy is art of teaching and Entomology is study 167. (d) Except Enmity, all other words represent Cordial
of insects. relationship.
132. (c) Silver -All except silver are alloys. 168. (b) All except Bat belong to the class of Aves (Birds), while
bat is a mammal.
133. (b) Puppet because except that all are media and News
related. 169. (c) Except Cardamom, all other are fruits.
134. (d) Nanda devi - All except Nanda devi are plateaus. 170. (a) Pool, Lake and Pond are water bodies.
135. (d) Andes - All except Andes are waterfalls. 171. (b) Except Hand, all others are sense organs.
136. (b) We Indians - All except We Indians are the books 172. (d) Editor is responsible for preparing a Magazine. Similarly,
written by 'Shobhaa De'. director is responsible for making a film.
137. (c) First is the part of second. 173. (c) Tongue is used to taste something. Similarly, leg is
138. (c) Second is necessary to acquired the first. used to walk.
139. (d) First indulges in the second. 174. (a) Smoke is emitted through chimney. Similarly, bullet is
140. (b) Second is the synonym of first. fired from gun.
141. (c) First is the result of second.
+1 +1
142. (c) All except lizard are insects, while lizard is a reptile.
175. (b)
143. (d) Potato is the only vegetable in the group which grows N M R Q
underground. 176. (d) As Cat has paw similarly Horse has hoof.
144. (c) Bronze – All others are metals. 177. (a) Flexible is antonym of Rigid. Similarly, Confidence is
145. (c) Pious – All others are synonyms. antonym of Diffidence.
146. (b) All except Bhad-ravati are places related to Gandhi ji. 178. (c) Mirage is an illusion caused by hot air conditions
147. (d) Except Brussels all the three are capital city. making one see something that is not there, especially
148. (c) Except vision all the three are parts of eyes. the appearance of a sheet of water on a hot road or in a
149. (d) Doddabetta is the mountain peak whereas all the three desert. Similarly,
are caves. Rainbow is an arch of seven colours formed in the sky
150. (a) Except Tibia all the three are related to brain. when the sun shines through rain.
151. (a) Saltoro kangri is the mountain peak where as all the 179. (c) Anaemia is the lack of blood. Similarly, Anarchy is the
three are mountain pass. lack of government.
152. (b) Anatomy is a branch of Zoology. 180. (a) Fresco is an art of painting that is done on freshly
Similarly, Paediatrics is a branch of medicine. spread moist lime plaster.
153. (d) Eccrinology is the study of secretions similarly, 181. (b) As, Infuenza is caused by virus.
selenography is the study of moon. Similarly, Ring worm is caused by Fungi.
154. (b) Virology deals with the effects of virus. 182. (d) Ravage is the different from the other words.
Similarly, semantics deals with the effects of language. 183. (b) Except disease, all other terms denote obstruction,
155. (c) Tectonics is the science dealing with the art of building. hindrance or interruption.
Similarly, taxidermy is the art of stuffing animals. 184. (b) Rooster, Gander and Peahen are birds. Buck is an
156. (d) First is result of second. animal.
157. (d) Except (d), all animals are females. 185. (d) Agreeable is different from the other three words.
158. (c) Except (c), all are capital cities. Agreeable (Adjective) means ‘pleasant’, ‘giving
159. (b) Except (b), every colour is a primary colour. pleasure’, ready to agree’.
EBD_8177
C-12 Analogy & Odd One Out

186. (b) Barrack is a large building or group of buildings where 211. (b) All except Ostrich are water birds.
soldiers live. 212. (c) Except, Automated Teller Machine, in all other
Principal is the head of school. Troupe is a group of bankings, there is no special instrument for use.
artists. 213. (d) All except Monastery are places of/worship, while
Chorous is a large group of singers. monastery is a place where monks stay.
187. (b) As, Infant of sheep is lamb. Similarly, Infant of Insect 214. (d) Parsley, basil, and dill are types of herbs.
is larva. Mayonnaise is not an herb.
188. (a) As, Monk antonyms is Nun. Similarly, Bachelor 215. (c) Tulip, rose, and daisy are all types of flowers. A bud
antonyms is Spinster. is not.
189. (d) Fear comes in Dark same as Trust comes due to 216. (a) Loaf, sourdough, and pumpernickel are types of
Honesty.
bread. A rye is not a bread type.
190. (c) As, Hungry needs food, Similarly, Weary needs rest.
Loaf - Bread that is shaped and baked in one piece
191. (b) As, Ampere is unit of current. Similarly, Newton is unit
and usually sliced before being eaten.
of force.
Pumpernickel - Dark, dense German bread made from
192. (c) Except July, All other month have 30 days.
coarsely ground whole-grain rye.
193. (b) House sparrow is a common type of bird.
Sourdough - Leaven for making bread, consisting
194. (b) Except Retina, all others are parts of a flower.
of fermenting dough, typically that left over from a
195. (d) In all other pairs, second is prepared by the first.
previous batch.
196. (d) In all other pairs, first is used to hold the second.
Rye - A wheat like cereal plant that tolerates poor
197. (c) As, Microphone makes sound louder. Similarly,
Microscope makes the object magnified. soils and low temperatures.
198. (b) Sound requires medium to travel and light can travel 217. (c) The scythe, knife, and saw are all cutting tools.
in vacuum. Pliers are tools, but they are not used for cutting.
199. (b) Country and its type of governance. 218. (c) Biology, chemistry, and zoology are all branches of
200. (b) We get education in school, Similarly Treatment is done science. Theology is the study of religion.
in hospital. 219. (b) Coffee goes into a cup and soup goes into a bowl.
201. (a) 5³=125 Choices (a) and (c) are incorrect because they are
7³=343 other utensils. The answer is not choice (d) because
202. (b) In all other pairs, the two words are antonyms of each the word food is too general.
other. 220. (b) An actor performs in a play. A musician performs
203. (d) In all other pairs, second is a collective group of the at a concert. Choices (a), (c), and (d) are incorrect
first. because none is people who perform.
204. (d) Ally is different in nature. 221. (c) The objects above the line are all things used by
205. (d) FQMV is different because there is no vowel in word. an artist. The objects below the line are all things
206. (d) Helper may be of any profession. used by a teacher.
207. (b) Pituitary is a gland in the brain.
222. (c) Scissors is to knife as pitcher is to watering can.
Similarly, thymus is a gland of the chest.
This relationship is about function. The scissors
208. (c) Forest is for future happenings and Regret is for past
actions. and knife are both used for cutting. The pitcher
209. (b) Family lives in a Home. Similarly, Colleagues works in and watering can are both used for watering.
office. 223. (b) One explores to discover; one researches to learn
210. (b) As, INDIA's National bird is PEACOCK. Similarly,
USA's National bird is Bald Eagle.
Series/Coding-Decoding C-13

Series/
2 Coding-Decoding
1. Which number comes next in this series ? [2011-II] 9. Z , S , W, O , T, K, Q, G, ?, ?
1 4 7 10 13 16 (a) N, D (b) N, C
(a) 17 (b) 19 (c) O, C (d) O, D
(c) 21 (d) None of these 10. BCD , RQP , LMN, TS?
2. Which number comes next in this series ? [2011-II] (a) U (b) V
0 1 3 6 10 15 (c) R (d) T
(a) 19 (b) 21 11. If A = C , B = D , C = E and so on, code the following:
(c) 26 (d) 32 ‘STUDENT’ [2014-I]
3. Look at the series 36, 34, 30, 28, 24, ..... what number should (a) VWVFGPV
come next ? [2011-II] (b) UUWFGPV
(a) 20 (b) 22 (c) UVWFGPV
(c) 23 (d) 26 (d) UVVFGPV
4. Look at the series 22, 21, 23, 22, 24, 23 ..... what number 12. If A = Z, B = Y and so on, code the following: [2014-I]
should come next ? [2011-II]
‘EDITOR’
(a) 22 (b) 24
(a) VWRGLI (b) VWRGVI
(c) 25 (d) 26 (c) VWVGLI (d) VWRLLI
5. Look at the series 53, 53, 40, 40, 27, 27 ..... what number
DIRECTIONS (Qs. 13-14) : a series is given, with one/two term
should come next ? [2011-II]
missing. Choose the correct alternative from the given ones that
(a) 12 (b) 14
will complete the series. [2019 - II]
(c) 27 (d) 53
13. 14, 19, 29, 49, 89, ?
6. Look at the series 8, 22, 8, 28, 8 ..... what number should
(a) 139 (b) 149
come next ? [2011-II]
(c) 159 (d) 169
(a) 9 (b) 34
14. 5, 21, 69, 213, 645, ? .
(c) 29 (d) 32
(a) 1670 (b) 1941
7. Look at the series 31, 29, 24, 22, 17 ..... what number should
come next ? [2011-II] (c) 720 (d) 1320
(a) 15 (b) 14 15. Fill the blank in the middle of the series or end of the series.
(c) 13 (d) 12 DEF, DEF2, DE2F2, _____, D2E2F3 [2020 - II]
DIRECTIONS (Qs. 8 – 10) : Complete the series [2014-I] (a) DEF3 (b) D3EF3
(c) D2E3F (d) D2E2F2
8. A, B, D, G, ?, P
(a) I (b) J
(c) K (d) L
EBD_8177
C-14 Series/Coding-Decoding

ANSWERS & EXPLANATIONS

+3 +3 +3 +3 +3 +3 12. (a) E D I T O R
1. (b)
¯ ¯ ¯ ¯ ¯ ¯
1 4 7 10 13 16 19
V W R G L I
2. (b) 0 1 3 6 10 15 21.
+1 +2 +3 +4 +5 +6 13. (d) The pattern is as follows :

3. (b) 36 34 30 28 24 22
–2 –4 –2 –4 –2 14 19 29 49 89 169

4. (c) 22 21 23 22 24 23 25
–1 +2 –1 +2 –1 +2 +5 +10 +20 +40 +80

5. (b) 53,55 40,40 27, 27 14 +5 +10 +20 +40


–13 –13 –13
+5 +10 +20
+6
6. (b) 8 22, 8, 28, 8, 34 +5 +10
+6

31 29 22 17 17 15
+5
7. (a) 24
–2 –5 –2 –5 –5 –2
So, 169 will complete the series.
A, B, D, G, K, P
14. (b) The pattern is as follows :
8. (c)
+1 +2 +3 +4 +5 21 – 5 = 16 Þ 16 × 3 = 48
69 – 21 = 48 Þ 48 × 3 = 144
–4 –4 –4 –4 213 – 69 = 144 Þ 144 × 3 Þ 432
645 – 213 Þ 432 Þ 432 × 3 = 1296
Z S W O T K Q G N C and
9. (b)
–3 –3 –3 –3 1941 – 645 = 1296
15. (d) In this series, the letters remain the same: DEF.
B C D, R Q P , L M N, T S R The subscript numbers follow this series:
10. (c) 111, 112, 122, 222, 223, 233, 333, ...
+1 +1 +1 +1 +1 +1 +1 +1
11. (c) S T U D E N T
¯ ¯ ¯ ¯ ¯ ¯ ¯
U V W F G P V
Logical Deduction / Statement & Assumption C-15

Logical Deduction/
3 Statement & Assumption
DIRECTIONS (Qs. 1 - 3) : Each of the following questions has four Assumption :
statements. Three are logically correct. Some of which may look (i) The Supreme Court is above the Executive.
factually absurd. Ignore this absurdity and look to the logical (ii) The Supreme Court is more active than the Executive.
corrections. Choose the statement which is wrong or doubtful: (iii) The Executive does not have sufficient powers.
[2014-I] (a) (i) & (ii) are implicit (b) (ii) & (iii) are implicit
1. (a) Birds fly in the air. Trees are birds. Therefore, trees fly (c) (i) & (iii) are implicit (d) Only (ii) is implicit
in the air.
DIRECTIONS (Qs. 6 - 10) : Each of these questions given below
(b) Some boys steal. All who steal are naughty. All naughty contains three elements. These elements may or may not have
are honest. Therefore, some boys are honest some inter- linkage. Each group of elements may fit into one of
(c) All girls like dance. Some girls are Indian. All Indians these diagrams at (a), (b), (c) or (d). You have to indicate the group
are artists. Therefore, some artists like dance. of elements which correctly fits into the diagrams. [2014-II]
(d) All liars are not thieves. All thieves are criminals. 6. Which of the following diagrams indicates the best relation
Therefore, all liars are criminals. between Judge, Thieves and Criminals ?
2. (a) Ramesh is tall. Ramesh is a boy. Therefore, boys are tall.
(b) All who can fly are animals. Some birds can fly. (a) (b)
Therefore, some birds are animals.
(c) Men live in houses. Houses grow on trees. Trees float
in water. Therefore, men float in water. (c) (d)
(d) All living things are mobile. Some non- living things 7. Which of the following diagrams indicates the best relation
are mobile. Therefore, some mobile are living and some between India, Haryana and World ?
non-living.
3. (a) All books can read. Some pencils are books. All pencils
are clever. Therefore, all clever cannot read. (a) (b)
(b) Some who fail are stupid. Some criminals are stupid.
Therefore, all criminals fail. (c) (d)
(c) Some liar are thieves. All thieves are criminals. There- 8. Which of the following diagrams indicates the best relation
fore, some liars are criminals. between Pigeon, Bird and Dog ?
(d) All that is given is black. Trees are green. Therefore,
trees are black. (a) (b)
DIRECTIONS (Qs. 4 - 5) : Consider the statement and assumptions
that follow. Which of these assumptions is/are implicit in the (c) (d)
statement: [2014-I]
9. Which of the following diagrams indicates the best relation
4.. Statement: “You won’t get sweets at any cost. I will not let between Earth, Sea and Sun ?
you eat things that are not good for your teeth” — a mother
tells her child (a) (b)
Assumption:
(i) The mother cares for her child.
(c) (d)
(ii) Sweets are not good for her child.
(iii) The mother has the authority to decide what her child 10. Which of the following diagrams indicates the best relation
is to eat. between Hockey, Football and Cricket ?
(a) (i) & (ii) are implicit (b) (ii) & (iii) are implicit
(c) (i) & (iii) are implicit (d) All are implicit (a) (b)
5. Statement: “Clearly, the judiciary cannot provide all answers.
But it seems the public, weary of an inactive Executive, is (c) (d)
turning to the Supreme Court as a last resort”—an article.
EBD_8177
C-16 Logical Deduction / Statement & Assumption

DIRECTIONS (Qs. 11 - 15) : Choose the most appropriate answer. (a) (b)
[2014-II]
11. Which one of the following is always with ‘Bargain’?
(c) (d)
(a) Exchange (b) Sumptuousness
(c) Triviality (d) Eloquence
12. Which one of the following a ‘Drama’ must have? 20. Which of the following diagrams indicates the best relation
between Males, Cousins and Nephews?
(a) Actors (b) Story
(c) Sets (d) Director (b)
(a)
13. A boy is sitting at the back seat of a car. When the driver
suddenly start moving the car (in forward direction) the boy
experiences a backward force ? (d)
(c)
(a) Always (b) Never
(c) Often (d) Sometimes
14. Which one of the following is always found in ‘Wonder’? DIRECTIONS (Qs. 21-25) : Each of these questions given below
(a) Crowd (b) Lumber contains three elements. These elements may or may not have
(c) Astonishment (d) Rustic some inter-linkage. Each group of elements may fit into one of
these diagrams (a), (b), (c) and (d). You have to indicate the
15. ‘Disclosure’ always involves group of elements which correctly fits into the diagrams.
(a) Agents (b) Display
(c) Exposition (d) Secrets 21. Which of the following diagrams indicates the best relation
between Females, Doctors, and Patients? [2015- II]
DIRECTIONS (Qs. 16 - 20) : Each of these questions given below
contains three elements. These elements may or may not have
some inter-linkage. Each group of elements may fit into one of (a) (b)
these diagrams at (a), (b), (c) or (d). You have to indicate the
group of elements which correctly fits into the diagrams.
[2015-I] (c) (d)
16. Which of the following diagrams indicates the best relation
between Doctors. Human Beings and Married People? 22. Which of the following diagrams indicates the best relation
between Doctors, Teachers and Engineers?
(a) (b)

(a) (b)
(c) (d)

(c) (d)
17. Which of the following diagrams indicates the best relation
Children, Naughty and Studious? 23. Which of the following diagrams indicates the best relation
(a) (b) between Steel, Wood and Building Material?

(a) (b)
(c) (d)

18. Which of the following diagrams indicates the best relation (c) (d)
between Thief, Criminal and Police ?
24. Which of the following diagrams indicates the best relation
(a) (b) between Males, Fathers and Human Being?

(c) (d) (a) (b)

19. Which of the following diagrams indicates the best relation (c) (d)
between Man, Worker and Garden?
Logical Deduction / Statement & Assumption C-17

25. Which of the following diagrams indicates the best relation DIRECTIONS (Qs. 31-34): Each of these questions given below
between Detectives, Spies and Men with Beards? contains three elements. These elements may or may not have
some inter-linkage. Each group of elements may fit into one of these
diagrams at (a), (b), (c) or (d). You have to indicate the group of
(a) (b) elements which correctly fits into the diagrams:- [2016 - II]
31. Which of the following diagrams indicates the best relation
between Mountains, Earth and Forests?
(c) (d)
(a) (b)
DIRECTIONS (Qs. 26-30): Each of these questions given below
contains three elements. These elements may or may not have some
inter-linkage. Each group of elements may fit into one of these
diagrams at (a), (b), (c) or (d). You have to indicate the group of (c) (d)
elements which correctly fits into the diagrams. [2016 -I]
26. Which of the following diagrams indicates the best relation 32. Which of the following diagrams indicates the best relation
between Biology, Zoology and Physics? between Sun, Planets and Earth?

(a) (b) (a) (b)

(c) (d) (c) (d)


27. Which of the following diagrams indicates the best relation
between Stamp, Pen and Chalk? 33. Which of the following diagrams indicates the best relation
between Graduate, Teacher and Player?
(a) (b)
(a) (b)
(c) (d)

28. Which of the following diagrams indicates the best relation (c) (d)
between Doctors, Human Beings and Married People'?
34. Which of the following diagrams indicates the best relation
(a) (b) between Leaf, Seed and Root?

(a) (b)
(c) (d)

29. Which of the following diagrams indicates the best relation


between Factory, Product and Machinery ? (c) (d)

(a) (b) DIRECTION (Qs. 35-39): Each of these questions given below
contains three elements. These elements may or may not have
some inter linkage. Each group of elements may fit into one of
(c) (d) these diagrams at (a), (b), (c), and (d). You have to indicate the
group of elements which correctly fits into the diagrams. [2017- I]
30. Which of the following diagrams indicates the best relation
between Singers, Athletes and Girls? 35. Which of the following diagrams indicates the best relation
between Travelers, Train and Bus?

(a) (b)
(a) (b)

(c) (d) (c) (d)


EBD_8177
C-18 Logical Deduction / Statement & Assumption

36. Which of the following diagrams indicates the best relation 42. Which of the following diagrams indicates the best relation
between Profit, Dividend and Bonus? between Earth, Sea and Sun ?

(a) (a) (b)


(b)

(c) (d)
(c) (d)
43. Which of the following diagrams indicates the best relation
37. Which of the following diagrams indicates the best relation between Hockey, Football and Cricket ?
between Women, Mothers and Engineers?
(a) (b)
(a) (b)

(c) (d)
(c) (d)
44. Which of the following diagrams indicates the best relation
38. Which of the following diagrams indicates the best relation between Iron, Lead and Nitrogen?
between Author, Lawyer and Singer?
(a) (b)
(a) (b)
(c) (d)

(c) (d) 45. Find out which of the diagrams given in the alternatives
correctly represents the relationship stated in the question.
39. Which of the following diagrams indicates the best relation Sharks, Whales, Turtles [2018 - I]
between Judge, Thieves and Criminals?

(a) (b) (a) (b)

(c) (d)
(c) (d)
DIRECTIONS (Qs. 40-44): Each of these questions given below
contains three elements. These elements may or may not have some
46. Indicate which figure will best represent the relationship
inter linkage. Each group of elements may fit into one of these amongst the three : [2018 - I]
diagrams at (a), (b), (c), and (d). You have to indicate the group of Legumes Seeds, Peas, Kidney Beans
elements which correctly fits into the diagrams. [2017 - II]
40. Which of the following diagrams indicates the best relation
between India, Haryana and World ? (a) (b)

(a) (b)
(c) (d)
(c) (d)
47. Find out the figure which best represents the relationship
41. Which of the following diagrams indicates the best relation among Garden, Rose and Jasmine. [2018 - I]
between Pigeon, Bird and Dog?

(a) (b)

(c) (d) (a) (b) (c) (d)


Logical Deduction / Statement & Assumption C-19

48. Identify the diagram that best represents the relationship (a) Only conclusion I is true
among classes given below : Christians, Catholics, Pope (b) Neither conclusion I nor II is true
[2018 - I] (c) Only conclusion II is true
(d) Both conclusions I and II are true
53. Statement:
No right is a left.
(a) (b)
All up are left.
Some down are up.
Conclusions:
I. No up is a right.
(c) (d) II. At least some down are left.
54. Statements:
Some cricketers are footballers
DIRECTIONS (Qs. 49-51): Each of the questions below contains
All footballers are boxers.
three elements. These three elements may or may not have some
linkage. Each group of the elements may fit into one of the diagrams Some boxers are players.
at (a), (b), (c) and (d). You have to indicate groups of elements in Conclusions:
each of the questions fit into which of the diagrams given below. I. At least some boxers are cricketers.
The letter indicating the diagram is the answer. [2018 - II] II. No player is a cricketer.
55. In the following figure, rectangle represents Opticians, circle
represents Art critics, triangle represents Riders and square
(a) (b) represents Boxes. Which set of letters represents Art critics
who are not Riders? [2019-I]

(c) (d)

49. Brick , House, Bridge


(a) BLEF (b) IJ
50. Student of law, Student of science, Men
(c) CHDG (d) EDGJ
51. Antisocial, Pickpockets, Kidnappers
56. Which one of the following diagram represents the correct
52. The following diagram represents the students who are
relationship among [2019-I]
singers, dancers and poets.
Professor, Male and Female.
Dancer
B C D
Singer (a) (b)
Q
M R
X (c) (d)

57. A’s birthday is on Friday 30th June. Find the day of the
Poets week on which B’s birthday in the same year if B was born
Study the diagram and identify the region which represents 15th November ? [2019-I]
the students who are both poets and singers but not (a) Tuesday (b) Wednesday
dancers. (c) Monday (d) Sunday
(a) B + C + D (b) M 58. Which one of the following diagram represents the correct
(c) Q + R (d) R + X relationship among [2019-I]
DIRECTIONS (Qs. 53 - 54): In this question two/three statements Pink, Blue and Fruit.
followed by two conclusions numbered I and II have been given.
You have to take the given statements to be true even if they (a) (b)
seem to be at variance with commonly known facts and then
decide which of the given conclusions logically follows the given
statements disregarding commonly known facts. [2018 - II] (c) (d)
EBD_8177
C-20 Logical Deduction / Statement & Assumption

59. Which one of the following diagram represents the correct 65. Priyank ranks 6th from the bottom and 28th from the top in a
relationship among [2019-I] class. How many students are there in the class?
Brain, Cerebrum, Liver and Human body. [2019 - II]
(a) 31 (b) 32
(c) 33 (d) 34
(a) (b)
66. Select the missing numbers from the given responses
[2019 - II]
(c) (d) 43 25 ?
21 40 35
DIRECTIONS (Qs. 60–69): In each question below are given 35 34 28
some statements followed by two conclusions numbered I and
II. You have to take the given statements to be true even if they (a) 36 (b) 46 (c) 40 (d) 26
seem to be at variance with commonly known facts. Read all the 67. Which of the following diagrams represents the correct
conclusions and then decide which of the given conclusions relationship between Doctors, Specialist Doctors and
logically follows/follow from the given statements, disregarding Patients? [2019 - II]
commonly known facts. [2019-I]
Give answer (a) (b)
(a) If only conclusion I follows.
(b) If only conclusion II follows.
(c) (d)
(c) If either conclusion I or II follows.
(d) If neither conclusion I nor II follows. 68. In the given figure, how many are musical toys?
[2019 - II]
60. Statements:
Electronic
All shirts are skirts. Musical
No skirt is top.
All tops are kurta. 33 17 18
14 28 11 36 Toys
Conclusions:
I. All shirts are kurta II. Some kurta are skirts. 25
61. Statements: (a) 53 (b) 61
All September are October. (c) 42 (d) 45
69. In the given figure, how many players are quick and fat?
No October is November. [2019 - II]
No November is December.
Conclusions: Players 68 32 36 Fat
I. Some September are not Novembers 30
31 33
II. No October is December. 81
62. Nithya is Sam’s Sister. Mogan is Sam’s Father. Selvan is
Rajan’s Son. Rajan is Mogan’s Brother. How is Nithya related
to Selvan? [2019 - II] Quick
(a) Daughter (b) Sister
(c) Cousin (d) Wife (a) 31 (b) 30 (c) 32 (d) 33
63. A boy running towards South, turns to his right and runs. 70. Pointing to a photograph, a man said, “I have no brother or
Then he turns to his right and finally turns to his left. sister but that man’s father is my father’s son.” Whose
Towards which direction is he running now? [2019 - II] photograph was it ? [2020 - I]
(a) East (b) West (a) His own (b) His son
(c) South (d) North (c) His father (d) His grandfather
64. Roshan is taller than Hardik who is shorter than Susheel. 71. A group of 1200 persons consisting of captains and soldiers
Niza is taller than Harry but shorter than Hardik. Susheel is is travelling in a train. For every 15 soldiers there is one
shorter than Roshan. Who is the tallest ? [2019 - II] captain. The number of captains in the group is ? [2020 - I]
(a) Roshan (b) Susheel (a) 70 (b) 75
(c) Hardik (d) Harry (c) 80 (d) 85
Logical Deduction / Statement & Assumption C-21

72. I. Mara runs faster than Gail. Study the diagram and identify the students who play all
II. Lily runs faster than Mara.
the three games.
III. Gail runs faster than Lily
(a) S+R (b) V+T
If the two statements are true, the third statement is
(c) S+T+V (d) S
[2020 - I]
(a) True (b) False 77. Which of the following diagrams indicates the best relation
(c) Uncertain (d) None of these between Hockey, Football and Cricket ? [2020 - I]
73. If Arun’s birthday is on May 25 which is Monday and his
sister’s birthday is on July 13. Which day of the week is his
sister’s birthday? [2020 - I] (a) (b)
(a) Monday (b) Wednesday
(c) Thursday (d) Friday
74. Read all the conclusions and then decide which of the given
conclusions logically follows from the given statements
disregarding commonly known facts. [2020 - I]
Statements:
Some Noise are steel (c) (d)
No steel is Blood
Some steel is Free
Conclusion: 78. In an organization of pollution control board, engineers are
I: Some noise are not Blood represented by a circle, legal experts by a square and
II: All Blood is noise environmentalist by a triangle. Who is most represented in
(a) If only conclusion I follows. the board as shown in the following figure? [2020 - I]
(b) If only conclusion II follows.
(c) If either conclusion I or II follows.
(d) If both conclusions I and II follow.
75. Which of the following diagrams indicates the best relation
between Profit, Dividend and Bonus ? [2020 - I]

(a) (b)
(a) Environmentalists
(b) Legal Experts
(c) Engineers with legal background
(c) (d)
(d) Environmentalists with Engineering background

76. The diagram given below represents those students who


play Cricket, Football and Kabaddi. [2020 - I]

Kabaddi

S T U

Football V

Cricket
EBD_8177
C-22 Logical Deduction / Statement & Assumption

ANSWERS & EXPLANATIONS


1. (d) 2. (a) 3. (b) 4. (d) 19. (d) Some workers are man. No man is garden.
5. (d)

criminals Males

6. (b) Thieves Judge


20. (a) Nephews Cousins

World
India
21. (c)
7. (d) Haryana

Females Doctors

Patients
Bird
Femals, Doctors and Patients are connected to some
extent so option (c) is correct.
8. (a) Pigeon Dog
22. (d) Doctors, Teachers and Engineers are different to each
other in their profession. So option (d) is correct.

Teachers Doctors
Earth

9. (a) sea sun


Engineers

23. (a) Steel and wood both are used as building material and
there is no relation between steel and wood .

Monkey building material

10. (b) Cricket

wood steel
Football

11. (a) Bargain is a thing bought for less than the usual price.
12. (b) A drama can not be performed without a story. 24. (c) All father are males but some human beings may not
13. (a) It occurs due to inertia of rest. When a car suddenly be males or father. So option (a) is correct.
starts the lower part of the boy’s body will be in the
motion while his upper will be at rest.
14. (c) Astonishment is a feeling of very great surprise. Human being
15. (d) A disclosure always has secrets. Males
16. (d) Some doctors may be married people. All doctor, married
people are human being.
17. (a) Children may be both naughty and studious. Fathers
18. (a) All thiefs are criminal. No criminal is police.
Logical Deduction / Statement & Assumption C-23

25. (d) Detectives, spies can be both men or women. Men 37. (a) All mothers are women and some mothers and some
detectives or spies can be both men with beard or men women may be engineers.
with beard so option (d) is correct. 38. (b) All the three are different professions.
39. (b) All the thieves are criminals while judge is different
Detectives Spies from these.
40. (d) Haryana is in India and India is in the World.
Men with beards 41. (a) All the pigeons are birds while dog is different from
26. (b) Zoology is a branch of Biology these.
So Outer circle representing Biology and Inner circle 42. (a) Sea is a part of Earth while Sun is different from these
representing Zoology. two.
Now, Biology and Physics are different branch of
science. 43. (b) All these three games are different from each other.
44. (b) All these three elements are different from each other
45. (c) Sharks belong to class pisces. Whale is a mammal and
Physics Turtle belongs to class reptiles.
Above diagram indicates the best relation b/w Biology,
Zoology and Physics.
Sharks Whale
27. (c) All are three different ways to write something.
28. (d) Doctors can be married people and both Doctors and
married people belong to Human Beings.

46. (b) Pea is different from kidney bean. But both are
Leguminous seeds.
29. (d) Product and Machinery are different from each other
but both are found in Factory.
Leguminous
30. (c) Some girls may be athletes and vice versa similarly Seeds
some girls may be singers and vice versa. Some athletes
may be singers and vice versa. Pea Kidney
Bean
31. (a) Forest

47. (a) Best representation of the relationship is :


Mountain Garden
Rose Jasmine
Earth
Q There are some forest at mountains also and they both 48. (b) Christians
are on earth Catholics

Pope
32. (b)

Sun
Earth
Planet
49. (a)
Q Earth belongs to the class of Planets. But, Sun is Bridge House
Brick
entirely different from the two.
33. (a)
34. (c) All are different parts of a tree.
35. (c) Bus and Train are different from each other but some
Men
Student Student
travelers travel by bus and some travel by train. 50. (b) of law of science
36. (b) Bonus and Dividend are different from each other. But
both these are parts of profit.
EBD_8177
C-24 Logical Deduction / Statement & Assumption

63. (b) Starting


Point

Antisocial
Pickpockets
51. (b) Kidnappers

64. (a)Roshan, Susheel > Hardik


Hardik > Niza > Harry
52. (b) According to diagram,
Roshan > Susheel
M is poet and singer but not dancer.
Roshan > Susheel > Hardik > Niza > Harry
Therefore, Roshan is the tallest.
up
65. (c) Priyank is 6th from the bottom 28th from top.
right down So, total number of students = 6 + 28 – 1 = 33
53. (d) left 66. (a) Each coloumn sum upto 99
\ 99 – (35 + 28) = 36
67. (b) Option (b) is correct.
68. (c) Total number of musical toys = 14 + 28 = 42.
69. (b) Total number of players are quick and fat = 30.
players 70. (b) Since the person who is telling has no brother or sister,
cricketer footballer
so his father son is he himself. So the man in the
54. (a) photograph is his son.
boxer
71. (b) Generally we may commit mistake of dividing 1200/15.
But out of 16 persons there is one captain.so, it will be
1200/16 = 75
55. (a) BLEF 72. (b) We know from the first two statements that Lily runs
56. (a) fastest. Therefore, the third statement must be false.
57. (b) According to question, 73. (a) Reference day : May 25th Monday
Required day = (31 + 31 + 30 + 31 + 15) ÷ 7 = 5 days Days from May 25th to July 13 = 6 + 30 +13 = 49
Hence, B’s birthday was on 5 days later of A’s birthday No of odd days : 49/7 = 0
Hence, Wednesday is the required day. Hence, his sister birthday is on Monday.
58. (d) 74. (a) only conclusion I follows.
59. (d)

60. (d)

So neither conclusion I nor II follows.

75. (b) Bonus and Dividend are different from each other. But
61. (a) both these are parts of profit.
76. (d) S indicates those students who play all three games.
So only conclusion I follows. 77. (b) All these three games are different from each other.
62. (c) Nithya is Sam’s Sister and Mogan is Sam’s Father Þ 78. (d) Environmentalists with Engineering background is
Nithya is Mogan’s Daughter. most represented in the board.
Selvan is Rajan’s Son and Rajan is Mogan’s Brother Þ
Selvan is Mogan’s Nephew.
So, Nithya is Selvan’s Cousin.
Analogy/
4 Odd One Out/Series
DIRECTIONS (Qs. 1 - 4) : This set of questions is based upon the DIRECTION (Q. 5) : Which symbol in the Answer Figure
continuation of figures in a logical manner. There is a sequence completes the sequence in the problem figure: [2011-II]
of figures depicting a change step by step. Select one of the
figure from the four choices shows as (a), (b), (c) & (d) which 5.
will continue the sequence. [2011-II]
1. Problem Figure

Answer Figure (a) (b) (c) (d)


DIRECTIONS (Q. 6) : There are certain common characteristics/
properties between the two problem figures. Select the figure from
amongst the Answer Figures which shows similar characteristics/
properties as shown by the problem figures. [2011-II]
(a) (b) (c) (d)
2. Problem Figure 6.

Answer Figure

(a) (b) (c) (d)


DIRECTIONS (Q. 7) : Your task here is to look at the series of
(a) (b) (c) (d) four figures and work out the rule which links them all. Choose
which of the three following figures obey that rule and identify
3. Problem Figure the one which does not. [2011-II]

7. +
+
+ +
Answer Figure
(a) (b) (c) (d)
DIRECTIONS (Qs. 8 - 12) : In each of the following questions
find out which of the answer figures (1), (2), (3) and (4) completes
(a) (b) (c) (d) the figure matrix [2012-I]
4. Problem Figure
8.

Answer Figure

(a) (b) (c) (d) (1) (2) (3) (4)


(a) 1 (b) 2 (c) 3 (d) 4
EBD_8177
C-26 Analogy/Odd One Out/Series

13.
9.

1 2 3 4 5

(a) (b) (c) (d)


14.

(1) (2) (3) (4)


1 2 3 4 5
(a) 1 (b) 2 (c) 3 (d) 4

10. (a) (b) (c) (d)


15.

1 2 3 4 5

(1) (2) (3) (4)


(a) 1 (b) 2 (c) 3 (d) 4 (a) (b) (c) (d)
16.
11.

1 2 3 4 5

(a) (b) (c) (d)


(1) (2) (3) (4) 17.
CC CC C
(a) 1 (b) 2 (c) 3 (d) 4 S S S S S

1 2 3 4 5
12.
C
C C C
S S S
S
(a) (b) (c) (d)
DIRECTIONS (Qs. 18 - 22) : There are two sets of figures namely
the Problem figures containing five figures 1, 2, 3, 4, 5 and answer
figures (a), (b), (c), (d). You have to select one figure from the
Answer figures which will continue the same series as given in
(1) (2) (3) (4) the Problem figures. [2013-I]

(a) 1 (b) 2 (c) 3 (d) 4 18.


DIRECTIONS (Qs. 13 - 17): There are two sets of figures namely =
the Problem figures containing five figures 1, 2, 3, 4, 5 and An-
swer figures (a), (b), (c), (d). Select one figure from the Answer (1) (2) (3) (4) (5)
figures which will continue the same series as given in the Prob- + +
lem figures. [2012-II]

(a) (b) (c) (d)


Analogy/Odd One Out/Series C-27

19.

25. (a) (b)


(1) (2) (3) (4) (5)

(c) (d)
(a) (b) (c) (d)
20.

26. (a) (b)


(1) (2) (3) (4) (5)

(c) (d)
(a) (b) (c) (d)
21.
DIRECTIONS (Qs. 27 - 30) : Each of the following questions
consists of five figures marked 1, 2, 3, 4 and 5 called the Problem
Figures followed by four other figures marked a, b, c and d
(1) (2) (3) (4) (5) called the Answer Figures. select a figure from amongst the answer
Figures which will continue the same series as established by
the five Problem Figures. [2014-II]
Problem Figures: Answer Figures:
(a) (b) (c) (d) 27.
22.
(1) (2) (3) (4) (5) (a) (b) (c) (d)

28. Problem Figures: Answer Figures:


(1) (2) (3) (4) (5)

(1) (2) (3) (4) (5) (a) (b) (c) (d)

(a) (b) (c) (d) Problem Figures: Answer Figures:


29.
DIRECTIONS (Qs. 23 - 26) : In these tests find the odd figure
out: [2014-I]
(1) (2) (3) (4) (5) (a) (b) (c) (d)

30. Problem Figures: Answer Figures:


23. (a) (b)

(1) (2) (3) (4) (5) (a) (b) (c) (d)


(c) (d) DIRECTIONS (Qs. 31 - 34) : Each of the following questions
consists of two sets of figures. Figures 1, 2, 3 and 4 constitute the
problem Set while figures (1) and (2). Establish a similar
relationship between figures (3) and (4) by selecting a suitable
24. (a) (b) figure from the Answer set that would replace the question mark
(?) in fig. (4). [2014-II]

31. Problem Figures: Answer Figures:

(c) (d) ?
(1) (2) (3) (4) (a) (b) (c) (d)
EBD_8177
C-28 Analogy/Odd One Out/Series

32. Problem Figures: Answer Figures: 41.

?
(a) (b) (c) (d)
(1) (2) (3) (4) (a) (b) (c) (d)
42.
33. Problem Figures: Answer Figures:

? (a) (b) (c) (d)


(1) (2) (3) (4) (a) (b) (c) (d)
43.
34. Problem Figures: Answer Figures:

? (a) (b) (c) (d)


DIRECTIONS (Qs. 44 - 48) : Each of the following questions
(1) (2) (3) (4) (a) (b) (c) (d) consists of two sets of figures. Figures 1, 2, 3 and 4 constitute the
DIRECTIONS (Qs. 35 - 38) : In each problem, out of the four Problem Set while figures a, b, c and d constitute the Answer Set.
figures marked (a) (b) (c) (d) three are similar in a certain There is a definite relationship between figures (1) and (2).
Establish a similar relationship between figures (3) and (4) by
manner. However, one figure is not like the other three. Choose
selecting a suitable figure from the Answer set that would replace
the figure which is different from the rest. [2014-II] the question mark (?) in fig. (4). [2015-I]
44.
35.
?

(1) (2) (3) (4)


(a) (b) (c) (d)

36.

(a) (b) (c) (d)


(a) (b) (c) (d) 45.

37. ?
(1) (2) (3) (4)

(a) (b) (c) (d)


38.
(a) (b) (c) (d)
46.
(a) (b) (c) (d)
?
DIRECTIONS (Qs. 39 - 43) : In each problem, out of the four
(1) (2) (3) (4)
figures marked (a) (b) (c) and (d) three are smililar in a certain
manner. However, one figure is not like the other three. Choose
the figure which is different from the rest. [2015-I]

39.
(a) (b) (c) (d)
47.

(a) (b) (c) (d) ?

40. (1) (2) (3) (4)

(a) (b) (c) (d)


(a) (b) (c) (d)
Analogy/Odd One Out/Series C-29

52. Problem Figures:


48.
?

(1) (2) (3) (4) (1) (2) (3) (4) (5)


Answer Figures:

(a) (b) (c) (d) (a) (b) (c) (d)


DIRECTIONS (Qs. 49 - 51) : Each of the following questions 53. Problem Figures:
consists of five figures marked 1, 2, 3, 4 and 5 called the Problem
Figures followed by four other figures marked a, b, c and d
called the Answer Figurs. Select a figure from amongst the Answer
Figures which will continue the same series as established by
the Five Problem Figures. [2015-I] (1) (2) (3) (4) (5)
Answer Figures:
49. Problem Figures :
S

(1) (2) (3) (4) (5)


(a) (b) (c) (d)
Answer Figures : 54. Problem Figures:
S

(a) (b) (c) (d) (1) (2) (3) (4) (5)


50. Problem Figures : Answer Figures:

(1) (2) (3) (4) (5) (a) (b) (c) (d)


Answer Figures :
55. Problem Figures:

(a) (b) (c) (d)


51. Problem Figures: (1) (2) (3) (4) (5)
Answer Figures

(1) (2) (3) (4) (5)


Answer Figures :
(a) (b) (c) (d)
56. Problem Figures:

(a) (b) (c) (d)


DIRECTIONS (Qs. 52 - 56) : Each of the following questions
(1) (2) (3) (4) (5)
consists of five figures marked 1, 2, 3, 4 and 5 called the Problem
Figures followed by four other figures marked (a), (b), (c) and Answer Figures:
(d) called the answer. Select a figure from amongst the answer
figures which will continue the some series as established by the
five Problem Figures: [2015- II]
(a) (b) (c) (d)
EBD_8177
C-30 Analogy/Odd One Out/Series

DIRECTIONS (Qs. 57 - 61) : In each problem, out of the four 63.


figures marked (a) (b), (c) and (d), three are similar in a certain
manner. However, one figure is not like the other three. Choose
the figure which is different from the rest. [2015-II]
(a) (b) (c) (d)
57. 64.

(a) (b) (c) (d) (a) (b) (c) (d)

58. 65.

(a) (b) (c) (d)


(a) (b) (c) (d) 66.

59. (a) (b) (c) (d)

67.
(a) (b) (c) ` (d)
(a) (b) (c) (d)
60. DIRECTIONS (Qs. 68 - 70) : In each problem, out of the four
figures marked (a) (b) (c) and (d), three are similar in a certain
manner. However, one figure is not like the other three. Choose
the figure which is different from the rest: [2016 - II]
(a) (b) (c) (d)

61. 68.
(a) (b) (c) (d)

(a) (b) (c) (d)


69.
DIRECTIONS (Q. 62 ): Each of the following questions consists
of five figures marked 1,2,3,4 and 5 called the Problem Figures (a) (b) (c) (d)
followed by four other figures marked a, b, c and d called the
Answer Figures. Select a figure from amongst the Answer Figures 70.
which will continue the same series as established by the five
Problem Figures. [2016 -I] (a) (b) (c) (d)
DIRECTIONS (Qs. 71 - 75) : Each of the following questions
62. Problem Figures: consists of five figures marked 1, 2, 3, 4 and 5 called the Problem
Figures followed by four other figures marked (a), (b), (c) and
(d) called the Answer Figures. Select a figure from amongst the
Answer Figures which will continue the same series as estab-
lished by the five Problem Figures: [2016 - II]
Answer Figures: 71. Problem Figures:

(1) (2) (3) (4) (5)


Answer Figures:
DIRECTIONS (Qs. 63 - 67): In each problem, out of the four
figures marked (a), (b) (c) and (d) three are similar in a certain
manner. However, one figure is not like the other three. Choose
the figure which is different from the rest. [2016-I]
(a) (b) (c) (d)
Analogy/Odd One Out/Series C-31

72. Problem Figures: 76. Select a figure from amongst the Answer Figures which will
continue the same series as established by the five Problem
Figures.
Problem Figures: Answer Figures:
(1) (2) (3) (4) (5)
Answer Figures:
(1) (2) (3) (4) (5) (a) (b) (c) (d)
77. Select a figure from amongst the Answer Figures which will
continue the same series as established by the five Problem
Figures.
(a) (b) (c) (d) Problem Figures: Answer Figures:
73. Problem Figures:

(1) (2) (3) (4) (5) (a) (b) (c) (d)


78. Select a figure from amongst the Answer Figures which will
continue the same series as established by the five Problem
(1) (2) (3) (4) (5) Figures.
Answer Figures: Problem Figures: Answer Figures:

(1) (2) (3) (4) (5) (a) (b) (c) (d)


(a) (b) (c) (d) 79. Select a figure from amongst the Answer Figures which will
74. Problem Figures: continue the same series as established by the five Problem
Figures.
Problem Figures: Answer Figures:

(1) (2) (3) (4) (5)


(1) (2) (3) (4) (5) (a) (b) (c) (d)
Answer Figures:
80. Select a figure from amongst the Answer Figures which will
continue the same series as established by the five Problem
Figures.
Problem Figures: Answer Figures:
(a) (b) (c) (d)
75. Problem Figures:
(1) (2) (3) (4) (5) (a) (b) (c) (d)
DIRECTIONS (Qs. 81 - 83): In each problem, out of the five
figures marked (a), (b), (c) and (d), three are similar in a certain
(1) (2) (3) (4) (5) manner. However, one figure is not like the other three. Choose
Answer Figures: the figure which is different from the rest. [2017 - I]
81. Choose the figure which is different from the rest.

(a) (b) (c) (d)


DIRECTIONS (Qs. 76 - 80): Each of the following questions (a) (b) (c) (d)
consists of five figures marked 1, 2, 3, 4 and 5 called the Problem
82. Choose the figure which is different from the rest.
Figures followed by four other figures marked (a), (b), (c) and
(d) called the Answer Figures. Select a figure from amongst the
Answer Figures which will continue the same series as
established by the five Problem Figures. [2017 - I]
(a) (b) (c) (d)
EBD_8177
C-32 Analogy/Odd One Out/Series

83. Choose the figure which is different from the rest. 89. Choose the figure which is different from the rest.

(a) (b) (c) (d)


(a) (b) (c) (d) 90. Choose the figure which is different from the rest.
DIRECTIONS (Qs. 84 - 88): Each of the following questions
consists of five figures marked 1, 2, 3, 4 and 5 called the Problem
Figures followed by four other figures marked (a), (b), (c) and
(d) called the Answer Figures. Select a figure from amongst the (a) (b) (c) (d)
Answer Figures which will continue the same series as 91. Choose the figure which is different from the rest.
established by the five Problem Figures. [2017 - II]
84. Select a figure from amongst the Answer Figures which will
continue the same series as established by the five Problem (a) (b) (c) (d)
Figures. 92. Choose the figure which is different from the rest.
Problem Figures: Answer Figures:

(a) (b) (c) (d)


93. Choose the figure which is different from the rest.
85. Select a figure from amongst the Answer Figures which will
continue the same series as established by the five Problem
Figures. (a) (b) (c) (d)
Problem Figures: Answer Figures:
DIRECTIONS (Qs. 94-97): Each of the following questions
consists of five figures marked A, B, C, D and E called the Problem
Figures followed by five other figures marked (a), (b), (c), and
(d) called the Answer Figures. Select a figure from amongst the
Answer Figures which will continue the same series as
86. Select a figure from amongst the Answer Figures which will
established by the five Problem Figures. [2018 - I]
continue the same series as established by the five Problem
94. Select a figure from amongst the Answer Figures which will
Figures.
continue the same series as established by the five Problem
Problem Figures: Answer Figures: Figures.
Problem Figures:

87. Select a figure from amongst the Answer Figures which will
continue the same series as established by the five Problem (A) (B) (C) (D) (E)
Figures. Answer Figures:
Problem Figures: Answer Figures:

(a) (b) (c) (d)


88. Select a figure from amongst the Answer Figures which will 95. Select a figure from amongst the Answer Figures which will
continue the same series as established by the five Problem continue the same series as established by the five Problem
Figures. Figures.
Problem Figures:
Problem Figures: Answer Figures:

(A) (B) (C) (D) (E)


DIRECTIONS (Qs. 89 - 93): In each problem, out of the five Answer Figures:
figures marked (a), (b), (c) and (d), three are similar in a certain
manner. However, one figure is not like the other three. Choose
the figure which is different from the rest. [2017 - II] (a) (b) (c) (d)
Analogy/Odd One Out/Series C-33

96. Select a figure from amongst the Answer Figures which will Answer Figures
continue the same series as established by the five Problem
Figures.
Problem Figures:

(a) (b) (c) (d)


100. Question Figure

(A) (B) (C) (D) (E)


Answer Figures:
Answer Figures

(a) (b) (c) (d)


97. Select a figure from amongst the Answer Figures which will (a) (b) (c) (d)
continue the same series as established by the five Problem
101. Question Figure
Figures.
Problem Figures:

Answer Figures
(A) (B) (C) (D) (E)
Answer Figures:

(a) (b) (c) (d)


DIRECTIONS (Qs. 102-103) : There are two sets of figure given.
(a) (b) (c) (d) There is a definite relationship between first two. Establish a similar
relationship between third and fourth by selecting a suitable figure
98. Find the missing figure from the given responses. [2018 - II] from answer that would replace the question mark . [2019 - II]
Question Figures: 102. Question Figure

Answer Figures
Answer Figures:

(a) (b) (c) (d)


103. Question Figure
(a) (b) (c) (d)
DIRECTIONS (Qs. 99-101) : Three complete and fourth blank
space is given, choose the set of figures which follows the rule ?
and would replace the blank space given in question figure from
the four alternatives given. [2019 - II]
Answer Figures
99. Question Figure

(a) (b) (c) (d)


EBD_8177
C-34 Analogy/Odd One Out/Series

DIRECTIONS (Qs. 104-105): In the following four figures (a), DIRECTION (Qs. 107): There are two sets of figure given. There
(b), (c) and (d) have been given in each question. Of these four is a definite relationship between first two. Establish a similar
figures, three figures are similar in some way and one figure is relationship between third and fourth by selecting a suitable
different. Select the figure which is different. [2020 - I] figure from answer that would replace the question mark.
[2020 - I]
107. Question Figure

104. (a) (b)


?

Answer Figures

(c) (d)

(a) (b) (c) (d)


DIRECTION (Q. 108) : In Question, one part of a geometrical

105. (a) RUN (b) UNR figure is as question figure and the other one is among the four
answer figures (a), (b), (c) and (d). Find the figure that completes
the geometrical figure. [2020 - I]
108. Question Figure

(c) NKU (d) RNU


DIRECTION (Qs. 106): There are three question figure and the
space for the fourth figure is left blank. The question figures are Answer Figures
in a series. Find out one figure from among the answer figures
given which occupies the blank space for the fourth figure and
completes the series. [2020 - I]
(a) (b)
106. Question Figures

(c) (d)
Answer Figures

(a) (b)

(c) (d)
Analogy/Odd One Out/Series C-35

ANSWERS & EXPLANATIONS


1. (d) 2. (c) 38. (a) All others have four parts.
3. (a) Each step, horizontal lines gets rotated and increased 39. (c) In all other options line with arrow is shifting three
by 1 and vertical lines rotated as well decreased by 1. steps forward clockwise, while in option (c) it is shifting
4. (c) two steps forward clockwise.
5. (c) Figure is rotated by 90° every time. 40. (d) In option (d) curves in vertical line are not following
6. (d) 7. (c) pattern.
8. (d) In each row, the first unit is deleted in the third unit. 41. (a) In all other options arrow cuts on the line and dot is
9. (d) Answer figure (d) will complete the matrix. shifting 90° anti clockwise.
10. (d) Answer figure (d) will complete matrix. 42. (a) Designs are not following the pattern.
11. (b) Answer figure (b) will complete matrix.
43. (d)
12. (b) Answer figure (b) will complete matrix.
13. (a) 14. (a) 15. (c) 16. (d) 17. (d) 44. (a)
18. (a) ‘V’ shifts by half and rotate by 45° and another figure
shift alternatively by 45° and 90° and figure also changes.
19. (c)
20. (d) In all even number of figures outer most line increases Pattern follows (1) to (2)
by one shift by 60° and innermost line increases by and after they change the diagonal of the respective
one in even figures and shift by 60° in odd figures. image have one less than shapes.
21. (b) Figure rotates by 90° and everytime one extra line is 45. (d) 46. (c)
added to figure. 47. (d) Lower portion of the figure (1) remains same while upper
22. (a) 23. (c) portion gets inverted in the figure (2).
24. (b) In this figure shaded regions covers all the three circles 48. (c) Figure (1) gets inverted by 180° then its mirror image is
and in rest figures only two circles. formed in 2nd figure. Hence (c) is the correct choice.
25. (d) 26. (d) 27. (d) 49. (d) •( ) line is rotating 135° anticlockwise.
28. (b) The whole figures inverts alternatively upside and
downside •( ) line is rotating 45° clockwise.
• figure which is 4th position in problem figure 1 change
its position 1st, 2nd, 3rd, 4th and again 1st places. •( ) line is changing its design at third and fifth step.
• T figure shifts 1st and 2nd place alternatively. 50. (c) Each branch is rotating 45° anticlock-wise and new
• Y figure goes from 2nd to 3rd, 3rd to 4th, 4th to 1st and branch is added in each step.
1st to 2nd 51. (d) Dots shifts two steps clockwise in each step, everytime
• T figurer shifts 3rd and 4th place alternatively. one arrow gets reduced in Horizontally and lines
changes its position also.
29. (a)
52. (a)
• change its directions after 2nd step, so (d) is
eliminated.
• changes its direction after 1st step and then 2nd
step so, (b) is eliminated.
• changs its direction after 2nd step so (c) is 
eliminated.
Hence, (a) is the answer.

30. (a) Upper arrow becomes half and rotates clockwise by
45° middle arrow rotates anticlockwise by 45°. (1) (2)
31. (c) Dots remain on the same place, the rest figures
becomes invert.
32. (b) Upper figure (=) shifts diagnally opposite direction.
Bottom figure ( m ) shifts 90° antinclockwise and
becomes invert.
33. (a) The whole figure is devided into eight sections. 
34. (d) The inner figure becomes outer and outer becomes 
inner and got shading.
35. (c)
(5)
36. (a) In all other figures bottom right corners have been
shaded. Same shifting pattern is following in fig (3) & fig (4)
37. (b) Arrows are not in the same direction. so option (a) is correct.
EBD_8177
C-36 Analogy/Odd One Out/Series

53. (a) 66. (d) Clearly every figure except (d) has pointed arrow in the
direction of curve.
54. (a) Fig (1) Fig (2)
Ex. (a) arrow in the direction of curve.
67. (a)

68. (a)

Q in all the three diagram except (a) is like


toward
Interchanging Position
with reverse in direction 69. (a) +
Same pattern is following in fig (3) & fig (4) so for fig (5)
×
Q Figure in Middle is in upward direction where as figure
option (a) is correct.
in Middle is in downward direction in remaining 3 figure.
55. (d) Fig (1) Fig (2)

70. (d)

is Rotating Clockwise around in Remaining 3.


71. (d)
72. (b)
longer needle is rotating 135° clockwise and shorter
needle is rotating 45° anticlockwise. Same pattern is
following in fig (3) and fig (4). So for fig (5) option (d) is
correct.
56. (c) Fig (1) Fig (2) ← ↑ In each step, it moves one side in anti-clockwise
direction.
= ↑ It moves allernate one-half side and half side in
each option.
↑ It moves alternate half side and one-half side in
each option.
In the figures ‘x’ is converting into ‘+’ sign and there is ↑ It moves one side and change into new element.
a interchange of circled design in clock-wise direction. So, answer is option (b).
Same pattern is following in fig (3) and fig (4) also. So 73. (c)
for figure (5) option (c) is correct. 74. (a) Step - I ↑ Arrow and curve are added.
57. (c) In figure (a), (b), (d) marked angles are equal. Only in Step - II ↑ Arrow rotates oppasite direction and curve
option (c) marked angles are not equal. moves outside. So, answer is option (a).
58. (b) All the three leaves (except dark one) are placed right 75. (d) 76. (c) 77. (d) 78. (d)
79. (c) 80. (b)
angled to each other in fig (a), (c) and (d). So option (b)
81. (a) The pins, equal in number to the number of sides in the
is different from others. main figure are attached to the midpoint of a side of the
59. (d) In option (a), (b) and (c), the first dot (.) is adjucent to main figure in case of figures (b), (c), and (d). In fig. (a),
darken triangle and second dot (.) is one step behind. these pins are attached to a vertex of the main figure
So option (d) is different. 82. (c) In all other figures, the two line segments are parallel to
60. (a) Option (a) is different from others. each other.
61. (b) In option (a), (c) and (d) dots (.) are placed on the 83. (d) In all other figures, the arrow and the V sign lie towards
midpoints of the edges but in option (b) dots are placed the black end of the main figure.
at the corners. 84. (a) 85. (c) 86. (b) 87. (d) 88. (a)
62. (b) 89. (d) All other figures are divided into equal parts.
63. (a) All except in option (a) are in the same direction. 90. (a) All other figures can be rotated into each other.
64. (a) The pins, equal in number to the number of sides in the 91. (b) Each one of the figures except fig. (b), consists of five
main figure are attached to the midpoint of a side of the arrowheads.
main figure in case of figures (2), (3), (4) and (5). In fig. 92. (d) Only in fig. (d), both the parallel lines are bent in the
(1), these pins are attached to a vertex of the main figure. same direction {i.e. towards the left).
65. (c)
Analogy/Odd One Out/Series C-37

93. (a) In all other figures, as we move from the innermost to 99. (b) In each next step number of leaves are increased by one
the outermost element, the numbers of sides of the on alternate side.
elements either increase or decrease in a sequence. 100. (d) Number of sides in each next figure is increased by 2.
94. (c) All the three symbols in the dice are rotating clockwise. 101. (b) Number of petals in each next flower is decreased by 1
So option (c)
102. (b) Line segments are removed from the bottom right corner.
95. (c) the arrow and small line inside the small square are
103. (b) Inner and outer shapes are interchanged and the inner
rotating constantly anti clockwise and clockwise
shape becomes unshaded.
respectively by 90°, 45°, 90°, 45°,... and 45°, 90°, 45°,
90°. So next figure would be option (c). 104. (b) Number of circles is equal to the number of sides in
96. (d) In each step, the CW-end element moves to the ACW- the geometric figure enclosed by the circle. In option
end position. (b) a quadrilateral have four sides, so four circles
97. (c) In each step, one line segment is lost from the CW-end should be there but it is only three circles.
of the outer element and a new line segment appears at 105. (c) Letter ‘R’, ‘N’ and ‘U’ are present in option (a), (b) and
the ACW-end. Also, the inner 'L' shaped element rotates (d). But in option (c) letter N, K and U are present.
90°CW in each step. 106. (c)
98. (c) In each subsequent figure one dot is added and 107. (b) The second figure of first pair is the mirror image of
alternatively dots become white. Again, in each
first figure.
subsequent figure one arrow is deleted.
108. (a)
EBD_8177
C-38 Completion Of Figure/Embedded Figure

Completion Of
5 Figure/Embedded Figure
DIRECTIONS (Qs. 1 - 9) : The hidden figures test is designed to
perceive simple figures in complex drawings. At the top of each 4.
section are five figures lettered a, b, c, d, and e. Below these are
several numbered drawings. You must determine which lettered
figure is embedded in each of the numbered drawings. [2011-I]
a b c d e

a b c d e
5.

1.

a b c d e
a b c d e

a b c d e

2.

6.

a b c d e a b c d e
3.

7.

a b c d e
a b c d e
Completion Of Figure/Embedded Figure C-39

14.

(X) (1) (2) (3) (4)


(a) 1 (b) 2
8. (c) 3 (d) 4
DIRECTIONS (Qs. 15 - 19) : In each of the following questions
you are given a figure (X) followed by four alternative figures
(1), (2), (3) and (4) such that figure (X) is embedded in one of
a b c d e them. Trace out the alternative figure which contains fig. (X) as
its part. [2012-I]

15.

9. (X) (1) (2) (3) (4)


(a) 1 (b) 2
(c) 3 (d) 4

a b c d e 16.

DIRECTIONS (Qs. 10 - 14) : In each of the following questions


select a figure from amongst the four alternatives, which when (X) (1) (2) (3) (4)
placed in the blank space of figur66666e (X) would complete (a) 1 (b) 2
the pattern. [2012-I] (c) 3 (d) 4

10. 17.

(1) (2) (3) (4) (X) (1) (2) (3) (4)


(X)
(a) 1 (b) 2
(a) 1 (b) 2 (c) 3 (d) 4
(c) 3 (d) 4
18.
11.

(X) (1) (2) (3) (4)


(X) (1) (2) (3) (4)
(a) 1 (b) 2
(a) 1 (b) 2 (c) 3 (d) 4
(c) 3 (d) 4
19.
12.

(X) (1) (2) (3) (4)


(X) (1) (2) (3) (4) (a) 1 (b) 2
(c) 3 (d) 4
(a) 1 (b) 2 DIRECTIONS (Qs 20 - 24) : The original figure is embedded or
(c) 3 (d) 4 hidden inone of the answer figures (a), (b), (c) and (d). Select
the alternative that carries the correct figure which clearly shows
13. the embedded portion of the original figure.

(X) (1) (2) (3) (4) 20.

(a) 1 (b) 2
(c) 3 (d) 4 (X) (a) (b) (c) (d)
EBD_8177
C-40 Completion Of Figure/Embedded Figure

21.

(X) (a) (b) (c) (d)


(a) (b) (c) (d)
22. 28.

(X) (a) (b) (c) (d)


(X)
23.

(X) (a) (b) (c) (d)


24. (a) (b) (c) (d)
29.

(X) (a) (b) (c) (d)


(X)
DIRECTIONS (Qs. 25 - 29) : In this portion, a figure is given as
an original figure (X) followed by four answer figures. The
original figure is embedded or hidden in one of the answer figures
(a, b, c & d). Select the alternative that carries the correct figure
which clearly shows the embedded portion of the original figure.
[2013-I] (a) (b) (c) (d)
25. DIRECTIONS (Qs. 30 - 34) : In each of the following questions,
a part of the figure is missing. Find out from the given option (a),
(b), (c) or (d), the right figure to fit in the missing figure (X).
[2013-I]

(X) 30.
?

(a) (b) (c) (d) (X)


26.

(a) (b) (c) (d)


(X) 31.

?
(a) (b) (c) (d)
27. (X)

(X) (a) (b) (c) (d)


Completion Of Figure/Embedded Figure C-41

32.
?
(a) (b) (c) (d)

(X) ?
34.

(a) (b) (c) (d) (X)

33.
?

(a) (b) (c) (d)


(X)

DIRECTIONS (Qs. 35-38): In these tests you will find an Incomplete Figure and four Answer Figures. You have to select one
diagram from the Answer Figures which fits into the blank column in Incomplete Figure in order to complete it: [2014-I]

Incomplete Figure

35. Answer Figures

?
(a) (b) (c) (d)

Incomplete Figure
36.
Answer Figures
?
(a) (b) (c) (d)

Incomplete Figure
37.
Answer Figure

(a) (b) (c) (d)


EBD_8177
C-42 Completion Of Figure/Embedded Figure

Incomplete Figure
38. Answer Figures

(a) (b) (c) (d)

DIRECTIONS (Qs. 39 - 41) : In these tests Figure X is Hidden in the option figures. Find the correct option. [2014-I]
39.

X (a) (b) (c) (d)


40.

X (a) (b) (c) (d)


41.

X (a) (b) (c) (d)

DIRECTIONS (Qs. 42 - 44) : In each of the following quesions, 46.


select a figure from amongst the four alternatives which when ?
placed in the blank space of figure (X) would complete The
pattern. [2014-II]
42. (X) (a) (b) (c) (d)
47..
?
(X) (a) (b) (c) (d) ?
43.
(X) (a) (b) (c) (d)
48.
?
(X) (a) (b) (c) (d) ?
44.
(X) (a) (b) (c) (d)
?
(X) (a) (b) (c) (d) 49. ?
DIRECTIONS (Qs. 45 - 49) : In each of the following questions,
select a figure from amongst the four alternatives, which when
placed in the blank space of figure (X) would complete the (X) (a) (b) (c) (d)
pattern. DIRECTIONS (Qs. 50 - 54) : Each of the following questions
45. consists of two sets of figures. Figures 1, 2, 3 and 4 constitute the
? Problem Set while figures (a), (b), (c) and (d) constitute the
Answer Set. There is a definite relationship between figures (1)
and (2). Establish a similar relationship between figures (3)
and (4) by selecting a suitable figure from the Answer Set that
(X) (a) (b) (c) (d) would replace the question mark (?) in fig (4). [2015- II]
Completion Of Figure/Embedded Figure C-43

50.

(1) (2) (3) (4) (a) (b) (c) (d)

51.

(1) (2) (3) (4) (a) (b) (c) (d)


52.

(1) (2) (3) (4) (a) (b) (c) (d)


53.

(1) (2) (3) (4) (a) (b) (c) (d)


54.

(1) (2) (3) (4) (a) (b) (c) (d)

DIRECTIONS (Qs. 55 - 59) : In each of the following questions, 59.


select a figure from amongst the four alternatives, which when
placed in the blank space of figure (X) would complete the
pattern. [2015- II]

55.
(X) (a) (b) (c) (d)
DIRECTIONS (Qs. 60 - 63): Each of the following questions
consists of five figures marked 1,2,3,4 and 5 called the Problem
Figures followed by four other figures marked a, b, c and d
called the Answer Figures. Select a figure from amongst the
(X) (a) (b) (c) (d)
Answer Figures which will continue the same series as
established by the five Problem Figures. [2016 -I]
56. 60. Problem Figures:

(X) (a) (b) (c) (d)


57.
Answer Figures:

(X) (a) (b) (c) (d)


61. Problem Figures:
58.

(X) (a) (b) (c) (d)


EBD_8177
C-44 Completion Of Figure/Embedded Figure

Answer Figures:

66. ?
(1) (2) (3) (4)

62. Problem Figures:

(a) (b) (c) (d)


Answer Figures:
67.

(1) (2) (3) (4)

63. Problem Figures:

(a) (b) (c) (d)

68.
Answer Figures:

(1) (2) (3) (4)

DIRECTIONS (Qs. 64 - 68): Each of the following questions


consists of two sets of figures. Figures 1, 2, 3 and 4 constitute the (a) (b) (c) (d)
Problem Set while figures a, b, c and d constitute the Answer Set. DIRECTIONS (Qs. 69 - 72): In each of the following questions,
There is a definite relationship between figures (1) and (2). select a figure from amongst the four alternatives, which when
Establish a similar relationship between figures (3) and (4) by placed in the blank space of figure (X) would complete the
selecting a suitable figure from the Answer set that would replace pattern. [2016 -I]
the question mark (?) in figures. [2016 -I]

69.
64.
(X) (a) (b) (c) (d)

(1) (2) (3) (4)


70.

(X) (a) (b) (c) (d)

(a) (b) (c) (d)


71.
65.
(X) (a) (b) (c) (d)
(1) (2) (3) (4)
72.

(a) (b) (c) (d) (X) (a) (b) (c) (d)


Completion Of Figure/Embedded Figure C-45

DIRECTIONS (Qs. 73 - 77): In each of the following questions, 79. Select a suitable figure from the four alternatives that would
figure (X) is given followed by four alternative figures (a), (b), complete the figure matrix.
(c) and (d). Figure (X) is embedded in one of the alternative
figures. Trace out the alternative which contains figure (X) as its
part. [2016 - II]

73.

(X)
(a) (b) (c) (d)
80. Select a suitable figure from the four alternatives that would
complete the figure matrix.
(a) (b) (c) (d)
74.

(X)

(a) (b) (c) (d)


(a) (b) (c) (d) 81. Select a suitable figure from the four alternatives that would
75. complete the figure matrix.

(X) (a) (b) (c) (d)


76.

(X) (a) (b) (c) (d) (a) (b) (c) (d)


77. 82. Select a suitable figure from the four alternatives that would
complete the figure matrix.

(X) (a) (b) (c) (d)


DIRECTIONS (Qs. 78 - 82): In each of the following questions,
find out which of the answer figures (a), (b), (c) and (d) completes
the figure matrix? [2017 - I]
78. Select a suitable figure from the four alternatives that would
complete the figure matrix. (a) (b) (c) (d)
DIRECTIONS (Qs. 83 - 85): Each of the following questions
consists of two sets of figures. Figures (1), (2), (3) and (4)
constitute the Problem Set while figures (a), (b), (c), and (d)
constitute the Answer Set. There is a definite relationship between
figures (1) and (2). Establish a similar relationship between figures
(3) and (4) by selecting a suitable figure from the Answer Set that
would replace the question mark (?) in fig. (4). [2017 - I]
(a) (b) (c) (d)
EBD_8177
C-46 Completion Of Figure/Embedded Figure

83. Select a suitable figure from the Answer Figures that would 90. Identify the figure that completes the pattern.
replace the question mark (?).
Problem Figures: Answer Figures:

(X) (a) (b) (c) (d)


(1) (2) (3) (4) (a) (b) (c) (d) DIRECTIONS (Qs. 91 - 94): Each of the following questions
84. Select a suitable figure from the Answer Figures that would consists of two sets of figures. Figures (1), (2), (3) and (4)
replace the question mark (?). constitute the Problem Set while figures (a), (b), (c), and (d)
Problem Figures: Answer Figures: constitute the Answer Set. There is a definite relationship between
figures (1) and (2). Establish a similar relationship between figures
(3) and (4) by selecting a suitable figure from the Answer Set that
would replace the question mark (?) in fig. (4). [2017 - II]
(1) (2) (3) (4) (a) (b) (c) (d) 91. Select a suitable figure from the Answer Figures that would
replace the question mark (?).
85. Select a suitable figure from the Answer Figures that would Problem Figures: Answer Figures:
replace the question mark (?).
Problem Figures: Answer Figures:
(1) (2) (3) (4) (a) (b) (c) (d)
92. Select a suitable figure from the Answer Figures that would
replace the question mark (?).
(1) (2) (3) (4) (a) (b) (c) (d)
Problem Figures: Answer Figures:
DIRECTIONS (Qs. 86 - 90): In each of the following questions,
select a figure from amongst the four alternatives, which when
placed in the blank space of figure (X) would complete the
(1) (2) (3) (4) (a) (b) (c) (d)
pattern. [2017 - II]
93. Select a suitable figure from the Answer Figures that would
86. Identify the figure that completes the pattern. replace the question mark (?).
Problem Figures: Answer Figures:

(X) (a) (b) (c) (d) (1) (2) (3) (4) (a) (b) (c) (d)

87. Identify the figure that completes the pattern. 94. Select a suitable figure from the Answer Figures that would
replace the question mark (?).
Problem Figures: Answer Figures:

(1) (2) (3) (4) (a) (b) (c) (d)


(X) (a) (b) (c) (d)
DIRECTIONS (Qs. 95-98) : From the given answer figures, select
the one in which the question figure is hidden/embedded.
88. Identify the figure that completes the pattern.
[2018 - I]
95. Question Figure:

(X) (a) (b) (c) (d)


89. Identify the figure that completes the pattern.
Answer Figures:

(X) (a) (b) (c) (d)


(a) (b) (c) (d)
Completion Of Figure/Embedded Figure C-47

96. Question Figure 100. What comes next in the series? [2018 - I]
Question Figures :

Answer Figures Answer Figures :

(a) (b) (c) (d)


(a) (b) (c) (d) 101. Identify the figure that will complete the pattern.
97. Question Figure [2018 - II]
Question Figure

Answer Figures

(a) (b) (c) (d)


98. Question Figure:

(a) (b)

Answer Figures :
(c) (d)

(a) (b)
102. In a row of 64 girls, Anu is 17th from the left. Jagrati is 11th
to the right of Anu. What is Jagrati's position from the right
end of the row? [2018 - II]
(a) 36th (b) 37th
(c) (d) (c) 38th (d) 40th
103. Consider the following figure: [2018 - II]
99. Find the missing figure of the series from the given responses.
[2018 - I]
Question Figures :

?
Answer Figures :
Which of the following alternatives should replace the
question mark ?

(a) (b) (c) (d) (a) (b) (c) (d)


EBD_8177
C-48 Completion Of Figure/Embedded Figure

104. Which figure completes the statement? [2018 - II] Answer Figure:

(a) 1 (b) 3
(c) 4 (d) 5
105. Which one of the Answer Figures shall complete the given (a) (b) (c) (d)
question figure? [2018 - II] DIRECTIONS (Qs. 108-110): Find the missing figure in the
Question Figures: series from the given answer figures. [2018 - II]
108. Question Figure:

Answer Figures:
Answer Figure:

(a) (b) (c) (d)


106. Which answer figure will complete the pattern in the question
(a) (b) (c) (d)
figure? [2018 - II]
109. Question Figure:
Question Figure:

Answer Figure:

Answer Figure:

(a) (b) (c) (d)


110. Question Figure:

(a) (b) (c) (d)


107. Which answer figure will complete the pattern in the question
figure? [2018 - II] Answer Figure:
Question Figure:

(a) (b) (c) (d)


Completion Of Figure/Embedded Figure C-49

DIRECTIONS (Qs. 111-113): Choose a right figure from the set (a) 1 (b) 2
of answer figures which would replace the question mark (?) (c) 3 (d) 4
[2018 - II] 115. Find out the alternative figure which contains figure (X) as
111. Question Figure: its part.

(X) (1) (2) (3) (4)


Answer Figure: (a) 1 (b) 2
(c) 3 (d) 4
116. Which answer figure will complete the pattern in the
following question figure? [2019-I]

(a) (b) (c) (d)


112. Question Figure:
(a) (b)

(c) (d)
117. Which figure completes the statement? [2019-I]
Answer Figure:
Is to as

is to
(a) (b) (c) (d)
113. Question Figure:
(a) (b)

(c) (d)

Answer Figure: 118. Look at the patterns in the squares and understand their
relationship to one another so as to fill in the square with
missing symbols. [2019-I]

(a) (b) (c) (d)


DIRECTIONS (Qs. 114-115): In each of the following questions,
you are given a figure (X) followed by four alternative figures
(1), (2), (3) and (4) such that figure (X) is embedded in one of
them. Trace out the alternative figure which contains fig. (X) as
its part. [2018 - II]
114. Find out the alternative figure which contains figure (X) as
its part.
(a) (b)

.
(c) (d)
(X) (1) (2) (3) (4)
EBD_8177
C-50 Completion Of Figure/Embedded Figure

119. Replace ‘?’ by the appropriate figure from the given options. 122. Question Figures:
[2019-I]

Answer Figures:

(a) (b) (c) (d)


(a) (b)
123. Question Figures:

(c) (d) Answer Figures:

120. Which answer figure will complete the question figure?


(a) (b) (c) (d)
[2019-I]
124. Question Figures:

Answer Figures:

(a) (b) (c) (d)


DIRECTIONS (Qs. 125-127): Choose a right figure from the set
of answer figures which would replace the question mark (?)
[2019-I]
125. Question Figures:
(a) (b)

Answer Figures:
(c) (d)

121. Which answer figure will complete the question figure? (a) (b) (c) (d)
[2019-I] 126. Question Figures:

Answer Figures:

(a) (b) (a) (b) (c) (d)


127. Question Figures:

(c) (d) Answer Figures:

DIRECTIONS (Qs. 122-124) : Complete the series of figures, by


selecting correct answer figure from the given responses.
(a) (b) (c) (d)
[2019-I]
Completion Of Figure/Embedded Figure C-51

DIRECTIONS (Qs. 128-130) : In questions 29 to 31 there is a

?
problem figure, a part of which is missing. Observe the answer
figures (a), (b), (c), (d) and find out the answer figure which 131.
without changing the direction, fits in the missing part of the
problem figure in order to complete the pattern in the problem
figure. [2019 - II]

(a) (b)

128.
?
(c) (d)

(a) (b)

132. ?
(c) (d)

(a) (b)

129.
?
(c) (d)

(a) (b) DIRECTIONS (Qs. 133-134): In the following questions there is a


problem figure, a part of which is missing. Observe the answer
figures (a), (b), (c), (d) and find out the answer figure which
without changing the direction, fits in the missing part of the
(c) (d) problem figure in order to complete the pattern in the problem
figure. [2020 - I]

? 133.

130.

(a) (b)

(a) (b)
(c) (d)

DIRECTIONS (Qs. 131-132) : There are three problem figures


and the space for the fourth figure is blank.The problem figures (c) (d)
are in a series. Find out one figure from among the answer
figures, which occupies the blank space for the fourth figure and
completes the series. [2019 - II]
EBD_8177
C-52 Completion Of Figure/Embedded Figure

DIRECTION (Q. 135): Question figure is followed by four answer


134.
figures namely (a), (b), (c) and (d). Find out the correct answer
figure from the given alternatives in which Question figure is
hidden. [2020 - I]
135. Question Figure

(a) (b)
Answer Figures

(c) (d)

(a) (b) (c) (d)

ANSWERS & EXPLANATIONS


1. (e) 2. (a) 3. (c) 4. (d) 5. (d) 53. (c)
6. (a) 7. (c) 8. (a) 9. (e) 10. (d) Dark
11. (c) 12. (b) 13. (c) 14. (c) 15. (a)
16. (b) 17. (d) 18. (b) 19. (b) 20. (d)
21. (c) 22. (b) 23. (b) 24. (c) 25. (b)
26. (d) 27. (b) 28. (d) 29. (a) 30. (c)
31. (d) 32. (b) 33. (b) 34. (a) 35. (a) 
36. (b) 37. (c) 38. (d) 39. (d) 40. (c)
41. (a) 42. (b) 43. (b) 44. (d) 45. (b)
46. (d) 47. (d) 48. (c) 49. (a) 50. (d)
51. (b) Fig (1) Fig (2) change in direction

according to above explaination option (c) is correct.


54. (d) From figure (1) to (2)
symbol moves 90° clockwise
symbol moves 90° anticlockwise
symbol moves 180° clockwise or anticlockwise
symbol moves 180° clockwise or anticlockwise
and is replaced by another symbol.
180° rotation
sequence occurs from figure (3) to (4) so, (d) is
So option (b) is correct. the correct answer.
52. (a) Fig (1) Fig (2)
55. (d) 56. (c) 57. (d) 58. (b)
59. (c) 60. (c) 61. (c) 62. (c)
63. (b)
64. (d) All the diagonals of the figure have been drawn. View
answer workspace report discuss in forum.
65. (d) Diagram shifted 45° anti-clock wise and every thing
Rotation of dark leaf is 135° (clock wise). Rotation of got reversed.
white leaf in 135° (anticlockwise). So for fig (3) option
(a) is correct.
Ex. and
Completion Of Figure/Embedded Figure C-53

66. (b)
67. (d) Arrow of first figure goes to position of pointed arrow
while the pointed arrow moves to the position of arrow 88. (d)
with 180° change in angle.
68. (c) Figure got reversed with one more line-segment adding
in figure (2) and one-circle changes into two circle.
69. (b) 70. (d) 71. (d)
89. (d)
72. (b)

73. (d)

74. (c) 90. (b)

91. (b) The figure gets rotated through 180°.


75. (d)
92. (c) The trapezium gets vertically inverted and move to the
middle right position; the pin rotates 90°CW and moves
76. (d)
to the lower-right position; the third element rotates
135°ACW.
77. (d) 93. (c) The figure rotates 45°ACW; the circle changes colour
(turns black if initially white and vice-versa). The 'L'-
shaped element shifts to the other side of the main figure.
78. (d) The third figure in each row comprises of parts which
are not common to the first two figures. 94. (c) The rightmost element enlarges to become the outer
79. (b) In each row, the second figure is obtained from the element; the leftmost element becomes the inner-lower
element and the middle element becomes the inner-
first figure by adding two mutually perpendicular line
upper element.
segments at the centre and the third figure is obtained
95. (c)
from the first figure by adding four circles outside the
main figure. 96. (c)
80. (c) In each column, the second figure (middle figure) is
obtained by removing the upper part of the first figure
(uppermost figure) and the third figure (lowermost
figure) is obtained by vertically inverting the upper
part of the first figure.
81. (c) The third figure in each row comprises of the parts
common to the first two figures. 97. (b) Option (b) is the hidden/ embedded figure.
82. (b) The third figure in each row comprises of parts which 98. (a)
are not common to the first two figures.
83. (c) The half-shaded leaf rotates 135oACW and the
unshaded leaf rotates 135oCW.
84. (a) The upper element is converted to an element similar 99. (c)
to the lower elements and each one of the lower
elements is converted to an element similar to the upper 100. (c) The series represents continuous alphabets starting
element. from K. Hence, N is the right answer.
85. (b) Each one of the upper elements is replaced by an 101. (c)
element similar to the lower element(s) and each one 102. (b) According to question,
of the lower elements is replaced by an element similar
to the upper element(s). Required Number = 64 – (17 + 11) + 1 = 37th
103. (d)
104. (a) In PF(1), top half darkened rectangle turned 900
86. (d) clockwise, middle half darkened rectangle turned anti-
clockwise 900 and bottom half darkened rectangle
turned clockwise by 900. So turn the rectangles in
PF(C), clockwise, anti-clockwise, and clockwise. So
87. (c) correct Option (a)
105. (b) 106. (d) 107. (a) 108. (a) 109. (b)
EBD_8177
C-54 Completion Of Figure/Embedded Figure

110. (b) 111. (c) 112. (d) 113. (d)

114. (b)
130. (b)

115. (c)
131. (c) In every step, shaded region moves 90° clockwise.
116. (c) 117. (b) 118. (a) 132. (c) One small circle is added on the outer side of the
119. (c) Similar figure reappears in every fourth step and each semicircle in every step.
time a figure reappears, it rotates through 90oACW.
133. (a)
120. (c) 121. (b) 122. (c) 123. (d) 124. (c)
125. (b) 126. (d)
127. (d)

134 (b)
128. (c)

135. (c)
129. (a)
Spatial Ability/Image
6 Analysis/Rotated Figure

DIRECTIONS (Qs. 1-9) : This test is designed to test your ability to visualize and manipulate objects in space. In each question, you
are shown a picture of a block. To the right of the pictured block, there are five choices shown as a, b, c, d and e. Select the choice
containing a block that is just like the pictured block at the left although turned in a different position. [2011-I]
a A
b c
B
d
C
e
D
f
E

1.

a Ab c
B Cd eD Ef

2.

a Ab Bc Cd De Ef

3.

a b c d e f

4.

a Ab Bc Cd De Ef

5.

a b c d e f

6.

a b c d e f
EBD_8177
C-56 Spatial Ability/Image Analysis / Rotated Figure
A B C D E

7.

a b
A B
c C
d D
e fE

8.

a b c d e f

9.

DIRECTIONS (Qs. 10-12) : Your task here is to look at the target


4 3
figure and decide which of the rotated figures below is identical to 2
12.
it. If you do not think any of the figures is same as the target shape
then choose the answer option (d) ‘none of these’. [2011-II]
5 5 2

5
10.
(a) (b) (c) (d)
DIRECTIONS (Qs 13-17) : Find out which of the figures (1), (2), (3)
and (4) can be formed from the pieces given in figure (X). [2012-I]

(a) (b) 13.

(X) (1) (2) (3) (4)


(a) 1 (b) 2
(c) (d) None of these (c) 3 (d) 4
14.

11.
(X) (1) (2) (3) (4)
(a) 1 (b) 2
(c) 3 (d) 4
(a) (b)
15.

(c) (d) None of these


(X) (1) (2) (3) (4)
(a) 1 (b) 2
(c) 3 (d) 4
Spatial Ability/Image Analysis / Rotated Figure C-57

From amongst the figures marked (a), (b), (c) and (d), select the
16.
figures which satisfies the same conditions of placement of the
dot as in fig. (X). [2013-I]

(X) (1) (2) (3) (4)

(a) 1 (b) 2
(c) 3 (d) 4 23.
17. Find out how the key figure (X) will look like after rotation.
(X)

(X) (1) (2) (3) (4)

(a) 1 (b) 2
(c) 3 (d) 4 (a) (b) (c) (d)

DIRECTIONS (Qs 18-22) : From amongst the figures marked


(a), (b), (c) and (d), select the figure which satisfies the same
conditions of placement of the dot as in fig. (X) [2012-II]

24.

18. (X)
(X) (a) (b) (c) (d)

19. (a) (b) (c) (d)


(X) (a) (b) (c) (d)

25.

20.
(X)
(X) (a) (b) (c) (d)

21.
(a) (b) (c) (d) (a) (b) (c) (d)
(X)

26.
22.
(X) (a) (b) (c) (d)
(X)
DIRECTIONS (Qs. 23-27) : The following situations involve a
cluster of three or more geometrical figures, having one or more
dots placed at any point inside the cluster. This cluster is followed
by a set of four alternative figures each composed of a cluster of
the same type of figures. Now, for each dot we have to observe the
region in which it is enclosed i.e. in which of the geometrical
(a) (b) (c) (d)
figure this region is common.
EBD_8177
C-58 Spatial Ability/Image Analysis / Rotated Figure

27. [2013-I]

(X)

(a) (b) (c) (d)


DIRECTIONS (Qs.28-31) :In these tests find which code matches the shape or pattern given at the end of each questions. [2014-I]

LS RQ LM LQ
28. (a) (b) (c) (d)
JS RM LM JQ RS ?

29.
SN DZ SM SZ
(a) (b) (c) (d)
DM KN KZ SZ DN ?

PR JR GS JK
30. (a) (b) (c) (d)
GR PK JS GK PS ?

31.
BF GB BG CG
(a) (b) (c) (d)
AF BG AH CG CH ?

DIRECTIONS (Qs. 32 - 36) In each of the following questions,


select a figure from amongst the four alternatives, which satisfies
the same conditions of placement of the dots as in figure (X): 34.
[2016 - II]
(X) (a) (b)

32.

(X) (a) (b)


(c) (d)

(c) (d) 35.

33. (X) (a) (b)

(X) (a) (b)

(c) (d)
(c) (d)
Spatial Ability/Image Analysis / Rotated Figure C-59

45. A piece of paper is folded and cut. From the figures given,
indicate how it will appear when opened [2019-I]
36.

(X) (a) (b)


(a) (b)

(c) (d)
(c) (d)
DIRECTIONS (Qs. 37-41) : In each of the following questions, DIRECTIONS (Qs. 46-47): In each of the following questions,
select a figure from amongst the four alternative, which satisfies you are given a figure (X) followed by four alternative figures
the same conditions of placement of the dots as in figure (X): (1), (2), (3) and (4) such that figure (X) is embedded in one of
[2018 - I] them. Trace out the alternative figure which contains fig. (X) as
its part. [2019-I]
37. 46. Find out the alternative figure which contains figure (X) as
its part.
(X) (a) (b) (c) (d)

38.
(X) (1) (2) (3) (4)
(a) 1 (b) 2
(X) (a) (b) (c) (d) (c) 3 (d) 4
47. Find out the alternative figure which contains figure (X) as
39.
its part.

(X) (a) (b) (c) (d)

40. (X) (1) (2) (3) (4)


(a) 1 (b) 2
(X) (a) (b) (c) (d) (c) 3 (d) 4
DIRECTIONS (Qs. 48-50) : In the following four figures (a), (b),
(c) and (d) are given, three are similar in a certain manner.
41. However, one figure is not like the other three. Choose the figure
(X) (a) (b) (c) (d) which is different from the rest. [2019 - II]
DIRECTIONS (Qs. 42-44) : Each of the questions , out of the four
figures marked (a), (b), (c) and (d). three are similar in a certain
manner. However, one figure is not like the other three. Choose 48.
the figure which is different from the rest:- [2018 - I]
(a) (b) (c) (d)
42.

49.
(a) (b) (c) (d)
43. (a) (b) (c) (d)

(a) (b) (c) (d) 50.


44.
(a) (b) (c) (d)

(a) (b) (c) (d)


EBD_8177
C-60 Spatial Ability/Image Analysis / Rotated Figure

DIRECTIONS (Qs. 51): Question figure is followed by four DIRECTIONS (Qs. 52): There is a question figure and four
alternatives (a), (b), (c) and (d). Select the figure from Answer answer figures marked (a), (b), (c) and (d) are given. Select the
figure which is exactly similar to Question figure. [2020 - I] answer figure which is exactly the mirror image of the question
figure when the mirror is held at XY. [2020 - I]
51. Question Figure
52. Question Figure
X

Answer Figures

Y
Answer Figures

(a) (b) (c) (d)


(a) (b) (c) (d)

ANSWERS & EXPLANATIONS


1. (d) 2. (b) 3. (e) 4. (a) 5. (d) 6. (c) 35. (d)
7. (d) 8. (e) 9. (b)
10. (c) Figure is rotated by 180°.
11. (c) Figure is rotated by 90°.
12. (d)
13. (c) All of the components of figure (X) are present in the One point in the figure liesin triangle, square and circle,
figure (3) while other point lies in triangle and square.
14. (c) All of the components of figure (X) are present in the \ Answer figure is (d).
figure (3) 36. (c)
15. (c) All of the components of figure (X) are present in the
figure (3)
16. (a) All of the components of figure (X) are present in the
figure (1) One point is in circle and other in circle and triangle.
17. (c) When the key figure (X) will be rotated it will look like \ Answer figure is (c).
figure (3). The three faces are in the following manner 37. (d) 38. (c) 39. (c) 40. (d) 41. (d)
in clock wise direction. 42. (b) Each one of the figures except fig. (b), is obtained by
A® B® X the lateral inversion of an English alphabet.
18. (c) 19. (c) 20. (a) 21. (d) 22. (a) 23. (d) 43. (d) Only in fig. (d), the pin passes through a vertex of each
24. (c) 25. (c) 26. (d) 27. (b) 28. (d) 29. (c) one of the two elements.
44. (a) In all other figures, one of the dots lies outside the
30. (b) 31. (a)
triangle as well as the circle.
32. (d) 45. (c)

46. (d)

Since one point is in circle and Rectangle while other


point is in square, triangle and Circle.
33. (a) 47. (b)

48. (a) Except option (a), in all others, inner most figure (part)
and outer most figure (part) are similar.
Since one point is in triangle and Circle and other in 49. (c) Except option (c), all other options can be obtained by
triangle and square answer figure is (a). rotating each other.
34. (c) 50. (c) Except option (c), all others form a right angle.
51. (c)
52. (c) Mirror imge of question figure is given in option (c)
Section-D : General Awareness

1 History/Civics & Polity

1. The First Battle of Panipat was fought between [2011-I] 11. The person who is regarded as the greatest law giver of
(a) Akbar & Hemu ancient India is [2012-I]
(b) Babur and Ibrahim Lodhi (a) Panini (b) Kautilya
(c) Akbar & Rana Sanga (c) Manu (d) Patanjali
(d) Ahmad Shah Abdali & Marathas 12. The immortal fame of Ashoka largely rests upon [2012-I]
2. Fa-hien visited India during the reign of [2011-I] (a) his conversion to Buddhism and its propagation.
(a) Chandra Gupta Maurya (b) Bindusara (b) his policy for the welfare of his subjects.
(c) Chandra Gupta II (d) Bimbisara (c) his work in the sphere of politics and moral teaching.
3. Gandhiji’s first experience with Satyagraha came up in (d) his extensive conquests.
[2011-I] 13. Gandhiji started Dandi March [2012-I]
(a) Dandi (b) Champaran (a) to demonstrate against the British Empire.
(c) Bengal (d) Natal (b) to break the salt law.
4. During whose tenure as the viceroy of India were the great (c) to boycott foreign goods.
Martyrs Bhagat Singh, Sukhdev and Rajguru hanged ? (d) None of the above.
[2011-I] 14. The first war of Indian Independence began on 10 May 1857 at
(a) Lord Curzon (b) Lord Irwin [2012-I]
(c) Lord Minto (d) Lord Chelmford (a) Meerut (b) Jhansi
5. The boundary between China and India is known as (c) Barrackpore (d) Delhi
[2011-I] 15. Who among the following had discovered the Bramhi Script
(a) Mc Mohan Line (b) Radcliffe Line in 1838 ? [2012-II]
(c) Hindenberg Line (d) Line of Control (a) Sir William Jones (b) Dr. Rajendra Lal Mitra
6. The famous Grand Trunk (GT) Road from Peshawar to (c) Dr. Bhaw Dagi (d) Jones Prinsep
Kolkata was built by [2011-II] 16. Who was known as father of administration in medieval
(a) Akbar (b) Ashok India ? [2012-II]
(c) Sher Shah Suri (d) Chandragupta (a) Akbar (b) Sher Shah Suri
7. The Ashoka Pillar whose Lion Capitol (Carving) was adopted (c) Humayun (d) Aurangzeb
by the Government of India as National Emblem is situated 17. Brahma Samaj was founded by [2012-II]
at [2011-II] (a) Raja Rammohan Roy (b) Jawaharlal Nehru
(a) Varanasi (b) Puri (c) William Carey (d) Jonathan Duncan
(c) Prayag (d) Sarnath 18. Who discharges the function of the President when va-
8. The Preamble of the Constitution of India was prepared by cancy occur In the office of President & Vice President si-
[2011-II] multaneously, owing to removal, death, resignation or th
(a) Member of Constituent Assembly incumbent or otherwise ? [2012-II]
(b) BR Ambedkar (a) Chief Justice of High Court
(c) Jawaharlal Nehru (b) Chief Justice of India
(d) Dr. Radhakrishna
(c) Speaker of Lok Sabha
9. The two great revolutionaries who threw a bomb in
(d) Chairman of Rajya Sabha
Legislative Assembly were [2011-II]
19. Who accorded the title ‘Mahatma” to MK Gandhi ?
(a) Bhagat Singh & BK Dutt
[2013-I]
(b) Bhagat Singh & Chandrashekhar Azad
(a) Sardar Patel (b) Nehru
(c) Chandrashekar Azad & Bismil
(c) Sarojini Naidu (d) Rabindranath Tagore
(d) Bhagat Singh & Ashfak Ullah Khan
20. The Non co-operation Movement started in which year ?
10. Which one of the following is the exclusive power of the
[2013-I]
Lok Sabha ? [2011-II]
(a) 1900 (b) 1921
(a) To introduce Money Bill.
(c) 1940 (d) 1935
(b) To ratify declaration of Emergency.
21. During the Middle Ages education was confined only to
(c) To impeach the President.
(d) To pass No Confidence Motion against Council of (a) Kshatriyas (b) Brahmins
Ministers. (c) Peasants (d) Shudras
EBD_8177
D-2 History/Civics & Polity

22. Who among the following was responsible for the spread 34. 'Satyameva Jayate' is borrowed from which of the following ?
of Buddhism in Sri Lanka ? [2013-I] [2014-II]
(a) Ashoka (b) Mahavira (a) Mundaka Upanishad (b) Mahabharat
(c) Parsavanath (d) Chandra Gupta Maurya (c) Ramayana (d) Arthshashtra
23. Who was the Governor-General of India during the 35. The opposition part status is accorded to a political party in
‘Sepoy Mutiny’? [2014-I] the Lok Sabha only if it captures at least [2015-I]
(a) Lord Dalhousie (b) Lord Harding (a) 5% Seats (b) 10% Seats
(c) Lord Canning (d) Lord Lytton. (c) 15% Seats (d) 20% Seats
24. Which of the following statements is incorrect ? [2014-I] 36. How many Vice Presidents are elected at the start of its each
(a) Goa attained full statehood in 1987 regular session of UN General Assembly? [2015-I]
(b) Diu is an island in the Gulf of Khambhat (a) Nine (b) Fifteen
(c) Daman & Diu were separated from Goa by the 56 th (c) Two (d) Twenty one
Amendment of the Constitution of India 37. Who among the following was the Congress President at
(d) Dadar & Nagar Haveli were under French colonial Madras Session of 1927 when it boycotted the Simon
rule till 1954. Commission? [2015-I]
25. Who among the following has been called the ‘Napoleon of (a) Maulana Abul Kalam Azad
India’? [2014-I] (b) MA Ansari
(a) Ashoka (b) Samudragupta (c) Lala Lajpat Rai
(c) Chandragupta (d) Harshavardhana (d) Subhash Chandra Bose
26. Who is known as the ‘Grand Old Man of India’? [2014-I] 38. Why did Kalinga prove to be a turning point in the life of
(a) Dadabhai Naoroji (b) Gopal Krishna Gokhale Ashoka? [2015-I]
(c) Bal Gangadhar Tilak (d) A.O. Hume (a) Ashoka annexed Kalinga
27. Which amongst the following has the power to regulate the (b) It was the starting point of the expansion of his empire
right of citizenship in India ? [2014-I] (c) Ashoka became a zealous Buddhist
(a) Union Cabinet (b) Parliament (d) It enabled Mauryan Empire to reach its climax.
(c) Supreme Court (d) Law Commission 39. Which of the following wings was not part of the espionage
28. Kalhana’s ‘Rajatarangini’ is a history of [2014-I] system described by Kautilya? [2015-I]
(a) Kashmir (b) Harsha’s reign (a) Crime Branch (b) Special Branch
(c) Rajasthan (d) Chandragupta’s reign (c) Political Branch (d) None of these
29. The Constitution of India was promulgated on January 26, 40. Alauddin Khilji did not build [2015-I]
1950 because [2014-I] (a) Siri Fort (b) Tomb of Jalaluddin
(a) This day was being celebrated as the Independence (c) Hauz-i-Alai (d) Jamaat Khana Masjid
Day since 1929 41. Which of the following dynasties was ruling over North
(b) This was desired by the farmers of India India at the time of Alexander's invasion? [2015-I]
(c) The British did not want to leave India earlier than this (a) Nanda (b) Maurya
date (c) Sunga (d) Kanva
(d) It was an auspicious day 42. Todar Mal was associated with ______ in Akbar's durbar.
30. Which of the following rulers had the title 'Kaviraja' [2015- II]
[2014-II] (a) Music (b) Literature
(a) Kumaragupta (b) Chandragupta (c) Finance (d) Law Reforms
(c) Skandagupta (d) Samudragupta 43. Mahatma Gandhi left South Africa to return to India in
31. The Viceroy who divided Bengal by following the divide [2015- II]
and rule policy was:- [2014-II] (a) 1911 (b) 1915
(a) Lord Curzon (b) Lord Ripon (c) 1917 (d) 1919
(c) Lord Lytton (d) Lord Mayo 44. Who was the Viceroy when the Simon Commission visited
32. The right to vote in the national elections in India is based India? [2015- II]
on the principle of [2014-II] (a) Lord Chelmsford (b) Lord Reading
(a) Restricted franchise (b) Hereditary privileges (c) Lord Irwin (d) Lord Wellington
(c) Property qualifications (d) Universal adult suffrage 45. Who was the person behind conversion of East India
33. In which respect have the Centre-State relations been Company from a trading company into a regional power?
specifically termed as municipal relations ? [2014-II] [2016- I]
(a) Centre's control of the State in the legislative sphare (a) Lord Warren Hastings
(b) Centre's control of the State in Financial matter (b) Lord Clive
(c) Centre's control of the State in the administrative sector (c) Lord Dalhousie
(d) Centre's control of the State in the planning process (d) Lord Wellesley
History/Civics & Polity D-3

46. Who was the Viceroy when the Simon Commission visited (c) Viscount Wavell
India? [2016- I] (d) Marquess of Linlithgow
(a) Lord Chelmsford (b) Lord Reading 60. Who was the first tirthankara of Jainism? [2017 - I]
(c) Lord Irwin (d) Lord Wellington (a) Rishabhanatha (b) Mahavira
47. Mahatma Gandhi left South Africa to return to India in (c) Parshvanatha (d) Sambhavanatha
[2016- I] 61. Who was the 23rd tirthankara of Jainism? [2017 - I]
(a) 1911 (b) 1915 (a) Mahavira (b) Neminatha
(c) 1917 (d) 1919
(c) Sambhavanatha (d) Parshvanatha
48. The Battle of Plassey was fought in [2016- I]
62. Who was the first governor general of India? [2017 - I]
(a) 1757 (b) 1782
(c) 1748 (d) 1764 (a) Warren Hastings (b) John Macpherson
49. The language of discourses of Gautama Buddha was (c) John Shore (d) Alured Clarke
[2016- I] 63. What was the old name of Myanmar? [2017 - I]
(a) Bhojpuri (b) Magadhi (a) Burma (b) Newar
(c) Pali (d) Sanskrit (c) Nippon (d) Sarawak
50. Bajirao was appointed as Peshwa at the age of 20 yrs under 64. Who was the first NSA of India? [2017 - I]
the reign of [2016 - II] (a) Brajesh Mishra (b) J N Dixit
(a) Sambhaji (b) Chhatrapati Shahu (c) M. K. Narayanan (d) Shivshankar Menon
(c) Rajaram II of Satara (d) Rajaram Chhatrapati 65. Mandal Commission was established by which of the
51. Besides the Qutab Mosque and Qutab Minar, Qutab-ud-din following Prime Ministers of India? [2017 - I]
Aibak is said to have built [2016 - II] (a) Jawahar Lal Nehru (b) Morarji Desai
(a) Adhai-din-ka jhopra mosque at Ajmer (c) Rajiv Gandhi (d) VP Singh
(b) Khirki masjid at Jahan-Panath 66. Where is Lothal, a prominent city of the ancient Indus Valley
(c) Mausoleum of Khan-i-Jahan Tilangani Civilization, located? [2017 - II]
(d) Kila-i-Kuhna mosque at Delhi (a) Gujarat (b) Rajasthan
52. Indian Ruler who defeated British in their early stage of rule
(c) Punjab (d) Madhya Pradesh
in India was. [2016 - II]
67. Which of the following places was chosen by Gandhiji to
(a) Maharaja Ranjit Singh (b) Mahadaji Scindhia
start his first Satyagrah? [2017 - II]
(c) Hyder Ali (d) Tipu Sultan
(a) Ahmedabad (b) Champaran
53. The largest concentration of Harappan sites has been found
along the [2016 - II] (c) Gaya (d) Porbandar
(a) Sutlej (b) Ghaggar- Hakra 68. The first meeting of Indian National Congress was held at
(c) Indus (d) Ravi which place? [2017 - II]
54. Resolution of Purna Swaraj was passed on 26 January 1930 (a) Bombay (b) Delhi
at [2016 - II] (c) Nagpur (d) Ahemdabad
(a) Haripur (b) Lahore 69. When was Indian National Congress founded?
(c) Karachi (d) Calcutta [2018 - I]
55. Fundamental Rights are enshrined in the Constitution of (a) 18 December, 1885 (b) 28, December, 1885
India in [2016 - II] (c) 11, September, 1901 (d) 11, September, 1903
(a) Part I (b) Part III 70. Who is the author of 'India divided'? [2018 - I]
(c) Part IV (d) Part V (a) Jawahar Lal Nehru (b) Shashi Tharoor
56. What is the old name of USA? [2017 - I] (c) Rajendra Prasad (d) Kapil Sibbal
(a) United Colonies 71. Tashkent agreement was signed by _______. [2018 - I]
(b) Turtle Islands
(a) Sardar Patel and John Mathai
(c) Americas
(b) Lal Bahadur Shahstri and Ayub Khan
(d) Great America
(c) Maulana Abul Kalam Azad and Rajendra Prasad
57. Which religion was founded by Akbar? [2017 - I]
(a) Bahai (b) Taoism (d) Rajendra Prasad and Jawahar Lal Nehru
(c) Shintoism (d) Din-e-Ilahi 72. Who has written Panchtantra? [2018 - I]
58. Who was elected as the Prime Minister of India in the (a) Vaishnu Sharma
election held after the death of Rajiv Gandhi? [2017 - I] (b) Munshi Prem Chand
(a) V. P. Singh (b) Chandra Shekhar (c) Subhadra Kumari Chauhan
(c) P. V. Narasimha Rao (d) H. D. Deve Gowda (d) Maithali Saran Gupt
59. Who was the first Governor General of India after 73. In which city was the first British factory established in
Independence? [2017 - I] India. [2018 - I]
(a) C. Rajagopalachari (a) Kedarpuram (b) Machhilipatnam
(b) Lord Mountbatten (c) Agra (d) Panji
EBD_8177
D-4 History/Civics & Polity

74. Who was the Governor General of India during formation of 88. Which dynasty established the Ajanta & Ellora caves?
Indian National Congress? [2018 - I] [2019 - II]
(a) Lord Dufferin (b) Lord Mountbattern (a) Khilji dynasty (b) Shung dynasty
(c) Lord Minto (d) Lord William Bentinck (c) Hindu satvahan dynasty (d) Tughlaq dynasty
75. Find the odd one out: [2018 - I] 89. Which text of the Vedas refers to Medicine?
(a) Plassey (b) Sarnath [2019 - II]
(c) Haldighati (d) Panipat (a) Rigveda (b) Samveda
76. In 1954 which French settlements joined to India?[2018 - I] (c) Yajurveda (d) Atharvaveda
(a) Pondicherry (b) Dutch 90. Who was the founder of Mughal Dynasty? [2019 - II]
(c) Goa (d) Chennai (a) Akbar (b) Humayun
77. In which year Goa is taken by India? [2018 - II] (c) Babur (d) Aurangzeb
(a) November 1949 (b) December 1961 91. What was the Actual name of Ghiyas -ud -Din -Tughlaq?
(c) August 1962 (d) July 1963 [2019 - II]
78. When did the Jallianwala Bagh Massacre take place? (a) Ghazi Malik (b) Salim
[2018 - II] (c) Muhammad bin Tughlaq (d) Khurram
(a) 24 February 1919 (b) 10 March 1919 92. The Khilafat Movement was merged with which movement?
(c) 13 April 1919 (d) 24 July 1927 [2019 - II]
79. What was the term used for measurement of land in the (a) Non-cooperation movement
Delhi Sultanate period? [2018 - II] (b) Civil disobedience movement
(a) Kismat-i-Ghalla (b) Ghalla Bakshi (c) Dandy March
(c) Masahat (d) Ghazi (d) Quit India movement
80. Who was the founder of Brahmo Samaj? [2018 - II] 93. Who rejected the Muslim League’s demand for Pakistan?
(a) Debendranath Tagor (b) Rammohan Roy [2019 - II]
(c) Keshab Chandra Sen (d) Dayanand Saraswati (a) Muslim league (b) Cabinet mission
81. When did India become a member of Asian Development (c) INC (d) Muhammad Ali Jinnah
Bank? [2018 - II] 94. Who established the Indian National Congress?
(a) 1966 (b) 1972 [2019 - II]
(c) 1978 (d) 1982 (a) Dada Bhai Noroji
82. Which war was fought between Sher Shah and Humayun in (b) Mahatma Gandhi
the year 1540AD? [2018 - II] (c) Subhash Chandra Bose
(a) Battle of Chausa (b) Battle of Bhojpur (d) A.O Hume
(c) Battle of Guzargh (d) Battle of Kannauj
95. Article 343 is related to which? [2020 - I]
83. The language of Ashoka’s Inscription? [2019-I]
(a) Official languages (b) Election Commission
(a) Sanskrit (b) Tamil
(c) National Emergency (d) Supreme Court
(c) Prakrit (d) Parthian
84. Shuddhi Movement was run by? [2019-I] 96. Who among the following introduced preamble in
(a) Arya Samaj (b) Brahmo Samaj constituent Assembly? [2020 - I]
(c) Prarthana Samaj (d) None of these (a) Dr. B R Ambedkar
85. The founder of Stavahana Dynasty? [2019-I] (b) Dr. Rajendra Prasad
(c) Sardar Ballabh Bhai Patel
(a) Satakarni (b) Simuka
(d) Jawahar Lal Nehru
(c) Pulumavi (d) Kanha
86. Who is the chairman of the constitution drafting committee– 97. Metallic coin first used in India in? [2020 - I]
[2019-I] (a) Bihar and eastern U.P. (b) Southern India
(a) Alladi Krishnaswami Ayyar (c) Western India (d) Central India
(b) N. Gopalaswami 98. Akbar Buland Darwaza victory over? [2020 - I]
(c) B.R. Ambedkar (a) Gujarat (b) Asirgarh fort
(d) K.M Munshi (c) Panipat Battle (d) Chittorgarh fort
87. UN was established on? [2019-I] 99. Dravidian Art associated with? [2020 - I]
(a) 1944 (b) 1945 (a) Sangam period (b) Mughal Period
(c) 1942 (d) 1946 (c) Gupta Period (d) Maurya Period
History/Civics & Polity D-5

ANSWERS & EXPLANATIONS


1. (b) The First Battle of Panipat, on 21 April 1526, was fought 11. (c)
between the invading forces of Babur and the Lodi 12. (a) Ashoka converted gradually to Buddhism beginning
Empire. It took place in north India and marked the about 263 BCE at the latest. He was later dedicated to
beginning of the Mughal Empire. This was one of the the propagation of Buddhism across Asia, and estab-
earliest battles involving gunpowder firearms and field lished monuments marking several significant sites in
artillery. Ibrahim Lodi died on the field of battle along the life of Gautama Buddha.
with 15,000 of his troops. 13. (b) The Salt March, also mainly known as the Salt
2. (c) Fahien visited India in the early fifth century AD. He is Satyagraha, began with the Dandi March on 12 March
said to have walked all the way from China across icy 1930. It was a direct action campaign of tax resistance
desert and rugged mountain passes. Fahien's visit to and nonviolent protest against the British salt mo-
India occurred during the reign of Chandragupta II. nopoly in colonial India, and triggered the wider Civil
3. (b) The first Satyagraha revolutions inspired by Mahatma Disobedience Movement.
Gandhi in the Indian Independence Movement 14. (a) The Indian Rebellion of 1857 began as a mutiny of
occurred in Champaran district of Bihar on 1916. sepoys of the East India Company's army on 10 May
4. (b) 1857, in the cantonment of the town of Meerut, Utar
5. (a) The McMahon Line is a line agreed to by Britain and Pradesh.
Tibet as part of the Simla Accord, a treaty signed in 15. (d) The script was deciphered in 1837 by Jones Prinsep,
1914. It is the effective boundary between China and an archaeologist, philologist, and official of the British
India, although its legal status is disputed by the East India Company.
Chinese government. The line is named after Sir Henry 16. (a)
McMahon, foreign secretary of the British-run 17. (a) Brahma Samaj is the societal component of Brahmoism,
Government of India and the chief negotiator of the a monotheistic reformist and renaissance movement of
convention at Simla. It extends for 550 miles (890 km) Hindu religion. It was started at Calcutta on 20 August
from Bhutan in the west to 160 miles (260 km) east of 1828 by Raja Ram Mohan Roy and Debendranath
the great bend of the Brahmaputra River in the east, Tagore.
largely along the crest of the Himalayas. 18. (b) When President Zakir Hussain died in office, the Vice
6. (c) The Grand Trunk Road is one of Asia's oldest and President VV Giri, acted as the President. However,
longest major roads. For more than two millennia, it Mr. Giri resigned as the Vice President. Then the Chief
has linked the eastern and western regions of the Indian Justice Hidayatullah became the acting President of
subcontinent, connecting South Asia with Central Asia. India. The most senior judge of the Supreme Court
It runs from Chittagong, Bangladesh west to Howrah, became the acting Chief Justice of India. When the
West Bengal in India, across north India into Peshawar, newly elected President took office a month later,
up to Kabul, Afghanistan. The predecessor of the Justice Hidayatullah again became the Chief Justice of
modern road was rebuilt by Sher Shah Suri, who India.
renovated and extended the ancient Mauryan route in 19. (d) Rabindranath Tagore bestowed the title 'mahatma' to
the 16th century. M.K.Gandhi.
7. (d) The Lion Capital of Ashoka is a sculpture of four Indian 20. (b) The Non-cooperation movement was a significant
lions standing back to back, on an elaborate base that phase of the Indian struggle for freedom from British
includes other animals. A graphic representation of it rule. It was led by Mohandas Gandhi and was
was adopted as the official Emblem of India in 1950. It supported by the Indian National Congress.
was originally placed atop the Ashoka pillar at the 21. (b) In ancient India, during the Vedic period from about
important Buddhist site of Sarnath by the Emperor 1500 BC to 600 BC, most education was based on the
Ashoka, in about 250 BCE. Veda (hymns, formulas, and incantations, recited or
8. (b) The preamble of the Constitution of India was prepared chanted by priests of a pre-Hindu tradition) and later
by B R Ambedkar. Hindu texts and scriptures.Education, at first freely
9. (a) Seeking revenge for the death of Lala Lajpat Rai at the available in Vedic society, became over time more
hands of the police, Bhagat Singh was involved in the discriminatory as the caste system, originally based
murder of British police officer John Saunders. He on occupation, evolved, with the brahman (priests)
eluded efforts by the police to capture him. Soon after, being the most privileged of the castes.
together with Batukeshwar Dutt, he undertook a 22. (a) Ashoka sent his only daughter Sanghamitra and son
successful effort to throw two bombs and leaflets inside Mahindra to spread Buddhism in Sri Lanka (then known
the Central Legislative Assembly while shouting the as Tamraparni).As a Buddhist emperor, Ashoka
slogan of revolution. believed that Buddhism is beneficial for all human
10. (a) Money Bills can be introduced only in Lok Sabha (the beings as well as animals and plants, so he built a
directly elected 'people's house' of the Indian Parliament).
EBD_8177
D-6 History/Civics & Polity

number of stupas, Sangharama, viharas, chaitya, and Kalhana in 12th century CE. The work generally records
residences for Buddhist monks all over South Asia the heritage of Kashmir, but 120 verses of Rajatarangini
and Central Asia. describe the misrule prevailing in Kashmir during the
23. (c) A major cause of resentment that arose ten months reign of King Kalash, son of King Ananta Deva of
prior to the outbreak of the Rebellion was the General Kashmir. Although the earlier books are inaccurate in
Service Enlistment Act of 25 July 1856. As noted above, their chronology, they still provide an invaluable source
men of the Bengal Army had been exempted from over- of information about early Kashmir and its neighbors
seas service. Specifically they were enlisted only for in the north western parts of the Indian subcontinent,
service in territories to which they could march. Gover- and are widely referenced by later historians and
nor-General Lord Dalhousie saw this as an anomaly, ethnographers.
since all sepoys of the Madras and Bombay Armies 29. (a) This day was being celebrated as the Independence
and the six "General Service" battalions of the Bengal Day since 1929
Army had accepted an obligation to serve overseas if 30. (d) Samudragupta, ruler of the Gupta Empire (c. 335 - c. 375
required. As signed into effect by Lord Canning, CE), and successor to Chandragupta I, is considered to
Dalhousie's successor as Governor-General, the Act be one of the greatest military geniuses in Indian history.
required only new recruits to the Bengal Army to ac- He was the third ruler of the Gupta Dynasty, who ushered
cept a commitment for general service. However, serv- in the Golden Age of India.His title of Kaviraja (King of
ing high-caste sepoys were fearful that it would be poets) is justified by various poetical compositions.
eventually extended to them, as well as preventing sons 31. (a) The decision to effect the Partition of Bengal was
following fathers into an Army with a strong tradition announced in July 1905 by the Viceroy of India, Lord
of family service. Curzon. The partition took place in October 1905 and
24. (d) To keep the British at bay and to enlist their support separated the largely Muslim eastern areas from the
against the Moghuls, the Marathas, who had founded largely Hindu western areas.
their own empire/kingdom made friends with the Por- 32. (d) The democratic system in India is based on the principle
tuguese and signed with them a treaty in 1779. Under of Universal Adult Suffrage. All citizens of India who
this, the Maratha-Peshwa agreed that the Portuguese are 18 years of age as on 1st January of the year for
would be allowed to collect revenues from Dadra and which the electoral roll is prepared are entitled to be
Nagar Haveli which consisted of 72 villages (then registered as a voter in the constituency where he or
known as parganas, now referred to as district places).It she ordinarily resides. Only persons who are of
was annexed by India from Portugal on 2 August 1954. unsound mind and have been declared so by a
The people of the territory established free administra- competent court or disqualified due to 'Corrupt
tion of Dadra and Nagar Haveli, which was finally Practices' or offences relating to elections are not
merged into the Union of India in 1961. entitled to be registered in the electoral rolls. The right
25. (b) Samudragupta, ruler of the Gupta Empire (c. 335 - c. 375 to vote is irrespective of caste, creed, religion or gender.
CE), and successor to Chandragupta I, is considered 33. (d) Centres control of the State in the planning process.
to be one of the greatest military geniuses in Indian 34. (a) Satyameva Jayateis a mantra from the ancient Indian
history. He was the third ruler of the Gupta Dynasty, scripture Mundaka Upanishad. Upon independence
who ushered in the Golden Age of India. He was per- of India, it was adopted as the national motto of India.
haps the greatest king of Gupta dynasty. He was a It is inscribed in Devanagari script at the base of the
benevolent ruler, a great warrior and a patron of arts. national emblem. The emblem and the words
His name appears in the Javanese text `Tantrikamandaka'. "Satyameva Jayate" are inscribed on one side of all
26. (a) Dadabhai Naoroji (4 September 1825 - 30 June 1917), Indian currency. The emblem is an adaptation of the
known as the Grand Old Man of India, was a Parsi Lion Capital of Ashoka which was erected around 250
intellectual, educator, cotton trader, and an early In- BC at Sarnath, near Varanasi in the north Indian state
dian political and social leader. He was a Member of of Uttar Pradesh.
Parliament (MP) in the United Kingdom House of Com- 35. (b) A political party is officially accorded the status of an
mons between 1892 and 1895, and the first Asian to be opposition party in Lok Sabha, only if it secures at
a British MP. Naoroji is also credited with the founding least 10 percent of the seats.
of the Indian National Congress, along with A.O. Hume 36. (d) Twenty one Vice Presidents are elected at the start of
and Dinshaw Edulji Wacha. His book Poverty and Un- each regular session of General Assembly.
British Rule in India brought attention to the draining 37. (b) M. A. Ansari was the Congress President at the Madras
of India's wealth into Britain. session of 1927, when it boycotted the Simon
27. (b) Nothing in the foregoing provisions of this Part shall Commission.
derogate from the power of Parliament to make any 38. (c) Ashoka invaded Kalinga in 261 B. C. In this war more
provision with respect to the acquisition and termination than 2 lakh people died, wounded and made prisoners
of citizenship and all other matters relating to citizenship. in war. Such a huge carnage and massacre of human
28. (a) Rajatarangini is a metrical historical chronicle of north- lives and the sufferings of the wounded made a deep
western Indian subcontinent, particularly the kings of impression on Ashoka's mind. So he decided to spare
Kashmir, written in Sanskrit by Kashmiri Brahman his life to the spread of Buddhism around the world.
History/Civics & Polity D-7

39. (c) Indian National Congress under the influence of Bhagat


40. (d) The Jama 'at-Khana-Masjid or Khilji mosque was built Singh and other revolutionaries on 19 December 1929,
in 1325 by Khizr Khan, son of Alauddin Khilji while all resolving the Congress and Indian nationalists to fight
the other monuments were built by Allauddin Khilji. for Purna Swaraj, or complete self-rule independent of
41. (a) During the invasion Alexander, Nanda dynasty was the British Empire.
ruling the North India in the Magadha empire. 55. (b) Fundamental rights are contained in Part III of
42. (c) Raja Todar Mal was the finance minister of the Mughal Constitution of India. It guarantees civil liberties such
empire during Akbar's reign. that all Indians can lead their lives in peace and harmony
43. (b) Mahatma Gandhi returned to India from South Africa as citizens of India.
in 1915. 56. (a) In 1776, the Continental Congress formally declared
44. (c) Lord Irwin was the viceroy of India when Simon the name of the new nation to be the "United States"
Commission visited India. of America, replacing the term "United Colonies," which
45. (b) Lord Clive converted the East India Company into had been in general use. Americas is the term used for
regional power after winning the battle of Plassey. the continents of North and South America. Turtle
46. (c) The Simon commission was established under the Islands is a group of seven islands in the province of
guidance of the then vice president Lord Irwin by the Tawi-Tawi in the Southern Philippines.
council of India. 57. (d) Din-e-Ilahi was founded by Mughal emperor Akbar in
47. (c) Mahatma Gandhi returned to India from South Africa 1582 AD, intending to merge the best elements of the
permanently in 1917. religions of his empire. Bahaullah was the founder of
48. (a) The Battle of Plassey, 23 June 1757, was a decisive the Bahai faith. There is no known founder of
British East India Company victory over the Nawab of Shintoism. Lao-Tzu was a Chinese philosopher credited
Bengal and his French allies, establishing Company with founding the philosophical system of Taoism.
rule in South Asia which expanded over much of the 58. (c) P. V. Narasimha Rao (21 June 1991-16 May 1996) was
Indies for the next 190 years. The battle took place at elected as the 9th Prime Minister of India in the election
Palashi, Bengal, on the river banks of the Bhagirathi held after the death of Rajiv Gandhi on 21 May 1991.
River, about 150 km north of Calcutta, near Narasimha Rao succeeded Prime Minister Chandra
Murshidabad, then capital of undivided Bengal. The Shekhar.
belligerents were Siraj-ud-daulah, the last independent 59. (b) Lord Mountbatten was the last Viceroy of India (1947)
Nawab of Bengal, and the British East India Company. and the first Governor-General of independent India
49. (c) (1947-48). Marquess of Linlithgow (1936-43) and
50. (b) Bajirao served as Peshwa to the fifth Maratha Viscount Wavell (1943-47) served as Governor Generals
Chhatrapati Shahu from 1720 until his death. Bajirao is before independence. C. Rajagopalachari served as
credited with expanding the Maratha Empire, especially second Governor General of Independent India from
in North India, which contributed to its reaching a 1948-50.
zenith during his son's reign twenty years after his 60. (a) Tirthankara is a saviour and spiritual teacher of the
death. dharma. Rishabhanatha was the first tirthankara of
51. (a) Adhai Din Ka Jhonpra is a mosque in the Ajmer city of Jainism. Mahavira, Parshvanatha and Sambhavanatha
Rajasthan, India. It was commissioned by Qutb-ud- were 24th, 7th and 3rd tirthankara, respectively.
Din-Aibak, on orders of Muhammad Ghori, in 1192 CE. 61. (d) Parshvanatha was the 23rd tirthankara of Jainism. He
It was completed in 1199 CE, and further beautified by is the earliest Jain Tirthankara who is generally
Iltutmish of Delhi in 1213 CE. The mosque was acknowledged as a historical figure. Mahavira,
constructed on the remains of a Sanskrit college, with Parshvanatha and Sambhavanatha were 24th, 7th and
materials from destroyed Hindu and Jain temples. 3rd tirthankara, respectively.
52. (c) In the First Anglo-Mysore War (1767-69), the British 62. (a) Warren Hastings (20 October 1773- 1 February 1785)
were defeated by Hyder Ali in a number of battles. was the first governor general of British-ruled India.
This was followed by the Second Anglo-Mysore War Other governor generals were John Macpherson (1
(1780-84), which resulted in stalemate but also included February 1785-12 September 1786), John Shore (28
the Battle of Pollilur, where the British first encountered October 1793-18 March 1798) and Alured Clarke (18
the hugely effective Mysorean rockets. March 1793-18 May 1798).
53. (b) The largest concentration of Harappan site has been 63. (a) The ruling military changed Burma's name to Myanmar
found along Ghaggar-Hakra River, which is an in 1989, a year after thousands were killed in the
intermittent, endorheic river in India and Pakistan that suppression of a popular uprising. Rangoon also
flows only during the monsoon season. Over 530 became Yangon. Japan's old name was Nippon. Sarawak
Harappan sites (of the more than 800 known sites, not is a state in Malaysia. Nepalese are also called Newar.
including Late Harappan or OCP) are located on the 64. (a) Brajesh Mishra was the first National Security Advisor
Ghaggar-Hakra. (NSA) of India who served from November 1998 to
54. (b) Resolution of Purna Swaraj was passed on 26 January May 2004. Other former NSAs were J N Dixit (2004-
1930 at Lahore. Purna Swaraj declaration, or Declaration 2005), M. K. Narayanan (2005-2010) and Shivshankar
of the Independence of India, was promulgated by the Menon (2010-2014).
EBD_8177
D-8 History/Civics & Polity

65. (b) The Mandal Commission was established in India in the Drafting Committee elected B.R Ambedkar as its
1979 by the Janata Party government under Prime Chairman.
Minister Morarji Desai, to identify the socially or 87. (b)
educationally backward. 88. (c) Ajanta & Ellora caves monuments were built during
66. (a) Lothal is one of the most prominent cities of the ancient Hindu dynasties such as the Rashtrakuta dynasty,
Indus valley civilization, located in the Bh a l region of which constructed part of the Hindu and Buddhist
the modern state of Gujarat. The city was discovered caves, and the Yadava dynasty, which constructed a
in 1954. number of the Jain caves. This dynasty is mentioned
67. (b) The Champaran Satyagraha of 1917, in the Champaran as Andhras in the puranas was ruling in Deccan region.
district of Bihar, India during the period of the British 89. (d) Here, the word Atharvanas meaning the procedure for
Raj, was the first Satyagraha movement started by everyday life and veda means knowledge so,
Mohandas Gandhi and it was considered a major revolt Atharvaveda provides knowledge about medicine
in the Indian Independence Movement. used for many diseases.
68. (a) Under the Presidentship of Vyomesh Chandra Banerjee, 90. (c) Babur founded the Mughal Empire in India. He was a
the first meeting of Indian National Congress was held central Asian ruler (1526-1530) in India and was
in Bombay in 1885. The meeting was attended by 72 descended from Turco-Mangol conqueror Timur, on
English-educated people that included advocates, his father side and, from chengiz khan from his mother's
traders and zamindars. side.
69. (b) 70. (c) 71. (b) 72. (a) 73. (b) 74. (a) 91. (a) The actual name of Ghiyas -ud -Din -Tughlaq was Gazi
75. (b) 76. (a) malik that means 'fighter for islam'was the founder of
77. (b) The Annexation of Goa was the process in which the Tughluq dynasty in India.
Republic of India annexed the former Portuguese 92. (a) Mahatma Gandhi had a central role in the juxtaposition
Indian territories of Goa, Daman, and Diu, starting with of the Khilafat Movement with India's freedom
the "armed action" carried out by the Indian Armed struggle, primarily the Non Cooperation Movement.
Forces in December 1961. This was the first Hindu-Muslim protest against British
78. (c) The Jallianwala Bagh massacre, also known as the rule, this movement became the symbol of Hindu-
Amritsar massacre, took place on 13 April 1919 when a Muslim unity.
crowd of nonviolent protesters, along with Baishakhi 93. (b) The Cabinet Mission rejected the Muslim League's
pilgrims, who had gathered in Jallianwala Bagh, demand for Pakistan. Because,
Amritsar, Punjab, were fired upon by troops of the The formation of Pakistan would not solve the
British Indian Army under the command of Colonel communal minorities problem. Even after the creation
Reginald Dyer. of Pakistan, 37.93% non Muslims would still be in the
79. (c) 80. (b) North-West zone of Pakistan and 48.31% would be in
81. (a) India became a member of the Asian Development Bank the North-East zone.
(ADB) as a founding member in 1966. The Bank is 94. (d) On 28 December 1885, the Indian National Congress
engaged in promoting economic and social progress was founded by A.O Hume.
of its developing member countries (DMCs) in the Asia 95. (a) Article 343 is related to official languages, this article
Pacific Region. is enshrined in Part 17 of the Constitution of India.
82. (d) Battle of Kannauj (1540 A.D.) - Sher Shah Suri defeated 96. (a) The Assembly adopted the Preamble as presented by
Humayun. chairman of the Drafting Committee Dr. B R Ambedkar
83. (c) Prakrit, Greek, and Aramaic . The Constituent Assembly debated the Preamble on
84. (a) Arya samaj was founded by the sannyasi Dayanand 17th October 1949. The debates around the Preamble
Saraswati on 10 April 1875. Members of the Arya Samaj revolved around the name of India and inclusion of
believe in one God and reject the worship of idols. It is ‘God’ and ‘Gandhi’.
an Indian Hindu reform movement. 97. (a) The first metallic coin used in India was in
85. (b) Simuka was an Indian king belonging to the Satavahana 98. (a) Buland Darwaza or the “Door of victory”, was built in
dynasty. He is mentioned as the first king in a list of 1601 A.D. by Mughal emperor Akbar to commemorate
royals in a Satavahana inscription at Nanaghat. his victory over Gujarat. It is the main entrance to the
86. (c) The Drafting Committee had seven members: Alladi Jama Masjid at Fatehpur Sikri.
Krishnaswami Ayyar, N. Gopalaswami; B.R. Ambedkar, 99. (a) Dravidian architecture or the South Indian temple style
K.M Munshi, Mohammad Saadulla, B.L. Mitter and is an architectural idiom in Hindu temple architecture
D.P. Khaitan. At its first meeting on 30th August 1947, that emerged in the southern part of the Indian sub-
continent during the Sangam period.
Geography / General
2 Science
1. Which one of the following soils is most suitable for cotton 12. The sun rises in Arunachal Pradesh two hours before it
cultivation [2011-I] does in Dwaraka in Gujarat. This is because the former is
(a) Red soil (b) Black soil [2012-I]
(c) Loamy soil (d) Laterite soil (a) higher in elevation than Dwaraka and the earth rotates
2. Equinox means two days in a year when day and night are from West to East.
almost equal. If March 21 is an equinox which is the next ? (b) situated further North than Dwaraka and the earth
[2011-I] rotates from West to East.
(a) 09 October (b) 31 August (c) situated further East (about 30º Longitude) than
(c) 23 September (d) 03 November dwaraka and the earth rotates from West to East.
3. Vitamin necessary to prevent prolonged bleeding is (d) situated about 30º East of Dwaraka and the earth rotates
[2011-I] from West to East.
(a) Vitamin A (b) Vitamin E 13. Srinagar is situated on the banks of the river [2012-I]
(c) Vitamin D (d) Vitamin K (a) Ravi (b) Sutlej
4. The term ‘Carbon Credit’ is associated with [2011-I] (c) Jhelum (d) Chenab
14. The areas in India that receive approximately an average of
(a) Coal reserve of a nation
more than 200 cms of rainfall annually are [2012-I]
(b) Reduction of Green House Gas emissions
(a) Meghalaya, Assam, Nagaland, Arunachal Pradesh.
(c) Fossil Fuel reserve
(b) Odisha, Madhya Pradesh, Gujarat, Maharashtra.
(d) Amount of CO2 an individual emits in a year
(c) Meghalaya, Assam, Rajasthan, Jammu & Kashmir.
5. India tops the world in production of [2011-II]
(d) Meghalaya, Delhi, Punjab, Rajasthan.
(a) Aluminium (b) Copper 15. Isotherms are imaginary lines drawn on a map which con-
(c) Chromite (d) Mica nect places of equal [2012-I]
6. DPT vaccine does not give protection to a child from (a) Atmospheric pressure (b) Humidity
[2011-II] (c) Rainfall (d) Temperature
(a) Tetanus (b) Polio 16. Jim Corbett National Park is situated in which state ?
(c) Diphtheria (d) Whooping Cough [2012-I]
7. What will be the colour of a red rose when it is seen through (a) Arunachal Pradesh (b) Himachal Pradesh
green glass ? [2011-II] (c) Andhra Pradesh (d) Uttaranchal
(a) White (b) Black 17. Which one of the following statement regarding the sun is
(c) Pink (d) Brown correct ? [2012-I]
8. Which one of the following crops enriches nitrogen (a) The sun is composed mainly of hydrogen.
content in the soil ? [2011-II] (b) Its energy is generated by nuclear collision in its interior.
(a) Pea (b) Sunflower (c) It is calculated that the sun consumes about a trillion
(c) Potato (d) Wheat pounds of hydrogen every second.
9. Which of the following periodical winds blowing from sea (d) All of the above.
to land cause summer monsoon in India ? [2011-II] 18. Supersonic speed is speed greater than the speed of sound
(a) East West (b) North East (in air at sea level) that is to say around______ miles/hour.
(c) South West (d) South East [2012-I]
10. India’s permanent research station ‘Dakshin Gangotri” is (a) 760 (b) 860
situated in the [2011-II] (c) 960 (d) 1060
(a) Great Himalayas (b) Indian Ocean 19. An aeroplane rises because [2012-I]
(c) Arabian Sea (d) Antarctica (a) of upward reaction of air.
11. Which one of the following is the busiest ocean route in the (b) the density of air above the plane is less than below it.
world ? [2011-II] (c) the pressure above its wings is less than the pressure
(a) Indian Ocean (b) Pacific Ocean below them.
(d) its nose points upwards.
(c) North Atlantic Ocean (d) South Atlantic Ocean
EBD_8177
D-10 Geography/General Science
20. Rocks formed on the solidification of molten matter are called 31. River Satluj originates from [2013-I]
[2012-I] (a) Northern slopes of the Kailash range
(a) Metamorphic (b) Sedimentary rocks (b) Spring at Verinag
(c) Volcanic rocks (d) Igneous rocks (c) Rakas Lake
21. River Jhelum emerges from [2012-II] (d) Amarkantak plateau
(a) Northern slopes of the Kailash range 32. Limonitic ore is the ore of which metal ? [2013-I]
(b) Spring at Verinarg (a) Iron (b) Aluminium
(c) Rakas Lake (c) Zinc (d) Cobalt
33. Black revolution relates to which of the following ?
(d) Amarkantak plateau
[2013-I]
22. Haematite ores is the ore of which metal ? [2012-II] (a) Self-dependence in foodgrains production
(a) Iron (b) Aluminium (b) Self-dependence in milk production
(c) Zink- (d) Cobalt (c) Self-dependence in petroleum/crude oil
23. Green revolution relates to which of the following ? (d) None of these
[2012-II] 34. __________ is a thyroid hormone which controls the bal-
(a) Self-dependence in foodgrains production ance of calcium in the body [2013-I]
(b) Self-dependence in milk production (a) Calcitonin (b) Thyroxine
(c) Self-dependence in petroleum crude oil production (c) Calmodulin (d) All of these
(d) None of the above 35. The cell wall in plants is interrupted by narrow pores carrying
24. Vertebrates have two endocrine glands associated with the fine strands of cytoplasm which interlink the contents of
brain, namely [2012-II] the cells. These strands are called:- [2013-I]
(a) Thyroid, Thymus (b) Pituitary, Pancreas (a) Plasmohole (b) Microvilli
(c) Pituitary, Pineal (d) Pancreas, Pineal (c) Plasmodesmata (d) Plasmalemma
25. The layer common to two adjacent plant cells called Middle 36. The transport phenomenon occurs only in __________
Lamella is composed of [2012-II] state of a gas and is __________. [2013-I]
(a) Calcium Phosphate (b) Calcium Sulphate (a) non-equilibrium, irreversible
(b) non-equilibrium, reversible
(c) Calcium Carbonate (d) Calcium Pectate (c) equilibrium, irreversible
26. With the increase of the effective nuclear charge, the size of (d) equilibrium, reversible
the atom or ion [2012-II] 37. Which of the following compounds form nitrites with
(a) increases nitrous acid ? [2013-I]
(b) decreases (a) Primary amines (b) Secondary amines
(c) remain the same, since it has no bearing on size (c) Tertiary amines (d) All of these
(d) it will depend on period to period and group to group 38. On which river is Washington DC situated ? [2014-I]
27. On which of the following statements, is the kinetic theory (a) Potomac (b) Irrawaddy
of matter base ? [2012-II] (c) Mississippi (d) Hudson
(a) Matter is made up of molecules 39. When body is accelerated: [2014-I]
(b) Molecules are in rapid motion (a) Its velocity never changes
(c) Molecules experience forces of attraction between one (b) Its speed will always changes
another (c) Its direction always changes
(d) All of the above (d) Its speed may or may not change
28. When heated with chloroform, secondary amines and 40. Which of the following is not a unit of energy ? [2014-I]
tertiary amines [2012-II] (a) Calorie (b) Joule
(a) gives isocyanides (c) Electron volt (d) Watt
(b) gives cyanides 41. The Baglihar Hydroelectric power project in J & K is built
across the river. [2014-I]
(c) do not give isocyanides (a) Beas (b) Chenab
(d) Both (a) and (b) (c) Jhelum (d) Sutlej
29. The elements which have low value of ionization potential 42. Which is the longest bone in the human body ? [2014-I]
are strong [2013-I] (a) Fibula (b) Radius
(a) oxidising agents (c) Stapes (d) Femur
(b) reducing agents
(c) oxidising and Reducing agents depending upon the 43. A US team of scientists has found that the mechanism
reactants responsible for the ageing process is located [2014-I]
(d) none of these (a) inside the face (b) inside the skin
30. Term ‘Visible Horizon’ in astronomy is defined as [2013-I] (c) inside the brain (d) inside the heart
(a) The circle of contact of the earth and the cone of visual 44. The outer most layer of the Sun is known as [2014-I]
rays passing through the meridian of the place (a) Corona (b) Photosphere
(b) The circle of contact of the earth and the cone of visual (c) Chromosphere (d) Granule
rays not passing through the meridian of the place 45. Which one of the following pairs of water bodies are
(c) The circle of contact of the earth and the cone of visual
rays passing through the point of observation connected by the Suez Canal ? [2014-I]
(d) The circle of contact of the earth and the cone of visual (a) Indian Ocean- Pacific Ocean
rays not passing through the point of observation (b) Mediterranean sea- Black Sea
Geography/General Science D-11

(c) Mediterranean Sea-Red Sea 58. Which one of the following is the petroleum wax? [2016-I]
(d) Atlantic Ocean- Pacific Ocean (a) Paraffin wax (b) Jonoba wax
46. Which of the following is not correctly matched ?[2014-I] (c) Carnauba wax (d) Bees wax
(a) Indonesia - Jakarta (b) Maldives - Male 59. Artificial rain is produced by seeding clouds with[2016 - II]
(c) North Korea - Seoul (d) Zimbabwe - Harare (a) Silver Iodide (b) Potassium Nitrate
47. Algae often float on surface of water during day but sink (c) Copper Sulphate (d) Silver Nitrate
during night due to:- [2014-II] 60. Which of the following parts of India receives rainfall from
(a) evolution and trapping of oxygen bubbles during the retreating monsoon? [2016 - II]
day in their photosynthesis process (a) North East India (b) Tamil Nadu Coast
(b) Becoming light as they consume most of their food in (c) Mahanadi Delta (d) Malabar Coast
the night
(c) warming action of sun during the day 61. Which one of the following best explains the occurrence of
the solar eclipse? [2016 - II]
(d) Release of absorbed air by warming of water
(a) Position of the Moon between Sun & Earth
48. On which river is Berlin city situated ? [2014-II]
(b) Position of the Moon between Sun & Earth on a new
(a) Potomac (b) Irrawaddy
moon
(c) Rhine (d) Spree
(c) Both (a) & (b)
49. What is the splash and burn agriculture' in Indonesia called
(d) None of these
as ? [2014-II]
62. Heavy water is called heavy because [2016 - II]
(a) Jhoom cultivation (b) Roke cultivation
(a) It is denser than ordinary water
(c) Milpa cultivation (d) Ladang cultivation (b) It is an oxide of deuteron
50. Which theory gave birth to the French Revolution and the (c) It has a heavy (or bad) taste
Revolution in America ? [2014-II] (d) It has a heavier isotope of hydrogen
(a) Legal theory of rights 63. Which of the following soils is most conducive for the
(b) Theory of natural rights growth of cotton? [2016 - II]
(c) Social welfare theory (a) Alluvial (b) Red
(d) Historical theory of rights (c) Laterite (d) Black
51. 38th parallel is a boundary line between _____ [2014-II] 64. The largest Oil Field of Gujarat is in [2016 - II]
(a) United States and Canada (a) Senand (b) Kadi
(b) Pakistan and India (c) Ankleshwar (d) Kalol
(c) Turkey and Cyprus 65. Which is the largest diamond producing country in the
(d) South and North Korea world? [2017 - I]
52. The branch of science that studies cells is called [2015-I] (a) Russia
(b) Botswana
(a) Cytology (b) Entomology
(c) Democratic Republic of Congo
(c) Homoplastic (d) Hormonolgy
(d) Australia
53. Kaziranga National Park is famous for [2015-I] 66. Which Indian state is the largest producer of banana?
(a) One-horned Rhinos [2017 - I]
(b) Tigers (a) Tamil Nadu (b) Gujarat
(c) Swamp Dears (Barasingha) (c) Maharashtra (d) Andhra Pradesh
(d) Elephants 67. Satyajit Ray was a [2017 - I]
54. Animals active at night are called [2015-II] (a) Politician (b) Dancer
(a) Diurnal (b) Nocturnal (c) Filmmaker (d) Tabla player
(c) Parasites (d) Nacto-diurnal 68. Mandal Commission was established by which of the
55. Natural radioactivity was discovered by [2015-II] following Prime Ministers of India? [2017 - I]
(a) Marie Curie (b) Earnest Rutherford (a) Jawahar Lal Nehru (b) Morarji Desai
(c) Henry Bacquerel (d) Enrico Fermi (c) Rajiv Gandhi (d) VP Singh
56. How many days moon takes to revolve around the earth? 69. Which Cuban athlete is called 'horse'? [2017 - I]
[2015-II] (a) Dayron Robles (b) Iván García
(c) Alberto Juantorena (d) Héctor Herrera
1 1 70. Which is the deepest trench in the world? [2017 - I]
(a) 26 (b) 27
3 3 (a) Tonga Trench (b) Mariana Trench
(c) Romanche Trench (d) Kermadec Trench
2 1
(c) 28 (d) 29 71. Keoladeo National Park is located in which state?[2017 - II]
3 2 (a) Punjab (b) Tamil Nadu
57. In which atmospheric layer are the communication satellites (c) Rajasthan (d) Karnataka
located? [2015-II] 72. On which river the Baglihar Dam is built? [2017 - II]
(a) Stratosphere (b) Ionosphere (a) Brahmaputra (b) Ganga
(c) Troposphere (d) Mesosphere (c) Chenab (d) Godavari
EBD_8177
D-12 Geography/General Science

73. How many countries are there in Asia? [2018 - I] 89. Agra city was founded by - [2019-I]
(a) 26 (b) 39 (c) 42 (d) 48 (a) Sikandar Lodhi (b) Babar
74. Speed of wind measured by __________. [2018 - I] (c) Akbar (d) Shah Jahan
(a) Speedometer (b) Spectrometer 90. The visible part of the sun is called – [2019-I]
(c) Hydrometer (d) Anemometer (a) Chromosphere (b) Photosphere
75. Salal project is on the river_________. [2018 - I] (c) Corona (d) Core
(a) Godavri (b) Ganga 91. Ozone layer located in which layer - [2019-I]
(c) Chenab (d) Mahanadi (a) Stratosphere (b) Troposphere
76. Guwahati is on the bank of which river? [2018 - I] (c) Mesosphere (d) Exosphere
(a) Ganga (b) Barak 92. What is the Capital of Colombia? [2019 - II]
(c) Brahmaputra (d) Teesta (a) Bogota (b) Caracas
77. What is the Radcliffe Line? [2018 - II] (c) Quito (d) Bangkok
(a) Boundary demarcation line between India and Pakistan. 93. Which Desert is not located in Asia? [2019 - II]
(b) Boundary demarcation line between India and Nepal. (a) Gobi desert (b) Thar desert
(c) Boundary demarcation line between India and China. (c) Arabian desert (d) Sahara desert
(d) Boundary demarcation line between Indian and 94. Which lines run parallel to the equator? [2019 - II]
Afghanistan.
(a) Longitudes (b) Latitudes
78. Riga is the capital of which country? [2018 - II]
(c) Slanting lines (d) straight lines
(a) Latavia (b) Estonia
95. Who Invented Electricity? [2019 - II]
(c) Lithuania (d) Belarus
(a) Benjamin franklin (b) Thomas Edison
79. Where is Indira Point located? [2018 - II]
(c) Michael Faraday (d) Nicola Tesla
(a) Nicobar Islands (b) Lakshadweep
96. Which gas results in Brown Cloud Atmosphere?[2019 - II]
(c) Kerala Coast (d) Tamil Nadu Coast
(a) Carbon dioxide (b) Hydrogen
80. Which of the given Vitamin is responsible for blood clotting?
[2018 - II] (c) Sulphur dioxide (d) Fossil fuels and biomass
(a) Vitamin K (b) Vitamin A 97. Which of the following is a non-ferrous element? [2019 - II]
(c) Vitamin E (d) Vitamin C (a) Iron (b) Silver
81. Which of the following act is known as the Black Act? (c) Gold (d) Copper
[2018 - II] 98. The Sun occupies how much percentage of area of Solar
(a) Regulating Act of 1773 system? [2019 - II]
(b) Charter Act of 1813 (a) 70 (b) 99.8
(c) Vernacular Press Act 1878 (c) 60.8 (d) 80
(d) Rowlatt Act 1919 99. Tropic of Cancer does not pass through which country?
82. Taseometer is an instrument to measure? [2018 - II] [2020 - I]
(a) Intensity (b) Strains (a) Iran (b) India
(c) Sea waves (d) Speed of Storm (c) Algeria (d) Egypt
83. Which island is located between Russia and Japan? 100. Hargobind Khurana won the model prize in? [2020 - I]
[2018 - II] (a) Dermatology (b) Physiology
(a) Kuril Islands (b) St Helena Island (c) Nephrology (d) Oology
(c) Ascension Island (d) Curieuse Island 101. Prime meridian is also known as? [2020 - I]
84. Which is the capital of Estonia? [2018 - II] (a) Arctic Meridian (b) Latin Meridian
(a) Tallinn (b) Vilnius (c) Greenwich Meridian (d) Antarctic Meridian
(c) Minsk (d) Riga
102. Retreating monsoon occurs during which month? [2020 - I]
85. Which Continent has the largest coastline? [2019-I]
(a) November (b) September
(a) Asia (b) North America
(c) October (d) December
(c) South America (d) Africa
103. Which of the following Mountain passes are not in India?
86. When our Constitution was adopted? [2019-I]
[2020 - I]
(a) 26 November 1949 (b) 26 January 1949
(a) Rohtang Pass (b) Khyber Pass
(c) 26 January 1950 (d) 26 November 1950
(c) Baralachala Pass (d) Lipulekh Pass
87. The Hardest substance on the Earth? [2019-I]
(a) Iron (b) Silver 104. Nautical Mile is unit of? [2020 - I]
(c) Diamond (d) Lead (a) Mass (b) Work
88. Oldest Mountain range in India? [2019-I] (c) Length (d) Energy
(a) Himalaya (b) Aravali 105. Which rivers flow into the Arabian Sea? [2020 - I]
(c) Satpura (d) Nilgiri (a) Ganga (b) Narmada
(c) Tapi (d) Both b & c
Geography/General Science D-13

ANSWERS & EXPLANATIONS


1. (b) Black soil is most suitable for cotton cultivation, locally 14. (a) Meghalaya, Assam, Nagaland, Arunachal Pradesh.
called regard or black cotton soils, and internationally Mawsynram, the wettest place on earth (annual rain-
known as 'tropical black earths' or 'tropical chernozems' fall of 1,141 cm approx), is a small village in Meghalaya's
have been developed by the weathering of the Deccan Khasi Hills near Shillong. Cherrapunji, now the second
lava in majorparts of Maharashtra, western Madhya wettest place on earth, is located 10 km from
Pradesh, Gujarat, Andhra Pradesh, Karnataka, Mawsynram and has the distinction of having just one
Rajasthan, Tamil Nadu and Uttar Pradesh. season the year round - monsoon. It receives about
2. (c) An equinox occurs twice a year, around 20 March and 1,087 cm of rain annually.
22 September. If march 21 is an equinox then next 15. (d) Isotherm, line drawn on a map or chart joining points
equinox will be 23 september. with the same temperature.
3. (d) Vitamin K is a group of structurally similar, fat-soluble 16. (d) Jim Corbett National Park is the oldest national park in
vitamins that the human body needs for modification India and was established in 1936 as Hailey National
of certain proteins that are required for blood Park to protect the endangered Bengal tiger. It is lo-
coagulation, and in bone and other tissue. cated in Nainital district of Uttarakhand(Earlier
4. (b) The term Carbon Credit is associated with Reduction Uttaranchal) and was named after Jim Corbett who
of Green House Gas emissions in the atmosphere. played a key role in its establishment. The park was the
5. (d) The British Geological Survey reported that as of 2005, first to come under the Project Tiger initiative.
Koderma district in Jharkhand state in India had the 17. (d)
largest deposits of mica in the world. 18. (a) Supersonic speed speed is approximately 343.2 m/s,
6. (b) DPT (also DTP and DTwP) refers to a class of 1,125 ft/s, 768 mph, 667 knots, or 1,235 km/h.
combination vaccines against three infectious diseases
19. (c)
in humans: diphtheria, pertussis (whooping cough),
and tetanus. 20. (d) Igneous rock is formed through the cooling and solidi-
7. (b) Black, because red and green are two primary colours fication of magma or lava.
which when mixed together gives black colour in terms 21. (b) Verinag is approximately 80 km from Srinagar, by road,
of light and wavelength. at an elevation of 1,876 m. It is believed that the
8. (a) Many legumes (alfalfa, clover, peas, beans, lentils, eponymous Verinag spring is the chief source of the
soybeans, peanuts and others) contain symbiotic river Jhelum. There is an octagonal base at the spring,
bacteria called Rhizobia within root nodules of their surrounded by a covered passage.
root systems.These bacteria have the special ability of 22. (a) Hematite, also spelled as haematite, is the mineral form
fixing nitrogen from atmospheric, molecular nitrogen of iron oxide, one of several iron oxides. Hematite
(N2) into ammonia (NH3). crystallizes in the rhombohedral lattice system, and it
9. (c) The southwestern summer monsoons occur from June has the same crystal structure as ilmenite and
through September. The moisture-laden winds from the corundum.
Indian Ocean rush in to the subcontinent. These winds, 23. (a)
rich in moisture, are drawn towards the Himalayas. The 24. (c) The endocrine system refers to the collection of glands
Himalayas act like a high wall, blocking the winds from
of an organism that secrete hormones directly into the
passing into Central Asia, and forcing them to rise. As the
circulatory system to be carried toward a distant target
clouds rise their temperature drops and precipitation occurs.
organ. The major endocrine glands include the pineal
10. (d) Dakshin Gangotri was the first scientific base station
gland, pituitary gland, pancreas, ovaries, testes, thyroid
of India situated in Antarctica, part of the Indian
gland, parathyroid gland, hypothalamus,
Antarctic Program. It is located at a distance of 2,500
gastrointestinal tract and adrenal glands.
kilometres from the South Pole.
11. (c) The North Atlantic sea route, linking the US and 25. (d) The middle lamella is a pectin layer which cements the
Canada to Europe, is very busy as well. cell walls of two adjoining cells together. Plants need
12. (c) Situated further East (about 30º Longitude) than this to give them stability and so that they can form
Dwaraka and the earth rotates from West to East. plasmodesmata between the cells. It is the first formed
13. (c) Srinagar is the summer capital of the Indian State of layer which is deposited at the time of cytokinesis. The
Jammu and Kashmir. It is situated in the Kashmir Valley cell plate that is formed during cell division itself
and lies on the banks of the Jhelum River, a tributary of develops into middle lamella or lamellum. The middle
the Indus. The city is famous for its gardens, lakes and lamella is made up of calcium and magnesium pectates.
houseboats. It is also known for traditional Kashmiri In plants, the pectins form an unified and continuous
handicrafts and dry fruits. layer between adjacent cells.
EBD_8177
D-14 Geography/General Science

26. (d) The effective nuclear charge is the net positive charge 39. (b)
experienced by an electron in a multi-electron atom. 40. (d) The watt is a derived unit of power in the International
The term "effective" is used because the shielding System of Units, named after the Scottish engineer
effect of negatively charged electrons prevents higher James Watt. The unit defined as one joule per second,
orbital electrons from experiencing the full nuclear measures the rate of energy conversion or transfer.
41. (b) Baglihar Dam, also known as Baglihar Hydroelectric
charge by the repelling effect of inner-layer electrons.
Power Project, is a run-of-the-river power project on
The effective nuclear charge experienced by the outer the Chenab River in the southern Doda district of the
shell electron is also called the core charge. It is possible Indian state of Jammu and Kashmir.The project is esti-
to determine the strength of the nuclear charge by mated to cost USD $1 billion. The first phase of the
looking at the oxidation number of the atom. Baglihar Dam was completed in 2004. With the second
27. (c) 28. (b) phase completed on 10 October 2008, Prime Minister
29. (b) Elements with a low ionization energy tend to be Manmohan Singh of India dedicated the 900-MW
reducing agents and form cations. Baglihar hydroelectric power project to the nation.
30. (c) In astronomy the horizon is the horizontal plane 42. (d) The head of the femur articulates with the acetabulum
through (the eyes of) the observer. It is the fundamental in the pelvic bone forming the hip joint, while the distal
plane of the horizontal coordinate system, the locus of part of the femur articulates with the tibia and patella
points that have an altitude of zero degrees. While forming the knee joint. By most measures the femur is
the strongest bone in the body. The femur is also the
similar in ways to the geometrical horizon, in this context
longest bone in the body.
a horizon may be considered to be a plane in space, 43. (c) The US team of scientists found the mechanism in the
rather than a line on a picture plane. hypothalamus- which is located deep inside the brain-
31. (c) The Sutlej is sometimes known as the Red River. It is and showed that it is responsible for the ageing pro-
the easternmost tributary of the Indus River. Its source cess. Scientists carried out a series of experiments to
is Lake Rakshastal in Tibet. From there, it flows at first find that they could extend the lives of mice by a fifth,
west-northwest for about 260 kilometres (160 mi) to the without the problems such as animals suffering from
Shipki La pass, entering India in Himachal Pradesh state. muscle weakness, bone loss or memory problems as-
It then turns slightly, heading west-southwest for about sociated with old age.
360 kilometres (220 mi) to meet the Beas River near 44. (b) The visible surface of the Sun, the photosphere, is the
Makhu, Firozpur district, Punjab state. layer below which the Sun becomes opaque to visible
32. (a) Limonite is an iron ore consisting of a mixture of light. Above the photosphere visible sunlight is free to
propagate into space, and its energy escapes the Sun
hydrated iron(III) oxide-hydroxides in varying
entirely.
composition. 45. (c) The Suez Canal is an artificial sea-level waterway in
33. (c) Black revolution is related to self dependence in Egypt, connecting the Mediterranean Sea and the Red
petroleum/crude oil. Sea.
34. (a) The thyroid also produces calcitonin, which plays a 46. (c) Seoul is the capital of South Korea. Pyongyang is the
role in calcium homeostasis. capital of North Korea.
35. (c) Plasmodesmata (singular: plasmodesma) are 47. (a) The reason of algae float to the surface during the day
microscopic channels which traverse the cell walls of & sink at night is due to photosynthesis. In Day time,
plant cells and some algal cells, enabling transport and the algae is producing oxygen. When enough Oxygen
communication between them. Plasmodesmata evolved is produced during the day , it gets trapped in bubbles
independently in several lineages, and species that and it can lift the clumps up to the surface. In night,
have these structures. this oxygen is consumed and CO2 is produced. So algae
sinks.
36. (a) The aim of statistical mechanics is the interpretation
48. (d) Berlin is the capital city of Germany. Berlin is located in
and prediction of the observed macroscopic properties northeastern Germany on the River Spree, it is the center
of matter in terms of the mechanical properties of the of the Berlin-Brandenburg Metropolitan Region.
constituent molecules and the nature of the interaction 49. (a) In Indonesia the 'splash and burn agriculture' is called
among them. It is restricted to the non-equilibrium as jhoom cultivation.
statistical mechanics of non-reacting gases that is to 50. (b) Theory of natural rights gave birth to the French
the theory of transport phenomena. Revolution and the Revolution in America.
37. (d) 51. (d) The 38th parallel north is a circle of latitude that is 38
38. (a) The Potomac River is located along the mid-Atlantic degrees north of the Earth's equatorial plane. It crosses
coast of the United States and flows into the Chesa- Europe, the Mediterranean Sea, Asia, the Pacific Ocean,
peake Bay. The river (main stem and North Branch) is North America, and the Atlantic Ocean. The 38th parallel
approximately 405 miles (652 km) long, with a drainage north formed the border between North and South
Korea prior to the Korean War.
area of about 14,700 square miles (38,000 km²). The
52. (a) The branch of science that studies cells is called
river forms part of the borders between Maryland and Cytology.
Washington, D.C., on the left descending bank and 53. (a) Kaziranga National Park is a national park in the
West Virginia and Virginia on the river's right descend- Golaghat and Nagaon districts of the state of Assam,
ing bank.
Geography/General Science D-15

India. A World Heritage Site, the park hosts two-thirds 69. (c) Alberto Juantorena is a former Cuban racer who is
of the world's great one-horned rhinoceroses. called El Caballo (the horse). Juantorena had been born
54. (b) Animals active at night are called Nocturnal. with flat feet that caused feet and back problems, and
55. (c) Henry Becquerel is associated with the discovery of he had to have corrective surgery in 1977. At the 1976
Radioactivity.
Summer Olympics, he became the only athlete to win
1 both the 400 and 800 m Olympic titles.
56. (b) The Moon taken 27 days to revolve around the earth.
3 70. (b) Mariana Trench or Marianas Trench is the deepest
57. (b)
part of the world's oceans. It is located in the western
58. (a) Paraffin wax is obtained from petroleum by dewaxing
light lubricating oil stocks. It is used in candles, wax Pacific Ocean, an average of 200 kilometres to the east
paper, polishes, cosmetics, and electrical insulators. It of the Mariana Islands, in the Western Pacific East of
assists in extracting perfumes from flowers, forms a Philippines.
base for medical ointments, and supplies a waterproof 71. (c) Keoladeo National Park, also called Keoladeo Ghana
coating for wood. In wood and paper matches, it helps National Park, is located in Bharatpur, Rajasthan. It is a
to ignite the matchstick by supplying an easily famous avifauna sanctuary that hosts thousands of
vaporized hydrocarbon fuel.nm birds, especially during the winter season. Over 230
59. (a) Cloud seeding is a form of weather modification, aimed species of birds are known to be resident. It is also a
at increasing precipitation. The most common major tourist centre with scores of ornithologists
chemicals used for cloud seeding include silver iodide, arriving here in the hibernal season.
potassium iodide and dry ice (solid carbon dioxide). 72. (c) Baglihar Dam, also known as Baglihar Hydroelectric
60. (b) When the retreating monsoon blows from the northeast Power Project, built on the Chenab River in the
across the Bay of Bengal, it picks up a significant amount southern Doda district of the Indian state of Jammu
of moisture, which is subsequently released after and Kashmir. This project was conceived in 1992,
moving back onto the peninsula. Thus, from October approved in 1996 and construction began in 1999. The
to December the coast of Tamil Nadu state receives at first phase of the Baglihar Dam was completed in 2004.
least half of its roughly 40 inches (1,000 mm) of annual 73. (d) 74. (d) 75. (c) 76. (c)
precipitation. 77. (a) The Radcliffe Line was the boundary demarcation line
61. (b) A solar eclipse is a type of eclipse that occurs when between the Indian and Pakistani portions of the
the Moon passes between the Sun and Earth, and when Punjab and Bengal provinces of British India. It was
the Moon fully or partially blocks the Sun. This can named after its architect, Sir Cyril Radcliffe, who, as
happen only at new moon when the Sun and the Moon the joint chairman of the two boundary commissions
are in conjunction. for the two provinces, received the responsibility to
62. (d) Heavy water or deuterium oxide is called heavy water equitably divide 175,000 square miles (450,000 km2) of
because it is composed of heavier isotope of hydrogen territory with 88 million people.
called deuterium. 78. (a)
63. (d) Most conducive soil for cotton is blacks soil because 79. (a) Indira Point is a village in the Nicobar district at Great
black oils are highly argillaceous, very fine grained and Nicobar Island of Andaman and Nicobar Islands, India.
dark, and contain a high proportion of calcium and It is located in the Great Nicobar tehsil. It is the location
magnesium carbonates. of the southernmost point of India's territory.
64. (c) The largest oil field of Gujarat is Ankleshwar. Oil at this 80. (a) Vitamin K affects the clotting mechanism by being
field is available at depths varying from 1,000 to 1,200 essential for the production of four distinct clotting
metres. It has a capacity of 2.8 million tonnes per annum. factors: prothrombin, factors VII, IX and X.
65. (a) Russia is the largest diamond producing country of 81. (d) 82. (a) 83. (a)
the world. In 2015, Russian miners extracted an 84. (a) Tallinn became the capital of an independent Estonia.
estimated 41.9 million carats of diamonds. After World War II started, Estonia acceded to the
66. (a) Tamil Nadu is the largest banana producing state in Soviet Union (USSR) in 1940, and later occupied by
India followed by Gujarat, Maharashtra and Andhra Nazi Germany from 1941 to 1944.
Pradesh. 85. (a) Asia, the largest continent, stretches from the eastern
67. (c) Satyajit Ray (2 May 1921 - 23 April 1992) was an Indian Mediterranean Sea to the western Pacific Ocean. There
filmmaker and author, widely regarded as one of the are more than 40 countries in Asia. Some are among
greatest filmmakers of the 20th century. He was born in the most-populated countries in the world, including
the city of Calcutta into a Bengali Brahmo family. China, India, and Indonesia.
68. (b) The Mandal Commission was established in India in 86. (a) 87. (c)
1979 by the Janata Party government under Prime 88. (b) The Aravalli Range is the oldest range of fold mountains
Minister Morarji Desai, to identify the socially or in India. The estimated age of Aravalli Range according
educationally backward. to geologists is about 350 million years. As for now the
EBD_8177
D-16 Geography/General Science

Aravalli Range of mountains is approximately 692 km in Emirates, Oman, India, Bangladesh, Myanmar, China,
the north–west part of India. It starts from Delhi and Taiwan, Mexico, Bahamas, Mauritania and Mali.
pass throungh southern Haryana and across the states 100. (b) Har Gobind Khorana was an Indian-American biochem-
of Rajasthan which ends in Gujarat. ist. While on the faculty of the University of Wiscon-
89. (a) Modern Agra was founded by Sikandar Lodhi in the sin–Madison, he shared the 1968 Nobel Prize for Phys-
16th century. Emperor Akbar built the Agra fort and iology or Medicine with Marshall W. Nirenberg and
Fatehpur Sikri near Agra. Robert W. Holley for research that showed.
90. (b) The boundary between the Sun’s interior and the solar 101. (c) Prime meridian is also known as Greenwich meridian,
atmosphere is called the photosphere. It is what we imaginary line used to indicate 0° longitude that pass-
see as the visible “surface” of the Sun. es through Greenwich, a borough of London, and ter-
91. (a) Stratosphere extends upwards from the tropopause to minates at the North and South poles. An internation-
about 50 km. It contains much of the ozone in the al conference held in Washington, D.C., in 1884 desig-
atmosphere. The increase in temperature with height nated “the meridian passing through the centre of the
occurs because of absorption of ultraviolet (UV) transit instrument at the Observatory of Greenwich as
radiation from the sun by this ozone. the initial meridian for longitude.”
92. (a) Caracas is the capital of Venezuela, Quito is the capital
102. (c) The highest frequency of the cyclones is in the month
of Ecuador, Bangkok is the capital of Thailand. So,
of October and the first half of November.
Bogota is capital of Colombia.
93. (d) Sahara desert · During the months of October-November, the
The Sahara desert is located in Africa continent and south-west monsoon winds become weaker and
remaining continents are located in Asia continent. start to retreat from the skies of North India. This
94. (b) Latitude phase of the monsoon is known as the retreating
Lines of latitude or parallels run parallel to the equator. monsoon.
95. (a) Benjamin Franklin was the one to prove that lightning 103. (b) The Khyber Pass is a mountain pass in the northwest
was a form of electricity. of Pakistan, on the border with Afghanistan. It con-
96. (d) Emissions associated with the combustion of fossil nects the town of Landi Kotal to the Valley of
fuels and biomass.Brown cloud atmosphere is formed Peshawar at Jamrud by traversing part of the Spin Ghar
by aerosol. These aerosols are mainly made of black mountains.
carbon, and organic carbon other aerosols include 104. (c) A nautical mile is a unit of length used in both air and
sulphates and nitrates, scatter solar radiation back to marine navigation, and for the definition of territorial
space .black carbon absorbs solar radiation resulting waters.
in solar heating of the atmosphere.
97. (d) Non-ferrous metals include aluminium, copper, lead, 105. (d) Narmada rises from Amarkantak Plateau near Anuppur
nickel, tin, titanium and zinc, and alloys such as brass. district. It forms the traditional boundary between
Metals or alloys that does not contain iron (ferrite) are North India and South India and draining through the
called non ferrous metals examples include aluminium, Gulf of Khambhat into the Arabian Sea.
copper, lead, nickel, tin, titanium, zinc and alloy such · The Tapti River (or Tapi) is a river in central India
as brass. between the Godavari and Narmada rivers. It is
98. (b) The sun lies at the heart of the solar system, where it draining through the Gulf of Khambhat into the
is by far the largest object. It holds 99.8 percent of the Arabian Sea.
solar system's mass and is roughly 109 times the · Ganga is a trans-boundary river of Asia which
diameter of the Earth - about one million Earths could flows through India and Bangladesh. The 2,525
fit inside the sun.
km river rises in the western Himalayas in the Indi-
99. (a) Starting at the prime meridian and heading eastward, an state of Uttarakhand, and flows south and east
the Tropic of Cancer passes through 16 countries: Al- through the Gangetic Plain of India an d
geria, Niger, Libya, Egypt, Saudi Arabia, United Arab Bangladesh, eventually emptying into the Bay of
Bengal.
3 Miscellaneous

1. The winner of the highest number of gold medals in an 12. ‘Duckworth Lewis Rule’ is used in the game of [2011-II]
Olympic game is [2011-I] (a) Lawn Tennis (b) Cricket
(a) Mark Spitz (b) Matt Biondi (c) Basketball (d) Rugby
(c) Michael Phelps (d) Jenny Thompson 13. Dronacharya award is given for outstanding contribution
2. ‘Agha Khan Cup’ is associated with the game of [2011-I] in the field of [2011-II]
(a) Football (b) Hockey (a) Sports (b) Sarv Shiksha Abhiyan
(c) Badminton (d) Cricket (c) Anganwadi (d) Music
3. Usain Bolt, the 100 meters race world record holder, is from 14. The first Olympic Games were held in 1896 at [2011-II]
which country ? [2011-I] (a) Rome (b) Athens
(a) Jamaica (b) U.S.A. (c) Paris (d) London
(c) Canada (d) Nigeria 15. Indigenously build supersonic cruise missile is known as
4. Only two cricket players have taken 10 wickets in an innings. [2011-II]
(a) Brahmos (b) Prithvi
One is Anil Kumble. The other is [2011-I]
(c) Nag (d) Astra
(a) Richard Hadlee (b) Muttiah Muralidharan
16. Defence Services Staff College is located at [2011-II]
(c) Jim Laker (d) Andy Roberts
(a) Khadakvasala (b) Secunderabad
5. The highest Indian gallantry award which could be given in (c) Dehradun (d) Wellington
peace time is [2011-I] 17. Who authored the book “Freedom at Midnight”?[2011-II]
(a) Ashok Chakra (a) Salman Rushdie
(b) Param Vir Chakra (b) Charles Dickens
(c) Kirti Chakra (c) Mahatma Gandhi
(d) Param Vishisht Seva Medal (d) Larry Collins and Dominique Lapierre
6. The most successful Satellite Launch Vehicle of Indian 18. Who was the first Indian woman to climb Mount Everest ?
Space Programme to launch commercial satellites is known [2011-II]
as [2011-I] (a) Junko Tabei (b) Bachendri Pal
(a) SLV (b) ASLV (c) Dola Banerjee (d) Sanamacha Chanu
(c) PSLV (d) GSLV 19. The United Nations Organisations responsible to maintain
7. The name of indigenously built Light Combat Aircraft is international peace & security, was established on 24
[2011-I] October in the year [2011-II]
(a) Tejas (b) Chakra (a) 1920 (b) 1945
(c) Vajra (d) Trishul (c) 1939 (d) 1942
8. Who authored the book ‘Train to Pakistan’? [2011-I] 20. Yoga sutra was written by [2012-I]
(a) Salman Rushdie (b) Khushwant Singh (a) Vatsyayana (b) Patanjali
(c) Mulk Raj Anand (d) Vikram Seth (c) Bhartrihari (d) Maharshi Mahesh
9. The famous classical dance form of Andhra Pradesh is 21. The pioneer of Atomic energy in India is [2012-I]
[2011-I] (a) Homi J Bhabha (b) Vikram Sarabhai
(a) Kathakali (b) Kuchipudi (c) C.V. Raman (d) C.K. Naidu
(c) Mohini Attam (d) Yakshaagna 22. Which one of the following statements regarding FIFA
10. ‘Borlaug Award’ is given every year to an Indian scientist World Cup 2010 is not correct ? [2012-I]
for outstanding contribution in the field of [2011-I] (a) South Africa became the first host nation to fail to
(a) Medicine (b) Space qualify for the tournament’s second round.
(b) The tournament was the culmination of a qualification
(c) Applied Science (d) Agriculture
process that began in August 2007.
11. UNHCR, an organisation of United Nations, was established
(c) This is the first time that the tournament was hosted
to provide/promote [2011-I]
by an African nation.
(a) Primary Education (b) Health and Culture
(d) Zakumi, the official mascot for the FIFA World Cup,
(c) Relief (d) Protection to refugees 2010 is an African bush elephant.
EBD_8177
D-18 Miscellaneous

23. The first Indian to win the World Amateur Snooker 35. The official song ‘WakaWaka’ of FIFA World Cup held in
Championship is [2012-I] the year 2010 at South Africa was sung by which popular
(a) Om Agarwal (b) Geet Sethi singer ? [2013-I]
(c) Michael Ferreira (d) Wilson Jones (a) Madonna (b) Destiny Child
24. The first person to win the Arjuna award for badminton is (c) Shakira (d) Rehanna
[2012-I] 36. In the year 2011, the ace badminton player Saina Nehwal
was honoured with which sports award ? [2013-I]
(a) Pullela Gopichand (b) Prakash Padukone
(a) Dronacharya award
(c) Nandu Natekar (d) Farook Engineer
(b) Rajiv Gandhi Khel Ratna award
25. The sport which requires the largest field is [2012-I] (c) Arjun award
(a) Football (b) Cricket (d) None of these
(c) Hockey (d) Polo 37. Karrar is the unmanned bomber aircraft of [2013-I]
26. The National Institute of Oceanography is located at (a) Iraq (b) Iran
[2012-I] (c) Turkey (d) Pakistan
(a) Trivandrum (b) Panaji 38. Who is the author of book ‘Keeping the Faith: Members of
(c) Cochin (d) Mangalore a Parliamentarian’? [2013-I]
27. With which sport is Lewis Hamilton associated ?[2012-II] (a) David Omand (b) Raja Shehadeh
(a) Golf (b) Hockey (c) Raghav Bahl (d) Somnath Chatterjee
(c) Billiards (d) F – 1 39. Who is the cricketer bestowed with an honorary commission
28. From which country does the top seeded Tennis player in the Territorial Army ? [2013-I]
Rafael Nadal hail from ? [2012-II] (a) Sachin Tendulkar (b) Kapil Dev
(a) France (b) Germany (c) Virendra Sehwag (d) Harbhajan Singh
40. NREGA stands for [2013-I]
(c) Spain (d) Russia
(a) National Revenue Engagement Guarantee Association
29. Which Indian Boxer won the gold medal in 60 kg category in
(b) National Revenue Employment Guarantee Act
Asian game and became the youngest to win a boxing gold
(c) National Rural Employment Guarantee Association
for India ? [2012-II] (d) National Rural Employment Guarantee Act
(a) Vijendra Singh (b) Vikas Krishan 41. Which of the following is not an agency of UN ? [2014-I]
(c) Ranjan Sodhi (d) Somdev (a) World Bank
30. In 2010 Leander Paes in partnership with Cara Black won (b) International Atomic Energy Agency
the mixed doubles title of [2012-II] (c) Universal Postal Union
(a) Australian Open Tennis Championship (d) None of the above
(b) Wimblendon Open Tennis Championship 42. Which kind of missile is BRAHMOS ? [2014-I]
(c) Both of the above (a) Medium range ballistic missile
(d) None of the above (b) Supersonic cruise missile
31. Nuclear Submarine Akula has been handed over to India (c) Short range tactical missile
recently by [2012-II] (d) Ultrasonic cruise missile
(a) France (b) Germany 43. Which Indian sportsperson was appointed as ‘Messenger
(c) USA (d) Russia of Peace’ in 2001 by the UN ? [2014-I]
32. Who is the author of book 'The Fragrance of Forgotten (a) Prakash Padukone (b) Vishwanathan Anand
(c) Sachin Tendulkar (d) VijayAmritraj
Years' ? [2012-II]
44. Indian Women’s hockey team secured which of the following
(a) David Omand (b) Bilkees Latif
position in Asian Champions Trophy 2013 ? [2014-I]
(c) Pranab Bardhan (d) Jagat S. Mehta
(a) First (b) Second
33. MNREGA stands for [2012-II] (c) Third (d) Fourth
(a) Mahatma Gandhi National Revenue Engagement Guar- 45. Which among the following is not a gallantry medal ?
antee Association [2014-I]
(b) Maharaja National Revenue Employment Guarantee (a) Ashok Chakra (b) Arjuna Award
Act (c) Param Vir Chakra (d) Shaurya Chakra
(c) Mahanagar National Rural Employment Guarantee Act 46. Indian Institute of Science, Bangalore was founded by
(d) Mahatma Gandhi National Rural Employment Guaran- [2014-I]
tee Act (a) CV Raman (b) Jamsetji Tata
34. Wayne Rooney, the famous footballer, hails from which (c) Vikram Sarabhai (d) None of these
country ? [2013-I] 47. Who is the author of 'Train to Pakistan':- [2014-II]
(a) Italy (b) Brazil (a) Arun Shourie (b) J.N. Dixit
(c) Argentina (d) England (c) Khushwant Singh (d) Ismat Chugtai
Miscellaneous D-19

48. The commonwealth Games Relay Baton has traditionally (a) Marshal Tito
contained ? [2014-II] (b) Mikhail Gorbachev
(a) sand from the last host city (c) Khan Abdul Ghaffar Khan
(b) a message from the Head of the Commonwealth (d) Abdul Wali Khan
(c) The commonwealth Games Motto 62. Sir CV Raman was awarded Nobel Prize for his work
(d) sand from each competing nation connected with which of the following phenomenon of
49. When is the World Poetry Day recognized by the UNESCO radiation? [2015-I]
observed ? [2014-II] (a) Scattering (b) Diffraction
(a) 02 February (b) 30 December (c) Interference (d) Polarisation
(c) 21 March (d) 14 February 63. In which city is headquarters of Asian Development Bank
50. Who was the first Commander in chief of Indian Air Force in located? [2015-I]
Independent India ? [2014-II] (a) Manila (b) Singapore
(a) Gen KM Kariyappa (c) Bangkok (d) Jakarta
(b) Field Marshal Sam Manekshaw 64. K-15 missile is [2015-I]
(c) Major Stringer Lawrence (a) Submarine launched Ballistic Missile (SLBM)
(d) Thomas Walker Elmhirst (b) Inter Continental Ballistic Missile (ICBM)
51. Who was the first woman to climb Mount Everest ? (c) Medium Range Ballistic Missile (MRBM)
[2014-II] (d) Short Range Ballistic Missile (SRBM)
(a) Junko Tabei (b) Tenzing Norgay 65. India agreed to UN Chief Ban ki-Moon's offer to remain as a
(c) Aarti Pradhan (d) Bachendri Pal member of the advisory board of one of the following
52. Who founded the Asiatic Society of Bengal in Kolkata? recently. [2015-I]
[2014-II] (a) UNCCT (b) UNICEF
(a) Warren Hastings (b) John Shore (c) UNEP (d) UNCTAD
(c) Sir William Jones (d) Lord Cornwallis 66. Who was the first Indian to win an individual medal in
53. The youngest mountain range in the world is:- [2014-II] Olympics? [2015-I]
(a) Himalayas (b) Alps (a) Milkha Singh (b) PT Usha
(c) Andes (d) Rockies (c) Karnam Malleshwari (d) KD Jadhav
54. Who amongst the following in the author of the book 'A 67. Which of the following Intercontinental Ballistic Missiles
Bend in the River? [2015-I] (ICBMs) is under development in India? [2015-I]
(a) Chetan Bhagat (b) VS Naipaul (a) Agni-I (b) Agni-II
(c) Kiran Desai (d) Anita Desai (c) Agni-IV (d) Agni-VI
55. 'Long Walk to Freedom' is a book written by [2015-I] 68. Who among the following was adjudged as the Most
(a) Sonia Gandhi (b) LK Advani Valuable Plaer of the 17th Asian Games held at Incheon, South
(c) Nelson Mandela (d) Benazir Bhutto Korea? [2015-I]
56. Which sports personality has been awarded the honorary (a) Mary Kom of India (b) Kosuke Hagino of Japan
rank of Group Captain by the IAF? [2015-I] (c) Ning Zetao of China (d) None of these
(a) Kapil Dev (b) Sania Mirza 69. Think-tank of Government of India that replaced Planning
(c) Saina Nehwal (d) Sachin Tendulkar Commission on 01 January 2015 is [2015- II]
57. Which county among the following has been declared Ebola- (a) NITI Vakya (b) NITI Dharma
free by WHO? [2015-I] (c) NITI Vajpayee (d) NITI Aayog
(a) Sierra leone (b) Liberia 70. What is the name of 'Innovative Council' proposed in the
(c) Nigeria (d) Guinea Rail Budget 2015 to promote innovation? [2015- II]
58. How many Gold medals did India win in the Incheon Asian (a) Rail Tech (b) Sankalp
Games held in Oct 2014? [2015-I] (c) Kayakalp (d) Navachar
(a) 10 (b) 11 71. Which city will host the 2022 CommonWealth Games?
(c) 12 (d) 8 [2015- II]
59. Who has been appointed as the new Finance Secretary of (a) Gold Coast, Australia
India? [2015-I] (b) Birmingham, United kingdom
(a) Arvind Mayaram (b) Rajiv Mehrishi (c) Lusaka, Zambia
(c) Kaushik Basu (d) Dinesh Gupta (d) Nairobi, Kenya
60. Which among the following is India's first long range 72. Serena Williams defeated _______ to win the women's
subsonic cruise missile? [2015-I] singles title of Australian Open 2015. [2015- II]
(a) Agni II (b) Prithvi (a) Venus Williams (b) Sania Mirza
(c) Dhanush (d) Nirbhay (c) Maria Sharapova (d) Ana Ivanovic
61. The highest civilian award of India 'Bharat Ratna' has been 73. Lines joining places of equal temperature are called [2015- II]
awarded to only two foreigners so far. One of them is Nelson (a) Isotherms (b) Isohyets
Mandela. The other is [2015-I] (c) Isomers (d) Isobars
EBD_8177
D-20 Miscellaneous

74. Sir C.V. Raman was awarded Noble Prize for his work (a) Sarnath lions (b) Gir lions
connected with which of the following phenomenon of (c) Khajurao (d) Ajanta caves
radiation? [2015- II] 87. Which of the following books is written by Arnold Toynbee?
(a) Scattering (b) Diffraction [2016 -I]
(c) Interference (d) Polarization (a) Mankind and Mother Earth
75. Recently tested Agni-V missile is capable of carrying nuclear (b) Tom Sawyer
payload of [2015- II] (c) The Future of Freedom
(a) 1000 Kg. (b) 1500 Kg. (d) The Moon's Last Sigh
(c) 2000 Kg. (d) 2500 Kg. 88. Who among the following is not a recipient of the 'Bharatiya
76. Vinod Mehta an eminent Indian journalist and founder editor- Jnanpith' Award'? [2016 -I]
in-chief of 'Outlook' published his autobiography by the (a) Ashapurna Devi (b) MF Hussain
name of [2015- II] (c) Raghupati Sahay (d) VK Gokak
(a) Lucknow Boy A Memoir 89. Which city will host the 2022 Common Wealth Games?
(b) One Life is Not Enough [2016 -I]
(c) Accidental Prime Minister (a) Gold Coast, Australia (b) Durban, South Africa
(d) The Substance and the Shadow (c) Lusaka, Zambia (d) Nairobi, Kenya
77. The book, 'Born Again on the Mountain' is authored by 90. Which of the following Asian countries celebrates its
[2015- II] Independence Day on 4th February? [2016 -I]
(a) Pooja Rani (b) Arunima Sinha (a) Sri Lanka (b) Bahrain
(c) Bachendri Pal (d) Urbashi Lal (c) Cambodia (d) Bangladesh
78. Which of the following Asian countries celebrates its 91. The present chairman of ISRO is [2016 -I]
Independence Day on 4th February? [2015- II] (a) MYS Prasad (b) Shailesh Nayak
(a) Sri Lanka (b) Bahrain (c) AS Kiran Kumar (d) K Radhakrishnan
(c) Cambodia (d) Bangladesh 92. Mangalyan, Mars Orbiter Mission (MOM) was launched in
79. Who has been awarded the DadaSaheb Phalke Award for [2016 -I]
the year 2014? (a) Jan 2013 (b) Oct 2012
(a) Pran (b) Shashi Kapoor (c) Oct 2013 (d) Nov 2013
(c) Dilip Kumar (d) Amitabh Bachhan 93. 'Ruhr of India" known for its rich coal deposits is in the State of
80. The Russian name of INS Vikramaditya is [2015- II] [2016 -I]
(a) Admiral Groshkov (a) Uttar Pradesh (b) Jharkhand
(b) Admiral Gorbachev (c) West Bengal (d) Bihar
(c) Admiral Nakhimov 94. The Indian space programme began in [2016 -I]
(d) Admiral Petr Bezobrazov (a) 1961 (b) 1962
81. The present chairman of ISRO is [2015- II] (c) 1965 (d) 1 969
(a) MYS Prasad (b) Shailesh Nayak 95. 'Bachpan Bachao Andolan (BBA), an organisation to protect
(c) AS Kiran Kumar (d) K Radhakrishnan the rights of children was founded by [2016 -I]
82. Indian-American music composer, Ricky Kej has won the (a) Amir Khan (b) Kailash Satyarthi
Grammy Award for the album [2015- II] (c) Om Prakash Gurjar (d) Santa Sinha
(a) In the Lonely Hour (b) Morning Phase 96. The book, 'On My Terms: From the Grassroots to the
(c) Beyonce (d) Winds of Samsara Corridors of Power,' is the autobiography of [2016 - II]
83. The book, "A China Passage' was written by [2016 -I] (a) Lal Thanhawla (b) Ram Jethmalani
(a) MJ Akbar (b) Jagmohan (c) Sharad Pawar (d) Okram Ibobi Singh
(c) Anees Jung (d) John Kenneth Galbraith 97. 21st Conference of Parties (COP21) on Climate Change was
84. Global 500 awards are given for outstanding achievement in held in Nov-Dec 2015 at [2016 - II]
which of the following fields? [2016 -I] (a) London (b) Singapore
(a) Population Control (c) Paris (d) New York
(b) Elimination of illiteracy 98. Who was honoured with the Order of Australia, the highest
(c) Campaigning against AIDS civilian award in January this year? [2016 - II]
(d) Protection of environment (a) Yashwant Sinha (b) Vashu Bhagnani
85. Pulitzer Prize is awarded for the outstanding work in the (c) Sajeev Koshy (d) Sanjeev Kapoor
field of [2016 -I] 99. Who among the following was awarded India's highest
(a) Science and Technology peacetime gallantry award, Ashoka Chakra posthumously
(b) Environmental Studies on 26 January 2016? [2016 - II]
(c) Literature and Journalism (a) Lance Naik Mohan Nath Goswami
(d) International Understanding (b) Lance Naik Albert Ekka
86. The national emblem. viz four lions standing back to back is (c) Lance Naik Hanamanthappa Koppad
an adaptation from which of the following:- [2016 -I] (d) Lance Naik Hemraj
Miscellaneous D-21

100. Highest Civilian Award, Bharat Ratna was conferred in 2015 115. What was the score of Dipa Kamarakar in the final round in
to [2016 - II] Rio Olympics? [2017 - I]
(a) Atal Bihari Bajpayee (b) LK Advani (a) 15.253 (b) 15.966
(c) Jai Prakash Narain (d) Madan Mohan Malviya (c) 15.066 (d) 15.216
101. The indigenously developed Navigational Satellite system 116. On which of the following rivers the Salal project was
is known as [2016 - II] launched? [2017 - I]
(a) IRGPS (b) Galileo (a) Narmada (b) Godavari
(c) IRNSS (d) GLONASS (c) Chenab (d) Mahanadi
102. Which cricketer has been conferred with Col C K Naidu life 117. Which port in Iran has been built by India? [2017 - I]
time achievement award in 2015? [2016 - II] (a) Chabahar Port (b) Bandar Abbas
(a) Bishen Singh Bedi (b) Kapil Dev (c) Bandar Anzali (d) Bandar Khomeini
(c) Mohinder Amarnath (d) Syed Kirmani 118. Which port in Pakistan has been built by China? [2017-I]
103. Who among the following will head the Judicial Committee (a) Karachi Port
on 'One Rank One Pension (OROP)' scheme? [2016 - II] (b) Gwadar Port
(a) Y V Reddy (b) L Narasimha Reddy (c) Port Muhammad Bin Qasim
(c) Sanjay Mishra (d) N S Reddy (d) None of these
104. The mascot of the South Asian Games conducted in 119. Which of the following is a classical dance of Kerala?
February 2016 was [2016 - II] [2017 - I]
(a) Chikor (b) Shera (a) Bharatanatyam (b) Kuchipudi
(c) Tikhor (d) Appu (c) Sattriya (d) Mohiniyattam
105. Who among the following shooters clinched the 50 m pistol 120. Which Indian sportsperson has been appointed as a member
gold at the ISSF World Cup in Bangkok in Mar 2016 [2016 - II] of the International Olympic Committee? [2017 - I]
(a) Jitu Rai (b) Pang Wei (a) Devendra Jhajharia (b) Saina Nehwal
(c) Wang Zhiwei (d) Potent Warren (c) Abhinav Bindra (d) Sakshi Malik
106. Which country will host Commonwealth Games in 2018? 121. Who wrote the epic poem 'Padmavat '? [2017 - I]
[2016 - II] (a) Malik Muhammad Jayasi (b) Tulsidas
(a) New Zealand (b) Australia (c) Kalidas (d) Asaga
(c) Scotland (d) Malaysia 122. Aladdin is the character of which of the following books?
107. India has inked a deal with France to acquire fighter aircraft, [2017 - I]
Dassault Rafale. Which is the other French fighter aircraft (a) The One Thousand and One Arabian Nights
in the inventory of Indian Air Force? [2016 - II] (b) Cinderella
(a) MiG 21 (b) Mirage 2000 (c) Snow White
(c) AJT Hawk (d) Su-30 Mkl (d) The Frog Prince
108. Deepika Padukone has received Filmfare award 2016 for the 123. Where is the office of the International Court of Justice
Best Actress in the movie [2016 - II] located? [2017 - I]
(a) Bajirao Mastani (b) Happy New Year (a) Hague (b) Washington
(c) Tamasha (d) Piku (c) Moscow (d) Venice
109. Who was the Indian Army Chief in October 2016?[2017 - I] 124. What is sin bin in ice hockey game? [2017 - I]
(a) Bipin Rawat (b) Dalbir Singh (a) Penalty box (b) Hockey sticks
(c) Bikram Singh (d) V K Singh (c) Dust bin (d) Rest room
110. Who was the Indian Navy Chief in October 2016?[2017 - I] 125. Who was the first cricket player to be declared out in a test
(a) Nirmal Kumar Verma (b) Devendra Kumar Joshi cricket match by third umpire? [2017 - I]
(c) Robin K Dhowan (d) Sunil Lanba (a) Rahul Dravid (b) Saurabh Ganguly
111. Yarlung Zangbo is the name of which river? [2017 - I] (c) Sachin Tendulkar (d) VVS Laxman
(a) Brahmaputra (b) Ganga 126. Satyajit Ray was a [2017 - I]
(c) Narmada (d) Mahanadi (a) Politician (b) Dancer
112. Which is the highest tower in the world? [2017 - I] (c) Filmmaker (d) Tabla player
(a) Tokyo Skytree (b) KVLY-TV mast 127. Which Cuban athlete is called 'horse'? [2017 - I]
(c) Burj Khalifa (d) Abraj Al Bait Towers (a) Dayron Robles (b) Iván García
113. Who defeated PV Sindhu in the final in Rio Olympics? (c) Alberto Juantorena (d) Héctor Herrera
[2017 - I] 128. Which of the following films won Oscar in the best film
(a) Carolina Marin (b) Michelle Li category in 2017? [2017 - I]
(c) Akane Yamaguchi (d) None of these (a) Moonlight (b) Captain Fantastic
114. In which South Korean city the Asian Games 2014 were (c) Loving (d) Nocturnal Animals
concluded? [2017 - I] 129. Which navy ship was decommissioned in 2017?[2017 - II]
(a) Incheon (b) Jecheon (a) INS Tarangini (b) INSV Mhadei
(c) Jeongeup (d) Namwon (c) INS Viraat (d) INS Tir
EBD_8177
D-22 Miscellaneous

130. Who won the Nobel Prize for the novel "Old Man and the 146. Who was the Indian Air Chief in November 2016?[2017 - II]
Sea"? [2017 - II] (a) Birender Singh Dhanoa (b) Arup Raha
(a) Alice Munro (b) Mo Yan (c) Anil Kumar Browne (d) Pradeep Vasant Naik
(c) Bob Dylan (d) Ernest Hemingway 147. Who won the Nobel Peace Prize in 2016? [2017 - II]
131. What was the old name of Myanmar? [2017 - II] (a) Juan Manuel Santos
(a) Burma (b) Newar (b) International Campaign to Abolish Nuclear Weapons
(c) Nippon (d) Sarawak (c) Tunisian National Dialogue Quartet
132. Aladdin is the character of which of the following books? (d) Malala Yousafzai
[2017 - II] 148. Pugilist is a term used for player of a game. Identify the
(a) The One Thousand and One Arabian Nights game. [2018 - I]
(b) Cinderella (a) Hockey (b) Badminton
(c) Snow White (c) Boxing (d) Billiards
(d) The Frog Prince 149. Theyyam is a tradition of which state? [2018 - I]
133. Where is the office of the International Court of Justice (a) Tamil Nadu (b) Karnataka
located? [2017 - II] (c) Andhra Pradesh (d) Kerala
(a) Hague (b) Washington 150. How many hurdles are there in 400m race? [2018 - I]
(c) Moscow (d) Venice (a) 6 (b) 8
134. What is sin bin in ice hockey game? [2017 - II] (c) 10 (d) 12
(a) Penalty box (b) Hockey sticks 151. In which two years were Asian games held in India?
(c) Dust bin (d) Rest room [2018 - I]
135. Which is the deepest trench in the world? [2017 - II] (a) 1951, 1982 (b) 1982, 2011
(a) Tonga Trench (b) Mariana Trench (c) 1951, 1983 (d) 1954, 1981
(c) Romanche Trench (d) Kermadec Trench 152. Who won 4 back to back titles in badminton? [2018 - I]
136. Who was the first cricket player to be declared out in a test (a) Pankaj Adwani (b)Prannoy Kumar
cricket match by third umpire? [2017 - II] (c) K. Srikanth (d)Mahesh Bhupti
(a) Rahul Dravid (b) Saurabh Ganguly 153. When was modern Olympic started? [2018 - I]
(c) Sachin Tendulkar (d) VVS Laxman (a) 6th April, 1986 (b) 6th April, 1896
137. Who was the first NSA of India? [2017 - II] (c) 11th July, 1807 (d) 4th June, 1907
(a) Brajesh Mishra (b) J N Dixit 154. What is the name of first artificial satellite of USA?
(c) M. K. Narayanan (d) Shivshankar Menon [2018 - I]
138. Satyajit Ray was a [2017 - II] (a) Explorer 1 (b) Discovery
(a) Politician (b) Dancer (c) Titan (d) Atlantis
(c) Filmmaker (d) Tabla player 155. How many states does tropic of cancer pass through?
139. Mandal Commission was established by which of the [2018 - I]
following Prime Ministers of India? [2017 - II] (a) 6 (b) 7
(a) Jawahar Lal Nehru (b) Morarji Desai (c) 8 (d) 9
(c) Rajiv Gandhi (d) VP Singh 156. Which Indian made the national record in long jump?
140. Which Cuban athlete is called 'horse'? [2017 - II] [2018 - I]
(a) Dayron Robles (b) Iván García (a) Ankit Sharma (b) Devendra Jhajharia
(c) Alberto Juantorena (d) Héctor Herrera (c) Anju Bobby George (d) Nayana James
141. Michael Ferreira is related to which sports? [2017 - II] 157. Where is NATO headquarter located? [2018 - I]
(a) Car racing (b) Badminton (a) Ottawa, Canada (b) Maxico city, Mexico
(c) Hockey (d) Billiards (c) Brussels, Belgium (d) New York, USA
142. Where is headquarter of World Health Organization (WHO) 158. Ruyder cup is associated with which sport? [2018 - I]
located? [2017 - II] (a) Men's golf (b) Soccer
(a) Hague (b) Washington (c) Badminton (d) Basketball
(c) Geneva (d) Beijing 159. In which year was the first FIFA world cup held? [2018 - I]
143. Who was named ICC Cricketer of the year 2016?[2017 - II] (a) 1931 (b) 1930
(a) Viraat Kohli (b) Ravichandran Ashwin (c) 1940 (d) 1935
(c) Steve Smith (d) Mitchell Johnson 160. Who is the author of 'Sleeping of Jupiter'? [2018 - I]
144. Which of the following states won Vijay Hazare Trophy (a) Jhumpa Lahiri (b) Anuradha Roy
2016-17? [2017 - II] (c) Shushma Swaraj (d) Vikram Seth
(a) Tamil Nadu (b) West Bengal 161. What is the maximum Number of timeouts in Volleyball that
(c) Kerala (d) Delhi a team can take? [2018 - II]
145. Bharatanatyam is the classical dance of which state? [2017 - II] (a) Maximum of 1 (b) Maximum of 2
(a) Kerala (b) Andhra Pradesh (c) Maximum of 3 (d) Maximum of 4
(c) Tamil Nadu (d) Karnataka
Miscellaneous D-23

162. Who was the captain Indian hockey team in 1928? 177. Number of Players in Basket Ball? [2019-I]
[2018 - II] (a) 11 (b) 9
(a) Lal shah Bokhari (b) Dhyan Chand (c) 7 (d) 5
(c) Kishan Lal (d) Jaipal Singh 178. ISRO’s Satish Dhawan Space Center is located at which
163. Who among the following was the first posthumous place? [2019-I]
recipient of Bharat Ratna? [2018 - II] (a) Andhra Pradesh (b) Telangana
(a) B.R. Ambedkar (b) Lal Bahadur Shastri (c) Tamil Nadu (d) Odisha
(c) K. Kamraj (d) M.G. Ramachandran 179. The Author of the book “The Golden Threshold”?
164. Baltic cup is related to which game? [2018 - II] [2019-I]
(a) Football (b) Hockey (a) R.K Narayan (b) Sarojini Naidu
(c) Volleyball (d) Badminton (c) Jhumpa Lahiri (d) Arundhati Roy
165. Hook pass is related to which of the following game? 180. Who is the Father of the Indian Space Program?
[2018 - II] [2019-I]
(a) Football (b) Hockey (a) Vikram Ambalal Sarabhai (b) A. P. J. Abdul Kalam
(c) Volleyball (d) Basketball (c) Homi Jehangir Bhabha (d) Satyendra Nath Bose
166. Who wrote Hindu Sanskrit Book Natya Shastra? 181. Uber cup is related to which sport? [2019-I]
[2018 - II] (a) Cricket (b) Badminton
(a) Bharata Muni (b) Manu Rishi (c) Football (d) Tennis
(c) Yagyawalak (d) Ashwagosh 182. Who is the youngest grandmaster in India? [2019-I]
167. JAXA is an aerospace agency of which of the following (a) Parimarjan Negi (b) D. Gukesh
country? [2018 - II] (c) Krishnan Sasikiran (d) Surya Shekhar
(a) Russia (b) China 183. From which country India bought C-17 transport aircraft?
(c) United Kingdom (d) Japan [2019-I]
168. "Double fault" is related to which of the following sport? (a) Iran (b) France
[2018 - II] (c) USA (d) Germany
(a) Rugby (b) Tennis 184. Asian games maximum number of times? [2019-I]
(c) Basket ball (d) Football (a) Japan (b) China
169. 'Lona' is the term associated with which sports? (c) Thailand (d) India
[2018 - II] 185. How many total medals have been won by Micheal Phelps
(a) Kho Kho (b) Kabaddi in Olympic games? [2019 - II]
(c) Lawn Tennis (d) Badminton (a) 28 (b) 26
170. Women who won first medal in Olympic in India was?
(c) 24 (d) 32
[2018 - II]
186. Duckworth-Lewis-Stern or DLS method is related to which
(a) Mary Kom (b) Jwala Gutta
game? [2019 - II]
(c) Karnam Malleswari (d) Sakshi Malik
(a) Football (b) Cricket
171. Who is the author of the book "Underground"?
(c) Hockey (d) Basket ball
[2018 - II]
187. Which cricketer reached fastest 10000 runs in ODI?
(a) Colson Whitehead (b) Khaled Hosseini
[2019 - II]
(c) Nora Roberts (d) Dean Koontz
(a) Sachin Tendulkar (b) Rahul Dravid
172. In 4 × 100 m race, which nation holds the record? [2019-I] (c) Virat Kohli (d) Rohit Sharma
(a) Nigeria (b) USA 188. Where is the Headquarter of WHO located? [2019 - II]
(c) Kenya (d) Jamaica (a) New York, America (b) Geneva, Switzerland
173. Which game is Geet Sethi associated with? [2019-I] (c) London, England (d) Paris, France
(a) Squash (b) Table Tennis 189. What is the distance between the stumps of both sides in
(c) Billiards (d) Golf cricket? [2019 - II]
174. In which state of India “Than Ta Dance” is related? (a) 20 Yards (b) 24 Yards
[2019-I] (c) 30 Yards (d) 22 Yards
(a) Meghalaya (b) Manipur 190. Which of the following book was authored by Vikram Seth?
(c) Assam (d) Sikkim [2019 - II]
175. Who was awarded first Bharat Ratana? [2019-I] (a) A suitable boy (b) The God of small things
(a) M. Visvesvaraya (b) Sarvepalli Radhakrishnan (c) Train to Pakistan (d) A history of sikhs
(c) Govind Ballabh Pant (d) Rajendra Prasad 191. Who among the following writer of the Mother India Book?
176. The first Modern Olympic held where and which city? (a) Hans Aanrud [2020 - I]
[2019-I] (b) Katherine Mayo
(a) France (b) Britain (c) Rachel Aaron
(c) Cuba (d) Greece (d) Ben Aaronovitch
EBD_8177
D-24 Miscellaneous

192. Mallet term is related to which sports? [2020 - I] 198. Dr. M.S. Subbulakshmi related with? [2020 - I]
(a) Cricket (b) Tennis (a) Hindustani Music (b) Dhrupad
(c) Hockey (d) POLO (c) Kathak (d) Carnatic Classical Music
193. Who among the following first person to take hat-trick in a 199. Which case Ram Prasad Bismil related? [2020 - I]
WorldCup? [2020 - I] (a) Alipore conspiracy (b) Kakori conspiracy
(a) Chetan Sharma (b) Hardik Pandya (c) Kanpur conspiracy (d) Meerut conspiracy
(c) Md. Shami (d) Jasprit Bumrah 200. What is the capital of Ghana? [2020 - I]
194. Capital of Barbados? [2020 - I] (a) Tirana (b) Tbilisi
(a) Andorra la Vella (b) Manama (c) Accra (d) Addis Ababa
(c) Bridge town (d) Nassau 201. Who wrote the book Gone with the Wind? [2020 - I]
195. How many number of bodies in UNO? [2020 - I] (a) Anand Neelakantan (b) Gita Sahgal
(a) Two (b) Four (c) Douglas Noel Adams (d) Margaret Mitchell
(c) Six (d) Seven 202. Where is Unido Headquarters? [2020 - I]
196. Irani Cup is related to which of the game ? [2020 - I] (a) New York (b) Brussels
(a) Badminton (b) Football (c) Vienna (d) Rome
(c) Hockey (d) Cricket 203. How many players are there in kabaddi? [2020 - I]
197. Cryolite is mainly found in which country ? [2020 - I] (a) Six (b) Eleven
(a) Canada (b) Iceland (c) Five (d) Seven
(c) Green land (d) Norway

ANSWERS & EXPLANATIONS


1. (c) Michael Fred Phelps (born June 30, 1985) is an American 6. (c) PSLV- The Polar Satellite Launch Vehicle, commonly
swimmer and 14-time Olympic gold medallist (the most known by its abbreviation PSLV, is an expendable
by any Olympian), who currently holds seven world launch system developed and operated by the Indian
records in swimming. He holds the record for the most Space Research Organisation (ISRO).
gold medals won at a single Olympics; a total of eight,
7. (a) The HAL Tejas is a 4+ generation, multirole light fighter
surpassing Mark Spitz, also a swimmer. Overall, Phelps
developed by India.
has won 16 Olympic medals: six gold and two bronze at
Athens in 2004, and eight gold at the 2008 Summer 8. (b) Train To Pakistan is a historical novel by Khushwant
Olympics in Beijing. Singh, published in 1956. It recounts the Partition of
India in August 1947.
2. (a) The Agha Khan Gold Cup was played in Dhaka,East
Pakistan(Bangladesh) which invited top club sides 9. (b) Kuchipudi is a Classical Indian dance from Andhra
from leading football playing nations to compete. Pradesh, In9dia.
3. (a) Usain St. Leo Bolt (born 21 August 1986) is a Jamaican 10. (d) The Borlaug Award is an award recognition conferred
sprinter widely regarded as the fastest person ever. by a fertilizer company, Coromandel International, for
outstanding Indian scientists for their research and
4. (c) Jim Laker and Anil Kumble are the only two players
contributions in the field of agriculture and
who have taken 10 wickets in an innings. James "Jim"
environment. The award was created in 1972 and named
Charles Laker (9 February 1922 - 23 April 1986) was a
in honour of Nobel Laureate Norman E. Borlaug. It
cricketer who played for England in the 1950s. Laker
carries a cash prize of Rs 500,000, a gold medal, and a
was the first player to take all 10 wickets in a Test
citation.
match innings, ten for 53 in the Australians' second
11. (d) The Office of the United Nations High Commissioner
innings of the fourth Ashes Test at Old Trafford in
for Refugees (UNHCR), also known as the UN Refugee
1956 (the only other bowler to take all 10 wickets is
Agency, is a United Nations agency mandated to
Anil Kumble of India in 1999).
protect and support refugees at the request of a
5. (a) The Ashoka Chakra is the peace time equivalent of the government or the UN itself and assists in their
Param Vir Chakra, and is awarded for the "most voluntary repatriation, local integration or resettlement
conspicuous bravery or some daring or pre-eminent to a third country. Its headquarters are in Geneva,
valour or self-sacrifice" other than in the face of the Switzerland and is a member of the United Nations
enemy. Development Group.
Miscellaneous D-25

12. (b) The Duckworth-Lewis method is a mathematical 25. (d) The playing field is 300 yards (274 metres) long by 160
formulation designed to calculate the target score for yards (146 metres) wide, the approximate area of nine
the team batting second in a limited overs cricket match American football fields. The playing field is carefully
interrupted by weather or other circumstances. maintained with closely mowed turf providing a safe,
13. (a) Dronacharya Award is an award presented by the fast playing surface. Goals are posts which are set eight
government of India for excellence in sports coaching. yards apart, centred at each end of the field.
14. (b) The 1896 Summer Olympics, officially known as the 26. (b) The National Institute of Oceanography (NIO) is one
Games of the I Olympiad, was a multi-sport event held of 37 constituent laboratories of the CSIR - Council of
in Athens, Greece, from 6 to 15 April 1896. Scientific and Industrial Research, an autonomous re-
15. (a) The BrahMos has been developed as a joint venture search organization in India. The institute has its head-
between the Defence Research and Development quarters in the coastal state of Goa i.e. Panaji, and re-
Organization (DRDO) of India and the Federal State gional centres in Kochi, Mumbai and Vizag.
Unitary Enterprise NPO Mashinostroyenia (NPOM) of 27. (d) Lewis Carl Davidson Hamilton, MBE is a British
Russia under BrahMos Aerospace. The missile is Formula One racing driver from England, currently
named after two rivers, the Brahmaputra and the racing for the Mercedes AMG team. He is the 2008
Moskva. Formula One World Champion. Hamilton was born in
16. (d) One of the oldest military institutions in India, it was Stevenage, Hertfordshire.
founded in 1905 as the Army Staff college in Deolali 28. (c) Rafael "Rafa" Nadal Parera (born 3 June 1986) is a
(near Bombay), relocated to its present home in Spanish professional tennis player and the current
Wellington Cantonment in The Nilgiris District of Tamil world No. 1. Nadal has won 13 Grand Slam singles titles,
Nadu, India. the 2008 Olympic gold medal in singles, a record 27
17. (d) Freedom at Midnight (1975) is a book by Larry Collins ATP World Tour Masters 1000 and a record 15 ATP
and Dominique Lapierre. It describes the events in the World Tour 500 tournaments.
Indian independence movement in 1947-48, beginning 29. (a)
with the appointment of Lord Mountbatten of Burma 30. (b) Leander Paes and Cara Black won the last senior title
as the last viceroy of British India, and ending with the of Wimbledon 2010 by beating Wesley Moodie and
death and funeral of Mahatma Gandhi. Lisa Raymond 6-4 7-6 (7-5) in the mixed doubles.
18. (b) Bachendri Pal (born 24 May 1954) is an Indian 31. (d)
mountaineer, who in 1984 became the first Indian 32. (b)
woman to reach the summit of Mount Everest.
33. (c) MGNREGA stands for Mahatma Gandhi National Rural
19. (d) Employment Guarantee Act offers to guarantee hundred
20. (b) Yoga sutra was written by Patanjali. days of wage-employment in a year to a rural
21. (a) Homi J. Bhabha was the eminent scientist who played household.
a key role in the development of the Indian atomic en- 34. (d) Wayne Mark Rooney is an English footballer who plays
ergy program. He is also considered as the father of as a forward for Manchester United and the England
India's nuclear program. He also established the Atomic national team. Aged nine, Rooney joined the youth
Energy Commission of India in 1948. team of Everton, for whom he made his professional
22. (d) The official mascot for the 2010 World Cup was Zakumi, debut in 2002 at the age of 16.
an anthropomorphised African leopard with green hair, 35. (c) The official song of the 2010 World Cup "Waka Waka"
presented on 22 September 2008. was performed by the Colombian singer Shakira and
23. (d) "Wilson Jones" was the first to win the world amateur the band Freshlyground from South Africa, and is sung
billiards title. He won this title in 1958, he won it in in both English and Spanish. The song is based on a
Sydney, Australia. He was awarded the Arjuna Award traditional African soldiers' song, "Zangalewa".
in 1962, the Padma Shri Award in 1965, and the 36. (b) Saina Nehwal was awarded the "Rajiv Gandhi Khel
Dronacharya Award in 1996. Ratna award" which is India's highest award for
24. (c) The first successful Badminton players to win the excellence in the field of sports, following her terrific
award was N.M. Natekar who won it in 1961 and Ms. achievements in badminton in the year 2010.
Meena Shah followed him to win it in the very next 37. (b) Karrar is an unmanned combat air vehicle produced
year in 1962. for the military of Iran. According to reports, the UCAV
can bomb targets at high speed. It is the first long-
EBD_8177
D-26 Miscellaneous

range unmanned aerial drone manufactured in Iran.The losing to hosts Japan by a solitary goal in the summit
long-range drone was unveiled on August 23, 2010 - one clash of the event at Kakamigahara, Japan.
day after the activation of the nuclear reactor in Bushehr. 45. (b) The Arjuna Awards were instituted in 1961 by the gov-
38. (d) ernment of India to recognize outstanding achievement
39. (b) On 24 September 2008 Kapil Dev joined the Indian in National sports. The award carries a cash prize of `
Territorial Army and was commissioned as a Lieutenant 500,000, a bronze statuette of Arjuna and a scroll.
Colonel by General Deepak Kapoor, Chief of the Army 46. (b) Indian Institute of Science (IISc) is a public university
Staff. He joined as an honorary officer. for scientific research and higher education located in
40. (d) It is an Indian law that aims to guarantee the 'right to Bengaluru (formerly Bangalore), India. Established in
work' and ensure livelihood security in rural areas by 1899 with active support from Jamshetji Tata it is also
providing at least 100 days of guaranteed wage locally known as the "Tata Institute". It acquired the
employment in a financial year to every household status of a Deemed University in 1958. IISc is widely
whose adult members volunteer to do unskilled manual regarded as India's finest institution in its field, and
work. has made significant contribution to advanced
computing, space, and nuclear technologies.
41. (b) The World Bank is a United Nations international fi-
nancial institution that provides loans to developing 47. (c ) Train To Pakistan is a historical novel by Khushwant
countries for capital programs. The World Bank is a Singh, published in 1956. It recounts the Partition of
component of the World Bank Group, and a member of India in August 1947. Instead of depicting the Partition
the United Nations Development Group. in terms of only the political events surrounding it,
Singh digs into a deep local focus, providing a human
The International Atomic Energy Agency (IAEA) is an
dimension which brings to the event a sense of reality,
international organization that seeks to promote the
horror, and believability.
peaceful use of nuclear energy, and to inhibit its use
for any military purpose, including nuclear weapons. 48. (b) The Queen's Baton Relay is one of the great traditions
The IAEA was established as an autonomous organi- of the Commonwealth Games, having started at the
zation on 29 July 1957. Though established indepen- Games in Cardiff, Wales, in 1958. The Baton is now as
dently of the United Nations through its own interna- much a part of the Commonwealth Games tradition as
tional treaty, the IAEA Statute, the IAEA reports to the torch is part of the Olympics.The relay traditionally
both the United Nations General Assembly and Secu- begins with a commencement ceremony at Buckingham
rity Council. Palace, London, which coincides with the city's
Commonwealth Day festivities. There Her Majesty
The Universal Postal Union is a specialized agency of
Queen Elizabeth II entrusts the baton containing Her
the United Nations that coordinates postal policies
'message to the athletes' to the first honorary relay
among member nations, in addition to the worldwide
runner. The relay concludes at the Opening Ceremony,
postal system.
as the final relay runner hands the baton back to Her
42. (b) BrahMos is a supersonic cruise missile that can be Majesty, or Her representative, and the message is read
launched from submarines, ships, aircraft or land. It is aloud. At that moment the Games begin.
a joint venture between Republic of India's Defence
49. (c) In November 1999, UNESCO designated World Poetry
Research and Development Organisation (DRDO) and
Day to be held on March 21 each year. The organization
Russian Federation's NPO Mashinostroeyenia who
recognized the important role of poetry in the arts and
have together formed BrahMos Aerospace Private
in cultures throughout the world and over time. It also
Limited.It is the world's fastest cruise missile in opera-
wanted the day to promote the efforts of small
tion. The missile travels at speeds of Mach 2.8 to 3.0.
publishers with regard to publishing poetry.
The land-launched and ship-launched versions are al-
ready in service, with the air and submarine-launched 50. (d) Air Marshal Sir Thomas Walker Elmhirst was the first
versions currently in the testing phase. Commander-in-Chief of the Indian Air Force. He was
Air Chief from 15 August 1947 to 21 February 1950. It
43. (d) On 9 February 2001 Vijay Amritraj was appointed UN
was Air Marshal Elmhirst, who insisted that the Indian
Messenger of Peace. He has been a committed advo-
Air Force be an independent service under no control
cate to people in need, devoting his time to raising
of the Army.
awareness on the issues of drugs and HIV/AIDS and
in raising funds to fight the spread of AIDS worldwide. 51. (a) Junko Tabei is a Japanese mountain-climber who, on
May 16 , 1975, became the first woman to reach the
44. (b) Indian women's hockey team had to be content with a
summit of Mount Everest.
silver medal in third Asian Champions Trophy after
Miscellaneous D-27

52. (c) The Asiatic Society was founded by Sir William Jones strengthen United Nations' counter-terrorism expertise.
on 15 January 1784 in a meeting presided over by Sir The Centre engages with the over 30 CTITF entities
Robert Chambers, the Chief Justice of the Supreme Court with expertise on a broad spectrum of counter terrorism
at the Fort William in Calcutta, then capital of the British related issues.
Raj, to enhance and further the cause of Oriental research. 66. (d) Khashaba Dadasaheb Jadhav (born January 15, 1926-
53. (a) Himalayas is one of the youngest mountain ranges in August 14, 1984) was an Indian athlete. He is best
the world, situated in the northern border of India and known as a wrestler who won a bronze medal at the
spread across six Asian countries -India, Pakistan, 1952 Summer Olympics in Helsinki.
Bhutan, Afghanistan, China and Nepal. The Himalayas 67. (d) Agni-VI is an intercontinental ballistic missile being
contains some of the highest peaks in the world that developed by the Defence Research and Development
includes Mount Everest, Karakoram and Organisation (DRDO) for the use of the Indian Armed
Kanchenjunga. Forces. Agni-VI will be a three-stage intercontinental
54. (b) 55. (c) 56. (d) ballistic missile, which is in the hardware development
57. (c) Nigeria is considered free of Ebola transmission by phase, after its design phase was completed.
WHO. 68. (b) Kosuke Hagino, 20, won medals in all seven events he
58. (b) India won 57 medals (11 gold, 10 silver, 36 bronze). competed in, winning gold in the men's 200m freestyle,
200m and 400m individual medleys, and the 800m
59. (b)
freestyle relay, in addition to one silver in the 400m
60. (d) Nirbhay is the first Indian long-range subsonic cruise freestyle and two bronze medals in the 100m and 200m
missile developed by Defence Research & Development backstroke events.
Organisation (DRDO). This missile can be launched
69. (d) NITI Aayog is the Government of India think tank
from multiple platforms like, air, land. Nirbhay missile's
established by PM Narendra Modi to replace Planning
flight test was successfully completed in October 2014.
Commission.
61. (c) Khan Abdul Gaffar Khan was another foreigner who
70. (c) 'Kayakalp' is the innovative council of Indian Railways for
received Bharat Ratna. He was the foremost 20th-
innovation, technology development and manufacturing.
century leader of the Pashtuns (a Muslim ethnic group
of Pakistan and Afghanistan), who became a follower 71. (b) Birmingham, United Kingdom will host the 2022
of Mahatma Gandhi and was called the "Frontier Commonwealth Games.
Gandhi." 72. (c) Serena Williams won the 2015 Australian Open
62. (a) Sir Chandrasekhara Venkata Raman, was an Indian (Women's singles) by defeating Maria Sharapova.
physicist, whose ground breaking work in the field of 73. (a) Isotherm is a line on the map connecting points having
light scattering earned him the 1930 Nobel Prize for the same temperature at a given time.
Physics. He discovered that, when light traverses a 74. (a) Sir C. V. Raman's ground breaking work in the field of
transparent material, some of the deflected light light scattering for which he was awarded the 1930
changes in wavelength. This phenomenon is now called Nobel Prizes for physics.
Raman scattering and is the result of the Raman Effect.
75. (b) 76. (a)
63. (a) The Asian Development Bank is a r egional
77. (b) Arunima Sinha wrote the book "Born again on the
development bank established on 22 August 1966
mountain", which was launched by PM Narendra Modi
which is headquartered in Metro Manila, Philippines,
in 2014.
to facilitate economic development in Asia.
78. (a) Sri Lanka celebrates its Independence Day of 4th
64. (a) K-15 Sagarika is a nuclear-capable submarine-launched
February.
ballistic missile with a range of 700 kilometres (435 mi).
It belongs to the K Missile family and forms a part of 79. (b) Indian actor and producer Shashi Kapoor was awarded
India's nuclear triad, and will provide retaliatory nuclear the DadaSaheb Phalke Award for the year 2014.
strike capability. 80. (a) Admiral Gorshkov is the Russian name of INS
65. (a) India agreed to remain a member of the UNCCT at the Vikramaditya.
offer of UN Chief Ban Ki Moon. The UNCCT was 81. (c) A. S. Kiran Kumar is the current chairman of Indian
established in 2011 within the United Nations Counter- Space Research Organisation (ISRO).
Terrorism Implementation Task Force (CTITF), under 82. (d) Ricky Kej won the Grammy at the 57th Annual Grammy
the leadership of the CTITF, to assist in meeting awards for his album Winds of Samsara, a collaboration
capacity-building needs of Member States, and to with South African flautish Wouter Kellerman.
EBD_8177
D-28 Miscellaneous

83. (d) The book "A China Passage" was written by John on the Australia Day. Other two recipients of the award
Kenneth Galbraith in which he wrote about his visit to were Chennupati Jagadish, an eminent professor at
China between September 4th to September 23rd 1972. Australian National University (ANU) in Canberra, and
84. (d) Global 500 awards are the awards given by Global 500 Jay Chandra, an eye doctor in New South Wales.
Environment Forum for outstanding performance in the 99. (a) President Pranab Mukherjee has conferred the
environment protection. country's highest peacetime gallantry award Ashok
85. (c) The Pulitzer Prize is an award for achievements in Chakra to Lance Naik Mohan Nath Goswami in January
newspaper and online journalism, literature, and musical 2016. He belonged to 9 PARA (Special Force) and laid
composition in the United States. down his life while fighting LeT militants in Kupwara
district of Jammu and Kashmir.
86. (a) The National emblem, viz four lions standing back to
back is an adaptation from Sarnath Lion Capital of 100. (a & d) Bharat Ratna in 2015 was given to freedom fighter
Ashoka on 26th January 1950. and educationist Madan Mohan Malviya and former
Indian Prime Minister Atal Bihari Bajpayee.
87. (a) The famous book Mankind and Mother Earth is written
by prominent world historian Arnold Joseph Toynbee 101. (c) The Indian Regional Navigation Satellite System (IRNSS)
which summarizes his Magnum Opus earlier in his with an operational name of NAVIC is an autonomous
career with an afterword cautioning against Nuclear regional satellite navigation system that provides
Proliferation. accurate real-time positioning and timing services. The
system at-present has a constellation of 7 satellites, with
88. (b) MF Hussain did not get Bharatiya Jnanpith award.
two additional satellites on ground as stand-by.
89. (b) Birmingham, United Kingdom will be the host of 2022
102. (d) Former India stumper Syed Kirmani was chosen for the
commonwealth game.
prestigious Col. C.K. Nayudu Lifetime Achievement
90. (a) Sri Lanka celebrates its independence day on 4th award for 2015. He has served as chairman of the senior
February. national selection committee and vice-president of the
91. (c) The present chairman of ISRO is Alur Seelin Kiran Karnataka State Cricket Association (KSCA).
Kumar. 103. (b) Justice (Retd.) L Narasimha Reddy, former Chief Justice
92. (d) Mangalayan. Mars Orbit Mission was launched on 5 of Patna High Court, headed the Judicial Committee
November 2013 by the Indian Space Research which looked into the implementation of One Rank One
Organisation (ISRO). Pension (OROP) scheme.
93. (b) Ruhr of India is known for its rich coal deposits in 104. (c) Tikhor, the baby Indian Rhino, was unveiled to the
Jharkhand. public as the official mascot for the 2016 South Asian
94. (b) Indian Space programmes began in 1962. Games. According to the organising committee, Tikhor
95. (b) Bachpan Bachao Andolan was founded by Kailash carried the message of Peace, Progress, and Prosperity
Satyarthi to protect the right of children in the South Asian region. This was in line with the
official motto, "Play for Peace, Progress and
96. (c) Indian politician Sharad pawar is the author of the book
Prosperity".
'On My Terms: From the Grassroots to the Corridors of
Power'. Through his five- decade-long career, during 105. (a) Jitu Rai won gold medal at the ISSF World Cup held in
which he has never lost an election, he has served as Bangkok in March 2016. Jitu reached the final by
chief minister of Maharashtra four times. He has often finishing third at the end of the 60 shot qualifying series
bucked the trend, preferring policy and pragmatism with a score of 562.
over populism. 106. (b) The 2018 Commonwealth Games, commonly known as
97. (c) 21st Conference of Parties (COP21) on Climate Change Gold Coast 2018 will be held on the Gold Coast,
was held in Paris, France, from 30 November to 12 Queensland, Australia, between 4 and 15 April 2018. It
December 2015. It was the 21st yearly session of the will be the fifth time Australia has hosted the
Conference of the Parties (COP) to the 1992 United Commonwealth Games.
Nations Framework Convention on Climate Change 107. (b) Dassault Mirage 2000 is a French multirole, single-
(UNFCCC) and the 11th session of the Conference of engine fourth-generation jet fighter manufactured by
the Parties (CMP) to the 1997 Kyoto Protocol. The Dassault Aviation. It was designed in the late 1970s as
Conference negotiated the Paris Agreement. a lightweight fighter to replace the Mirage III for the
98. (c) Sajeev Koshy, a dentist in Melbourne, received the French Air Force. The sale of US F-16s to Pakistan
Order of Australia medal for the year 2016 announced prompted India to enter talks with France regarding
Miscellaneous D-29

the purchase of the Mirage 2000. In October 1982, the 117. (a) Chabahar Port is a seaport in Chabahar located in
country placed an order with Dassault for 36 single- southeastern Iran, on the Gulf of Oman. The port will
seat Mirage 2000Hs and 4 twin-seat Mirage 2000THs. make way for India to bypass Pakistan in transporting
108. (d) Indian actress Deepika Padukone received Filmfare Award goods to Afghanistan using a sea-land route.
2016 in the Best Actress category for her film 'Piku'. 118. (b) Gwadar Port is a deep-sea port situated on the Arabian
109. (b) Dalbir Singh served as the Chief of the Indian Army Sea at Gwadar in Balochistan province of Pakistan. The
from 1 August 2014 to 31 December 2016. Current Army port features prominently in the China-Pakistan
Chief is Bipin Rawat (31 December 2016-present). Other Economic Corridor (CPEC) plan, and is considered to
former chiefs were Bikram Singh (1 June 2012-31 July be a link between the ambitious One Belt, One Road
2014) and V K Singh (1 April 2010-31 May 2012). and Maritime Silk Road projects.
110. (d) Sunil Lanba is the current Indian Navy Chief (31 May 119. (d) Mohiniyattam is a classical dance of Kerala. Sattriya,
2016-present). Other former chiefs were Robin K Kuchipudi, and Bharatanatyam are classical dances of
Dhowan (17 April 2014-31 May 2016), Devendra Kumar Assam, Andhra Pradesh, and Tamil Nadu, respectively.
Joshi (31 August 2012-26 February 2014) and Nirmal 120. (b) Indian shuttler Saina Nehwal has been appointed as a
Kumar Verma (31 August 2009-31 August 2012). member of the International Olympic Committee's (IOC)
111. (a) The Brahmaputra is a trans-boundary river which flows Athletes' Commission. The Athletes' Commission is
through China, India and Bangladesh. It is also known chaired by Angela Ruggiero and it comprises nine vice
by its Chinese name (Yarlung Zangbo) and by other presidents and 10 other members.
local Tibetan names. Ganga is a trans-boundary river 121. (a) Padmavat is an epic poem written in 1540 CE by Malik
of Asia which flows through India and Bangladesh. Muhammad Jayasi in the Awadhi language. The poem
Narmada, also called the Rewa, is a river in central India describes the historic siege of Chittor byAlauddin
and the fifth longest river in the Indian subcontinent. Khalji in 1303 CE.
Mahanadi is a major river in East Central India, which 122. (a) 'One Thousand and One Nights' is a collection of
flows through the states of Chhattisgarh and Odisha. Middle Eastern folk tales compiled in Arabic during
112. (c) Burj Khalifa in Dubai (828 mts) is the tallest building in the Islamic Golden Age. It is often known in English as
the world followed by Tokyo Skytree in Tokyo (643 the Arabian Nights. Aladdin is a famous character of
mts), KVLY-TV mast in Blanchard (628.8 mts) and Abraj the book.
Al Bait Towers in Mecca (601 mts). 123. (a) The office of the International Court of Justice is located
113. (a) Spanish badminton player Carolina Marin defeated PV in Hague, the Netherlands.
Sindhu in the final in Rio Olympics to win Gold medal. 124. (a) A penalty box or sin bin is the area in ice hockey where
PV Sindhu won a silver medal. Canadian player a player sits to serve the time of a given penalty.
Michelle Li was defeated by PV Sindhu in the round of
125. (c) Sachin Tendulkar became the first batsman to be
16 in the women's singles badminton playdowns at the
dismissed (run out) by using television replays in the
2016 Summer Olympics.
second day of the test match in November 1992 between
114. (a) The 2014 Asian Games were held in Incheon, South South Africa and India in Kingsmead, Durban. Karl
Korea from September 19 - October 4, 2014, featuring Liebenberg was the third umpire with Cyril Mitchley
439 events in 36 sports. Incheon was the third city in the on-field umpire.
South Korea after Seoul (1986) and Busan (2002) to
126. (c) Satyajit Ray (2 May 1921 - 23 April 1992) was an Indian
host the Asian Games.
filmmaker and author, widely regarded as one of the
115. (c) Dipa Kamarakar's score was 15.066 in the final round in greatest filmmakers of the 20th century. He was born in
Rio Olympics. Scores of artistic gymnasts- Simone the city of Calcutta into a Bengali Brahmo family.
Biles (USA), Maria Paseka (Russia) and Giulia
127. (c) Alberto Juantorena is a former Cuban racer who is
Steingruber (Switzerland)-were 15.966, 15.253 and
called El Caballo (the horse). Juantorena had been born
15.216, respectively.
with flat feet that caused feet and back problems, and
116. (c) Salal Dam, also known as Salal Hydroelectric Power he had to have corrective surgery in 1977. At the 1976
Station, is a power project on the Chenab River in the Summer Olympics, he became the only athlete to win
Reasi district of Jammu and Kashmir. Sardar Sarovar both the 400 and 800 m Olympic titles.
Dam is built on the Narmada River near Navagam,
128. (a) Moonlight won Oscar in the best film category in 2017.
Gujarat. Upper Indravati hydro power station is located
The film is directed by Barry Jenkins, based on Tarell
on Godavari. Hirakud Dam is built across the Mahanadi
Alvin McCraney's unpublished semi-autobiographical
River in Odisha.
EBD_8177
D-30 Miscellaneous

play In Moonlight Black Boys Look Blue. It stars 139. (b) The Mandal Commission was established in India in
Trevante Rhodes, André Holland, Janelle Monáe, 1979 by the Janata Party government under Prime
Ashton Sanders, Jharrel Jerome, Naomie Harris, and Minister Morarji Desai, to identify the socially or
Mahershala Ali. educationally backward.
129. (c) INS Viraat was a Centaur-class aircraft carrier of the 140. (c) Alberto Juantorena is a former Cuban racer who is
Indian Navy. The last British-built ship serving with called El Caballo (the horse). Juantorena had been born
the Indian Navy, the ship was the oldest aircraft carrier with flat feet that caused feet and back problems, and
in service in the world. The ship was completed and he had to have corrective surgery in 1977. At the 1976
commissioned in 1959 as the Royal Navy's HMS Summer Olympics, he became the only athlete to win
Hermes. Viraat was formally decommissioned on 6 both the 400 and 800 m Olympic titles.
March 2017. 141. (d) Michael Ferreira, also called "the Bombay Tiger", is
130. (d) Ernest Hemingway won the Nobel Prize in Literature notable amateur player of English billiards from India,
for his novel "Old Man and the Sea" in 1954. The novel and a three-time Amateur World Champion. He
was published in 1952 which tells the story of Santiago, participated in the Indian National Billiards
an aging Cuban fisherman who struggles with a giant Championship in 1960 for the first time, and in 1964
marlin far out in the Gulf Stream off the coast of Florida. represented India in the World Amateur Billiards
131. (a) The ruling military changed Burma's name to Myanmar Championship (WABC) held in New Zealand, where
in 1989, a year after thousands were killed in the he progressed to the semi-finals.
suppression of a popular uprising. Rangoon also 142. (c) The World Health Organization (WHO) is a specialized
became Yangon. Japan's old name was Nippon. Sarawak agency of the United Nations that monitors
is a state in Malaysia. Nepalese are also called Newar. international public health. It was established on 7 April
132. (a) 'One Thousand and One Nights' is a collection of 1948, headquartered in Geneva, Switzerland.
Middle Eastern folk tales compiled in Arabic during 143. (b) Indian bowler Ravichandran Ashwin won Sir Garfield
the Islamic Golden Age. It is often known in English as Sobers Trophy for ICC Cricketer of the Year 2016. He
the Arabian Nights. Aladdin is a famous character of has become the third India player and 12th player overall
the book. to win the prestigious Sir Garfield Sobers Trophy after
133. (a) The office of the International Court of Justice is located being named as the ICC Cricketer of the Year 2016.
in Hague, the Netherlands. 144. (a) 2016-17 Vijay Hazare Trophy was the 15th season of
134. (a) A penalty box or sin bin is the area in ice hockey where the Vijay Hazare Trophy. It was contested by the 28
a player sits to serve the time of a given penalty. domestic cricket teams of India. Tamil Nadu won the
tournament, beating Bengal by 37 runs in the final.
135. (b) Mariana Trench or Marianas Trench is the deepest
part of the world's oceans. It is located in the western 145. (c) Bharatanatyam is a major genre of Indian classical dance
Pacific Ocean, an average of 200 kilometres to the east that originated in Tamil Nadu. Traditionally,
of the Mariana Islands, in the Western Pacific East of Bharatanatyam has been a solo dance that was
Philippines. performed exclusively by women, and expressed Hindu
religious themes and spiritual ideas, particularly of
136. (c) Sachin Tendulkar became the first batsman to be
Shaivism, but also of Vaishnavism and Shaktism.
dismissed (run out) by using television replays in the
second day of the test match in November 1992 between 146. (b) Arup Raha was the Indian Air Chief in November 2016.
South Africa and India in Kingsmead, Durban. Karl He served as Indian Air Chief from 31 December 2013
Liebenberg was the third umpire with Cyril Mitchley to 31 December 2016. He was also the Chairman of
the on-field umpire. Chiefs of Staff Committee, a post occupied by India's
senior-most military officer, who advises the
137. (a) Brajesh Mishra was the first National Security Advisor
government and ensures jointsmanship in the armed
(NSA) of India who served from November 1998 to
forces.
May 2004. Other former NSAs were J N Dixit (2004-
2005), M. K. Narayanan (2005-2010) and Shivshankar 147. (a) Juan Manuel Santos Calderón is a Colombian politician
Menon (2010-2014). and the President of Colombia, in office since 2010. He
was the sole recipient of the 2016 Nobel Peace Prize for
138. (c) Satyajit Ray (2 May 1921 - 23 April 1992) was an Indian
his efforts negotiating a peace treaty with the FARC-
filmmaker and author, widely regarded as one of the
guerilla in the country.
greatest filmmakers of the 20th century. He was born in
the city of Calcutta into a Bengali Brahmo family. 148. (c) 149. (d) 150. (c) 151. (a) 152. (c) 153. (b)
Miscellaneous D-31

154. (a) 155. (c) 156. (a) 157. (c) 158. (a) 159. (b) India. It houses the Satish Dhawan Space Centre, one
160. (b) of the two satellite launch centres in India.
161. (b) Maximum of 2 time outs per game for a team is allowed. 179. (b)
Time out lasts 30 seconds. In official international 180. (a) Vikram Ambalal Sarabhai was an Indian scientist and
competitions two 60 seconds technical time-out are innovator widely regarded as the father of India’s
used when the leading team reaches the 8 or 16 point space programme. Sarabhai received the Shanti Swarup
mark. Bhatnagar Medal in 1962. The nation honoured him
162. (d) Jaipal Singh Munda (3 January 1903 - 20 March 1970) by awarding Padma Bhushan in 1966 and Padma
born in a Munda tribal family in Jharkhand, was a Vibhushan in 1972.
politician, prolific writer and sportsman. He captained 181. (b)
the Indian field hockey team to clinch gold in the 1928 182. (b) As per the latest data of the World Chess Federation;
Summer Olympics in Amsterdam. Lal Shah Bokari , D. Gukesh has become India’s youngest Grandmasters
Dhyan Chand and Kishan Lal also became the captain at 12 years, 7 months and 17 days with this
of Indian Hockey Team later. achievement, Gukesh has overtaken Praggnanandhaa
163. (b) who held the record at 12 years and 10 months in June
164. (a) The Baltic Cup is an international football competition 2018.
contested by the national teams of the Baltic States - 183. (c) 184. (a)
Estonia, Latvia and Lithuania. Finland has also 185. (a) Michael Phelps is an American swimmer. He has record
participated as a guest twice in this game. of winning 28 individual medals in Olympic include
165. (d) (23 gold, 3 silver and 2 bronze) during (2004 - 2016).
166. (a) Bharata Muni was an ancient Indian theatrologist and
musicologist who wrote the Natya Shastra, a 186. (b) This method is based on mathematical calculation is
theoretical treatise on ancient Indian dramaturgy and applied in case when the cricket match is interrupted
histrionics, especially Sanskrit theatre. Bharata is by weather or other circumstances.
considered the father of Indian theatrical art forms.
187. (c) Virat Kohli has become the fastest to reach 10000 ODI
167. (d) 168. (b) 169. (b) runs breaking Sachin Tendulkar's record. He creates
170. (c) The first Indian woman to ever win an Olympic medal this record in only 205 innings, 54 innings less than
was Karnam Malleswari who won a bronze medal at Sachin.
the Sydney Olympics in the Women's 69 kg category 188. (b) WHO is an acronym that stands for the World Health
in Weightlifting. Organization, which is a body of the United Nations
171. (a) Colson Whitehead is an American novelist. He is the that is tasked with matters of international public health.
author of six novels, including his debut work, the The body, which is part of the United Nations
1999 novel The Intuitionist, and The Underground Development Group, was established 70 years ago on
Railroad, for which he won the 2016 National Book April 7, 1948. Which has its headquarters in Geneva,
Award for Fiction and the 2017 Pulitzer Prize for Fiction. Switzerland.
173. (d) 173. (c) 174. (b) 189. (d) Two sets of wickets shall be pitched opposite and
175. (c) Rajagopalachari, Sarvepalli Radhakrishnan, C. V. parallel to each other at a distance of 22 yards/20.12 m
Raman were awarded the Bharat Ratana Award in the between the centres of the two middle stumps.
year 1954, after the effect of the constitution. 190. (a) Vikram Seth's famous writing include the travel book
176. (d) In 1896, the first modern Olympic was played in Athens, From Heaven Lake (1983), the verse novel The Golden
Greece. Gate (1986), and the epic novel A Suitable boy (1993).
177. (d) Basketball is a team sport in which two teams, most 191. (b) The book ‘Mother India’ authored by Katherine Mayo
commonly of five players each, opposing one another published in 1927 became a sensation in the Western
on a rectangular court. The only major sport strictly of world.
U.S. origin, basketball was invented by James Naismith 192. (d) Terms used in Polo : Chukker ; Bunder ; Mallet.
(1861–1939) on or about December 1, 1891. 193. (a) India’s paceman, Chetan Sharma, will forever have the
178. (a) Sriharikota is a barrier island off the Bay of Bengal honour of taking the first-ever Cricket World Cup hat-
coast located in the Nellore district of Andhra Pradesh, trick.
EBD_8177
D-32 Miscellaneous

194. (c) Capital of Barbados is Bridge town, Currency: conspiracy (1925). The Kakori Conspiracy (or Kakori
Barbadian dollar. Barbados is an island country in the train robbery or Kakori Case) was a train robbery that
Lesser Antilles of the West Indies. took place between Kakori and, near Lucknow, on 9
195. (c) The United Nations System consists of the United August 1925 durin g the Indian Independence
Nations, and the six principal organs of the United Movement against the British Indian Government.
Nations: the General Assembly, Security Council, 200. (c) The capital of Ghana is Accra , Currency: Ghanaian
Economic and Social Council (ECOSOC), Trusteeship cedi. Ghana ,officially the Republic of Ghana, is a
Council, International Court of Justice (ICJ), and the country located along the Gulf of Guinea and Atlantic
UN Secretariat. Ocean, in the subregion of West Africa.
196. (d) Trophy of Cricket: Deodhar Trophy, lrani Trophy, G. 201. (d) Margaret Mitchell was an American novelist. After a
D. Birla Trophy, C. K. Naydu Trophy etc. broken ankle immobilized her in 1926, Mitchell started
197. (c) Cryolite (Na3AlF6, sodium hexafluoroaluminate) is an writing a novel that would become Gone With the Wind.
uncommon mineral identified with the once-large Published in 1936, Gone With the Wind made Mitchell
deposit at Ivittuut on the west coast of Greenland, an instant celebrity and earned her the Pulitzer Prize.
depleted by 1987. 202. (c) The United Nations Industrial Development
198. (d) Dr M S Subbulakshmi was a legendary Carnatic Organization (UNIDO), is a specialized agency in the
vocalist. She was born on 16 September 1916 to veena United Nations system, headquartered in Vienna,
player Shanmukavadiver Ammal and stage performer Austria.
Subramania Iyer. 203. (d) Kabaddi is a game of 2 teams of 7 players each, where
199. (b) Ram Prasad Bismil, Ashfaqulla Khan and Roshan Singh one team becomes the raiders and the other team anti
were hanged to death for their involvement in Kakori raiders.
1 Practice Set
INSTRUCTIONS
1. This practice set comprises four sections. Section A : Verbal Ability in English; Section B : Numerical Ability; Section C :
Reasoning and Military Aptitude; Section D : General Awareness.
2. The set will consist of 100 questions and each questions will be of 3 marks.
3. Each questions have four options, of which one is correct. The students are advised to read all the options thoroughly.
4. There is one-third negative marking in the set.

Time: 2 hrs. Max. Marks: 300


Ti

SECTION-A : VERBAL ABILITY IN ENGLISH DIRECTIONS (Qs. 4 - 8) : Select the most appropriate word
from the options against each number :
DIRECTIONS (Qs. 1 - 3) : Read the following passage carefully
and answer the questions given below it : One fine morning a (4) man knocked at the doors of the home for
the aged run by nuns. He told the nun in charge that as he was (5)
One day an army group won a land battle against the enemy. to Delhi, he wanted to leave his servant-maid to the (6) of the
The commander feared that the enemy’s powerful air force might nuns. He assured the nun of sending some money every month
bomb his camp that night in revenge. So, he ordered all lights to (7) she was an orphan. The nun (8) her saying that she had got an
be put out at 7:00 pm. At midnight the commander went round excellen master.
inspecting the camp. Seeing a light in a tent, he entered it. His 4. (a) gentle (b) bad
(c) nice (d) good
son, an officer under him, was writing a letter. The son explained 5. (a) moved (b) shifted
that he was writing to his mother about his brave deeds in battle. (c) changed (d) transferred
The commander told his son to add to his letter that by the time 6. (a) care (b) home
his mother received the letter he would have been shot dead for (c) custody (d) protection
indiscipline. 7. (a) because (b) and
(c) though (d) if
1. The commander went round the camp at midnight because he
8. (a) loved (b) praised
(a) was too tired from the day’s battle to go to sleep (c) consoled (d) condoled
(b) wished to check if his soldiers had obeyed his order
(c) was too worried about the next day’s battle DIRECTIONS (Qs. 9 - 10) : Choose the one which best expresses
the meaning of the given word and mark it in the Answer Sheet.
(d) wished to check if enemies had entered his camp
2. The commander entered his son’s tent because he 9. Wily
(a) Angry (b) Wise
(a) wished to see and talk to his son (c) Stupid (d) Cunning
(b) suspected that enemies had entered his tent 10. Temerity
(c) wished to send a message to his wife (a) Paucity (b) Verity
(d) had to punish any soldier who disobeyed his order (c) Audacity (d) Simplicity
3. The son was writing a letter because he DIRECTIONS (Qs. 11 - 13) : Choose the word opposite in
meaning to the given word and mark it in the Answer Sheet.
(a) wanted to write to his mother about his father’s brave
deeds in battle 11. Fabricate
(a) Unearth (b) Construct
(b) loved his mother so much that he had to write to her
(c) Demolish (d) Renovate
(c) was eager to tell his mother about his own deeds 12. Gregarious
(d) did not care for orders, since his father was the (a) Sociable (b) Societal
commander (c) Unsociable (d) Solitary
EBD_8177
PS-2 Practice Set - 1

13. Pragmatic
(a) Indefinite (b) Vague 25. While in conversation with a high military officer
a
(c) Optimistic (d) Idealistic
DIRECTIONS (Qs. 14 - 18) : Each of these questions has an he told me that at the headquarters nothing was known .
idiomatic expression followed by four options. Choose the one b c d
closest to its meaning. The subject should come before the verb.
14. In the blues: SECTION-B : NUMERICAL ABILITY
(a) Being colourful
(b) Melancholy and low spirited 26. 465 coins consists of 1 rupee, 50 paise and 25 paise coins.
(c) Behave like a lord Their values are in the ratio 5: 3 : 1. The number of each type
(d) Cheerful and happy of coins respectively is
(a) 155, 186, 124 (b) 154, 187, 124
15. See eye to eye:
(a) State at each other (b) Agree (c) 154, 185, 126 (d) 150, 140, 175
27. A car covers four successive 6 km stretches at speeds of
(c) Depend on (d) Make an effort
25 kmph, 50 kmph, 75 kmph and 150 kmph respectively. Its
16. Talk shop:
average speed over this distance is
(a) Talk about one’s profession
(a) 25 kmph (b) 50 kmph
(b) Talk about shopping
(c) 75 kmph (d) 150 kmph
(c) Ridicule 28. Kabir buys an article with 25% discount on its marked price.
(d) Treat lightly He makes a profit of 10% by selling it at ` 660. The marked
17. Bad blood: price is
(a) Infected blood (b) Ill-feeling (a) ` 600 (b) ` 685
(c) Unfaithful (d) Suspicion (c) ` 700 (d) ` 800
18. Wear one’s heart on one’s sleeve:
29. In a class of 250 students, 75.8% took French and 49.4%
(a) Lure passionately (b) Do the right thing took Latin. How many students took both French and Latin ?
(c) Show one’s feelings (d) Be intimate (a) 189.0 (b) 123.0
DIRECTIONS (Qs. 19-22) : In the following questions, a word (c) 63.0 (d) 90.0
has been spelt in four different ways, one of which is correct. 30. A is 3 times more efficient than B. Hence, he takes 60 days
Choose the correctly spelt word. less in painting a room. In how many days, work will be
19. (a) Dysentary (b) Dysantery completed, if A and B both work together ?
(c) Dysentry (d) Dysentery (a) 30 days (b) 45 days
20. (a) Rejevanation (b) Rejuvenation
(c) Rejvenation (d) Rejuenation 1 1
(c) 22 days (d) 17 days
21. (a) accomodate (b) acommodate 2 2
(c) accommodate (d) accommodat 31. In a family, a couple has a son and daughter. The age of the
22. (a) vegeterian (b) vegetarian father is three times that of his daughter and the age of the
(c) vegetarean (d) vegitarean son is half of his mother. The wife is nine years younger to
her husband and the brother is seven years older than his
DIRECTIONS (Qs. 23-25) : In each of the following sentences
sister. What is the age of the mother ?
four words or phrases have been underlined. Only one under-
(a) 40 years (b) 45 years
lined part in each sentence is not acceptable in Standard (c) 50 years (d) 60 years
English, Pick up the part a, b, c or d. 32. The monthly income of Komal and Asha are in the ratio of
4 : 3. Their monthly expenses are in the ratio of 3 : 2. However,
23. I have seen as bad or worse scenes of disorder at the both save ` 600 per month. What is their total monthly
a b income ?
(a) ` 8,400 (b) ` 5,600
English fair than in any other Australian mining town.
(c) ` 4,200 (d) ` 2,800
c d 33. A worker reaches his factory 3 minutes late if his speed from
24. The officers are now perfectly happy fishing, boating, his house to the factory is 5 km/hr If he walks at a speed of
6 km/hr, then he reaches the factory 7 minutes early. The
a b
distance of the factory from his house is
shooting, playing cricket and other sports . (a) 4 km (b) 5 km
c d (c) 6 km (d) 7 km
Practice Set - 1 PS-3

34. One local and another express train were proceeding in 43. Which of the following pair of fractions adds up to a number
the same direction on parallel tracks at 29 km/hour and greater than 5 ?
65 km/hour respectively. The driver of the former noticed
that it took exactly 16 seconds for the faster train to pass by 13 11 11 8
(a) , (b) ,
him. What is the length of the faster train ? 5 6 4 3
(a) 60 m (b) 120 m
(c) 160 m (d) 240 m 7 11 5 3
(c) , (d) ,
35. A gardener increased the area of his rectangular garden by 3 5 3 4
increasing its length by 40% and decreasing its width by
20%. The area of the new garden 9 2 8 5
44. If the fractions , , , are arranged in ascending
(a) has increased by 20%. 13 3 11 7
(b) has increased by 12%.
order, then the correct sequence is :
(c) has increased by 8%.
(d) is exactly the same as the old area. 9 2 8 5 2 9 5 8
36. There are two candidates Bhiku and Mhatre for an election. (a) , , , (b) , , ,
13 3 11 7 3 13 7 11
Bhiku gets 65 % of the total valid votes. If the total votes
were 6,000, what is the number of valid votes that the other
2 8 5 9 5 8 2 9
candidate Mhatre gets if 25 % of the total votes were (c) , , , (d) , , ,
declared invalid ? 3 11 7 13 7 11 3 13
(a) 1575 (b) 1625 45. A cistern has three pipes, A, B and C. The pipes A and B can
(c) 1675 (d) 1525 fill it in 4 and 5 hours respectively and pipe C can empty it in
37. A sum of `5,000 lent on simple interest amounts to ` 5,700 2 hours. If the pipes are opened in an order at 1, 2 and 3
in two years. If the rate of interest is increased by four A.M., when will the cistern be empty?
percentage, then what would the sum amount to in the same
period ? (a) 3 P.M. (b) 7 P.M.
(a) ` 5,980 (b) ` 6,100 (c) 4 P.M. (d) 5 P.M.
(c) ` 5,900 (d) ` 6,300 SECTION-C : REASONING
38. Mira’s expenditure and savings are in the ratio 3 : 2. Her & MILITARY APTITUDE
income increases by 10%. Her expenditure also increases
by 12%. By how much % do her savings increase ? 46. Identify the diagram that best represents the relationship
(a) 7% (b) 9% among the classes given below :
(c) 10% (d) 13%
Liquids, Milk, River water
39. The difference between the simple interest and the
compound interest compounded annually at the rate of 12%
per annum on Rs 5000 for two years will be :
(a) ` 47.50 (b) ` 63 (a) (b)
(c) ` 45 (d) ` 72
40. A pipe can fill a cistern in 6 hours. Due to a leak in its
bottom, it is filled in 7 hours. When the cistern is full, in how
much time will it be emptied by the leak ?
(a) 42 hours (b) 40 hours
(c) 43 hours (d) 45 hours (c) (d)
41. A man in a train notices that he can count 21 telephone
posts in one minute. If they are known to be 50 metres apart,
then at what speed is the train travelling ? 47. Identify the diagram that best represents the relationship
(a) 45 km/h (b) 60 km/h among milk, goat, cow, hen?
(c) 63 km/h (d) 65 km/h Answer figures :
42. A and B run a 5 km race on a round course of 400 m. If their
speeds be in the ratio 5 : 4, how often does the winner pass
the other? (a) (b)
1 3
(a) 4 times (b) 2 times
2 4
1 1 (c) (d)
(c) 3 times (d) 2 times
2 2
EBD_8177
PS-4 Practice Set - 1

48. Identify the figure which best represents the relationship 58. Cassock : Priest :: ? : Graduate
among Tree, Plant, and House. (a) Cap (b) Tie
Answer figures : (c) Coat (d) Gown
59. Ornithologist : Bird :: Archealogist : ?
(a) Islands (b) Mediators
(a) (b)
(c) Archealogy (d) Aquatic
60. Cloth : Mill :: Newspaper : ?
(a) Editor (b) Reader
(c) (d) (c) Paper (d) Press
DIRECTIONS (Qs. 61-65) : choose the most appropriate answer:
49. Identify the diagram which represent the relationship among 61. Which one of the following is always in 'Sentiment'?
the following :- Capsules, Antibiotics, Injection.
(a) Cruelty (b) Insight
(c) Neutrality (d) Emotion
(a) (b) 62. Controversy always involves
(a) Dislike (b) Injustice
(c) Disagreement (d) Passion
63. What is found necessarily in newspaper?
(a) Date (b) Advertisement
(c) (d)
(c) News (d) Editor
64. If we are going early in the morning towards the south the
50. Identify the diagram which represents the best relationship sun will be visible at our left:
among athletes, football players and cricket-players. (a) Always (b) Never
(c) Often (d) Sometimes
65. Which one of the following is always associated with
(a) (b) 'justice'?
(a) Hypocrisy (b) Legitimate
(c) Magnanimity (d) Diminutiveness
DIRECTIONS (Qs. 66-69) : Two Sets of the figures are given.
One set of Question-figures and another set is of Answer-figures.
(c) (d)
Question-figures are arranged in a sequence. One figure from
the Answer figures is to be selected such that it can be placed
DIRECTIONS (Qs. 51-55) : In each of the following questions, after the series of Question-figures. Find the correct Serial
four words have been given out of which four are alike in some number of the selected Answer-figure.
manner, while the fourth one is different. Choose the word which 66. Question-Figures
is different from the rest.
51. (a) Pear (b) Apple
(c) Litchi (d) Orange
52. (a) Chameleon (b) Crocodile
Answer-Figures
(c) Alligator (d) Locust
53. (a) Cumin (b) Groundnut
(c) Cinnamon (d) Pepper
54. (a) Sleet (b) Mist
(c) Hailstone (d) Vapour (a) (b) (c) (d)
55. (a) Zinc (b) Iron 67. Question-Figures
(c) Aluminium (d) Mercury
DIRECTIONS (Qs. 56-60 ) : In each of the following questions
find out the alternative which will replace the question mark.
56. Microphone : Loud :: Microscope : ? Answer-Figures
(a) Elongate (b) Investigate
(c) Magnify (d) Examine
57. Poles : Magnet :: ? : Battery
(a) Cells (b) Power
(c) Terminals (d) Energy (a) (b) (c) (d)
Practice Set - 1 PS-5

68. Question-Figures 72. Problem-figures:


+ +
+ + ?
(1) (2) (3) (4)
Answer-Figures
Answer-figures:
+
+

(a) (b) (c) (d) (a) (b) (c) (d) (e)


69. Question-Figures

DIRECTIONS (Qs. 73 - 76) : In each of the following questions


four figures are given. One of these figures does not fit with the
rest of the figures. Find out that correct serial number.

Answer-Figures 73.

(1) (2) (3) (4)


(a) (b) (c) (d)
(a) (b) (c) (d) 74.
DIRECTIONS (Qs. 70 - 72): In the following questions there is
some relationship between the two figure on the left of (::) the
same relationship exists between the two terms on the right, of (1) (2) (3) (4)
which one is missing. Find the missing one from the given (a) (b) (c) (d)
alternative.
75.
70. Question-Figures

: :: : ? (a) (b) (c) (d)

A F Z B
Answer-Figures
76.

(a) (b) (c) (d)


DIRECTIONS (Qs. 77 - 80) : In each of the questions, there is a
(a) (b) (c) (d) big figure at the top having a vacant chamber. Identify one
71. Question-Figures figure from the given choices which will fit into the vacant
chamber.

: :: : 77.

Answer-Figures

(1) (2) (3) (4) (1) (2) (3) (4)


(a) (b) (c) (d)
(a) (b) (c) (d)
EBD_8177
PS-6 Practice Set - 1

86. The longest river of peninsular India is


78. (a) Godavari (b) Krishna
(c) Kaveri (d) Narmada
87. Minorities Rights Day is observed in India on
(a) 18th December (b) 23rd December
(c) 5th September (d) 1st December
(1) (2) (3) (4) 88. "Sattriya Nritya” recognised as a classical dance form of
(a) (b) (c) (d) India by the Sangeet Natak Akademi only in 2000, originated
from
79.
(a) Tripura (b) Assam
(c) Karnataka (d) Gujarat
89. Which one of the following National Park/Sanctuary is not
in Rajasthan ?
(a) Sariska National Park
(b) Sambar Wildlife Sanctuary
(a) (b) (c) (d)
(c) Rajaji National Park
80.
(d) Rhanthambore National Park
? 90. Which of the following is associated with Panchayati Raj ?
(a) Nanavati Commission
(b) Balwant Rai Mehta Committee
(c) Librahan Commission
(a) (b) (d) Shah Commission
91. ‘Pehli Udaan’ is a name given to
(c) (d) (a) Launching of Air Asia in India
(b) SBI's Savings Account Scheme for children
SECTION-D : GENERAL AWARENESS (c) Proposed Bullet Train in India
81. Under which Article of the Constitution can an individual (d) Satellite sent to Mars
move to the Supreme Court directly in case of any violation 92. Jean Tirole has won Nobel Prize in which of the below
of Fundamental Rights ? category?
(a) Article 31 (b) Article 32
(a) Literature (b) Physics
(c) Article 28 (d) Article 29
(c) Chemistry (d) Economics
82. The Dandi March of Gandhi-is an example of
93. What does ‘Ozone Layer’ absorb?
(a) Non-Coopefation (b) Direct Action
(c) Boycott (d) Civil Disobedience (a) g-rays (b) Infrared rays
83. Which one of the following inscriptions relate to the (c) Ultraviolet rays (d) X-rays
Chalukya king, Pulakesin II ? 94. Who among the following is the author of "The scatter here
(a) Nasik (b) Maski is too great"?
(c) Hathigumpha (d) Aihole (a) Kedarnath Singh (b) Bilal Tanweer
84. The filtration unit of kidney is
(c) Deepti Kapoor (d) Mahesh Rao
(a) yellow fiber (b) axon
95. The Pilotless target aircraft, fabricated at the Aeronautical
(c) nephron (d) neuron
Development Establishment, Bengaluru, is:
85. Yeast is an important source of
(a) Lakshya (b) Cheetah
(a) protein (b) vitamin B
(c) invertase (d) vitamin C (c) Nishant (d) Arjun
Practice Set - 1 PS-7

96. Which of the following statements are true about the impact 98. Which one of the following is not a line of demarcation
of Alexander’s invasion of India? between two countries ?
(a) Alexander destroyed the power of petty states in the (a) Durand Line (b) Mac Mahon Line
north (c) Plimsoll Line (d) Maginot Line
(b) The invasion opened up four distinct routes by land 99. Who among the following first propounded the idea of
and sea Basic Education?
(c) Greek settlements such as Buoukephala on the Indus (a) Jawahar Lal Nehru (b) Raja Ram Mohan Roy
were established
(c) Mahatma Gandhi (d) Dayanand Saraswati
(d) His invasion helps as to build Indian chronology for
subsequent events on a definite basis. 100. Which of the following Countries will the host for the 2020
97. The Blue Revolution is related with Olympic Games?

(a) Fish production (b) Food grain production (a) South Africa (b) Netherlands
(c) Oilseed production (d) Milk production (c) Japan (d) Italy

ANSWERS & SOLUTIONS


1. (b) The commander had ordered the lights to be put off by 26. (a) The ratio of number of coins = 5 : 6 : 4
7 : 00 pm, he took a round to check if his soldiers had
obeyed his orders or not. 465
\ The number of one rupee coins = ´ 5 = 155
2. (d) Refer to the last two lines of the passage. 5+6+4
3. (c) As written in the passage. 465
4. (a) 5. (d) 6. (c) 7. (a) 8. (c) The number of 50 paise coins = ´ 6 = 186
5+6+ 4
9. (d) 'Wily' means 'cunning' which also means skillful or
clever. 465
The number of 25 paise coins = ´ 4 = 124
10. (c) 'Audacity' best expresses the meaning of 'temerity' 5+ 6+ 4
which also means 'arrogance'
Total Distance Covered
11. (c) Fabricate means make up something artificial or untrue 27. (b) Average Speed =
Total Time Taken
while demolish means destroy completely which is just
opposite. 6+6+6+6 24
12. (c) Gregarious denotes tending to form a group with others = Þ
6 6 6 6 é1 1 1 1 ù
+ + + 6ê + + +
ë 25 50 75 150 úû
of the same species and unsociable is the opposite. 25 50 75 150
13. (d) Pragmatic means concerned with practical matters while
idealislic means bounded with limits. 24 ´ 150
= Þ 50 km/hr
14. (b) ‘Blues’ refer to bad moods, melancholy, low spirit, ill 6 ´ 12
temper.
28. (d) Let the marked price be ` x.
15. (b) Seeing eye to eye means approving of each other or
agreeing with each other. 3
Q C.P. = (x – 25% of x) = x
16. (a) This means to talk about one’s profession in a dry, 4
boring way. æ 3x 3x ö 33
Þ S.P. = ç + 10% of ÷= x
17. (b) Bad - blood reefers to the element of animosity in a è 4 4 ø 40
person and, thus, ill - feeling. 33
18. (c) This means to express openly one’s desires or But, x = 660 Þ x = 800.
40
emotions.
19. (b) 20. (d) 21. (c) 22. (b)
23. (a) The underlined portion should be deleted. 29. (c) Fr L
24. (d) and indulging in other sports.
25. (b) I was told
AÈB =A + B- AÇB
EBD_8177
PS-8 Practice Set - 1

Þ 100 = 75.8 + 49.4 – A Ç B 35. (b) Let initial dimensions be, l & b \ Final length is 1.4 l
Þ A Ç B = 125.2 – 100 = 25.2 Final breadth is 0.8 b
Students who took both 25.2% of 250 = 63 \ Final area is = 1.4 l × 0.8 b
30. (c) Let A takes ‘n’ days to paint the room. = 1.12 lb = lb + 12% of lb
\ Area is increased by 12%.
So B will take 3n days to paint the room.
Þ 3n – n = 60 40 ´ ( -20 )
or n = 30 Alternate Method : + 40 – 20 +
100
Þ A = 30, B = 90 = 20 – 8 = 12%
1 1 4 2 Therefore, the area of the new garden increased by
A & B will do + = = work in a day 12%
30 90 90 45
36. (a) Total votes = 6000
45 1 Invalid votes = 25% of 6000
So they will complete the work in = 22 days \ Valid votes = 75% of 6000 = 4500
2 2
31. (d) Let the mother’s age be y years. Bhiku gets = 65% of 4500 = 2925 votes
\ Mhatre gets = 4500 – 2925 = 1575 votes.
\ The age of father = (y + 9) years
37. (b) S.I. = 5700 – 5000 = ` 700
y
The age of son = years S.I. ´ 100 700 ´ 100
2 \R= = = 7% per annum
Pr incipal ´ Time 5000 ´ 2
æy ö New rate = 11%
The age of daughter = ç - 7 ÷ years
è2 ø 5000 ´ 11 ´ 2
Now according to the given condition, \ S.I. = = ` 1100
100
æy ö \ Amount = ` (5000 + 1100) = ` 6100
(y + 9) = 3 ç - 7 ÷ 38. (a) Suppose her income is 5. So his expenditure after 12%
è2 ø
increase becomes 3.36 and income after 10% increase
3 y - 42 becomes 5.5. Net increase in saving is 0.14. So
Þ y+9 =
2 percentage increase in savings is 7 per cent.
Þ 2y + 18 = 3y – 42 39. (d) Required difference
Þ y = 60 years é 2 ù 5000 ´ 12 ´ 2
32. (c) Let monthly income of Komal and Asha be 4x and 3x æ 12 ö
= ê5000ç1 + ÷ - 5000ú -
Also, let Monthly expenses of Komal and Asha be 3y êë è 100 ø úû 100
and 2y.
Now, 4x – 3y = 600 ......(i) æ 28 28 ö
= 5000ç ´ - 1÷ - 1200
3x – 2y = 600 .....(ii) è 25 25 ø
Solving (i) and (ii), x = 600 and y = 600
\ Total monthly income = (4 + 3)(600) = ` 4200 æ 784 - 625 ö
= 5000 ç - 1200 = ` 72
33. (b) Let the worker takes t min originally and distance of è 625 ÷ø
the factory from his house be x. 40. (a) Part of the capacity of the cistern emptied by the leak
(t + 3) 5 ( t - 7 ) 6 æ1 1ö 1
Now, x= = in one hour = ç - ÷ = of the cistern.
60 60 è 6 7 ø 42
( t + 3) 5 (t - 7 ) 6 The whole cistern will be emptied in 42 hours.
=
60 60 41. (b) Distance between the 1st and 21st posts
5t + 15 = 6t – 42 = (21 – 1) × 50 = 1000 m
t = 57 Therefore, the speed of train = 1 km/min = 60 km/h
( 57 + 3) 5 42. (d) Given, ratio of speeds of A and B is 5 : 4.
\ x = = 5 km. \ B makes 4 rounds when A makes 5 rounds.
60
34. (c) Relative speed of faster train with respect to the slower Now, distance covered by A in 5 rounds
( 65 - 29 ) ´ 1000 æ 400 ö
train =
3600
= 10 m/s = ç 5× ÷ = 2km
è 1000 ø
\ Length of the faster train = Relative speed × time and distance covered by B in 4 rounds
taken
= 10 × 16 = 160 m æ 400 ö
= ç 4× km = 1.6 km
è 1000 ÷ø
Practice Set - 1 PS-9

It is clear that in 5 hours, A passes Some cricket players may be football players and vice-
B only once. (i.e., 1 time). versa.
In other words, in covering 2 km, A pases B 1 time.
æ1 ö
\ In covering 5 km, A passes B in ç ´ 5÷ times
è2 ø Athletes Football players

1
i.e., 2times.
2
Cricket players
43. (b) Let us check each of the options here starting with (a)
13 11 133
(a) + = <5
5 6 30 51. (d) Orange is the only citrus fruit in the group.
11 8 65 52. (d) All except Locust are reptiles, while locust is an insect.
(b) + = >5 53. (b) All except Groundnut are spices.
4 3 12
44. (b) Correct asecending order is 54. (d) All except Vapour are different forms of precipitation.
55. (d) Mercury is the only liquid metal in the group.
2 9 5 8
< < < 56. (c) As Microphone makes sound louder similarly
3 13 7 11
Microscope makes the object magnified.
1 1 9 57. (c) As magnet has poles similarly battery has terminals.
45. (d) Part of the tank filled in 2 hours = + =
4 5 20 58. (a) Priest wears cassock while Graduate wears gown.
Part of the tank emptied in an hour when all three are 59. (c) As Ornithologist is a specialist of Birds similarly
1 1 1 1 Archealogist is a specialist of Archealogy.
opened = + - = -
4 5 2 20 60. (d) As Cloth is made in a Mill, similarly Newspaper is
14 printed in press.
First two hours fill the tank = 61. (d) Sentiment is a feeling especially based on emotions.
20
62. (c) Disagreement always creates controversy.
14
\ Time taken = ´ 20 = 14 hours 63. (c) A news paper always consists news.
20
64. (a) Early in the morning the sun is in the direction of east.
It means 5 p.m.
If we are going towards the south, our face will be in
46. (c) Liquids the direction of South and our left hand will be in the
direction of east.
Hence if we go early in the morning towards the south
Milk the sun will always be visible at our left.
River water 65. (b) Legitimate is related to law or to rules.
66. (b) By visualizing the figure, we get option (b) is correct.
Milk 67. (a) By visualizing the figure, we get option (a) is correct.
47. (c) Cow Goat Hen 68. (d) By visualizing the figure, we get option (d) is correct.
69. (b) Each time line rotates in + 45°, + 90°, + 135°, + 180°, +
225° clockwise direction. Hence, option (b) is correct.
70. (d) By visualizing the figure, we get option (d) is correct.
Tree 71. (a) By visualizing the figure, we get option (a) is correct.
72. (d)
48. (c) Plant House
73. (d) By visualizing the figure, we get option (d) is correct.
74. (a) In each figure semicircle is in inside the square and
placed diagonally opposite to each other. Hence option
49. (c) Capsules are different from injection but both are uses (a) is not fit to the pattern.
as antibiotics. 75. (c) By visualizing the figure, we get option (c) is correct.
50. (b) Some athletes may be football players and vice-versa. 76. (a) Going by mirror image, we get option (a) is correct.
Some athletes may be cricket players and vice-versa. 77. (a) 78. (d) 79. (a)
Some athletes may be both football players and cricket
players.
EBD_8177
PS-10 Practice Set - 1

Eastern Ghats region, however, the river flows between


80. (d) steep and precipitous banks, its width contracting
until it flows through a deep cleft only 600 feet (180
metres) wide, known as the Gorge.
87. (a) Minorities Rights Day is observed in India in 18th
December. Minorities Rights Day is celebrated on 18th
December every year. The day is celebrated to protect
rights of the minorities communities as well as bringing
the better understanding among religious minorities
in India.
81. (b) Under Article 32 of the Constitution, an individual 88. (b) "Sattriya Nritya" is a classical dance form of India
can directly move to the Supreme Court in Case of and has originated in Assam. Sattriya or Sattriya
any violation of fundamental rights. Fundamental Nritya , is one among the eight principal classical
Rights are those rights which are essential for the Indian dance traditions. In the year 2000, the Sattriya
growth of an individual's personality and are enjoyed dances of Assam received recognition as one of the
by every citizen irrespective of caste, color, creed, race eight classical dance forms of India.
and sex. 89. (c) 90. (b) 91. (b)
82. (d) The Dandi March of Gandhi was an important part of
92. (d) Jean Tirole has won Nobel Prize for his analysis of
the Indian Independence Movement.It was a direct
action campaign of tax resistance and non-violent market power and regulation.
protest against British saltmonopoly and triggered the 93. (c) The ozone layer is a layer in Earth's atmosphere that
wider Civil Disobedience Movement. absorbs most of the Sun's UV radiation. It contains
83. (d) Aihole inscription is found at Aihole in Karnataka state relatively high concentrations of ozone (O3), although
India, was written by the Ravikriti,court poet of it is still very small with regard to ordinary oxygen,
Chalukya king,Pulakeshin II who reigned from 610 to and is less than ten parts per million, the average ozone
642 CE.The poetic verses of Ravikirti,in praise of the
concentration in Earth's atmosphere being only about
king, can be read in the Meguti temple,dated 634CE.
0.6 parts per million. The ozone layer is mainly found
84. (c) The filtration unit of kidney is known as nephron.
Kidneys filter the nitrogenous waste products of the in the lower portion of the stratosphere from
body through nephron and throw them out in the form approximately 20 to 30 kilometres (12 to 19 miles) above
of urine. Kidneys and skin are the chief organs of Earth, though the thickness varies seasonally and
excretion. geographically.
85. (b) Yeast is an important source of vitamin B. Yeasts is 94. (b) Bilal Tanweer has been awarded with Shakti Bhatt First
eukaryotic microorganisms classified in the kingdom Book Prize- 2014 for his novel "The scatter here is too
Fungi, with 1,500 species (estimated to be 1% of all
great".
fungal species). Yeasts are unicellular, although some
95. (a) 96. (c)
species with yeast forms may become multicellular
97. (a) The Blue Revolution is related with fish production.
through the formation of strings of connected budding
cells known as pseudohyphae, or false hyphae, as 98. (c) Plimsol line is not a line of demarcation between two
seen in most molds. countries.
86. (a) Godavari is the longest river of peninsular India. From 99. (c) The first major attempt in curriculum reconstruction in
its source to the Eastern Ghats, the Godavari River India was made in 1937 when Gandhiji propounded
flows through gentle, somewhat monotonous terrain, the idea of Basic Education.
along the way receiving the Darna, Purna, Manjra, 100. (c)
Pranhita, and Indravati rivers. Upon entering the
Practice Set - 2 PS-11

2 Practice Set
INSTRUCTIONS
1. This practice set comprises four sections. Section A : Verbal Ability in English; Section B : Numerical Ability; Section C :
Reasoning and Military Aptitude; Section D : General Awareness.
2. The set will consist of 100 questions and each questions will be of 3 marks.
3. Each questions have four options, of which one is correct. The students are advised to read all the options thoroughly.
4. There is one-third negative marking in the set.

Time: 2 hrs. Max. Marks: 300


Ti

SECTION-A : VERBAL ABILITY IN ENGLISH DIRECTIONS (Qs. 9 - 11) : Choose the word which is nearly
opposite in meaning to the given word :
DIRECTIONS (Qs. 1 - 5) : Select the most appropriate word
from the options against each number : 9. Knack :
(a) Talent (b) Dullness
Auctions are public (1) of goods, conducted by (c) Dexterity (d) Balance
an (2) auctioneer. He encourages buyers to (3) higher 10. Pernicious :
(a) Prolonged (b) Ruinous
prices and finally names the (4) bidder as the buyer of the (c) Ruthless (d) Beneficial
goods. This is called ‘knocking down’ the goods, for when the 11. Opulence :
bidding ends the auctioneer (5) a small hammer on a table in (a) Luxury (b) Transparency
(c) Weath (d) Poverty
front of him.
1. (a) sale (b) marketing DIRECTIONS (Qs. 12 - 16) : Read the following passage carefully
(c) promotion (d) viewing and answer the questions given below it :
Freedom has assuredly given us a new status and new
2. (a) authoritative (b) allowed
opportunities. But it also implies that we should discard selfishness,
(c) authentic (d) approved laziness and all narrowness of outlook. Our freadom suggests toil
3. (a) bid (b) buy and creation of new values for old ones. We should so discipline
(c) get (d) bargain ourselves as to be able to discharge our new responsibilities
4. (a) smartest (b) highest satisfactorily. If there is any one thing that needs to be stressed
(c) biggest (d) strongest more, than any other in the new set-up, it is that we should put into
5. (a) bangs (b) thrashes action our full, capacity, each one of us in productive effort - each
one of us in his own sphere, however humble. Work, unceasing
(c) smashes (d) hits
work, should now be our watch-word. Work is wealth, and service
DIRECTIONS (Qs. 6 - 8) : Choose the word which is nearest in is happiness. Nothing else is. The greatest crime in India today is
meaning to the given word : idleness. If we root out idleness, all our difficulties, including even
6. Apprise : conflicts, will gradually disappear.
(a) Praise (b) Inform 12. Anyone can free himself from the clutches of difficulties, if he
(c) Conceal (d) Assess (a) eliminates narrow outlook
7. Periodic : (b) fulfils his responsibilities
(a) Infrequent (b) Continuous (c) discards idleness
(c) Occasional (d) Regular (d) discharges his obligations
8. Gruesome : 13. What has freedom undeniably offered to the citizens of India ?
(a) Sullen (b) Hideous (a) New opportunities (b) New outlook
(c) Exhausting (d) Insulting (c) New responsibilities (d) New values
EBD_8177
PS-12 Practice Set - 2

14. One thing needs to be stressed more than anything else in 28. A man sold an article at a loss of 20%. If he sells the article
this new set-up. It is that people should for ` 12 more, he would have gained 10%. The cost price of
(a) discard narrowness of outlook
the article is
(b) discipline themselves suitably
(c) work to their full capacity (a) ` 60 (b) ` 40 (c) ` 30 (d) ` 22
(d) substitute old values with new ones 29. What is the least fraction which, when added to or subtracted
15. Work should be the motto of our citizens. 29 15
(a) Resourceful (b) Incessant from + , will make the result a whole number ?
12 16
(c) Productive (d) Ingenious
16. Nothing else can give us joy except 31 31 17
21
(a) service (b) idleness (a) (b) (c) (d)
38 38 48 48
(c) wealth (d) freedom
30. Among the goods purchased by a trader, two-fifth were
DIRECTIONS (Qs. 17 - 21) : In the following questions, four
sold at 20% loss, and the reamaining were sold at a profit. If
alternatives are given for the Idiom/Phrase underlined. Choose on the whole 10% profit is made, what per cent of profit did
the alternative which best expresses the meaning of the Idiom/ the trader make on the remaining goods ?
Phrase and mark it in the Answer Sheet. (a) 15% (b) 20% (c) 30% (d) 40%
17. To be above board. 31. Two equal sums of money were invested, one at 4% and the
(a) To have a good height other at 4.5%. At the end of 7 years, the simple interest
(b) To be honest in any business deal received from the latter exceeded to that received from the
(c) Having no debts
former by ` 31.50. Each sum was :
(d) To try to be beautiful
18. To cry wolf. (a) `1,200 (b) ` 600 (c) ` 750 (d) ` 900
(a) To listen eagerly (b) To give false alarm 32. Three brothers A, B and C divided a sum among them selves
(c) To turn pale (d) To keep off starvation and their mother in the ratio 3 : 3 : 3 : 4. If their mother got
19. He is on the wrong side of seventy. ` 5,000 more than each son, how much amount did all the
(a) more than seventy years old three brothers get ?
(b) less than seventy years old (a) ` 15,000 (b) ` 65,000 (c) ` 30,000 (d) ` 45,000
(c) seventy years old 33. A train 100 metres long meets a man going in opposite
(d) eighty years old
20. To have an axe to grind. 1
Directions at 5 km/hr and passes him in 7 seconds. What
(a) a private end to serve (b) to fail to arouse interest 5
(c) to have no result (d) to work for both sides is the speed of the train in km/hr ?
21. To drive home. (a) 45 km/hr (b) 60 km/hr (c) 55 km/hr (d) 50 km/hr
(a) To find one’s root (b) To return to place of rest
34. If x + y > 5 and x – y > 3, then which of the following gives
(c) Back to original position
all possible values of x ?
(d) To emphasise
(a) x > 3 (b) x > 4 (c) x > 5 (d) x < 5
DIRECTIONS (Qs. 22 - 25) : In the following questions four 35. After having spent 35% of the money on machinery, 40%
words are given in each question, out of which only one word is on raw material and 10% on staff, a person is left with
correctly spelt. Find the correctly spelt word and indicate it in ` 60,000. The total amount of money spent on machinery
the Answer-Sheet by blackening the appropriate rectangle [ ]. and raw material is :
22. (a) garulous (b) garrulous (a) `1,76,000 (b) ` 1,70,000
(c) garullous (d) garrullous (c) ` 3,00,000 (d) ` 3,40,000
23. (a) marquee (b) markue 36. An aeroplane travels distances 2500 km, 1200km and 500km
(c) marquei (d) marquie at the rate of 500 km/hr, 400 km/hr, and 250 km/hr, respectively.
24. (a) puissant (b) puiscant The average speed is
(c) puiscent (d) puissent (a) 420 km/hr (b) 405 km/hr
25. (a) disconncerting (b) disconserting (c) 410 km/hr (d) 575 km/hr
(c) discuncerting (d) disconcerting 37. The population of a village is 10,000. If the population
increases by 10% in the first year, by 20% in the second
SECTION-B : NUMERICAL ABILITY year and due to mass exodus, it decreases by 5 % in the
third year, what will be its population after 3 years ?
26. Divide 50 into two parts so that the sum of their reciprocals (a) 13,860 (b) 11,540 (c) 12,860 (d) 12,540
is 1/12. 38. A plane left 30 min later than its scheduled time to reach its
(a) 28, 22 (b) 35, 15 (c) 20, 30 (d) 24, 36 destination 1500 km away. In order to reach in time it increa
27. Rakesh got 273 marks in an examination and scored 5% ses its speed by 250 km/h. What is its original speed ?
more than the pass %. If Lokesh got 312 marks, then by (a) 1000 km/h (b) 750 km/h
what % above the pass mark did he pass the examination? (c) 600 km/h (d) 800 km/h
(a) 20% (b) 27% (c) 25% (d) 15%
Practice Set - 2 PS-13

39. X and Y can do a piece of work in 30 days. They work 53. (a) Tonnes (b) Quintals
together for 6 days and then X quits and Y finishes the work (c) Kilometres (d) Kilograms
in 32 more days. In how many days can Y do the piece of 54. (a) Tomato (b) Carrot
work alone ? (c) Ginger (d) Potato
(a) 30 days (b) 32 days (c) 34 days (d) 40 days
55. (a) Verse (b) Rhyme
40. The ratio of two numbers is 3 : 4. If 5 is added to both the
(c) Couplet (d) Rhetoric
numbers, the ratio becomes 4 : 5. The product of the numbers
added to the sum of the squares of the numbers is DIRECTIONS (Qs. 56-60 ) : In each of the following questions
(a) 950 (b) 925 (c) 700 (d) 725 find out the alternative which will replace the question mark.
41. A cistern has a leak which would empty it in 8 hours. A tap 56. Ice : Coldness :: Earth : ?
is turned on which admits 6 litres a minute into the cistern (a) Weight (b) Jungle
and it is now emptied in 12 hours. The cistern can hold (c) Gravitation (d) Sea
(a) 7860 litres (b) 6840 litres 57. Physician : Treatment :: Judge : ?
(c) 8640 litres (d) None of these (a) Punishment (b) Judgement
42. If a% of x is equal to b% of y, then c% of y is what % of x ?
(c) Lawyer (d) Court
ac bc 58. Flow : River :: Stagnant : ?
(a) c% (b) % (c) % (d) abc%
b a (a) Rain (b) Stream (c) Pool (d) Canal
43. In a class, the average score of girls in an examination is 59. Country : President :: State : ?
73 and that of boys is 71. The average score of the whole (a) Governor (b) M.P
class is 71.8. The percentage of the girls in the class is : (c) Legislator (d) Minister
(a) 60% (b) 40% 60. Melt : Liquid :: Freeze : ?
(c) 1.8% (d) 18%
(a) Ice (b) Condense
44. Two numbers are such as that square of one is 224 less
(c) Solid (d) Force
than 8 times the square of the other. If the numbers are in
the ratio of 3 : 4, they are: DIRECTIONS (Qs. 61-63) : Consider the statement and
(a) 12, 16 (b) 6, 8 (c) 9, 12 (d) None of these assumptions that follow. Which of these assumption is/are implicit
in the statement
45. A train 300 m long is running at a speed of 90 km/hr. How
many seconds will it take to cross a 200 m long train running 61. Statement: The situation of this area still continues to be
tense and out of control. People are requested to be in their
in the opposite direction at a speed of 60 km/hr ?
homes only.
1 Assumptions:
(a) 7 sec (b) 60 sec (c) 12 sec (d) 20 sec
5 I. There had been some serious incidents.
II. People will not go to the office.
SECTION-C : REASONING III. Normalcy will be restored shortly.
& MILITARY APTITUDE (a) Only I is implicit
(b) Only I and II are implicit
DIRECTIONS (Qs. 46 - 47) : Complete the series.
(c) Only I and III are implicit
46. YVP, WTN, URL, ? (d) All are implicit
(a) SPJ (b) TQLS (c) VSP (d) SRJ 62. Statement: To improve the employment situation in India,
47. BD, FH, JL, NP, ? there is a need to recast the present educational system
(a) PQ (b) RS (c) SU (d) RT towards implementation of scientific discoveries in daily life.
48. If A = 1, ACE = 9, then ART = ? Assumptions:
(a) 29 (b) 38 (c) 10 (d) 39 I. The students after completing such education may be
49. If A = 26, SUN = 27, then CAT = _________ able to earn their livelihood.
(a) 27 (b) 72 (c) 57 (d) 58 II. This may bring meaning of education in the minds of
50. If O = 16, FOR = 42, then what is FRONT equal to? the youth.
(a) 61 (b) 65 (c) 73 (d) 78 III. The state may earn more revenue as more and more
people will engage themselves in self-employment.
DIRECTIONS (Qs. 51-55 ): Choose the word which is least like
(a) Only I and II are implicit
the other words in the group.
(b) Only III is implicit
51. (a) Skull (b) Appendix
(c) Only I and III are implicit
(c) Pelvis (d) Fibula
(d) None of these
52. (a) Island (b) Coast
63. Statement: Pramod decided to get the railway reservation
(c) Harbour (d) Oasis in May, for the journey he wants to make in July, to Madras.
EBD_8177
PS-14 Practice Set - 2

Assumptions: 68.
I. The railways issues reservations two months in advance.
II. There are more than one trains to Madras.
III. There will be vacancy in the desired class.
(a) Only I is implicit
(b) Only II and III are implicit
X (a) (b) (c) (d)
(c) Only I and III are implicit
(d) All are implicit 69.
DIRECTIONS (Qs. 64 & 65) : Each of the question below has
a set of 4 statements. Each statement has 3 segments. Choose
the alternative where the third segment can be logically
deduced, using both the preceding two, but not just from one
?
of them. X (a) (b) (c) (d)
64. (A) To forgive is divine. Divine facts are rare. Forgiveness
DIRECTIONS (Qs. 70 - 73) : In each of the questions, a part
is rare.
of the figure is given. Select one from the given four figures
(B) G is the brother of A and the father of L. B is the
in which that part is embedded.
wife of A. L is the daughter of G.
(C) Pepsi contains added flavour. Coke contains 70.
permitted colours. Pepsi and Coke are cold drinks.
(D) Some beer is wine and some beer is vodka. All wine
is vodka. All beer must be vodka or wine. (a) (b) (c) (d)
(a) Only A (b) A & B
(c) C & A (d) Only D 71.
65. (A) All Jadoo is Magic. Some magic is witchcraft. Some
Jadoo is witchcraft. (a) (b) (c) (d)
(B) Floor C is two storeys above floor D. Floor E & A
are adjacent and above the floor C. In the Storey
complex, Floor D is the ground Floor. 72.
(C) In the Kingdom of Lemon Grass, Tom Yarn is 400 km (1) (2) (3) (4)
(a) (b) (c) (d)
from Jom Yarn. Tom Kha is 300 km from Jom Yorn,
and away from Tom Yarn. Tom Kha and Tom Yarn
are 700 km apart. 73.
(D) Ford 1 km is a josh machine. Hyundai Santro is the
complete family car. There is little josh in Santro. (a) (b) (c) (d)
(a) A & C (b) Only B DIRECTIONS (Qs. 74 - 77) : In these tests find the odd figure
(c) A & D (d) B & C out.
DIRECTIONS (Qs. 66 - 69) : In each of the following questions,
figure X is given with a part missing.
Choose the alternative which will complete the missing part. 74.
66.
(a) (b) (c) (d)

? 75.

X (a) (b) (c) (d)


67. (a) (b) (c) (d)

76.
?
(1)
(a) (2)
(b) (c)(3) (d) (4)
X (a) (b) (c) (d)
Practice Set - 2 PS-15

85. Which General, who commanded the British forces against


the Americans in their War of Independence later became
77. Governor-General of India ?
(a) Dalhousie (b) William Bentinck
(c) Wellesley (d) Cornwallis
(a) (2)
(b) (c)(3) (d)(4) 86. The Constitutional Amendment Act that has introduced
safeguards against the misuse of proclamation of national
DIRECTIONS (Qs. 78 - 80) : In each of the questions, which one emergency is the
of the four numbered figures from the right side row will replace (a) 42nd Amendment Act (b) 43rd Amendment Act
the question mark in the left side row so as to maintain the (c) 44th Amendment Act (d) 45th Amendment Act
sequence ? 87. The Fundamental Rights can be suspended by the
(a) Governor (b) President
78. (c) Law Minister (d) Prime Minister
88. Prithvi Raj Chauhan was defeated in the Second Battle of
Tarain by
(a) Mahmud Ghazni (b) Muhammad Ghori
(c) Qutbuddin Aibak (d) Yalduz
89. The city of Prayag was named Allahabad - the city of God by
(a) Aurangzeb (b) Akbar
(c) Shahjahan (d) Bahadur Shah Zafar
(a) (b) (c) (d) 90. Which one of the following wars decided the fate of the
79. French in India ?
(a) Battle of Plassey (b) Battle of Wandiwash
(c) First Carnatic War (d) Battle of Buxar
91. Which one of the following is a warm ocean current ?
(a) Gulf Stream (b) Kurile
(c) Canary (d) Labrador
92. The main advantage of Rain Water Harvesting (RWH) is
(a) Avoid soil erosion
(b) Recharge ground water
(1)
(a) (2)
(b) (3)
(c) (4)
(d) (c) Avoid floods
80. (d) Reduce the loss of water
93. Who was the first posthumous recipient of Bharat Ratna?
(a) M.G. Ramachandran (b) B.R. Ambedkar
(c) K. Kamraj (d) Lal Bahadur Shastri
94. Which day is obser ved as “International Day of
Non-Violence”
(a) 1 st May (b) 2 nd October
(c) 24 th October (d) 30 th January
95. Lakshmibai National Institute of Physical Education (LNIPE)
(1)
(a) (b) (c) (d) is in:
(a) Patiala (b) Gwalior (c) Indore (d) Jhansi
SECTION-D : GENERAL AWARENESS 96. Amuktamalyada is the work of :
81. In context to India's defence structure ‘Agni missile’ is (a) Krishnadeva Raya (b) Vachcharaj
_____. (c) Kharavela (d) Allasani Peddana
(a) Surface-to-air (b) Air-to-air 97. Yakshagana is a folk dance-drama of:
(c) Air-to-surface (d) Surface-to-surface (a) Maharashtra (b) Karnataka
82. The programme of ‘Operation Flood’ was concentrated on (c) Gujarat (d) W. Bengal
(a) increasing irrigation facilities. 98. The ‘Panch Prayag’ which connotes the five sacred river
(b) flood control. confluences is in the state of ___________.
(c) increasing the milk production. (a) Himachal Pradesh (b) Madhya Pradesh
(d) increase the flood grains production. (c) Uttarakhand (d) Andhra Pradesh
83. Article 324 of the Indian Constitution deals with the 99. The ‘Yashsvini Health Insurance’ scheme is associated with
(a) imposition of President’s Rule in States. which of the following states?
(b) appointment of Finance Commission. (a) Punjab (b) Karnataka
(c) power s and functions of the Chief Election (c) Tamil Nadu (d) Andhra Pradesh
Commissioner. 100. Which of the following pairs is incorrect ?
(d) functions of the Union Public Service Commission. (a) Amirkhusro – Sarod
84. The founder of the Lodi Dynasty was (b) Bhim Sen Joshi – Vocal music
(a) Bahlul Lodi (b) Sikandar Shah Lodi (c) Utpal Dutt – Films
(c) Jalal Khan Lodi (d) Ibrahim Lodi (d) Shambhu Maharaj – Kathak
EBD_8177
PS-16 Practice Set - 2

ANSWERS & SOLUTIONS


1. (a) 2. (d) 3. (a) 4. (b) 5. (a) 6. (b) 30. (c) Let number of goods = 100
7. (d) 8. (b) 9. (b) 10. (d) 11. (d) 12. (c) C.P of each article = ` 1
13. (a) 14. (c) 15. (b) 16. (a) 2 æ 80 ö
17. (b) If somebody is above board, he/she is honest in any \ S.P. of ´ 100 = 40 article = ` ç 40 ´ ÷ = ` 32.
business deal. 5 è 100 ø
18. (b) To cry wolf means that someone is giving false alarm. Total S.P. = ` 110.
19. (a) If somebody is on the right/ wrong side of 30/ 40 etc \ S.P. of remaining 60 articles = ` (110 – 32) = ` 78
that means he/she is younger/ older than 30/ 40 etc. gain = (78 – 60) = `18
20. (a) If you have an axe to grind; that means you have a 18
private end to serve. \ Required percentage gain = ´ 100 = 30%
21. (d) If you drive something home, that means you are 60
making something completely clear to someone. She 31. (d) Let each sum be ` x. Then
didn't have to drive the point home. The movie had Difference of S.I. = ` 31.50
done that. 1
x´4 ´7 x´4´7
22. (b) Garrulous 2 - = 31.50
23. (a) Marquee 100 100
24. (a) Puissant 7 x 1 63
or ´ = or x = ` 900
25. (d) Disconcerting 100 2 2
1 1 1 32. (d) Let the total amount be x.
26 . (c) + =
x 50 - x 12 Sum of the ratios = 3 + 3 + 3 + 4 = 13
x2 – 50x + 600 = 0
x2 – 30x – 20x + 600 = 0 æ 4 3ö
\ ç – ÷ ´ x = 5000
x(x – 30) – 20(x – 30) = 0 è 13 13 ø
x = 30, 20 \ x = ` 65000
27. (a) Let passing marks be represented by p. \ Amount of three brothers
p × 1.05 = 273 9
p = 260 = ´ 65000 = ` 45000
13
312 – 260 33. (a) Let speed of train = x km/hr
Lokesh passing % = ´ 100 = 20%
260 Distance travelled by train
= Relative speed of train × Time
æ 80 ö 4
28. (b) S.P = C.P ç ÷ Þ S.P = C.P ...(1) 36
è 100 ø 5 100 m = (x + 5) km/hr × seconds
5
æ 110 ö 11
S.P + 12 = C.P ç ÷ Þ S.P = 10 C.P - 12 ...(2) 100 æ 36 1 ö
è 100 ø km = (x + 5) × ç ´ ÷ hrs
1000 è 5 3600 ø
From eqs. (1) and (2)
Þ x + 5 = 50
4 11 \ x = 45 km/hr
C.P = C.P - 12
5 10 34. (b) x+y>5 ... (i)
11 4
Þ C.P - C.P = 12 Þ C.P = `40
x-y>3 ... (ii)
10 5 Adding inequations (i) and (ii), we get
29 15 116 + 45 161 2x > 8 i.e. x > 4
29. (d) + = = 35. (c) Let the original money be ` 100.
12 16 48 48 Money spent on machinery = ` 35
161 31 192 Money spent on raw material = ` 40
Therefore, + = = 4 = a whole number Money spent on staff = ` 10
48 48 48
Money left = 100 – 85 = ` 15
161 17 If money left is ` 15, the original money = 100
And - = 3 = whole number
48 48 So, if money left is 60,000, the original money
31 17 17 100
Fraction and ; is the least fraction. = ´ 60, 000 = ` 400000
48 48 48 15
Clearly, the least fraction among the given fractions in Money spent on machinery and raw material
17 = 400000 ´ 75% = ` 300000
options is . 36. (a) Given, distan ces are 2500 km, 1200 km and
48 500 km.
Practice Set - 2 PS-17

Given, speeds are 500 km/h, 400 km/h and 250 km/h ax by y a
2500 1200 500 42. (b) = Þ =
\ Total time = + + 100 100 x b
500 400 250 cy ? x
= 5 + 3 + 2 = 10 hr. =
100 100
Total distance
\ Average speed = cy ca
Total time Þ = %
x b
2500 + 1200 + 500 4200 43. (b) Let the number of boys and girls be x and y,
= = respectively.
10 10
= 420 km/hr Then, the total score of boys = 71x
and the total score of girls = 73y
110 Now, average score = 71. 8
37. (d) Population after 1st year = ´ 10, 000 = 11000
100 71x + 73y
\ = 71.8
120 x+y
Population after 2nd year = 11000 ´ = 13200
100 Þ 71x + 73y = 71.8x + 71.8y Þ 0.8x = 1.2y
95 x 1.2 3
Population after 3rd year = 13200 ´ = 12,540 Þ = =
100 y .8 2
Hence, population after 3rd year = 12, 540.
38. (b) Let the original time be T hours and original speed be \ % of girls in the class = 2 ´ 100 = 40%
x km/h 5
44. (b) Given, ratio of numbers is 3 : 4
1500 \ The numbers are 3x and 4x.
=T ...(i)
x Now, according to the question
1500
=T-
30
...(ii) 16x 2 = 8(3x)2 - 224
x + 250 60
Solving equation (i) and (ii), we get Þ 16 x 2 = 72 x 2 - 224 Þ 56x 2 = 224
Speed of plane x = 750 or – 1000 (not possible) x= 2
\ x = 750 km/h \ Required numbers = 6, 8
45. (c) Relative speed = 90 + 60 = 150 km/hr.
æ 1 ö 1 Total distance to be covered = 300 + 200 = 500 m
39. (d) (x + y) ‘s 6 days’ work = ç ´ 6 ÷ = .
è 30 ø 5 500
Time required = ´ 3600 = 12 sec.
æ 1ö 4 150 ´1000
Remaining work = ç 1 - ÷ =
è 5ø 5 –2 –2 –2
4 YV P , W TN , U R L , S PJ
Now, work is done by y in 32 days. 46. (a)
5 –2 –2 –2
æ 5ö –2 –2 –2
Whole work will be done by y in ç 32 ´ ÷ = 40 days. +2 +2 +2 +2 +2
è 4ø
3x + 5 4 47. (d) B D; F H; J L; N P; R T
40. (b) According to the question =
4x + 5 5
15x + 25 = 16x + 20 +2 +2 +2 +2
Þ x = 25 – 20 = 5 48. (d) A = 1, A + C + E = 1 + 3 + 5 = 9
\ Numbers are 15 and 20 A + R + T = 1 + 18 + 20 = 39
\ Required answer = 15 × 20 + 152 + 202 = 925 49. (c) Reverse place value of A = 26
SUN = (8 + 6 + 13) = 27
1
41. (c) In 1 hour, empty part = th. \ CAT = (24 + 26 + 7) = 57
8 50. (d) O = 16, FOR = 42
When tap is turned on, then F=6 + 1 =7
1 R = 18 + 1 = 19
empty part in 1 hour = th . O = 15 + 1 = 16
12 N = 14 + 1 = 15
\ Part of cistern emptied, due to leakage in T = 20 + 1 = 21
1 1 3- 2 1 78
1 hour = - = = th
8 12 24 24 Then, FRONT = 78
Now, In 1 min, cistern fill = 6 lit 51. (b) All except Appendix are bones, while appendix is an
1 organ.
\ In hr, cistern fill = 6 lit. 52. (d) All except Oasis are features related to sea, while oasis
60
is related to desert.
\ Cistern can hold = 6 × 60 × 24 litre = 8640 litre.
EBD_8177
PS-18 Practice Set - 2

53. (c) All others are units for measuring weights. surpassing the USA in 1998, with about 17 percent of
54. (a) All except Tomato grow underground. global output in 2010-11, which in 30 years doubled
55. (d) All except Rhetoric are terms associated with poetry. the milk available per person, and which made dairy
56. (c) As effect of Ice is coldness similarly the effect of Earth farming India's largest self-sustainable rural
is gravitation. employment generator. All this was achieved not merely
57. (b) As Physician does the treatment similarly Judge by mass production, but by production by the masses.
delivers the judgement. 83. (c) power s and functions of the ch ief Election
58. (c) As Water of a River flows similarlywater of Pool is Stagnant. Commissioner
59. (a) As President is the nominal head of a country, similarly 84. (a) Bahlul Khan Lodi was the founder of Lodi dynasty of
Governor is the nominal head of a State.
the Delhi Sultanate in India upon the abdication of the
60. (c) As on melting, liquid is formed similarly on freezing
solid is formed. last claimant from the previous Sayyid rule.
61. (b) The statement mentions that situation in the area is 85. (b) Lieutenant-General Lord William Henry Cavendish-
tense. So, I is implicit. Since people have been Bentinck, GCB, GCH, PC, known as Lord William
requested not to go out and remain in homes for safety, Bentinck, was a British soldier and statesman. He
so II is implicit. It cannot be inferred when the normalcy served as Governor-General of India from 1828 to 1835.
will be restored. So, III is not implicit. 86. (c)
62. (a) The statement mentions that such education can 87. (b) The Fundamental Rights can be suspended during the
improve employment situation. So, both I and II are Emergency under Article 359 of the Constitution by
implicit. Nothing about the aspect of revenue collection the President of India.
is mentioned in the statement. So, III is not implicit. 88. (b) 1191 - First Battle of Tarain in which Prithviraj Chauhan
63. (a) Clearly, since Pramod decides to get the reservation in defeated Mohd. Ghori.1192 - Second Battle of Tarain
May for the journey in July, so I is implicit. The number in which Mohd.Ghori defeated Prithviraj Chauhan.
of trains to Madras or the position of vacancies in 89. (b) Emperor Akbar named Prayag as Allahabad - City of
different classes cannot be deduced from the given God- also called Allahabad in 1575 AD. The city of
statement. So, neither II nor III is implicit. Allahabad is situated at the confluence of three rivers
64. (a) Forgiveness Divine Rare - Ganga, Yamuna and the invisible Saraswati. Every
12th year when the waters are felt to be especially
We see that forgiveness is divine and divine facts purifying, Allahabad holds a much greater festival called
are rare. So, forgiveness is rare. Kumbh Mela. Built by Emperor Akbar in 1583 AD, the
65. (d) In (a), the middle term ‘magic’ is not distributed. Allahbad fort stands on the banks of the river Yamuna
Therefore no deduction of mediate inference can be near the confluence site i.e SANGAM.
established. For (b) first two Statements, we see that 90. (b) Battle of Wandiwash decided the fate of French in
the pattern of 5 storeys is as given.
India. Battle of Wandiwash, (Jan. 22, 1760), in the
E ® 4th (5th storey)
history of India, a confrontation between the French,
A ® 3rd (4th storey) under the comte de Lally, and the British, under Sir
C ® 2nd (3rd storey) Eyre Coote. It was the decisive battle in the Anglo-
– ® 1st (2nd storey) French struggle in southern India during the Seven
D ® Ground (1st storey) Years' War (1756-63).
91. (a) Gulf Stream is a warm ocean current. It flows along the
Clearly, D is the ground floor. North America and drifts towards western Europe, thus
As per the first two statements, the position of cities raising the temperature of western coast considerably.
is as follows: 92. (b) Recharging of ground water is the main advantage of
400 km 300 km rain water harvesting. Rainwater harvesting provides
an independent water supply during regional water
Tom Jom Tom restrictions and in developed countries is often used
Yarn Yorn Kha to supplement the main supply. It provides water when
Clearly, Tom Kha and Tom Yarn are 700 km apart. there is a drought, can help mitigate flooding of low-
66. (d) 67. (c) 68. (a) 69. (d)
lying areas, and reduces demand on wells which may
enable ground water levels to be sustained.
70. (d) This figure comes in 93. (d) Lal Bahadur Shastri was the first posthumous recipient
of Bharat Ratna in 1966. Lal Bahadur Shastri was the
71. (b) 72. (b) 73. (b) third Prime Minister of the Republic of India and a
74. (d) This figure not makes four equal or similar parts like
leader of the Indian National Congress party. Shastri
others.
75. (c) This figure use four spokes but others use only three. joined the Indian independence movement in the 1920s.
76. (a) ‘Oval’ is not a standard figure. 94. (b) 95. (b) 96. (a) 97. (b)
77. (b) Small figure partially covered by the big one. 98. (c) The Panch Prayag the five sacred river confluence is
78. (c) 79. (a) 80. (a) 81. (d) in the state of Uttarakhand. The five prayags are Vishnu
82. (c) Operation Flood in India, a project of the National Dairy Prayag, Nand Prayag, Karn Prayag, Rudra Prayag and
Development Board (NDDB) was the world's biggest Dev Prayag.
dairy development program which made India, a milk- 99. (b) Yashsvini is a health insurance scheme for the members
deficient nation, the largest milk producer in the world, of the cooperative bodies in Karnataka.
100. (a)
Practice Set - 3 PS-19

3 Practice Set
INSTRUCTIONS
1. This practice set comprises four sections. Section A : Verbal Ability in English; Section B : Numerical Ability; Section C :
Reasoning and Military Aptitude; Section D : General Awareness.
2. The set will consist of 100 questions and each questions will be of 3 marks.
3. Each questions have four options, of which one is correct. The students are advised to read all the options thoroughly.
4. There is one-third negative marking in the set.

Time: 2 hrs. Max. Marks: 300


Ti

SECTION-A : VERBAL ABILITY IN ENGLISH 4. Which of the following is meaning to word ‘dome’ as used
in passage?
DIRECTIONS (Qs. 1 - 4) : Read the following passage carefully (a) Floor (b) Arched part of ceiling
(c) Foot (d) None of these
and answer the questions given below it:
Over four hundred years after his death, scholars are still DIRECTIONS (Qs. 5 - 8) : Select the most appropriate word
travelling the mysteries of Michelangelo's art. Recently one from the options against each number :
mystery that was revealed was that his famous drawing of a Experienced climber Aron Ralston set out on a 5 hiking
pensive Cleopatra included a hidden drawing of a different adventure, which proved to be a 6 event. Despite it being a
Cleopatra on the reverse side. This hidden Cleopatra shows a common safety practice amongst climbers to inform others when
tormented woman, whose eyes stare out at the viewer and whose undertaking unaccompanied hiking trips, Aron had not
mouth is open, screaming in horror. The two images, drawn on 7 anyone of his plans. During his climbing adventure Aron’s
two sides of the same paper, can be viewed simultaneously. A right arm became pinned against the canyon wall by a 360 kg
boulder. Aron was unable to free himself, and after six days of
second mystery concerns Michelagelo's architectural plan for
being 8 he made the decision to break the bones in his forearm
the dome of St. Peter's Basilica in Rome. Did he intend for the and then amputate his arm below the elbow. Once free he made
dome to look like the model he built between 1558 and 1561 ? Or his way down a cliff and walked 8 km to seek assistance. Aron
did he change his mind after building the model and decide to survived.
elevate the dome in the way it is today? Scholars do not agree on 5. Which of these fits gap 7?
the answer. A third mystery about one of the greatest artists who (a) authorised (b) alerted
ever lived was why he destroyed hundreds or thousands of his (c) signalled (d) cautioned
drawings before he died. Did he feel they were unimportant? Did 6. Which of these fits gap 5?
he want posterity to see only his finished products? (a) supervised (b) solo
1. The dome of St. Peter's Basilica: (c) team (d) shared
(a) bears no relation to the one in the model 7. Which of these fits gap 8?
(b) was destroyed after the model was built (a) suppressed (b) captive
(c) is raised more than the one in the model (c) trapped (d) entangled
(d) follows the plan of the model 8. Which of these fits gap 6?
2. According to the passage, Michelangelo is : (a) beath-taking (b) fail-safe
(a) a private person (c) stimulating (d) life-changing
(b) one of the greatest artists in the world
(c) the most famous architect in Rome DIRECTIONS (Qs. 9 - 14) : Choose the word which can be
(d) screaming in horror substituted for the given words/sentence :
3. Why did Michelangelo destroy so many drawings before 9. That which has a double meaning
he died? (a) doubtless (b) uncertain
(a) Nobody knows (c) controversial (d) ambiguous
(b) They were unimportant 10. Incapable of making errors
(c) They were only drafts (a) infallible (b) incorrigible
(d) He had changed the drawings (c) impervious (d) inexplicable
EBD_8177
PS-20 Practice Set - 3

11. Governed by a sense of duty SECTION-B : NUMERICAL ABILITY


(a) conscious (b) sensible
(c) intelligent (d) conscientious 26. The prize money of ` 1,800 is divided among 3 students A,
B and C in such a way that 4 times the share of A is equal to
12. The depository where state records and documents are
6 times the share of B, which is equal to 3 times the share of
preserved
C. Then A’s share is
(a) museum (b) library
(a) ` 400 (b) ` 600
(c) emporium (d) archive
(c) ` 700 (d) ` 800
13. That which is no longer fashionable or in use
(a) unused (b) ancient 27. The average of 5 consecutive numbers is n. If the next two
numbers are also included, the average of the 7 numbers
(c) obsolete (d) old
will
14. Murder of a king
(a) increase by 2 (b) increase by 1
(a) homicide (b) fratricide
(c) remain the same (d) increase by 1.4
(c) regicide (d) parricide
28. The compound interest on a certain sum for two years is
DIRECTIONS (Qs. 15 - 20 ) : Which word or words explains the ` 618, whereas the simple interest on the same sum at the
meaning of the following idioms : same rate for two years is ` 600. The rate of interest per
15. To turn over a new leaf annum is
(a) To change completely one’s course of action
(a) 18% (b) 9%
(b) To shift attention to new problems
(c) To cover up one’s faults by wearing new marks (c) 6% (d) 3%
(d) To change the old habits and adopt new ones 29. If the numerator and the denominator of a proper fraction
16. To wrangle over an ass’s shadow are increased by the same quantity, then the resulting
(a) To act in a foolish way fraction is :
(b) To quarrel over trifles (a) always greater than the original fraction
(c) To waste time on petty things (b) always less than the original fraction
(d) To do something funny (c) always equal to the original fraction
17. All Agog (d) None of these
(a) Everybody (b) All ready 30. Out of 40 boys in a class, average weight of 30 is 60 kg and
(c) Restless (d) Almighty the average weight of the remaining is 56 kg. The average
18. To take with a grain of salt weight (in kilogram) of the whole class is
(a) To take with some reservation (a) 58.5 (b) 58
(b) To take with total disbelief (c) 57 (d) 59
(c) To take whole heartedly
1
(d) To take seriously 31. A CD was sold at a profit of 12 % . If it had been sold at a
19. Hobson’s choice 2
(a) Feeling of insecurity profit of 15%, it would have gained him ` 10 more. The cost
(b) Accept or leave the other prices of CD is (in `)
(c) Feeling of strength (a) 450 (b) 500
(d) Excellent choice (c) 400 (d) 550
20. To take through one’s hat 32. Mr. X’s salary is increased by 20%. On the increase, the tax rate
(a) To speak fluently (b) To talk nonsense is 10% higher. The percentage increase in his tax liability is :
(c) To talk wisdom (d) To speak at random (a) 20 (b) 22
DIRECTIONS (Qs. 21 - 25) : Choose the correct spelling of the (c) 23 (d) cannot be determined
given word. 33. Half the girls and one-third of the boys of a college reside in
21. (a) Efflorascence (b) Efflorescence the hostel. What fractional part of the student body is hostle
(c) Efllorescence (d) Eflorescence dwellers if the total number of girls in the college is 100 and
22. (a) Aliennate (b) Allienate
1
(c) Alienate (d) Alienatte is of the total student strength ?
4
23. (a) Forefiet (b) Forefeit
(c) Forfeit (d) Forfiet 3 5
(a) (b)
24. (a) Comemorate (b) Commemmorate 5 8
(c) Momemmorate (d) Commemorate
25. (a) Exemple (b) Exampel 2
(c) (d) None of these
(c) Example (d) Exampal 5
Practice Set - 3 PS-21

34. A train covers 180 km distance in 4 hours. Another train 44. The average monthly salary of employees, consisting of
covers the same distance in 1 hour less. What is the differ- officers and workers, of an organisation is ` 6000. The
ence in the distances covered by these trains in one hour ? average salary of an officer is ` 15000 while that of a worker
(a) 45 km (b) 9 km is ` 4000 per month. If there are a total 600 employees in the
(c) 40 km (d) None of these organisation, find the number of Workers.
35. A man sells an article at 5% profit. If he had bought it at 5% (a) 491 (b) 320
less and sold it for ` 1 less, he would have gained 10%. The (c) 400 (d) 432
cost price of the article is : 45. A car owner buys petrol at ` 7.50, ` 8.00 and ` 8.50 per
(a) ` 200 (b) ` 150 litre for three successive years. What approximately is his
(c) ` 240 (d) ` 280 average cost per litre of petrol if he spends ` 4000 each
36. A bag contains ` 216 in the form of one rupee, 50 paise and year ?
25 paise coins in the ratio of 2 : 3 : 4. The number of 50 paise (a) ` 8 (b) ` 9
coins is : (c) ` 7.98 (d) ` 8.50
(a) 96 (b) 144
(c) 114 (d) 141 SECTION-C : REASONING
37. A can do 50% more work as B can do in the same time. B & MILITARY APTITUDE
alone can do a piece of work in 20 hours. A, with help of B,
can finish the same work in how many hours ? DIRECTIONS (Qs 46- 48) : Choose the most appropriate word.
(a) 12 Hours (b) 8 Hours
46. 'Rabbit' is related to 'Burrow' in the same way as 'Lunatic' is
1
(c) 13 1 Hours (d) 5 Hours related to:
3 2 (a) Prison (b) Cell
38. Anil calculated that it will take 45 minutes to cover a distance (c) Barrack (d) Asylum
of 60 km by his car. How long will it take to cover the same 47. 'Smoke' is related to 'Pollution' in the same way as 'War' is
distance if the speed of his car is reduced by 15 km/hr? related to:
(a) 36 min (b) 55.38 min (a) Victory (b) Treaty
(c) 48 min (d) 40 min (c) Defeat (d) Destruction
39. A trader has a weighing balance that shows 1,200 gm for a 48. Walk' is related to 'Run' in the same way as 'Breeze' is related to:
kilogram. He further marks up his cost price by 10%. Then (a) Cold (b) Dust
the net profit percentage is (c) Wind (d) Air
(a) 32% (b) 23%
DIRECTIONS (Qs 49-50) : Each of the following questions has
(c) 31.75% (d) 23.5%
a group. Find out which one of the given alternatives will be
40. ` 6500 were divided equally among a certain number of
another member of the group or of that class.
persons. Had there been 15 more persons each would have
got ` 30 less. Find the original number of persons. 49. Wheat, Barley, Rice
(a) 45 (b) 50 (a) Food (b) Agriculture
(c) 55 (d) 48 (c) Farm (d) Gram
41. A number of friends decided to go on a picnic and planned 50. Clutch, Brake, Horn
to spend ` 96 on eatables. Four of them, did not turn up. As (a) Car (b) Scooter
a consequence, the remaining ones had to contribute ` 4 (c) Accident (d) Vehicle
each extra. The number of those who attended the picnic DIRECTIONS (Qs 51 - 55) : In each word of the following
was ? questions consists of pair of words bearing a relationship among
(a) 8 (b) 16 these, from amongst the alternatives, pick up the pair that best
(c) 12 (d) 24
illustrate a similar relationship.
42. Ravi’s salary is 150% of Amit’s salary. Amit’s salary is 80%
of Ram’s salary. What is the ratio of Ram’s salary to Ravi’s 51. Thermometer : Temperature
salary ? (a) Millimeter : Scale (b) Length : Breadth
(a) 1 : 2 (b) 2 : 3 (c) Solar Energy : Sun (d) Cardiograph : Heart rate
(c) 5 : 6 (d) 6 : 5 52. Sound : Muffled
43. ` 5,887 is divided between Shyam and Ram, such that (a) Moisture : Humid (b) Colour : Faded
Shyam’s share at the end of 9 years is equal to Ram’s share (c) Despair : Anger (d) Odour : Pungent
at the end of 11 years, compounded annually at the rate of
53. Platform : Train
5%. The share of Shyam is
(a) ` 2,088 (b) ` 2,000 (a) Aeroplane : Aerodrome (b) Hotel : Tourist
(c) ` 3,087 (d) None of these (c) Quay : Ship (d) Footpath : Traveller
EBD_8177
PS-22 Practice Set - 3

54. Scales : Fish (a) KORYBGJ (b) LMEYTPK


(a) Bear : Fur (b) Woman : Dress (c) KMPTYEL (d) KPTYELM
(c) Skin : Man (d) Tree : Leaves DIRECTION (Q. 65) : Find the odd number / letters / number
55. Numismatist : Coins pair from the given alternatives.
(a) Jeweller : Jewels 65. (a) Pathology (b) Geology
(b) Cartographer : Maps (c) Cardiology (d) Radiology
(c) Philatelist : Stamps
DIRECTIONS (Qs. 66-70) : Find out which of the alternatives
(d) Geneticist : Chromosomes (a), (b), (c) and (d) can be formed from the pieces given in box ‘X’.
DIRECTIONS (Qs. 56 - 60) : The following questions are based
on the patterns of three circles as shown in the following
diagrams. Each question has an item group having
relationship with a particular pattern of circles. Find out which
item group is related to which pattern of circles.

(X)

A B C D E
56. Mammals, Cows, Crows
(a) E (b) B (a) (b) (c) (d)
(c) C (d) D
57. Women, Mothers, Engineers 67.
(a) E (b) C
(c) D (d) A
58. Family, Sons, Daughters
(a) E (b) B
(X)
(c) C (d) A
59. Colour, Cloth, Merchant
(a) A (b) B
(c) C (d) D
60. Konark, Pushkar, India
(a) E (b) D (a) (b) (c) (d)
(c) C (d) B
68.
DIRECTIONS (Qs 61 & 62) : In these questions find the odd
word/number/ letters/number pair from the given alternatives.
61. (a) LPXOY (b) RQST
(c) FBDLX (d) MPONL
62. (a) 14 - 16 (b) 56 - 64 (a) (b) (c) (d)
(X)
(c) 77 - 88 (d) 80 - 93
63. Find the missing (?) figure?
Question figures 69.

. .
: . :: :? (a) (b) (c) (d)
Answer figures (X)

. . . . 70.

(a) (b) (c) (d)


64. In the following questions number of letters skipped in
between adjacent letters in the series increased by one. (a) (b) (c) (d)
(X)
Which of the following series observe the rule ?
Practice Set - 3 PS-23

DIRECTIONS (Qs 71 - 75) : In each of the following questions,


74.
a part of the figure is missing. Find out from the given option
(a), (b), (c) or (d) the right figure to fit in the missing figure (x).

71.

(X)

(X) (a) (b)

(a) (b)
(c) (d)

(c) (d)
75.

72.

(X)

(X)
(a) (b)

(a) (b)

(c) (d)
(c) (d)
DIRECTIONS (Qs 76 - 80) : From amongst the figures marked
(1), (2), (3) and (4), select the figure which satisfies the same
73. conditions of placement of the dots as in figure (X).

76.

?
(X) (1) (2) (3) (4)
(X) (a) 1 (b) 2
(c) 3 (d) 4

(a) (b)
77.

(X) (1) (2) (3) (4)


(c) (d) (a) 1 (b) 2
(c) 3 (d) 4
EBD_8177
PS-24 Practice Set - 3

78. Select the figure which satisfies the same conditions of 87. Which is NOT the name of the missile developed by the
placement of the dots as in Figure-X. Defense Research and Development Organisation (DRDO)?
(a) Shaurya (b) Pinaka
(c) Brahmos (d) Agni
88. ‘New Horizons’ spacecraft was launched by NASA to Study
which of the following Planet?
(a) Mars (b) Pluto
(X) (1) (2) (3) (4)
(c) Jupiter (d) Mercury
(a) 1 (b) 2 89. Which of the following is known as ‘Seven Pagodas’?
(c) 3 (d) 4 (a) Mahabalipuram temple (b) Karle caves
(c) Chaityas (d) Elephanta caves
79.
90. Who is the author of the book India 2020?
(a) Nibal Singh (b) R.K. Narayan
(c) Sidney Shelton (d) Dr. A.P.J.Abdul Kalam
(X) (1) (2) (3) (4) 91 When is the World Population Day observed?
(a) 1 (b) 2 (a) May 31 (b) October 4
(c) 3 (d) 4 (c) December 10 (d) July 11
92. The Government of India has decided to set up National
80.
Cultural Audio-Visual Archives at
(a) Jodhpur (b) Vadodara
(X) (1) (2) (3) (4) (c) Jalandhar (d) Ernakulam
(a) 1 (b) 2 93. Who has been crowned miss universe of the year 2019?
(c) 3 (d) 4 (a) Modison Anderson (b) Ashley Alvidrez
(c) Zozibini Tunzi (d) None of the above
SECTION-D : GENERAL AWARENESS 94. The study of lake is called
81. How many members can be nominated to both the Houses (a) Topology (b) Hydrology
of the Parliament by the President ? (c) Limnology (d) Potomology
(a) 14 (b) 16 95. Which countries are separated by the McMahon line?
(c) 10 (d) 12 (a) India and Bangladesh (b) India and Pakistan
82. Presidential form of government consists of the following : (c) China and Tibet (d) India and China
(a) Popular election of the President 96. The ‘Chipko Movement’ is related to
(b) No overlap in membership between the executive and (a) Wildlife preservation (b) Scientific agriculture
the legislature (c) Forest conservation (d) Deforestation
(c) Fixed term of office 97. Israel built "Spike", which has been decided to be bought
(d) All of the above by India is a/an
83. Who among the following introduced the Mansabdari (a) Radar system (b) Anti- aircraft missile
system ? (c) Stealth ship (d) Anti tank missile
(a) Jahangir (b) Shah Jahan 98. With how many permanent members of United Nations
(c) SherShah (d) Akbar Security Council India has civil nuclear pact signed?
84. Moraines are formed in (a) Two (b) Three
(a) Monsoon region (b) River deltas (c) Four (d) Five
(c) Arid regions (d) Glacial regions 99. What is the name given to military exercises conducted
85. Who has won the 55th Jnanpith Award 2019? between India and Nepal?
(a) AP Gour (a) Surya Kiran (b) Malabar
(b) Vikram Seth (c) Indra (d) Varuna Exercise
(c) Piyush Govind 100. The union government has announced a nationwide scheme
(d) Akkitham Achuthan Namboothiri “Rashtriya Gokul Mission” which aims to
86. Longest cell in human body is: (a) Eliminate diseases of cattle
(a) Blood cell (b) Bone cell (b) Increase milk production
(c) Nerve cell (d) Muscle cell (c) Curb slaughtering of cattle
(d) Protect the indigenous breed of cows
Practice Set - 3 PS-25

ANSWERS & SOLUTIONS


1. (c) This also is a factual question. Refer to the line “Did he 27. (b) Let the numbers be n – 2, n – 1, n, n + 1 and n + 2. Their
intend for the dome to look like the model he built..... average = n.
elevate the dome in the way it is today ?” This line clearly Next two consecutive numbers are n + 3 and n + 4.
suggests that the dome is raised more than the medal. Therefore the average of 7 consecutive numbers
2. (b) This is an inference question. This should be solved
(n - 2) + (n -1) + n + (n +1) + (n + 2) + (n + 3) + (n + 4)
by elimination. (a) is not the answer because the =
passage does not state the in Michelangelo was a 7
private person, it only question if he was a private 5n + 2n + 7
= = n +1
person. (b) is obviously implicit in the passage (c) is 7
not suggested by the passage. He did some 28 (c) Let P be the Principal amount and r be the rate of interest
architectural work in Rome, which is considered great C.I – S.I = 618 – 600 = 18
but the passage does not directly deem Michelangelo éæ 2 ù P´r´2
as the greatest arched of Rome (d) is absolutely wrong r ö
P êç1 + ÷ - 1ú - = 18
and is not in any way suggested by in passage. êëè 100 ø úû 100
3. (a) Because the passage raises many questions about the 2
fact that Michelangelo destroyed many of his paintings æ r ö
Pç = 18
è 100 ÷ø
...(1)
but cause is not mentioned in the passage.
4. (b)
2 Pr
5. (c) In the gap 7, 'Aron had not signalled anyone' is the Also, we have = 600 Þ Pr = 30000
right option. Other options simply do not fit in here. 100
6. (b) 'Solo' means 'any activity that is performed alone Put value of Pr in (1)
without assistance' which Aron did. Other options we get, r = 6, rate = 6%
'supervised' means under observation or under the 1
direction of a superintendent or overseer, team means 29. (a) Let us take a proper fraction, such as .
2
form a team and shared means have in common; held
or experienced in common. 1+ 2 3
Now, the new fraction = =
7. (c) In the gap 7, the word 'trapped' is the right option while 2+ 2 4
suppressed means kept from public knowledge by
various means; captive means a person who is confined; 3 1
Thus, >
especially a prisoner of war and entangled means deeply 4 2
involved especially in something complicated. 30. (d) Average weight of 30 boys = 60 kg
8. (d) In the context of paragraph, the 'life-changing' may rightly Þ Sum of weight of 30 boys = 1800
be fit in here as it changed Aaron's life with the amputation Average weight of 10 =56 kg
of one of his arms. Other options are not relevant. Þ Sum of weight of 10 boys = 560
9. (d) Ambiguous means having more than one possible Average weight of the whole class
meaning. Sum of weight of all boys
10. (a) If someone or something is infallible, that means they =
40
are incapable of failure or error.
11. (d) Conscientious is the one who is guided by or in sum of weight of 30 boys + sum of weight of 10 boys
accordance with conscience or sense of duty and right =
40
and wrong.
12. (d) 60×30+56×10
= = 59kg
13. (c) Obsolete means no longer in use 40
14. (c) regicide means the act of killing a king. 31. (c) Ist case :
15. (d) 16. (b) 17. (c) 18. (a) 19. (b) 20. (b)
21. (b) 22. (c) 23. (c) 24. (d) 25. (c) 25
100 + ´ C.P
100 ´ Profit% 2
26. (b) 4A = 6B Þ 2A = 3B Þ A : B = 3 : 2 S.P = ´ C.P. Þ S.P. =
6B = 3C Þ 2 B = C Þ B : C = 1 : 2 100 100
A : B : C 112.5
3 : 2 Þ S.P = CP ...(1)
1 : 2 100
IInd case :
3 : 2 : 4
100 + Profit % 100 + 15
3 3 S.P = ´ C.P. Þ (S.P + 10) = ´ C.P.
A’s share = ´ 1800 = ´ 1800 = 600 100 100
(3 + 2 + 4) 9
EBD_8177
PS-26 Practice Set - 3

115 36. (b) Let the no. of one rupee, 50 paise and 25 paise coins be
Þ (S.P+10) = C.P ...(2) 2x, 3x and 4x respectively.
100 According to question,
Dividing equation (1) by (2)
æ 3x 4x ö
S.P 112.5 100 ` ç 2x + + ÷ = ` 216
= (C.P) ´ è 2 4ø
S.P+10 100 115(C.P) 8x + 6x + 4x
Þ = 216 \ x = 48
æ 112.5 ö 4
S.P = ç ÷ (S.P + 10) \ Number of 50 paise coins = 48 × 3 = 144
è 115 ø
1
115 S.P = 112.5 SP + 1125 37. (b) In one hr. B finishes of the work.
S.P = 450 20
S.P ´ 100 450 ´100 1 3 3
\ C.P = = = 400 In one hr. A finishes ´ = of the work.
112.5 112.5 20 2 40
32. (b) Let the salary of Mr ‘X’ is Rs x and % of tax he has to 2+3 1
pay = y A+B finish = of the work in 1 hr..
40 8
xy Both of them will take 8 hrs. to finish the work.
So tax payable =
100 38. (b) D=S×T
On increase in salary, his salary becomes = 1.2x
æ 45 ö 60 ´ 60
% of tax he has to pay on increased salary = 1.1y 60 = S ´ ç ÷ hr ; S = Þ 80km/hr
So, tax payable on increased salary è 60 ø 45
1.1 y ´ 0.2 x 0.22 xy Now, new speed = 80 – 15 = 65 km/hr.
= = Distance 60
100 100 \ Time = = hr.
xy 0.22 xy Speed 65
Total tax payable on new salary = +
100 100
60
\ % increase in tax liability or ´ 60 min = 55.38 min.
65
æ 0.22 xy xy xy ö Hence, Time to taken by car to travel same distance is
çè + - ÷
100 100 100 ø 55.38 min.
= ´ 100 = 22%
xy 39. (a) The trader professes to sell 1200 kg but sells only
100 1000 kg.
So profit = 20%
33. (d) Total no. of students = 400; No. of girls = 100
Markup = 10%
\ No. of boys = 300
10 ´ 20
1 1 Total profit = 10 + 20 + = 32%
No. of hostel dwellers = ´ 100 + ´ 300 = 150 100
2 3
40. (b) Let the original no. of persons be x.
150 3 6500 6500
\ Required fraction = = Then, = + 30
400 8 x x + 15
34. (d) First train’s speed is 45km/hr. 6500 6500 + 30 x + 450
or =
æ Distance ö x x + 15
çè Using speed = ÷
Time ø or x 2 + 15 x - 3250 = 0
Second train’s speed is 60km/hr. or x = 50
Difference in the distance covered by these trains in 1 hr. 41. (a) Let initial number of friends to attend picnic = x.
is 15 km. 96 96
35. (a) Let the CP of the article be ` x. \ +4=
x x-4
105x
Then, SP = ` 96 + 4 x 96
100 =
x ( x - 4)
95x 105x
Now, new CP = ` and new SP = -1 Þ 96x + 4x2 – 384 – 16x = 96x Þ 4x2 – 16x – 384 = 0
100 100 Þ x2 – 4x – 96 = 0 Þ x2 – 12x + 8x – 96 = 0
According to the question Þ x(x – 12) + 8(x – 12) = 0 Þ (x + 8) (x – 12) = 0
105x 95 10 ´ 95x \ x = – 8, 12
-1- = where (x – 4) attended the picnic.
100 100 100 ´100
Hence the no. of friends who attended the picnic is 8.
\ x = ` 200
Practice Set - 3 PS-27

42. (c) Let the salary of Ram be ` 100. 53. (c) Second is the place where first stops.
Then, salary of Amit = ` 80 54. (c) As scales from an outer layer of fish similarly skin form
and salary of Ravi = 150% of 80 = ` 120 an outer layer of man.
Ratio of Ram’s salary to Ravi’s salary = 100 : 120 = 5 : 6 55. (c) As Numismatist collects coins similarly Philatelist
43. (c) Let Shyam’s share be x collects stamps.
According to question
9 11 Mammals
æ 5 ö æ 5 ö
x ç1 + = ( 5887 - x ) ç 1 +
è 100 ÷ø è 100 ÷ø 56. (b)
Cow
2
x æ 5 ö Crows
= ç1 + ÷
5887 - x è 100 ø
x
= 1.1025 \ x = ` 3087. 57. (c) Women Engineers
5887 - x
44. (a) Average monthly salary of employees = 6000
Let the number of officer = n 1
and the number of workers = n 2 Mothers
15000 n1 + 4000 n2 = 6000
15 n1 + 4 n2 = 3600 Family
n1 + n2 = 600
58. (c) Sons Daugthers
Þ 15 n1 + 4 n2 = 3600
4 n1 + 4 n2 = 2400

11n1 = 1200 59. (d)


Merchant
1200
n1 = Colour
11
= 109.09
» 109 Cloth

Number of offer = 109 India


\ Number of workers = (600 – 109) = 491
45. (c) Let average cost of petrol per litre be ` x 60. (c) Konark Manasarovar
12000
\ x=
4000 4000 4000
+ + 61. (b) Except (b) all others have five letters.
7.5 8 8.5 62. (d) Except (d) in both number in all others pairs are divided
12000 3 by same number.
3
= = = 63. (a) The middle element adjecents to the right side line after
æ 1 1 1 ö 10 + 1 + 10 2 1 2 rotating 90° anti-clockwise. The bottom element goes
4000 ç + + ÷ + +
è 7.5 8 8.5 ø 75 8 85 15 8 17 up on the top and becomes enlarge. The top element
becomes the inner figure of bottom element.
6120
= =` 7.98 per litre 64. (c) By options :
767
46. (d) As the dwelling place of 'Rabbit' is 'Burrow' in the same +2 +3 +4 +5 +6 +7
K ¾¾®M ¾¾®P ¾¾®T ¾¾®Y ¾¾®E ¾¾®L
way the dwelling place of 'Lunatic' is 'Asylum'.
47. (d) As 'smoke' leads to 'pollution' in the same way ''War'' 65. (b) As all terms given in question are medical terms except
leads to 'destruction'. geology.
48. (c) As fast mode of 'Walk' is 'Run' in the same way fast
mode of 'Breeze' is 'Wind'.
49. (d) All the terms given in the question are cereals and 66. (b)
gram is also one of the cereals.
50. (d) All these are parts of a vehicle.
51. (d) As temperature is measured from a thermometer in the
same way heart rate is measured with cardiograph. 67. (b)
52. (b) Second is the process of gradual disappearances of
the first. 68. (b) 69. (c) 70. (c)
EBD_8177
PS-28 Practice Set - 3

no region common to the circle and the rectangle only.


In fig. (3), there is no region common to the circle, the
square and the triangle only. Only fig. (4) consists of
71. (b) both the types of regions.
79. (a) In fig. (X), one of the dots lies in the square alone,
another dot lies in the triangle alone and the third dot
lies in the region common to the circle and the square.
In fig. (2) there is no region that lies in the square
alone, in fig. (3) there is no region that lies in the triangle
alone and in fig. (4) there is no region that lies in the
72. (d) region common to the circle and the square only. Only
fig. (1) consists of all the three types of regions.
80. (c) In fig. (X), one of the dots lies in the square alone,
another dot lies in the region common to the square
and the triangle only and the third dot lies in the region
common to the circle and the triangle. In fig. (1), there
is no region which lies in the square alone. In each of
the figures (2) and (4), there is no region common to
73. (d) the circle and the triangle only. Only, fig. (3) consists
of all the three types of regions.
81. (a) According to the Indian Constitution, 14 members can
be nominated to both the houses of parliament by the
President. This is the legislative power of the President
where he nominates 12 members to the Rajya Sabha
and if not adequately represented 2 Anglo-Indian
74. (a) members to the Lok Sabha.
82. (d) A presidential system is a republican system of
government where a head of government is also head
of state and leads an executive branch that is separate
from the legislative branch. The United States, for
75. (c)
instance, has a presidential system. Popular election
of President, no overlap in membership and fixed term
of office are the main criteria of Presidential form of
Government.
83. (d) Akbar introduced the Mansabdari system. This
system came under the military reforms of Akbar. Under
76. (c) In fig. (X), one of the dots lies in the region common to this system each officer was assigned a rank(mansab).
the square and the triangle and another dot lies in the Varying from 10 to 10,000, the mansab carried the
region common to the circle and the triangle. In each Zat(the personal status and salary) and Sawar (the
of the alternatives (1), (2) and (4), the region common number of cavalry men to be maintained.
to the square and the triangle lies within the circle. 84. (d) Moraines are formed in glacial regions. Moraine is
Therefore, in each of these figures, there is no region accumulation of rock debris (till) carried or deposited
common to the square and the triangle only. Only the by a glacier. The material, which ranges in size from
alternative (3) consists of a region common to the blocks or boulders (usually faceted or striated) to sand
square and the triangle only and another region and clay, is unstratified when dropped by the glacier
common to the circle and the triangle only. Hence, fig. and shows no sorting or bedding.
(3) is the answer. 85. (d) 86. (c)
77. (c) In fig. (X), one of the dots lies in the region common to 87. (d) Agni is a surface to surface missile developed in India
the circle and the triangle only, another dot lies in the under Integrated Guided Missile Programme.
circle alone and the third dot lies in the region common to 88. (b)
the circle and the square only. In fig. (1) there is no region 89. (a) Mahabalipuram temple is known as ‘Seven-Pagodas’.
common to the circle and the triangle only, in fig. (2), there 90. (d) 91. (d)
is no region common to the circle and the square and in 92. (a) Ministry of Culture and the Rupayan Sansthan has
fig. (4), there is no region which lies in the circle alone. signed an MoU to set up a National Cultural Audio-
Only, fig. (3) consists of all the three types of regions.
78. (d) In fig. (X), one of the dots lies in the region common to Visual Archives in Jodhpur with the aim to preserve
the circle and the rectangle only and the other dot lies cultural heritage of the country.
in the region common to the circle, the square and the 93. (c) 94. (c) 95. (d) 96. (d) 97. (d) 98. (c)
triangle only. In each of the figures (1) and (2), there is 99. (a) 100. (d)
4 Practice Set
INSTRUCTIONS
1. This practice set comprises four sections. Section A : Verbal Ability in English; Section B : Numerical Ability; Section C :
Reasoning and Military Aptitude; Section D : General Awareness.
2. The set will consist of 100 questions and each questions will be of 3 marks.
3. Each questions have four options, of which one is correct. The students are advised to read all the options thoroughly.
4. There is one-third negative marking in the set.

Time: 2 hrs. Max. Marks: 300


Ti

SECTION-A : VERBAL ABILITY IN ENGLISH (a) Love of family


(b) Love of compatriots
DIRECTIONS (Qs. 1 - 5) : Read the following passage carefully (c) The element of worship
and answer the questions given below it : (d) None of the above
Patriotism is a very complex feeling, built up out of primitive 5. Which one of the following statements is false?
instincts and highly intellectual convictions. There is love of home
(a) We tend to like our own countrymen better than we
and family and friends, making us peculiarly anxious to preserve
our own country from invasion. There is the mild instinctive liking like foreigners
for compatriots as against foreigners. There is pride, which is (b) Nations always stand for ideals that are important to
bound up with the success of the community to which we feel the human race
that we belong. There is a belief, suggested, by pride but reinforced (c) It is the religious element in patriotism that motivates
by history, that one's own nation represents a great tradition and us for sacrificing ourselves for our nation
stands for ideals that are important to the human race. But besides (d) Our pride of the community is bound with the
all these, there is another element, at once nobler and more open community's success
to attack, an element of worship, of willing sacrifice, of joyful
merging of the individual life in the life of the nation. This religious DIRECTIONS (Qs. 6 - 8) : Choose the word which is nearest in
element in patriotism is essential to the strength of the State, meaning to the given word :
since it enlists the best that is in most men on the side of national 6. DISCOMFIT
sacrifice.
(a) litigate (b) ease
1. A suitable title for the passage could be :
(a) Elements of Patriotism (c) conflict (d) frustrate
(b) Historical Development of a Nation 7. WRATH
(c) The role of Religion and History in Patriotism (a) violence (b) anger
(d) Religion and Patriotism (c) hatred (d) displeasing
2. Describing the element of worship "Open to attack", the 8. ABSTINENCE
author implies that it : (a) synchronic (b) torrential
(a) is unnecessary (c) restraint (d) gluttony
(b) leads to national sacrifice DIRECTION (Qs. 9 - 10) : Choose the word which is nearly
(c) has no historical basis opposite in meaning to the given word.
(d) cannot be justified on rational grounds 9. Insipid
3. The tone of the passage can best be described as :
(a) Tasty (b) Colourful
(a) critical (b) descriptive
(c) Colourless (d) Dull
(c) persuasive (d) analytical
10. Relinquish
4. Which of the following can clearly be grouped under
"intellectual convictions" the author mentions in the (a) Relish (b) Continue
opening sentence? (c) Vanish (d) Quench
EBD_8177
PS-30 Practice Set - 4

DIRECTIONS (Qs. 11 - 15) : In each of the following questions, (c) In a fighting mood
find out which part has an error. (d) Insuspense
11. Hasan plays (a) / both-cricket and billiards (b) /at the national 25. To cast pearls before a swine
level. (c) / No error. (d) (a) To spend recklessly
12. More you (a) / think of it, (b) / the worse it becomes.(c) / No (b) To spend a lot of money on the unkeep of domestic
error. (d) hogs
13. My father gave me (a) / a pair of binocular (b) / on my (c) To waste monkey over trifles
birthday. (c) / No error. (d)
(d) To offer to a person a thing which he cannot
14. Kalidas is (a) / a Shakespeare (b) /of India. (c) / No error. (d)
15. The teacher as well as his students, (a) / all left (b) / for the appreciate.
trip. (c) / No error. (d)
SECTION-B : NUMERICAL ABILITY
DIRECTIONS (Qs. 16 - 20) : Pick up the most effective word 26. A student was asked to divide a number by 6 and add 12 to
from the given words to fill in the blanks to make the sentence
the quotient. He, however, first added 12 to the number and
meaningfully complete.
then divided it by 6, getting 112 as the answer. The correct
16. The human mind seems to have built-in ________against answer should have been :
original thought. (a) 122 (b) 118
(a) prejudices (b) ideas (c) 114 (d) 124
(c) interests (d) safeguards 27. One-fourth of Nikhil’s money is equal to one-sixth of
17. The statue _________ a global symbol of freedom Yogesh’s money. If both together have Rs 600, what is the
(a) stands against (b) stands to difference between their amounts ?
(c) stands for (d) stands as (a) ` 160 (b) ` 240
18. A child is the future of a family _________ nation. (c) ` 200 (d) ` 120
(a) just as (b) as a 28. Ram spends 20% of his monthly income on his household
(c) like a (d) as well as of a expenditure, 15% of the rest on books, 30% of the rest on
19. If strict security measures were taken, the tragedy might clothes and saves the rest. On counting, he comes to know
have been ________ that he has finally saved ` 9,520. Find his monthly income.
(a) restrained (b) averted (a) ` 15,000 (b) ` 10,000
(c) removed (d) controlled (c) ` 20,000 (d) None of these
20. The deceased left _______ children. 29. The price of petrol is increased by 25%. How much per cent
(a) behind (b) for must a car owner reduce his consumption of petrol so as
(c) with (d) by not to increase his expenditure on petrol ?
DIRECTIONS (Qs. 21 - 25) : Which word or words explains the (a) 50% (b) 30%
meaning of the following idioms : (c) 25% (d) 20%
21. To take through one’s hat 30. If books bought at prices ranging from ` 150 to ` 300 are
(a) To speak fluently (b) To talk nonsense sold at prices ranging from ` 250 to ` 350, what is the greatest
(c) To talk wisdom (d) To speak at random possible profit that might be made in selling 15 books ?
22. To snap one’s fingers (a) ` 3,000 (b) Cannot be determined
(a) To speak abruptly (c) ` 750 (d) ` 4,250
(b) To accept immediately 31. The average of 30 numbers is 40 and that of other 40 numbers
(c) To grasp eagerly is 30. The average of all the numbers is
(d) To become contemptuous of 2
(a) 34.5 (b) 34
23. To take the bull by the horns 7
(a) To punish a person severly for his arrogance (c) 35 (d) 34
(b) To grapple courageously witha difficulty that lies in 32. A man sold two articles at ` 375 each. On one, he gains 25%
our way and on the other, he loses 25%. The gain or loss% on the
(c) To handle it by fierce attack whole transaction is :
(d) To bypass the legal process and take action accord-
1
ing to one’s own whims. (a) 6% (b) 4 %
6
24. To be in abeyance
(a) To be in trouble 1
(c) 7% (d) 6 %
(b) Dual minded 4
Practice Set - 4 PS-31

33. In a school, the ratio of boys to girls is 4 : 3 and the ratio of 43. A, B, C together earn ` 1450 and spend 60%, 65% and 70%
girls to teachers is 8 : 1. The ratio of student to teachers is : of their salaries respectively. If their savings are in the ratio
(s) 56 : 3 (b) 55 : 1 14 : 21 : 15, the salary of B is
(c) 49 : 3 (d) 56 : 1 (a) ` 500 (b) ` 600
34. R and S start walking each other at 10 AM at the speeds of (c) ` 450 (d) ` 750
3 km/hr and 4 km/hr respectively. They were initially 17.5 km 44. Rahul can row a certain distance downstream in 6 hours
apart. At what time do they meet ? and return the same distance in 9 hours. If the speed of
(a) 2 : 30 PM (b) 11 : 30 AM Rahul in still water is 12 km/hr, find the speed of the stream.
(a) 2 km/hr (b) 2.4 km/hr
(c) 1 : 30 PM (d) 12 : 30 PM
(c) 3 km/hr (d) Data inadequate
35. The length and breadth of a square are increased by 30% 45. Two trains, 130 m and 110 m long, are going in the same
and 20% respectively. The area of the rectangle so formed direction. The faster train takes one minute to pass the
exceeds the area of the square by: other completely. If they are moving in opposite directions,
(a) 46% (b) 66% they pass each other completely in 3 seconds. Find the
(c) 42% (d) 56% speed of each train.
36. The average weight of 12 crewmen in a boat is increased by (a) 38 m/sec, 36 m/sec (b) 42 m/sec, 38 m/sec
1 (c) 36 m/sec, 42 m/sec (d) None of these
kg, when one of the crewmen whose weight is 55 kg is
3 SECTION-C : REASONING
replaced by a new man. What is the weight of that new & MILITARY APTITUDE
men ?
(a) 58 (b) 60 DIRECTIONS (Qs. 46-50): In each of the following questions, certain
(c) 57 (d) 59 pairs of words are given, out of which the words in all pairs except one,
37. A sum of ` 8,000 lent on simple interest amounts to ` 8,800 bear a certain common relationship. Choose the pair in which the words
in two years. If the rate of interest is decreased by two are differently related.
percentage, then what would the sum amount to in the same 46. (a) Newspaper : Editor (b) Film : Director
period ? (c) Stamps : Philatelist (d) Book : Author
(a) ` 8,680 (b) ` 8,480 47. (a) Steel : Utensils (b) Bronze : Statue
(c) Duralumin : Aircraft (d) Iron : Rails
(c) ` 8,280 (d) ` 8,340
48. (a) Fish : Pisciculture (b) Birds : Horticulture
38. Two trains are 2 km apart and their lengths are 200 m and 300 (c) Bees : Apiculture (d) Silkworm : Sericulture
m. They are approaching towards each other with a speed 49. (a) Principal : School (b) Soldier : Barrack
of 20 m/s and 30 m/s, respectively. After how much time will (c) Artist: Troupe (d) Singer : Chorus
they cross each other ? 50. (a) Cow : Fodder (b) Crow : Carrion
(a) 50 s (b) 100 s (c) Poultry : Farm (d) Vulture : Prey
(c) 25/3 s (d) 150 s
39. When the price of a radio was reduced by 20%, its sale DIRECTIONS (Qs. 51-55):In each of the following questions, five
words have been given out of which four are alike in some manner, while
increased by 80%. What was the net effect on the sale?
the fifth one is different. Choose the word which is different from the rest.
(a) 44% increase (b) 44% decrease
(c) 66% increase (d) 75% increase 51. (a) Inch (b) Foot
40. Deepak and Sanjeet run a 10 km race on a round course of (c) Yard (d) Quart
200 m. If their speeds be in the ratio 5 : 4, how often does the 52. (a) Mercury (b) Bromine
(c) Aluminium (d) Sodium
winner pass the other ?
53. (a) Mariana (b) Nigar
(a) 8 times (b) 12 times
(c) Angel (d) Gersoppa
(c) 14 times (d) 10 times
54. (a) Arc (b) Diagonal
41. Two pipes P and Q would fill a cistern in 24 and 32 minutes,
(c) Tangent (d) Radius
respectively. Both pipes are kept open. When should the
55. (a) Granite (b) Lignite
first pipe be turned off so that the cistern may be just filled (c) Peat (d) Anthracite
in 16 minutes ?
(a) After 10 minutes (b) After 12 minutes DIRECTIONS (Qs. 56 - 60): Three words in bold letters are given in
each questions,which have something in common among themselves.
(c) After 14 minutes (d) None of these
Out of the four given alternatives, choose the most appropriate description
42. If the length of a certain rectangle is decreased by 4 cm and about these three words.
the width is increased by 3 cm, a square with the same area
56. Spinach : Fenugreek : Celery
as the original rectangle would result. The perimeter of the
(a) These are cactus plants.
original rectangle (in centimetres) is :
(b) These are wild flowers.
(a) 44 (b) 46
(c) These are wild plants.
(c) 48 (d) 50
(d) These are leafy vegetables.
EBD_8177
PS-32 Practice Set - 4

57. Pulpit : Pews : Steeple


(a) They are connected with a glacier valley.
(b) They are connected with church. (c) (d)
(c) The terms are connected with race-course.
(d) They are parts of a plant.
58. Petrol :Phosphorus : Cooking gas
Options: ?
(a) They are fuels.
63
(b) They are highly inflammable.
(c) They can`t be sold without permit.
(d) India has to import them. (X)
59. Chlorine : Fluorine : Iodine
(a) These are names of inert gases.
(b) These are gases at room temperature. (a) (b)
(c) These are transition elements.
(d) These are halogens.
60. Species : Genera : Family
(a) These are biological terms. (c) (d)
(b) These give information about living things for classification.
(c) These are traits of animal kingdom.
(d) These are groups of animals. 64.
DIRECTION (Qs. 61- 65) : In each of the following questions,
selecta figure from amongst the four alternatives which when
placed in the blank space of figure (X) would complete the
pattern.
?

61.
(a) (b)

(c) (d)

(X)
65.
?

(a) (b) (c) (d)

62.
? (a) (b)

(c) (d)
(X)

DIRECTIONS (Qs. 66-70): In each of the following questions there is


(a) (b) a specific relationship between the first and second figure. The same
relationship exists between the third and fourth figure which will replace
the blank column. Select the term from the alternatives given.
Practice Set - 4 PS-33

66.
•• ••
•• 70. ••

(a) (b)
(a) (b)

(c) (d)
(c) (d)

••
67. •• DIRECTIONS (Qs. 71- 75) : In each of the following questions
there are four figures which are alike in some respect and one is
different from others. Find out the odd figure in each questions
and indicate your answer.
(a) (b)

71.
(c) (d)

(a) (b) (c) (d)

••
68. •• 72.

(a) (b) (c) (d)


(a) (b)

73.

(c) (d)

(a) (b) (c) (d)

••
69. •• 74.

(a) (b) (c) (d)


(a) (b)

75.

(c) (d)
(a) (b) (c) (d)
EBD_8177
PS-34 Practice Set - 4

DIRECTION (Qs. 76 - 80) : Each of the following questions Answer Figures


consists of five problem figures marked A, B, C, D and E.From
the answer figures marked (a), (b), (c) and (d) select a figure
which will continue the series.
76. Problem Figures

(a) (b) (c) (d)


80. Problem Figures
(A) (B) (C) (D) (E)
Answer Figures
\

(A) (B) (C) (D) (E)


Answer Figures
(a) (b) (c) (d)
77. Problem Figures

(a) (b) (c) (d)

(A) (B) (C) (D) (E) SECTION-D : GENERAL AWARENESS


Answer Figures 81. The concept of Concurrent List in Indian Constitution is
borrowed from the Constitution of
(a) U.S.A. (b) Japan
(c) Canada (d) Australia
82. Which one of the following cities and the personalities
(a) (b) (c) (d) associated with their establishment is wrongly matched ?
78. Problem Figures (a) Calcutta - Robert Clive
(b) Pondicherry - Francis Martin
(c) Ahmedabad - Ahmad Shah I
(d) Madras - Francis Day
83. The Crimean War came to an end by the
(a) Treaty of St. Germain
(A) (B) (C) (D) (E) (b) Treaty of Trianon
(c) Treaty of Versailles
Answer Figures
(d) Treaty of Paris
84. The Himalayan mountain range is an example of
(a) Fold mountain (b) Volcanic mountain
(c) Residual mountain (d) Block mountain
85. The depletion of Ozone layer is mainly due to
(a) Chlorofluorocarbons (b) Volcanic eruptions
(a) (b) (c) (d)
(c) Aviation fuels (d) Radioactive rays
79. Problem Figures 86. The use of which of the following regional languages was
popularised by the Bhakti leader, Shankaradeva ?
(a) Bengali (b) Brijbhasha
(c) Avadhi (d) Assamese
87. Which part of the plant is used as 'saffron'?
(a) Petals (b) Stamens
(c) Style and Stigma (d) Sepals
(A) (B) (C) (D) (E)
Practice Set - 4 PS-35

88. Suspended colloidal particles in the water can be removed (c) College education
by the process of : (d) Education of children between 6-14 years
(a) Filtration (b) Adsorption 94. ‘Apna Khet, Apna Kaam’, a new scheme under MNREGA
(c) Absorption (d) Coagulation has been initiated in which state?
89. When and by whom were the Asokan inscriptions (a) Punjab (b) Rajasthan
deciphered for the first time ? (c) Uttra Pradesh (d) Madhya Pradesh
(a) 1787 - John Tower (b) 1825 - Charles Metcalfe 95. Which of the following places is famous for Chikankari work,
(c) 1837 - James Prinsep (d) 1810 - Harry Smith which is a traditional art of embroidery?
90. Which gupta emperor is said to have founded Nalanda
University? (a) Lucknow (b) Hyderabad
(a) Skandagupta (b) Buddhagupta (c) Jaipur (d) Mysore
(c) Purugupta (d) Kumaragupta I
91. Other than Annie Besant, who among the following also 96. The second highest Gallantry award is
launched a Home Rule Movement in India? (a) Mahavir Chakra (b) Vir Chakra
(a) Aurobindo Ghosh (c) Arjuna Award (d) Ashok Chakra
(b) Bal Gangadhar Tilak 97. The highest peace time gallantry award is
(c) Gopal Krishna Gokhale (a) Ashok Chakra (b) Param Vir Chakra
(d) Moti Lal Nehru. (c) Kirti Chakra (d) Vir Chakra
92. Three-tier system of Panchayati Raj consists of 98. Who has authored the book - "India and the Global Financial
(a) Gram Panchayat, Panchayat Samiti, Block Samiti Crisis"?
(b) Gram Panchayat, Block Samiti, Zila Parishad (a) Y. V. Reddy (b) Shankar Acharya
(c) Gram Panchayat, Panchayat Samiti, Zila Parishad (c) Rakesh Mohan (d) C. Rangarajan
(d) None of these 99. Which of the following is a pair names of the same game?
93. ‘Sarva Siksha Abhiyan’ is aimed at the education of which (a) Soccer - Football (b) Golf - Polo
of the following? (c) Billiards - Carrom (d) Volleyball – Squash
(a) Engineering and technical education 100. Which zone of a candle flame is the hottest ?
(b) Education of girls upto graduation level (a) Dark innermost zone (b) Outermost zone
(c) Middle luminous zone(d) Central zone

ANSWERS & SOLUTIONS


1. (a) To find a suitable title one should first find the central 4. (d) This also will be done through elimination. “Intellectual
theme of the passage-Patriotism of the four given conviction” will means beliefs regarding matters of the
options (a) is the most appropriate because the other intellect. None of the options given are intellectual
three options only talk about a particular idea matters, they are rather feelings. Therefore the answer
discussed in the passage whereas none of them is (d).
receives role importance all of them are mentioned and 5. (c) The element of “willing sacrifice” as mentioned in the
discuss therefore a general title elements of patriotism line “But besides all these,...... Of willing sacrifice, of
will include all failure and well be the most appropriate joyful merging of the individual life in life of nation”
title for the given passage does not necessarily emerge from the religious element.
2. (d) “Open to attack” would mean it can be questioned or Both element of worship and the passage does not
criticized. Coopering this meaning to the given options suggest that the former motivates the latter.
(a) is not necessarily true, (b) is wrong, (c) is also not 6. (d) If you are discomfited by something, it causes you to
necessarily true, (d) on the other hand is nearest to feel slightly embarrassed or confused. In other words,
this meaning. Even in the larger sense of the passage it, frustrates you.
(d) is the most possible answer. 7. (b) Wrath is extreme anger.
3. (d) Thus can be done through elimination (a) is incorrect, 8. (c) If you abstain from something, you deliberately do
passage is not critical of any thing, it is rather bent not do it. Abstinence, however, is a particular kind of
more positively towards patriotism. (B) is also incorrect abstaining —that from alcoholic drink, sex etc, often
because there is no description as such of a process. for health or religious reasons. If you abstain from
(c) is not true because the author is only giving an drinking, you do not get drunk. Abstinence does not
analysis of several elements of patriotism and not refer to “drink” only,
persuading the reader to believe anything persay. Thus 9, (a) The opposite of 'insipid' is 'tasty'. The word 'insipid'
(d) is the correct answer. means dull, boring or colorless.
10. (b) The opposite of 'relinquish' is 'continue'. Relinquish
means to 'give up'.
EBD_8177
PS-36 Practice Set - 4

11. (d) 47.6


12. (a) Add 'the' before 'more'. Here the sentence consists of \Q ´ x = 9520 Þ x = 20, 000
two clauses- Principal and Subordinate, where the 100
Principal clause should be given more stress by adding Hence, Ram’s monthly income = 20,000.
'the' before 'more'. 29. (d) Required consumption to be reduced
13. (b) Delete 'pair of' before binocular because the word 25 ´ 100 25 ´ 100
'binocular' itself suggests a pair. = = = 20%
100 + 25 125
14. (b) 'a' should be replaced with 'the'. Here Kalidas is not
30. (a) Least cost price = `(150×15) = `2250
Shakespeare but he is compared to Shakespeare.
Greatest selling price = `(350×15) = `5250
15. (b) Delete 'all' before 'left'. Here the usage of 'all' is
Required profit = `(5250 – 2250) = `3000
superfluous as 'the teacher as well as his students'
31. (b) Sum of 30 numbers = 30×40 = 1200
itself signifies everyone.
Sum of 40 numbers = 40 × 30 = 1200
16. (a) The word 'prejudice' will fill in the blank because here
it means something or opinion which is not based on 1200 + 1200 2400 2
Average of 70 numbers = = = 34
reason or experience and hence it seems to get 70 70 7
conflicted with original thoughts. 32. (d) In such type of question,
17. (d) The phrase 'stands as' will fill in the blank because it
means to signify.
Required % loss =
( 25)2
%
18. (d) The phrase 'as well as of a' will fill in the blank as 100
children are considered to shape the future of a nation
along with carrying the goodwill of a family. 625 1
= % = 6.25% = 6 %
19. (b) The word 'averted' will fill in the blank. It is also the 100 4
synonym for the word 'avoid' and hence it can be 33. (a) boys : girls girls : teacher
understood that if strict measures were taken then the 4 : 3 8 : 1
tragedy would have been avoided.
20. (a) The word 'behind' will fill in the blank. The sentence So, boys : girls : teacher
means that the deceased (the person who recently 4 : 3
died), left behind him two young children. 8 : 1
21. (b) 22. (d) 23. (b)
32 : 24 : 3
24. (d) 25. (d)
26. (a) Let the number be x. Then, as per the operation So, Student : teacher
undertook by the student, we have Þ (boys + girls) : teacher
(32 + 24) : 3
x + 112
= 112 Þ x = 660 56 : 3
6 34. (d) Since they are moving in opposite direction, therefore
660 their relative speed will be 4+3 = 7 km/hr.
Hence, the correct answer = + 12 = 122
6 d 17.5
Time = = = 2.5hrs.
27. (d) Let Nikhil has ` x and Yogesh has ` y s 7
According to question (where d is distance and s is speed).
1 1 \ They should meet at 12.30 PM.
x= y ...(i) 35. (d) Let the side of square = ‘x’
4 6
Area of square = x2
and x + y = 600 ...(ii)
From equation (i) and (ii), we get 130
New length of rectangle = x
Nikhil has ` x = ` 240 100
and Yogesh has ` y = ` 360
120
Difference between their amounts ` 360 – ` 240 = ` 120 New Breadth of rectangle = x
28. (c) Let Ram’s monthly income be ` 100 100
20 130 120
Household expenditure = 100 ´ = 20 Hence, Area of so formed rectangle = ´ ´ x2
100 100 100
Rest income = 100 – 20 = 80. 156 2
=
x
15 100
Books expenditure = 80 ´ = 12 . Therefore, area of rectangle exceeds the area of square
100
Rest income = 80 – 12 = 68. by 56%
36. (d) Weight of new crewmen
30 = Replace man weight + [No. of crew mewn ×
Clothes expenditure = 68 ´ = 20.4
100 increased average]
Rest saved income = 68 – 20.4 = 47.6 1
But finally saved income = 9520 = 55 + 12 ´ = 59 kg
3
Practice Set - 4 PS-37

37. (b) S.I. = 8800 – 8000 = ` 800 14 40x 35 ´ 14 y


S.I. ´ 100 = Þx=
800 ´ 100 21 35y 21 ´ 40
\R= = = 5% per annum
Pr incipal ´ Time 8000 ´ 2
21 35y 15 ´ 35y
New rate = 3% = Þ =z
15 30z 21 ´ 30
8000 ´ 3 ´ 2
\ S.I. = = ` 480 Put these value in eqn (i)
100
\ Amount = ` (8000 + 480) = ` 8480 35 ´ 14y 55 ´ 35y
+y+ = 1450
38. (a) Relative velocity = 20 + 30 = 50 m/s. 21 ´ 40 21 ´ 30
Distance = 2.5 kms. = 2500 m. 0.583y + y + 0.833y = 1450
t = 2500/50 = 50 s. 2.416y = 1450
39. (a) Let the original price be x and sale be of y units.
Then, the revenue collected initially = x × y 1450
\y= = ` 600.08
Now, new price = 0.8x, new sale = 1.8 y 2.416
Then, new revenue collected = 1.44xy 44. (b) Let the speed of the stream be x km/hr and distance
travelled be S km. Then,
0.44 xy
% increase in revenue = ´ 100 = 44% S S
xy = 6 and =9
40. (d) Given, ratio of speeds of Deepak and Sanjeet is 5 : 4. 12 + x 12 – x
\ Sanjeet makes 4 rounds when Deepak makes 5 rounds. 12 – x 6
Þ = Þ 108 – 9x = 72 + 6x
Now, distance covered by Deepak in 5 rounds 12 + x 9
æ 200 ö 36
= ç5× ÷ = 1 k.m Þ 15x = 36 Þ x = = 2.4 km / hr.
è 1000 ø 15
and distance covered by Sanjeet in 4 rounds 45. (b) Let the Speed of faster train be x and speed of slower
æ 200 ö train be y.
= ç 4× ÷ km = 0.8 km Now, when both the train move in same direction their
è 1000 ø relative speed = x – y
It is clear that in 5 rounds, Deepak passes Now, total distance covered = 130 + 110 = 240
Sanjeet only once. (i.e., 1 time). Now, distance = speed × time
In other words, in covering 1 km, Deepak pases Sanjeet
1 time. \ 240 = ( x– y) × 60 (Q1min = 60sec)
\ In covering 10 km, Deepak passes Sanjeet in (1 × 10) Þ x–y=4 …(1)
times When the trains move in opposite direction
i.e., 10 times. then their relative speed = x + y
\ 240 = ( x + y) × 3
1 2 Þ 80 = x + y …(2)
41. (b) Work of pipe P in 16 minutes = ´ 16 =
24 3 on solving eqn (1) and (2), we get x = 42 m/sec
1 1 and y = 38 m/sec
Similarly, work of pipe Q in 16 minutes = ´ 16 = 46. (c) In all other pairs, first is prepared by the second.
32 2 47. (d) In all other pairs, first is the alloy used to make the
1 1 second. (Iron is not an alloy but a metal.)
\ Remaining work of pipe Q = 1 – = 48. (b) In all other pairs, second is the name given to artificial
2 2
rearing of the first.
1 49. (a) In all other pairs, second is a collective group of the
Now, Time taken by pipe P = ´ 24 = 12 minutes.
2 first.
Hence, first pipe (i.e P) should be turned off after 12 50. (c) In all other pairs, second is the food over which the
minutes. first feeds.
42. (d) Let the length and breadth of the rectangle be x and 51. (d) All except Quart are units of measuring distances.
y cm, respectively. 52. (b) All except Bromine are metals, while bromine is a non-
Then, (x – 4) (y + 3) = xy Þ 3x – 4y = 12 metal.
........... (i) 53. (a) All except Mariana are waterfalls, while Mariana is a
Also, (x – 4) = (y + 3) [sides of square] trench.
Þx–y=7 .......... (ii) 54. (b) All except Diagonal are terms associated with circle.
From (i) and (ii), 55. (a) All except Granite are different types of coal, while
x = 16 and y = 9 granite is a rock.
Perimeter of the original rectangle = 2(x + y) = 50 cm 56. (d) These are leafy vegetables.
43. (b) Let the salaries of A, B and C are x, y and z respectively. 57. (b) They are connected with church.
x + y + z = 1450 …(i) 58. (b) They are highly inflammable.
14 : 21 : 15 = 40 % of x : 35% of y : 30% of z 59. (d) These are halogens.
EBD_8177
PS-38 Practice Set - 4

60. (b) These give information about living things for the last item is numbered 47) given in Part XI of the
classification. Constitution of India, concerned with relations
61. (d) between the Union and States. This part is divided
62. (a) between legislative and administrative powers. The
63. (b) legislative section is divided into three lists: Union
64. (c) List, State List and Concurrent List.
82. (a) Calcutta with Robert Clive is wrongly matched. Lord
Curzon was associated with Calcutta. Lord Curzon
felt that the Bengal province was too big to be
administered efficiently and so he wanted to split it
into two provinces, one of which had Dacca as its
capital.
65. (a)
83. (d) The Crimean War came to an end by the treaty of
Paris. Crimean War, (October 1853-February 1856), war
fought mainly on the Crimean Peninsula between the
Russians and the British, French, and Ottoman
Turkish, with support from January 1855 by the army
of Sardinia-Piedmont.The resulting Treaty of Paris,
signed on March 30, 1856, guaranteed the integrity of
66. (d) As 4 dots are formed with intersecting lines in the 4 Ottoman Turkey and obliged Russia to surrender
sided figure so 6 dots will form with intersecting lines southern Bessarabia, at the mouth of the Danube.
in the 6 sided figure. Again the dots will be in the 84. (a) The Himalayan Mountain Range is an example of fold
middle of each side as in Figure 2. mountain.They are known as fold mountains because
67. (a) A box surrounds the figure and the inner figure is the mountains extend for 2500 km in length in a series
removed, only boundary is left. of parallel ridges or folds and consist of three folds
68. (c) Shaded position is moved 45 degrees in anticlockwise namely Himadri, Himachal, Shiwalik.
direction. 85. (a) The depletion of Ozone layer is mainly due to
69. (b) Outer figure one side is increased and inner lines one chlorofluorocarbons. A chlorofluorocarbon is an
line is decreased and line position also change from organic compound that contains only carbon,
vertical to horizontal. chlorine, and fluorine, produced as a volatile derivative
70. (d) One side is increased in the figure and outer line taken of methane, ethane, and propane. They are also
inside and kept down opening upwards in ‘V’ shape. commonly known by the DuPont brand name Freon.
71. (b) All the figures have shaded circle placed opposite to 86. (d) Sankaradeva was an erudite scholar, a prolific writer, a
each other. Hence (b) is odd one out. versatile saint-poet of unlimited merit, a lyricist of
72. (b) In every shaded sector have three unshaded sector universal acceptance, a musician of high calibre, a
between them. Hence, (b) is odd one out. pioneer in the field of Assamese prose, drama and
73. (d) All images have double sided arrow except the last dramatic performances, a painter and above all the
one. greatest religious teacher-preacher-leader of the
74. (c) Every figure have some sides except (c). medieval Vaisnava movement in Assam which is rightly
75. (d) Each images have 3 lines intersecting while (d) have 4 known as the Sankaradeva Movement.
lines intersecting. 87. (c) Saffron, a spice derived from the dried stigmas of the
76. (d) Five line segments are added in each step to complete saffron crocus. (crocus sativus) a small plant about a
the squares in an ACW direction. foot tall. Each flower has three female parts. (stigmas)
77. (d) One of the pins gets inverted in each step. The pins two male parts. (stamens) each stigmas is red or dark
gets inverted sequentially from right to left. red in color towards the top and yellow towards the
78. (c) In one step, the existing element enlarges and a new bottom of the stigma, where it is attached to the flower.
element appears inside this element. In the next step, 88. (d) 89. (c) 90. (d)
the outer element is lost 91. (b) Two home Rule leagues were started in 1915-16 – one
79. (d) In each step, both the elements move one space (each under the leadership of Lokmanya Tilak at Poona and
space is equal to half-a-side of the square boundary) other under the leadership of Anne Basent and S.
downwards. Once any of the two elements reaches Subramanya Iyer at Madras.
the lowermost position, then in the next step, it reaches 92. (c) Three-tier system of Panchayati Raj consists of Gram
the uppermost position in the next column to the right. Panchayat, Panchayat Samiti and Zila Parishad.
80. (b) Similar figure repeats in every third step and each time 93. (d) ‘Sarva Siksha Abhiyan’ is aimed at the education of
a figure reappears it gets vertically inverted. children between 6-14 years.
81. (d) The concept of Concurrent List in Indian Constitution 94. (b) 95. (a) 96. (a)
is borrowed from the Constitution of Australia. The 97. (a) 98. (a) 99. (a)
Concurrent List or List-III is a list of 52 items(though 100. (c) Middle luminous zone of a candle flame is the hottest.
Practice Set - 5 PS-39

5 Practice Set
INSTRUCTIONS
1. This practice set comprises four sections. Section A : Verbal Ability in English; Section B : Numerical Ability; Section C :
Reasoning and Military Aptitude; Section D : General Awareness.
2. The set will consist of 100 questions and each questions will be of 3 marks.
3. Each questions have four options, of which one is correct. The students are advised to read all the options thoroughly.
4. There is one-third negative marking in the set.

Time: 2 hrs. Max. Marks: 300


Ti

SECTION-A : VERBAL ABILITY IN ENGLISH 4. What happened after the man had snipped off the remaining
bit of the cocoon?
DIRECTIONS (Qs. 1 - 5) : Read the following passages carefully (a) The butterfly died instantly
and answer the questions given below it : (b) The butterfly flew in to the air
A man found a cocoon of a butterfly. One day a small opening (c) The butterfly emerged easily
appeared; he sat and watched the butterfly for several hours as it (d) The butterfly thanked the man
struggled to force its body through that little hole. Then it seemed 5. What did the man not understand in his kindness and haste?
to stop making any progress. (a) It was nature's way of forcing fluid from the body of
Then the man decided to help the butterfly, so he took a pair of the butterfly into its wings.
scissors and snipped off the remaining bit of the cocoon. The butterfly (b) It was nature's way of forcing liquid from the body of
then emerged easily. But it had a swollen body and small, shrivelled the butterfly into its wings.
wings. The man continued to watch the butterfly because he expected (c) It was nature's way of forcing mobile from the body of
that, at any moment, the wings would enlarge and expand to be able the butterfly into its wings.
to support the body, which would contract in time. (d) It was nature's way of forcing fluid from the wings of
Neither happened! It never was able to fly.
the butterfly into its body.
What this man in his kindness and haste did not understand was
that the restricting cocoon and the struggle required for the DIRECTIONS (Qs. 6 - 11) : Find out which part of a sentence
butterfly to get through the tiny opening were nature's way of has an error :
forcing fluid from the body of the butterfly into its wings.
6. It was he who / came running in the house /
Sometimes struggles are exactly what we need in our life.
1. What did the man find? ( a) ( b)
(a) a coconut (b) a coco plum
(c) a cocoon (d) a coco palm with the news about the earthquake. / No Error.
2. What did the man see one day? ( c) ( d)
(a) One day he saw a wide opening appear
(b) One day he saw a large opening appear 7. Her mother does not approve of / her to go to the party /
(c) One day he saw a small insect go inside the opening (a) ( b)
(d) One day he saw a small opening appear
3. What did the man do to help the butterfly? without dressing formally. / No Error.
(a) The man took a pair of scissors and snipped off the ( c) ( d)
whole of the cocoon.
(b) The man took a pair of scissors and snipped off the 8. Riding across the battle field / the famous Bhishm /
remaining bit of the cocoon.
(c) The man took a hammer and cracked the remaining bit
(a ) ( b)
of the cocoon. saw a large number of dead warriors. / No Error.
(d) The man took a needle and snipped off the remaining
bit of the cocoon.
( c) ( d)
EBD_8177
PS-40 Practice Set - 5

My Aunt / was first / to get a degree / No Error. 19. Regular exercise is conducive__________ heath.
9. (a) in (b) to
(a) ( b) ( c) ( d) (c) for (d) of
Padmini had not rarely missed / 20. Can you please __________ my web site just before I
10.
publish it ?
(a) (a) find out (b) go through
a dance performance or festival since / (c) set out (d) look up
( b) DIRECTIONS (Qs. 21 & 22): Choose the word which is nearest
in meaning to the given word.
she was eight years old. / No Error.
21. ANIMATE
( c) ( d) (a) kill (b) dead
11. Krupa and Kavya studied / in the Delhi Public School / (c) energise (d) calm
22. NIGGARDLY
(a) ( b) (a) penurious (b) generous
and so does Kamya. / No Error. (c) liberal (d) nimbus
( c) ( d) DIRECTIONS (Qs. 23 - 25) : Choose the word which is nearly
opposite in meaning to the given word.
DIRECTIONS (Qs. 12 - 15) : Select the most appropriate word 23. INDIGENTLY
from the options against each number : (a) richly (b) awfully
I peered at the river through a gap in the roots of the strangler fig. (c) completely (d) diligency
The thick branches of the tree 12 me without really protecting 24. AUDACITY
me. I noticed the gentlest of ripples in the water and then something (a) quivering (b) patricide
13 began to emerge from its depths. An enormous head upon (c) bravado (d) cowardice
a slender neck rose about the surface. I looked into the beast’s 25. ELEVATION
cold, reptilian eyes. I could sense no mind or soul behind them. I (a) depression (b) deflation
had to refrain from letting out a frightened 14 ! The time for (c) depreciation (d) recession
retreating into the depths of the forest had arrived and I knew SECTION-B : NUMERICAL ABILITY
that it was 15 .
12. (a) either sooner rather than later 1 1
26. If 3x – = 6, then the value of 4x – is
(b) either now or never 4y 3y
(c) neither now or never (a) 2 (b) 4
(d) neither sooner nor later (c) 6 (d) 8
13. (a) magnificent yet terrifying 27. A shopkeeper marks an article at a price which gives a profit
(b) frightening yet scary of 25%. After allowing certain discount, the profit reduces
(c) wonderful yet grand
1
(d) amazing yet astonishing to 12 % . The discount percent is
14. (a) “Phew!” (b) “Aah!” 2
(c) “Aha!” (d) “Ouch!” (a) 12% (b) 12.5%
15. (a) were hiding (b) hiding (c) 10% (d) 20%
(c) hidden (d) was hiding 28. The smallest positive integer which when multiplied by 392,
gives a perfect square is
DIRECTIONS (Qs. 16 - 20) : Choose the word/words which (a) 2 (b) 3
complete the sentence : (c) 5 (d) 7
29. If A : B = 6 : 7 and B : C = 8 : 9. Find the value of C : A?
16. Let us quickly __________.
(a) muddle (b) huddle 9 16
(a) (b)
(c) hurdle (d) puddle 6 21
17. Rajesh’s car wasn’t __________ Ramesh’s, so we were too
exhausted by the time we reached home. 21 6
(c) (d)
(a) such comfortable (b) as comfortable as 16 9
(c) comfortable enough (d) so comfortable that 30. Out of a group of swans, 7/2 times the square root of the
18. I don’t suppose that Pramod will be elected __________ number is playing on the shore of the pond. The two
how hard he struggles as he is not completely supported remaining are inside the pond. What is the total number of
by the committee. swans?
(a) although (b) seeing as (a) 10 (b) 14
(c) no matter (d) however (c) 12 (d) 16
Practice Set - 5 PS-41

31. Anthony got 30% of the maximum marks in an examination 41. What is the missing figure in the expression given below ?
and failed by 10 marks. However, Amar who took the same 16 16 * 9 9 9
examination, got 40% of the total marks and got 15 more ´ - ´ + ´ =1
than the passing marks in the examination. What were the 7 7 7 7 7 7
passing marks in the examination ? (a) 1 (b) 7
(a) 35 (b) 250 (c) 4.57 (d) 32
42. If the ratio of boys to girls in a class is B and the ratio of girls
(c) 75 (d) 85
to boys is G, then 3 (B + G) is :
32. The length of the ractangular field is increased by 25%. By (a) equal to 3 (b) less than 3
what per cent must the width be reduced so that the area in
each case remains the same? 1
(c) more than 3 (d) less than
(a) 15% (b) 17% 3
(c) 18% (d) 20% 43. In a mixture of 45 litres, the ratio of milk and water is 4 : 1.
How much water must be added to make the mixture ratio
33. A machine is sold at a profit of 10%. Had it been sold for
3:2?
`40 less, there would have been a loss of 10%. What was
(a) 72 litres (b) 24 litres
the cost price ?
(c) 15 litres (d) 1.5 litres
(a) ` 320 (b) ` 200 44. Two men starting from the same place walk at the rate of
(c) ` 225 (d) ` 250 5 km/h and 5.5 km/h respectively. What time will they take
34. A train 100 metres long moving at a speed of 50 km/hr. crosses to be 8.5 km apart, if they walk in the same direction?
a train 120 metres long coming from opposite direction in 6 (a) 16 h (b) 8 h 30 min
sec. The speed of the second train is (c) 4h / 5min (d) 17 h
(a) 60 km/hr. (b) 82 km/hr. 45. If the cost of 12 pencils is equal to the selling price of 10
(c) 70 km/hr. (d) 74 km/hr. pencils, the profit percent in the transaction is :
35. A shopkeeper marks up his goods to gain 35%. But he allows 1 1
10% discount for cash payment. His profit on the cash (a) 16 % (b) 22 %
transaction in percentage, is 3 2
(c) 20% (d) 25%
(a) 1 (b) 25
13 SECTION-C : REASONING
2
& MILITARY APTITUDE
1 1
(c) 21 (d) 31 DIRECTIONS (Qs 46 -50) : The words in the bottom row are
2 2
related in the same way as the words in the top row. For each
1 item, find the word that completes the bottom row of words.
36. Dhiraj purchased 150 kg of rice. He sold rd of it at 10%
3 46. ant Fly bee
loss. At what per cent of profit must he sell the remaining hamster squirrel ?
rice so that he can make 10% profit on the whole ? (a) Spider (b) mouse
(a) 20% (b) 15% (c) rodent (d) cat
(c) 10% (d) None of these 47. Tadpole Frog Amphibian
37. The average monthly salary of employees, consisting of Lamb Sheep ?
officers and workers, of an organisation is ` 3000. The (a) Animal (b) Wool
average salary of an officer is ` 10,000 while that of a worker (c) Farm (d) mammal
is ` 2000 per month. If there are total 400 employees in the 48. Snow Mountain Ski
organisation, find the number of officers. Warmth Lake ?
(a) 60 (b) 50 (a) Sand (b) Swim
(c) 80 (d) 40 (c) Sunburn (d) Vacation
38. A man gains 10% by selling a certain article for a certain price. 49. Walk Skip Run
If he sells it at double the price, then the profit made is: Toss Pitch ?
(a) 120% (b) 60% (a) Swerve (b) Hurl
(c) 100% (d) 80% (c) Jump (d) Dance
39. A 4 cm cube is cut into 1 cm cubes. Find the percentage 50. Rule Command Dictate
decrease in surface area. Doze Sleep ?
(a) 200% (b) 94% (a) Snore (b) Govern
(c) 400% (d) 300% (c) Awaken (d) Hibernate
40. In what time will ` 500 give ` 50 as interest at the rate of 5% DIRECTIONS (Qs 51 - 53) : Three of the words will be in the
per annum on simple interest ? same classification, the remaining one will not be. Your answer
(a) 2 years (b) 3 years will be the one word that does NOT belong in the same classifi-
(c) 4 years (d) 5 years cation as the others.
EBD_8177
PS-42 Practice Set - 5

51. Which word does NOT belong with the others? DIRECTIONS (Qs. 61 - 65) : Which answer figure will complete
(a) book (b) index the pattern in the question figure?
(c) glossary (d) chapter
52. (a) unimportant (b) trivial 61. Question Figure :
(c) insignificant (d) familiar
53. (a) biology (b) chemistry
(c) theology (d) zoology
54. Yesterday I saw an ice cube which had already melted due
to heat of a nearby furnace.
Answer Figures :
(a) Always (b) Never
(c) Often (d) Sometimes
55. What is always in worry?
(a) Difficulty
(b) Unrest
(c) Non-cooperation (a) (b) (c) (d)
(d) Poignancy 62. Question Figure :
56. Which one of the following is always found in 'Remedy of
fault'?
(a) Punishment (b) Remedy
(c) Fault (d) Scolding
57. A mirror always
(a) Retracts (b) Distorts Answer Figures :
(c) Refracts (d) Reflects
58. Danger always involves
(a) Enemy (b) Attack
(c) Fear (d) Help
DIRECTIONS (Qs. 59 & 60) : In each of the question below is (a) (b) (c) (d)
given a statement followed by two assumptions numbered I and 63. Question Figure :
II. Consider the statement and the following assumptions and
decide which of the assumptions is implicit in the statement.
59. Statement: "If you trouble me, I will slap you." - A mother
warns her child.
Assumptions: Answer Figures :
I. With the warning, the child may stop troubling her.
II. All children are basically naughty.
(a) Only assumption I is implicit
(b) Only assumption II is implicit
(c) Either I or II is implicit
(d) Neither I nor II is implicit (a) (b) (c) (d)
60. Statement: The State government has decided to appoint 64. Question figure
four thousand primary school teachers during the next fi-
nancial year.
Assumptions:
I. There are enough schools in the state to accommo-
date four thousand additional primary school teach-
ers.
Answer Figure
II. The eligible candidates may not be interested to apply
as the government may not finally appoint such a large
number of primary school teachers.
(a) Only assumption I is implicit
(b) Only assumption II is implicit
(c) Either I or II is implicit
(a) (b) (c) (d)
(d) Neither I nor II is implicit
Practice Set - 5 PS-43

65. Question Figure :


68.

Answer Figures :

(1) (2) (3) (4)


(a) (b) (a) 1 (b) 2
(c) 3 (d) 4

69.
(c) (d)

DIRECTIONS (Qs. 66 - 70) : In each of the following questions,


find out which of the answer figures (1), (2), (3) and (4) completes
the figure matrix.

66.

(1) (2) (3) (4)


(a) 1 (b) 2
(c) 3 (d) 4

70.

(1) (2) (3) (4)


(a) 1 (b) 2
(c) 3 (d) 4

67.

(1) (2) (3) (4)


(a) 1 (b) 2
(c) 3 (d) 4
DIRECTIONS (Qs. 71-75) : In there tests find which code
matches the shape or pattern given at the end of each questions.

71.

(1) (2) (3) (4) SL OP ON TN SP ?


(a) 1 (b) 2 (a) TP (b) SN
(c) 3 (d) 4 (c) TL (d) TN
EBD_8177
PS-44 Practice Set - 5

72. 80. Select a suitable figure from the four alternatives that
would complete the figure matrix
Question figure :
AC TP GL GK AG ?
(a) AG (b) TA
(c) GA (d) TG
73.
?
Answer figures :

TK GR JS TR GS ?
(A) (B) (C) (D)
(a) GK (b) JK
(a) A (b) B
(c) RS (d) JR (c) C (d) D
74.
SECTION-D : GENERAL AWARENESS
81. Hiuen Tsang visited the Pallava kingdom during the reign of ?
(a) Narasimhavarman I (b) Mahendravarman I
(c) Paramesvarvarman II (d) Nandivarman II
CF TL CP EL EP ? 82. The first sermon of Buddha made at Saranath is called
(a) TF (b) LT (a) Dharmachakra Parivartan (b) Dharma Sansthapan
(c) TL (d) EL (c) Dharma Sabha (d) Maha Parinirvan
83. When Alexander invaded india, who were the rulers of
75.
Magadha ?
(a) Haryankas (b) Shishunagas
TK KC LP PK TP ? (c) Nandas (d) Maurya
(a) PC (b) PK 84. Bhakta Tukaram was a contemporary of which Mughal
(c) TP (d) LT emperor?
DIRECTIONS (Qs. 76 - 79) : In each of the following questions, (a) Babar (b) Akbar
find out how will the key figure (x) look lite after rotation. (c) Jahangir (d) Aurangzeb
76. 85. Who founded the Fort William College at Calcutta?
(a) Lord Cornwallis (b) Lord Ellenborough
(c) Lord Macaulay (d) Lord Wellesley
86. Who abolished the Dual Government of Bengal?
(a) Cornwallis (b) Robert Clive
(c) Warren Hastings (d) John Macfersson
(X) (a) (b) (c) (d) 87. In 1930, Mahatma Gandhi started Civil Disobedience
77.
Movement from:
(a) Sevagram (b) Dandi
(c) Sabarmati (d) Wardha
88. Which comet appears every 76 years?
(X) (a) (b) (c) (d) (a) Halley’s (b) Holme’s
78. (c) Donati’s (d) Alpha Centauri
89. The waterfall ‘Victoria’ is associated with the river
(a) Amazon (b) Missouri
(c) St. Lawrence (d) Zambezi
90. Which one of the following rights has been described by
(X) (a) (b) (c) (d) Dr. Ambedkar as 'The heart and soul of the constitution'?
79. (a) Rights of Equality
(b) Right to freedom
(c) Right to property
(d) Right to Constitutional Remedies
91. The electric charge is stored in a device called
(a) Inductor (b) Capacitor
(X) (a) (b) (c) (d) (c) Resister (d) Transformer
Practice Set - 5 PS-45

92. The folk dance ‘Chhau’ belongs to (a) The Narrow Road to the Deep North
(a) Odisha (b) Uttarakhand (b) The Sound of One Hand Clapping
(c) Jharkhand (d) Assam (c) Death of a River Guide
93. Which country is the winner of Davis Cup- 2019? (d) The Unknown Terrorist
(a) Serbia (b) USA 97. Patrick Madiano has been awarded with Nobel Prize in which
(c) Spain (d) Russia of the following category?
94. Who among the following players has received the Arjuna (a) Medicine (b) Economics
Award in 2019? (c) Physics (d) Literature
(a) Ajay Thakur (b) Mary Kom 98. Fertilization occurs normally in the
(c) M S Dhoni (d) Sardar Singh (a) Cervix (b) Vagina
(c) Fallopian tube (d) Uterus
95. Dr. APJ Abdul Kalam was formally conferred the title of
99. Which one of the following correctly describes AGNI ?
Honorary Professor by which of the following university?
(a) A fighter plane (b) A versatile tank
(a) Shanghai University (c) A long-range missile (d) A long-range gun
(b) University of Hong Kong 100. According to a resolution adopted by the United Nations
(c) Peking University General Assembly, ‘International Day of Peace’ is observed
(d) Tsinghua University every year on
96. Richard Flanagan recently won Man Booker Prize for which (a) September 1 (b) September 14
of the following book? (c) September 21 (d) None of these

ANSWERS & SOLUTIONS


1. (c) A cocoon synonym. In such cases we go for the word closest in
2. (d) One day he saw a small opening appear meaning. Both penurious (very poor) and niggardly
3. (b) The man took a pair of scissors and snipped off the imply a lack of money, though in different senses.
remaining bit of the cocoon. 23. (a) One who is indigent is very poor. The opposite of
4. (c) The butterfly emerged easily poor is rich.
5. (a) It was nature's way of forcing fluid from the body of 24. (d) Audacity is audacious behaviour. If you are audacious,
the butterfly into its wings you take risks in order to achieve something. So it is a
6. (b) 7. (b) 8. (d) 9. (b) 10. (a) 11. (c) kind of bravery. Now, the opposite of bravery is
12. (b) When you say 'either now or never', it means that you cowardice.
must do something immediately because you will not 25. (a) An elevation is a ‘piece of ground that is higher
get another opportunity. Other options are simply out (elevated) than the area around it. A depression in a
of context. surface is an area which is lower (depressed) than the
13. (a) When two pairs of words are connected with 'yet', parts surrounding it.
they should usually be in contrast; despite anything
to the contrary (usually following a concession); e.g. 1 1
26. (d) 3x – =6 Þ 3x = 6 +
He was a stern yet fair master. Other options are just 4y 4y
not relevant.
14. (a) In gap 11 the interjection 'phew!' rightly fits as it is 6 1 1
used to express relief, fatigue, surprise, or disgust x= + Þ x = 2+
3 4.3 y 12 y
which is also the case here. Other options are out of
context. Multiplying equation by 4 on both sides
15. (a) Here 'were hiding' is the right form of the verb. Other
options do not fit in here. 1 1
16. (b) 17. (b) 18. (c) 19. (b) 20. (b) 4x = 8 + Þ 4x – =8
3y 3y
21. (c) As an adjective, animate simply means “having life”.
For example: Plants and animals are animate objects. If 27. (c) Short cut method :
you animate something, you make it lively or more
cheerful. Thus you put energy into it. In other words, Profit ´ Discount
Net profit = Profit – Discount –
you energise it. 100
22. (a) If someone is niggardly, he shows lack of generosity.
Thus generous (b) would be its antonym. But we are 25 25×Discount
looking for a synonym. None of the words is a fit = 25 – Discount–
2 100
EBD_8177
PS-46 Practice Set - 5

Distance = Speed × Time


25 –5
– 25 = Discount
2 4 æ 220 ö æ 6 ö
ç ÷ km = (50 + x) km/hr. ´ ç ÷ hr
\ Discount % = 10% è 1000 ø è 3600 ø
28. (a) 392 × 2 = 784 = (28)2
220 ´ 3600
Hence, 2 can be multiplied by 392 which gives perfect 50 + x =
1000 ´ 6
square.
29. (c) A : B = 6 : 7 50 + x = 132
B: C= 8 :9 x = 132 – 50 = 82 km/hr
35. (c) Let the cost price be `100.
A B 6 8 48 \ Marked price is ` 135.
Þ ´ = ´ =
B C 7 9 63 At 10% discount, the customer has to pay
= Marked price – discount = 135 – 13.5 = 121.5.
63 21
\C: A= =
48 16 1
\ % profit = 21.5% = 21 %.
2
30. (d) Let total Number of Swans is x.
then, 36. (a) Let the C.P. of 150 kg of rice be `150.
\ S.P. of 50 kg of rice at 10%
7/2 x = x–2
90
7 x = 2x – 4 loss = ´ 50 = ` 45
100
Squaring both side For 10% of gain on the whole.
49x = 4x2 – 16x + 16
Þ 4x2 – 65x + 16 = 0 110
S.P. = 150 ´ = ` 165
Þ 4x2 – 64x – x + 16 = 0 100
Þ 4x (x – 16) – 1(x – 16) = 0 \ 100 kg rice should be sold for ` 120.
(4x – 1) (x – 16) = 0 \ Per cent gain = 20
\ x = 1/4, 16 37. (b) Let the number of officers be x.
So, total number of swans = 16 Number of workers = 400 – x
31. (d) Let the max. marks be x. Þ 10000 × x + 2000(400 – x) = 3000(400)
Then, according to question,
x × 30% + 10 = x × 40% – 15 Þ 10000x + 800000 – 2000x = 12,00,000
Þ x × 10% = 25 or x = 250 Þ 4x = 600 – 400 = 200 Þ x = 50
\ Number of officers = 50
30
Therefore, passing marks = 250 ´ + 10 = 85 38. (a) Let the cost price of an article be ` 100
100
then, S.P. = 100 + 10 = ` 110
32. (d) Area remains same
If S.P. = 2 × 110 = ` 220
25x
25 - x - =0 (220 - 100)
100 then, profit % = ´ 100 = 120%
100
x
Þ x+ = 25 39. (b) Surface area with side 4 cm = 6 × 42 = 96 cm2
4
Now, surface area with side 1 cm = 6 × 12 = 6 cm2
Þ 5x = 25 × 4
Decrease = 96 – 6 = 90 cm2
Þ x = 20%
33. (b) Let the cost price of machine be `100 90
SP of machine at a profit of 10% = ` 110 Decrease % = ×100 = 94%
96
SP of machine at a loss of 10% = ` 90
If SP is (110 – 90) = ` 20 less then CP = ` 100 500 ´ 5 ´ t
40. (a) Here, 50 = Þ t = 2 years
Therefore, if SP is ` 40 less then 100
100 41. (d) Let the missing figure in the expression be x.
CP = ´ 40 = ` 200
20 16 16 x 9 9 9
34. (b) Let speed of the second train = x km/hr. ´ - ´ + ´ =1
7 7 7 7 7 7
Relative speed of trains = (50 + x) km/hr.
Distance travelled by trains = (100 + 120) = 220 metres Þ 16 × 16 – 9x + 9 × 9 = 7 × 7
Practice Set - 5 PS-47

Þ 9x = 16 × 16 + 9 × 9 – 7 × 7 = 256 + 81 – 49 = 288 48. (b) The relationship above the line is that snow on a moun-
tain creates conditions for skiing. Below the line, the
Þ x = 228 = 32 relationship is that warmth at a lake creates conditions
9 for swimming.
42. (c) Let the number of boys be x and the number of girls 49. (b) Walk, skip, and run represent a continuum of move-
be y. ment: Skipping is faster than walking; running is faster
than skipping. Below the line, the continuum is about
æ x y ö 3(x 2 + y2 ) throwing: Pitch is faster than toss; hurl is faster than
Then, 3(B + G) = 3 ç + ÷ = ,
èy xø xy pitch.
50. (d) The words above the line show a continuum: Com-
Clearly, which is greater than 3. mand is more extreme than rule, and dictate is more
Q [ (a - b) 2 ³ 0 Þ a 2 + b 2 - 2ab ³ 0 extreme than command. Below the line, the continuum
is as follows: Sleep is more than doze, and hibernate is
a 2 + b2 more than sleep. The other choices are not related in
Þ a 2 + b 2 ³ 2ab Þ ³2] the same way
ab
51. (a) An index, glossary, and chapter are all parts of a book.
4 Choice a does not belong because the book is the
43. (c) Quantity of milk = 45 ´ = 36 litres whole, not a part.
5
52. (d) The first three choices are all synonyms.
1 53. (c) Biology, chemistry, and zoology are all branches of
Quantity of water = 45 ´ = 9 litres science. Theology is the study of religion.
5
Let x litres of water be added to make the ratio 3 : 2 54. (b) Since the ice cube had already melted due to the heat
of a nearby furnace so after this ice cannot remain as
36 3 ice cube.
Then, =
9+ x 2 55. (b) worry involves to keep thinking about unpleasant
things that creates always unsest.
Þ 72 = 27 + 3x Þ x = 15 litres
56. (c) Remedy of fault means solution for fault. So, fault is
44. (d) Relative speed = 5.5 – 5 = 0.5 km/h.
always found in it. Hence, option C is answer.
8.5 57. (d) A mirror always has refelection of an image.
Required time = = 17h 58. (c) Danger always creates fear.
0.5
59. (a) The mother warns her child with the expectation that
45. (c) Let CP of 12 pencils = SP of 10 pencils = ` 1 he would stop troubling her. So, I is implicit. The gen-
1 eral nature of children cannot be derived from the state-
Therefore, CP of 1 pencil = ` and ment. So, II is not implicit.
12
60. (a) Such decisions as given in the statement are taken
1 only after taking the existing vacancies into consider-
SP of 1 pencil = `
10 ation. So, I implicit while II isn't.

1 1 2 1 61 (d)
Profit on one pencil = - = =`
10 12 120 60

% profit = 1/ 60 ´ 100 = 20%


1/12
46. (b) The three above the line are all insects. The hamster
and squirrel are rodents, so the correct choice is b
because the mouse is also a rodent. The other three
choices are not rodents. 62. (d)
47. (d) The tadpole is a young frog; frogs are amphibians. 63. (c)
The lamb is a young sheep; sheep are mammals. Animal
(choice a) is incorrect because it is too large a grouping: 64. (d)
Animals include insects, birds, mammals, reptiles, and
amphibians. Choices b and c are incorrect because
they are not part of the progression.
EBD_8177
PS-48 Practice Set - 5

74. (a)
F
TF
65. (c)
T

66. (d) In each row, the second figure forms the innermost 75. (a)
and the outermost elements of the third figure and P
the first figure forms the middle element of the third PK
figure. K
67. (b) In each row, the second figure is obtained by shading
one of the four parts of the first figure and the third
76. (d) 77. (d) 78. (b) 79. (d)
figure is obtained by shading two out of the four parts
of the first figure. 80. (d) The third figure in each row comprises of parts which
are not common to the first two figures.
68. (d) In each row, there are three types of outer elements
(circle, triangle and square), three types of inner ele- 81. (a)
ments (circle, triangle and square) and three types of 82. (a) The first sermon of Buddha made at Sarnath is called
shadings in the inner elements (black, white and lines). Dharmachakra Parivartan.
69. (b) In each row, the second figure is obtained by rotating 83. (c) When Alexander invaded India, Nandas were the rulers
the first figure through 90°CW or 90° ACW and add- of Magadha.
ing a circle to it. Also, the third figure is obtained by 84. (c) Tukaram (1608–1650) was a Marathi Bhakti poet and a
adding two circles to the first figure (without rotating devotee of Lord Krishna. Time period of Jahangir was
the figure). 1605-1627.
70. (d) There are 3 types of faces, 3 types of hands and 3 85. (d) Lord Wellesley founded Fort William College at
types of legs. Each type is used once in each row! So, Calcutta.
the features not used in the first two figures of the 86. (b)
third row would together form the missing figure. 87. (c) On 12 March, 1930, Gandhi started his civil
disobedience movement by starting Dandi March from
71. (c) Sabarmati Ashram in Gujarat and reached Dandi on 6
April 1930 and broke the salt law.
88. (a)
TN T 89. (d) Victoria waterfalls is associated with the river Zambezi
TL which is situated in Africa.
90. (d) Right to 'constitutional Remedies' has been
described by Dr. Ambedkar as "The heart and soul of
SL L the constitution.
91. (b) Capacitor is a device which stors electric charge.
92. (c) 93. (c) 94. (a)
72. (d) T
95. (c) Peking University, one of the oldest Chinese
TG universities. conferred the title of Honorary Professor
G to Dr.Kalam.
96. (a) The Narrow Road to the Deep North is set in the
background of World War II and give details about life
73. (b) of Prisoners of War.
97. (d) Patrick Madiano who has been hailed as "Marcel
Proust of our time" has been awarded with the Nobel
Prize for Literature.
98. (c) 99. (c) 100. (c)

You might also like